Final Study Questions

¡Supera tus tareas y exámenes ahora con Quizwiz!

A nurse is collecting data from a client in the health clinic who is reporting epigastric pain. Which of the following statements made by the client should the nurse identify as being consistent with peptic ulcer disease? 1) "The pain is worse after I eat a meal high in fat." 2) "My pain is relieved by having a bowel movement." 3) "I feel so much better after eating." 4) "The pain radiates down to my lower back."

"I feel so much better after eating"

A nurse is reinforcing teaching to a client who is preoperative prior to a transurethral resection of the prostate (TURP). Which of the following statements indicates as an understanding of the information?

"I will feel the urge to urinate following this procedure."

A nurse is caring for a client who is scheduled for a colonoscopy. The client asks the nurse if there will be a lot of pain during the procedure. Which of the following responses should the nurse make? 1) "You shouldn't feel any pain since the local area is anesthetized." 2) "Most clients report more discomfort from the preparation than from the procedure itself." 3) "You may feel some cramping during the procedure." 4) "Don't worry; you won't remember anything about the procedure due to the effects of the medication."

"You may feel some cramping during the procedure"

A nurse is reinforcing teaching about the prostate-specific antigen (PSA) test with a client. Which of the following statements should the nurse make?

"You should not ejaculate for 24 hours after the PSA test"

The nurse is preparing a teaching plan for a client who is newly diagnosed with Type 1 diabetes mellitus. Which signs and symptoms should the nurse describe when teaching the client about hypoglycemia? A) Sweating, trembling, tachycardia. B) Polyuria, polydipsia, polyphagia. C) Nausea, vomiting, anorexia. D) Fruity breath, tachypnea, chest pain.

A) Sweating, trembling, tachycardia. Sweating, dizziness, and trembling are signs of hypoglycemic reactions related to the release of epinephrine as a compensatory response to the low blood sugar (A). (B, C, and D) do not describe common symptoms of hypoglycemia.

3. Following a patient's bone marrow aspiration, which of the following nursing interventions should a nurse anticipate? A. Application of firm pressure to the site B. Positioning the patient in a prone position C. Positioning the patient in a supine position D. Application of a warm, moist compress to the site

A. Application of firm pressure to the site After a bone marrow aspiration procedure, a nurse should apply pressure to the aspiration site until bleeding stops. Application of a warm, moist compress will not alter the potential for bleeding. Positioning the patient to assume a supine or prone position also will not address the need to control bleeding from the aspiration site.

An elderly male client tells the nurse that he often wakes up during the night. What action should the nurse implement first? A. Ask the client to describe what happens when he awakens B. Encourage the client to describe his bedtime routines and habits C. Instruct the client to keep a daily sleep journal for one week D. Reassure the client that sleep needs often decrease with age

A. Ask the client to describe what happens when he awakens

A client diagnosed with angina pectoris complains of chest pain while ambulating in the hallway. Which action should the nurse implement first? A. Support the client to a sitting position. B. Ask the client to walk slowly back to the room. C. Administer a sublingual nitroglycerin tablet. D. Provide oxygen via nasal cannula.

A. Assist in safely repositioning and then administer (C & D). Then the client can be escorted back to the room via wheelchair or stretcher (B).

The home care nurse provided self-care instructions for a client with chronic venous insufficiency caused y deep vein thrombosis. Which instructions should the nurse include in the client's discharge teaching plan? (select all that apply) A. Avoid prolonged standing or sitting B. Maintain the bed flat while sleeping C. Cross legs at knee but not at ankle D. Continue wearing elastic stockings E. Use recliner for long periods of sitting

A. Avoid prolonged standing or sitting D. Continue wearing elastic stockings E. Use recliner for long periods of sitting

The nurse initiates neurologic checks for a client who is at risk for neurologic compromise. Which manifestation typically provides the first indication of altered neurologic function? A. Change in level of consciousness B. Increasing muscular weakness C. Changes in pupil size bilaterally D. Progressive nuchal rigidity

A. Change in level of consciousness Rationale: A decrease or change in the level of consciousness (A) is usually the first indication of neurologic deterioration. (B and C) may also occur but are much less likely to be the first sign of neurologic compromise. (D) is often a sign of meningitis.

Before administering a parenteral nutrition through a central vein, the should confirm information from which sources? (select all that apply) A. Client's identification band B. Healthcare providers prescription C. Dietician's progress notes D. Solution label E. Measured residual volume F. Medication administration record

A. Client's identification band B. Healthcare providers prescription D. Solution label F. Medication administration record

In assessing a client 48 hours following a fracture, the nurse observes ecchymosis at the fracture site, and recognizes that hematoma formation at the bone fragment site has occurred. What action should the nurse implement? A. Document the extent of he bruising in the medical record B. Assign a UAP to take vital sign measurements q1h C. Advise the client that anticoagulation therapy may be needed. D. Call the lab to obtain a stat APTT and prothrombin time

A. Document the extent of he bruising in the medical record

During assessment of a client in the intensive care unit, the nurse notes that the client's breath sounds are clear upon auscultation, but jugular vein distention and muffled heart sounds are present. Which intervention should the nurse implement? A. Prepare the client for a pericardial tap. B. Administer intravenous furosemide (Lasix). C. Assist the client to cough and deep breathe. D. Instruct the client to restrict oral fluid intake.

A. The client is exhibiting symptoms of cardiac tamponade that results in reduced cardiac output. Treatment is pericardial tap. (B) is not a treatment. (C) is not priority. (D) Fluids are frequently increased but this is not as priority as (A).

The nurse initiates neurologic checks for a client who is at risk for neurologic compromise. Which manifestation typically provides the first indication of altered neurologic function? A.Change in level of consciousness B.Increasing muscular weakness C.Changes in pupil size bilaterally D.Progressive nuchal rigidity

A.Change in level of consciousness Rationale: A decrease or change in the level of consciousness is usually the first indication of neurologic deterioration. Options B and C may also occur but are much less likely to be the first sign of neurologic compromise. Option D is often a sign of meningitis.

While teaching a client with chronic obstructive pulmonary disease about nutritional needs, which type of food should the nurse recommend? A. Cruciferous vegetables. B. Poultry. C. Citrus fruits. D. Pasta.

Answer: B Shortness of breath and increased work of breathing can make maintenance of a healthy body weight difficult in clients with COPD. Lean, high protein foods, such as poultry, should be included in the diet of patients with COPD to decrease the likelihood of protein energy malnutrition.

Thirty-six hours after cesarean delivery, a client complains of nausea and bloating. Assessment reveals a distended abdomen and no bowel movement since delivery. What intervention should the nurse implement first? A. Increase fiber in diet B. Auscultate the abdomen C. Insert a rectal tube D. Encourage ambulation

B. Auscultate the abdomen

Which condition should the nurse anticipate as a potential problem in a female client with a neurogenic bladder? A.Stress incontinence B.Infection C.Painless gross hematuria D.Peritonitis

B. Infection Rationale: Infection is the major complication resulting from stasis of urine and subsequent catheterization. Option A is the involuntary loss of urine through an intact urethra as a result of a sudden increase in intraabdominal pressure. Option C is the most common symptom of bladder cancer. Option D is the most common and serious complication of peritoneal dialysis.

A client who had an emergency appendectomy is being mechanically ventilated, and soft wrist restraints are in place to prevent self extubation. Which outcome is most important for the nurse to include in the client's plan of care? A. Understands pain management scale B. Maintains effective breathing patterns C. Absence of ventilator associated pneumonia D. No injuries related to soft restraints occur

B. Maintains effective breathing patterns

The nurse observes that a postoperative client with a continuous bladder irrigation has a large blood clot in the urinary drainage tubing. What action should the nurse perform first? A. Determine the client's blood pressure and apical pulse rate B. Observe the amount of urine in the client's drainage bag C. Obtain a pulse oximeter to assess the client's oxygen saturation D. Review the medication record for recently administered medications

B. Observe the amount of urine in the client's drainage bag

Nursing interventions for the patient with aplastic anemia are directed toward the prevention of the complications of A. fatigue and dyspnea. B. hemorrhage and infection. C. thromboemboli and gangrene. D. cardiac arrhythmias and heart failure.

B. hemorrhage and infection. Hemorrhage from thrombocytopenia and infection from neutropenia are the greatest risks for the patient with aplastic anemia. The patient will experience fatigue from anemia, but bleeding and infection are the major causes of death in aplastic anemia.

While working in the emergency room, the nurse is exposed to a client with active tuberculosis. When should the nurse plan to obtain a tuberculin skin test? A) Immediately after the exposure. B) Within one week of the exposure. C) Four to six weeks after the exposure. D) Three months after the exposure.

C) Four to six weeks after the exposure. A tuberculin skin test is effective 4 to 6 weeks after an exposure (C), so the individual with a known exposure should wait 4 to 6 weeks before having a tuberculin skin test.

During the change of shift report, the charge nurse reviews the infusions being received by the clients on the oncology unit. The client receiving which infusion should be seen first?

C. Has the highest risk for respiratory depression and therefor should be seen first. (A) Risk of hypotension. (B) Lowest risk. (D) Risk of nephrotoxicity and phlebitis.

To assess for the presence of diaphragmatic breathing, what action should the nurse take? A. Attach an apnea monitor to the chest wall B. Auscultate the lung bases anteriorly C. Observe the movement of the abdomen D. Note any gaps between respirations

C. Observe the movement of the abdomen

A man who has a known problem with alcohol is accused of stealing from his employer. When he returns home that evening, he accuses his son of stealing from school, and physically abuses the child for what the father describes as the child's dishonest behavior. Which two defense mechanisms are being used by the father? A. Sublimation and displacement B. Denial and sublimation C. Projection and displacement D. Projection and denial

C. Projection and displacement

A client with chronic kidney disease is being discharged with continuous ambulatory peritoneal dialysis (CAPD). What is the priority nursing diagnosis the nurse should use when developing a discharge teaching plan for this client? A. Altered nutriton B. Impaired mobility C. Risk for sepsis D. Risk for injury

C. Risk for sepsis

The nurse is caring for a critically ill client with cirrhosis of the liver who has a nasogastric tube draining bright red blood. The nurse notes that the client's serum hemoglobin and hematocrit are decreased. What additional change in lab data should the nurse expect? A. Increased serum albumin B. Decreased serum creatinine C. Decreased serum ammonia D. Increased liver function tests

C. The breakdown of glutamine in the intestine and the increased activity of colonic bacteria from the digestion of proteins increases the ammonia levels in the clients with advanced liver disease, so removal of blood, a protein source, from the intestines results in reduced ammonia. (A, B, D) will not be significantly impacted by the removal of blood.

An older client is admitted with a diagnosis of bacterial pneumonia. Which symptom should the nurse report to the health care provider after assessing the client? A. Leukocytosis and febrile B. Polycythemia and crackles C. Pharyngitis and sputum production D. Confusion and tachycardia

D. Confusion and tachycardia Rationale: The onset of pneumonia in the older client may be signaled by general deterioration, confusion, increased heart rate, and/or increased respiratory rate (D). (A, B, and C) are often absent in the older client with bacterial pneumonia.

Dinoprostone (Prostin E-2) is prescribed for primigravida who had a missed spontaneous abortion. An increase in which finding should the nurse expect? A. Maternal temperature B. Rh antibody production C. Hemoglobin (Hgb) levels D. Contractions of the uterus

D. Contractions of the uterus

41. When planning patient teaching about emphysema, the nurse understands that the symptoms of emphysema are caused by which of the following? A. Hypertrophy and hyperplasia of goblet cells in the bronchi B. Collapse and hypoventilation of the terminal respiratory unit C. An overproduction of the antiprotease alpha1-antitrypsin D. Hyperinflation of alveoli and destruction of alveolar walls

D. Hyperinflation of alveoli and destruction of alveolar walls In emphysema, there are structural changes that include hyperinflation of alveoli, destruction of alveolar walls, destruction of alveolar capillary walls, narrowing of small airways, and loss of lung elasticity.

33. The nurse determines that the patient is not experiencing adverse effects of albuterol (Proventil) after noting which of the following patient vital signs? A. Oxygen saturation 96% B. Respiratory rate of 18 C. Temperature of 98.4° F D. Pulse rate of 76

D. Pulse rate of 76 Albuterol is a β2-agonist that can sometimes cause adverse cardiovascular effects. These would include tachycardia and angina. A pulse rate of 76 indicates that the patient did not experience tachycardia as an adverse effect.

Beginning with the vena cavae, what is the normal sequence of blood circulation through the heart? A. Mitral valve B. Left atrium C. Pulmonary semilunar valve D. Right atrium E. Tricuspid valve F. Aortic semilunar valve G. Left ventricle H. Right ventricle

D. Right atrium E. Tricuspid valve H. Right ventricle C. Pulmonary semilunar valve B. Left atrium A. Mitral valve G. Left ventricle F. Aortic semilunar valve

To reduce risk for hypovolemic shock in a postoperative client, which intervention is most important for the nurse to implement?

Ensure that the IV is infusion at the prescribed rate

A client with chronic gout reports pain in several joints and has tophi on the hands. The nurse develops a plan to manage the client's pain based on which understanding of the underlying pathology of these manifestations?

Excess uric acid results in urate crystal deposits in the tissues

The nurse is assessing a client who has herpes zoster. Which question will allow the nurse to gather further information about this condition?

Has everyone at home already had varicella?

A nurse is monitoring a client who has Graves' disease for the development of thyroid storm. the nurse should report which of the following findings to the provider?

Hypertension -Rationale: Thyroid storm patients will have an exaggerated condition of hyperthyroidism, associated with the development of a fever, hypertension, abdominal pain, and tachycardia

A client with stage IV bone cancer is admitted to the hospital for pain control. The client verbalizes continuous, severe pain of 8 on a 1 to 10 scale. Which intervention should the nurse implement?

administer opiod and non-opiod medication simultaneously

A nurse is caring for a client following an open reduction and internal fixation of a fractured femur. Which of the following findings is the nurse's priority? 1) Altered level of consciousness 2) Oral temperature of 37.7° C (100° C) 3) Muscle spasms 4) Headache

altered LOC

A client uses triamcinolone (Kenalog), a corticosteroid ointment, to manage pruritus caused by a chronic skin rash. The client calls the clinic nurse to report increased erythema with purulent exudate at the site. Which action should the nurse implement?

schedule an appointment for the client to see the health care provider

A nurse is caring for a client with Diabetes Insipidus (DI). Which data warrants the most immediate intervention by the nurse?

serum sodium of 185

A nurse is reinforcing teaching about an esophagogastroduodenoscopy with a client who has upper gastric pain. Which of the following statements should the nurse include in the teaching? 1) "A flexible tube is introduced through the nose during the procedure." 2) "During the procedure you are in a sitting position." 3) "You will remain NPO for 8 hours before the procedure." 4) "You will be awake while the procedure is performed."

"You will remain NPO for 8 hours before the procedure"

An adolescent receives a prescription for an injection of S-matriptan succinate, 4 mg subcutaneously for a migraine headache. Using a vial labeled, 6 mg/0.5 ml, how many ml should the nurse administer? (nearest hundredth)

0.33 ml

220.) A adult client with muscle spasms is taking an oral maintenance dose of baclofen (Lioresal). The nurse reviews the medication record, expecting that which dose should be prescribed? 1. 15 mg four times a day 2. 25 mg four times a day 3. 30 mg four times a day 4. 40 mg four times a day

1. 15 mg four times a day Rationale: Baclofen is dispensed in 10- and 20-mg tablets for oral use. Dosages are low initially and then gradually increased. Maintenance doses range from 15 to 20 mg administered three or four times a day.

115.) A client received 20 units of NPH insulin subcutaneously at 8:00 AM. The nurse should check the client for a potential hypoglycemic reaction at what time? 1. 5:00 PM 2. 10:00 AM 3. 11:00 AM 4. 11:00 PM

1. 5:00 PM Rationale: NPH is intermediate-acting insulin. Its onset of action is 1 to 2½ hours, it peaks in 4 to 12 hours, and its duration of action is 24 hours. Hypoglycemic reactions most likely occur during peak time.

142.) A health care provider has written a prescription for ranitidine (Zantac), once daily. The nurse should schedule the medication for which of the following times? 1. At bedtime 2. After lunch 3. With supper 4. Before breakfast

1. At bedtime Rationale: A single daily dose of ranitidine is usually scheduled to be given at bedtime. This allows for a prolonged effect, and the greatest protection of the gastric mucosa. **recall that ranitidine suppresses secretions of gastric acids**

156.) A nurse is reviewing the laboratory results for a client receiving tacrolimus (Prograf). Which laboratory result would indicate to the nurse that the client is experiencing an adverse effect of the medication? 1. Blood glucose of 200 mg/dL 2. Potassium level of 3.8 mEq/L 3. Platelet count of 300,000 cells/mm3 4. White blood cell count of 6000 cells/mm3

1. Blood glucose of 200 mg/dL Rationale: A blood glucose level of 200 mg/dL is elevated above the normal range of 70 to 110 mg/dL and suggests an adverse effect. Other adverse effects include neurotoxicity evidenced by headache, tremor, insomnia; gastrointestinal (GI) effects such as diarrhea, nausea, and vomiting; hypertension; and hyperkalemia.

116.) A nurse administers a dose of scopolamine (Transderm-Scop) to a postoperative client. The nurse tells the client to expect which of the following side effects of this medication? 1. Dry mouth 2. Diaphoresis 3. Excessive urination 4. Pupillary constriction

1. Dry mouth Rationale: Scopolamine is an anticholinergic medication for the prevention of nausea and vomiting that causes the frequent side effects of dry mouth, urinary retention, decreased sweating, and dilation of the pupils. The other options describe the opposite effects of cholinergic-blocking agents and therefore are incorrect.

93.) The client who is human immunodeficiency virus seropositive has been taking stavudine (d4t, Zerit). The nurse monitors which of the following most closely while the client is taking this medication? 1. Gait 2. Appetite 3. Level of consciousness 4. Hemoglobin and hematocrit blood levels

1. Gait Rationale: Stavudine (d4t, Zerit) is an antiretroviral used to manage human immunodeficiency virus infection in clients who do not respond to or who cannot tolerate conventional therapy. The medication can cause peripheral neuropathy, and the nurse should monitor the client's gait closely and ask the client about paresthesia. Options 2, 3, and 4 are unrelated to the use of the medication.

125.) A nurse is preparing to administer digoxin (Lanoxin), 0.125 mg orally, to a client with heart failure. Which vital sign is most important for the nurse to check before administering the medication? 1. Heart rate 2. Temperature 3. Respirations 4. Blood pressure

1. Heart rate Rationale: Digoxin is a cardiac glycoside that is used to treat heart failure and acts by increasing the force of myocardial contraction. Because bradycardia may be a clinical sign of toxicity, the nurse counts the apical heart rate for 1 full minute before administering the medication. If the pulse rate is less than 60 beats/minute in an adult client, the nurse would withhold the medication and report the pulse rate to the registered nurse, who would then contact the health care provider.

A client is receiving an infusion labeled Heparin Sodium 20,000 units in D5W 500 mL at the rate of 35 mL/hour. How many units of heparin is the client receiving per hour?

1400 unit

A client is currently receiving an infusion labeled Heparin Sodium 25,00 units in 5% Dextrose Injection 500 ml at 14 ml/hour. A prescription is received to change the rate of the infusion to 900 units of Heparin per hour. The nurse should set the infusion pump to deliver how many ml.hour?

18 ml/hour

A client is prescribed codeine USP 60 mg every 4 hours. The available medication is codeine USP 30 mg/tablet. How many tablets should the practical nurse administer? Fill in the blank.

2

168.) Colcrys (colchicine) is prescribed for a client with a diagnosis of gout. The nurse reviews the client's medical history in the health record, knowing that the medication would be contraindicated in which disorder? 1. Myxedema 2. Renal failure 3. Hypothyroidism 4. Diabetes mellitus

2. Renal failure Rationale: Colchicine is contraindicated in clients with severe gastrointestinal, renal, hepatic or cardiac disorders, or with blood dyscrasias. Clients with impaired renal function may exhibit myopathy and neuropathy manifested as generalized weakness. This medication should be used with caution in clients with impaired hepatic function, older clients, and debilitated clients. **Note that options 1, 3, and 4 are all endocrine-related disorders: Myxedema=Hypothyroidism**

137.) A nurse is reinforcing instructions for a client regarding intranasal desmopressin acetate (DDAVP). The nurse tells the client that which of the following is a side effect of the medication? 1. Headache 2. Vulval pain 3. Runny nose 4. Flushed skin

3. Runny nose Rationale: Desmopressin administered by the intranasal route can cause a runny or stuffy nose. Headache, vulval pain, and flushed skin are side effects if the medication is administered by the intravenous (IV) route.

95.) The nurse is reviewing the results of serum laboratory studies drawn on a client with acquired immunodeficiency syndrome who is receiving didanosine (Videx). The nurse interprets that the client may have the medication discontinued by the health care provider if which of the following significantly elevated results is noted? 1. Serum protein 2. Blood glucose 3. Serum amylase 4. Serum creatinine

3. Serum amylase Rationale: Didanosine (Videx) can cause pancreatitis. A serum amylase level that is increased 1.5 to 2 times normal may signify pancreatitis in the client with acquired immunodeficiency syndrome and is potentially fatal. The medication may have to be discontinued. The medication is also hepatotoxic and can result in liver failure.

The healthcare provider prescribes an IV solution of regular insulin (Hummulin-R) 100 units in 250 ml of 0.45% saline to infuse at 12 units/hour. The nurse should program the infusion pump to deliver how many ml/hour?

30 mL/hr

231.) A client admitted to the hospital gives the nurse a bottle of clomipramine (Anafranil). The nurse notes that the medication has not been taken by the client in 2 months. What behaviors observed in the client would validate noncompliance with this medication? 1. Complaints of hunger 2. Complaints of insomnia 3. A pulse rate less than 60 beats per minute 4. Frequent handwashing with hot, soapy water

4. Frequent handwashing with hot, soapy water Rationale: Clomipramine is commonly used in the treatment of obsessive-compulsive disorder. Handwashing is a common obsessive-compulsive behavior. Weight gain is a common side effect of this medication. Tachycardia and sedation are side effects. Insomnia may occur but is seldom a side effect.

225.) A nursing student is assigned to care for a client with a diagnosis of schizophrenia. Haloperidol (Haldol) is prescribed for the client, and the nursing instructor asks the student to describe the action of the medication. Which statement by the nursing student indicates an understanding of the action of this medication? 1. It is a serotonin reuptake blocker. 2. It inhibits the breakdown of released acetylcholine. 3. It blocks the uptake of norepinephrine and serotonin. 4. It blocks the binding of dopamine to the postsynaptic dopamine receptors in the brain.

4. It blocks the binding of dopamine to the postsynaptic dopamine receptors in the brain. Rationale: Haloperidol acts by blocking the binding of dopamine to the postsynaptic dopamine receptors in the brain. Imipramine hydrochloride (Tofranil) blocks the reuptake of norepinephrine and serotonin. Donepezil hydrochloride (Aricept) inhibits the breakdown of released acetylcholine. Fluoxetine hydrochloride (Prozac) is a potent serotonin reuptake blocker.

160.) Meperidine hydrochloride (Demerol) is prescribed for the client with pain. Which of the following would the nurse monitor for as a side effect of this medication? 1. Diarrhea 2. Bradycardia 3. Hypertension 4. Urinary retention

4. Urinary retention Rationale: Meperidine hydrochloride (Demerol) is an opioid analgesic. Side effects of this medication include respiratory depression, orthostatic hypotension, tachycardia, drowsiness and mental clouding, constipation, and urinary retention.

The nurse is interviewing a male client with hypertension. Which additional medical diagnosis in the client's history presents the greatest risk for developing a cerebral vascular accident (CVA)? A) Diabetes mellitus. B) Hypothyroidism. C) Parkinson's disease. D) Recurring pneumonia.

A) Diabetes mellitus. A history of diabetes mellitus poses the greatest risk for developing a CVA (A). (B, C, and D) may place the client at some risk due to immobility, but do not present a risk as great as (A).

21. A 45-year-old man with asthma is brought to the emergency department by automobile. He is short of breath and appears frightened. During the initial nursing assessment, which of the following clinical manifestations might be present as an early symptom during an exacerbation of asthma? A. Anxiety B. Cyanosis C. Hypercapnia D. Bradycardia

A. Anxiety An early symptom during an asthma attack is anxiety because he is acutely aware of the inability to get sufficient air to breathe. He will be hypoxic early on with decreased PaCO2 and increased pH as he is hyperventilating.

A nurse is admitting a 4-month-old who has respiratory syncytial virus (RIS). What intervention should the nurse implement first? A. Initiate contact precautions B. Place the infant under a mist tent C. Elevate aretial blood gases D. Obtain the infants vital signs

A. Initiate contact precautions

While caring for a client with a new onset of diabetes mellitus, which intervention is most important for the nurse to include in the client's plan of care? A. Observe client's glucose self-monitoring technique B. Provide written diabetic diet instructions C. Teach client how to read food labels D. Check accuracy of glucose monitoring equipment

A. Observe client's glucose self-monitoring technique

A young woman is preparing to leave for a 7-day boat trip. She requests a prescription for motion sickness, so the healthcare provider prescribes meclizine (Antivert). Which instruction should the nurse include in this client's teaching? A. Suck on hard candy for a dry mouth while taking this drug B. Sit upright for at least 30 minutes after taken this drug C. Avoid eating shellfish for 24 hours after taking this drug D. Do not drink caffeinated beverages while taking this drug

A. Suck on hard candy for a dry mouth while taking this drug

Which expected outcome statement should the nurse include in a teaching plan designed to assist a client with management of an acute attack of gout? A. The client will avoid use of alcohol in managing stress. B. The client will implement a high-purine daily dietary regime. C. The client will use local heat applications for acute pain. D. The client will stop antigout medication once pain subsides.

A. The client will avoid use of alcohol in managing stress. Gout, an error in purine metabolism or excretion that results in urate crystal disposition usually in a joint of the great toe, causes acute joint inflammation and severe pain. Alcohol consumption should be avoided because it increases production or uric acid and prevents excretion

The nurse is responding to telephone messages at a psychiatric day clinic. Which client situation requires immediate intervention by the nurse? A. The wife of a client with post-traumatic stress syndrome reports that her husband is threatening to kill her. B. A client with depression who is crying and tells the nurse that he has has suicidal thoughts C. A young adult diagnosed with a somatoform disorder reports having a severe headache that has become unbearable D. An adult heroin abuser who reports the onset of withdrawal and requests a refill for a prescription for methadone

A. The wife of a client with post-traumatic stress syndrome reports that her husband is threatening to kill her.

The nurse is caring for a client with a chest tube to water seal drainage that was inserted 10 days ago because of a ruptured bullae and pneumothorax. Which finding should the nurse report to the healthcare provider before the chest tube is removed? A. Tidal of water in the water seal chamber B. Bilateral muffled breath sounds at bases C. Temperature of 101 degrees F D. Absence of chest tube drainage for 2 days.

A. Tidal in the water seal chamber should be reported to the HPC to show that the chest tube is working properly. (B) may indicate hypoventilation from the chest tube and usually improves when the tube is removed. (C) indicates infection (D) is an expected finding.

24. The nurse is caring for a patient with an acute exacerbation of asthma. Following initial treatment, which of the following findings indicates to the nurse that the patient's respiratory status is improving? A. Wheezing becomes louder B. Vesicular breath sounds decrease C. Aerosol bronchodilators stimulate coughing D. The cough remains nonproductive

A. Wheezing becomes louder The primary problem during an exacerbation of asthma is narrowing of the airway and subsequent diminished air exchange. As the airways begin to dilate, wheezing gets louder because of better air exchange.

To find the infection site associated with acute lymphangitis, the nurse should look _____ to the inflammation. A. distal B. anterior C. proximal D. contralateral

A. distal The nurse should assess distal to swelling to locate the initial site of infection. Examining proximal, contralateral, or anterior to the inflammation does not describe swelling associated with infection.

A nurse is providing care to an adult female patient and observes that the Hb laboratory analysis result is 9 g/dl. Based on this finding, the nurse should expect to observe A. dyspnea. B. bradycardia. C. warm, dry skin. D. activity tolerance without complaint of fatigue.

A. dyspnea.Hb levels are used to determine the severity of anemia. Patients with moderate anemia (Hb 6 to 10 g/dL) may suffer from dyspnea, palpitations, diaphoresis with exertion, and chronic fatigue. Patients who are anemic usually have cool skin related to compensatory mechanism of mild vasoconstriction. Patients who are anemic experience tachycardia because of increased demands placed on the heart to meet overall metabolic requirements. Activity tolerance without complaint is not correct because patients with anemic conditions fatigue readily.

The nurse notices clear nasal drainage in a patient newly admitted with facial trauma, including a nasal fracture. The nurse should: A. test the drainage for the presence of glucose. B. suction the nose to maintain airway clearance. C. document the findings and continue monitoring. D. apply a drip pad and reassure the patient this is normal.

A. test the drainage for the presence of glucose. Clear nasal drainage suggests leakage of cerebrospinal fluid (CSF). The drainage should be tested for the presence of glucose, which would indicate the presence of CSF.

Which consideration is most important when the nurse is assigning a room for a client being admitted with progressive systemic sclerosis (scleroderma)? A.Provide a room that can be kept warm. B.Make sure that the room can be kept dark. C.Keep the client close to the nursing unit. D.Select a room that is visible from the nurses' desk.

A.Provide a room that can be kept warm. Rationale: Abnormal blood flow in response to cold (Raynaud phenomenon) is precipitated in clients with scleroderma. Option B is not a significant factor. Stress can also precipitate the severe pain of Raynaud phenomenon, so a quiet environment is preferred to option C, which is often very noisy. Option D is not necessary.

A client sustained a burn injury greater than 25% of total body surface with majority of it lower extremities during a house fire. During the acute phase of care, which intervention is most important for the practical nurse to implement?

Administer 0.5 mL of tetanus toxoid IM. Prevention of infection from Clostridium tetani by administering tetanus toxoid has the highest priority for care of a client in the acute phase of burn care.

Which information should the nurse include when giving discharge instructions to a client following a left eye cataract extraction with lens implant?

Administer a stool softener

Which action is most important for the nurse to implement to reduce the risk for deep vein thrombosis in a postoperative client?

Advise the client to perform leg exercises regularly

The practical nurse (PN) is reviewing high-risk factors for type 2 diabetes with a client? Which of the following characteristics are risk factors for the development of diabetes? (Select all that apply.)

Age 40 years or older African American Obesity Hispanic Risk factors for developing type 2 diabetes include being 40 years or older, African American, Hispanic, and obese.

A nurse is caring for a client who has partial-thickness and full-thickness burns of his head, neck, and chest. The nurse should recognize which of the following is the priority risk to the client? 1) Airway obstruction 2) Infection 3) Fluid imbalance 4) Contractures

Airway obstruction

A nurse is caring for a client who was admitted with major burns to the head, neck, and chest. Which of the following complications should the nurse identify as the greatest risk to the client? 1) Hypothermia 2) Hyponatremia 3) Fluid imbalance 4) Airway obstruction

Airway obstruction

A client with a 30 year history cigarette smoking has developed emphysema. Which pathophysiological mechanism results in the greatest interference with normal gas exchange for this client?

Alveolar wall destruction by proteolytic enzymes from macrophages

A client scheduled for open heart surgery is prescribed a transfusion of 4 units of packed red blood cells (RBCs). Which intervention is the best method to prevent a blood transfusion reaction?

An autologous transfusion The best method for preventing transfusion reaction is an autologous transfusion (the client's own blood). A client's blood is generally collected 4 weeks before a scheduled surgery.

The nurse is assigned to care for four clients. Which client is at an increased risk for metabolic alkalosis?

An older female client with small bowel obstruction who has vomited for 3 days

While auscultating the abdomen of a thin older-adult client, the nurse detects a pulsatile mass. What is the correct action for the nurse to take? A. Notify the physician. B. Call an emergency code. C. Document the findings. D. Roll the patient onto his right side.

Answer: A A pulsatile mass in a thin elderly male is suspicious for an abdominal aortic aneurysm. Due to the potential for leakage or rupture, the nurse should notify the

The nurse is administering IV fluid resuscitation to an elderly client diagnosed with sepsis. The nurse should be alert for which possible complication of this treatment? A. Shortness of breath. B. Facial droop. C. Decreased urine output. D. Confusion.

Answer: A IV fluid resuscitation is administered to clients experiencing extreme dehydration or sepsis. When administering IV fluid resuscitation to an elderly client, the nurse should monitor for symptoms of fluid overload, which include shortness of breath and respiratory compromise.

A parent of a child who was just diagnosed with Sickle Cell Disease (SCD) asks the nurse what cause their daughter to get this disease. Which is a correct response? A. Both biological parents have to have a copy of the faulty gene. B. One of the parents had to have a faulty dominant copy of the gene. C. The faulty gene is located on one of the parents X chromosome. D. Their father had to have a faulty gene located on his Y chromosome.

Answer: A Sickle cell disease is known as an autosomal recessive disease in which both of the parents have to have the recessive gene to pass the disease onto their offspring. There is a 1:4 chance of having a child with the disease; 1:4 a child not affected; 2:4 chance of a child being a carrier of the recessive faulty gene. With autosomal recessive gene diseases, sometimes the occurrence of a disease can skip a generation.

A client presents with pain that extends from the midepigastric region and radiates posteriorly to the right shoulder blade. Which condition should the nurse suspect? A. Cholecystitis. B. Appendicitis. C. Diverticulitis. D. Renal calculi.

Answer: A The gallbladder is a small, pear-shaped organ located in the right upper quadrant of the abdomen near the liver. Clients with inflammation of the gallbladder (cholecystitis) typically present with upper abdominal pain that radiates to the right shoulder or back.

The nurse is reviewing the bone marrow aspiration results of a client which revealed abnormal high amount of blast cells present. This client will most likely be diagnosed with which condition? A. Leukemia. B. Hemophilia. C. Hodgkin's Lymphoma. D. Autoimmune thrombocytopenic purpura.

Answer: A The procedure bone marrow aspiration which reveals abnormal high levels of immature white blood cells referred to as "blast" cells is indicative of leukemia.

A client has been diagnosed with an ankle sprain. The nurse should anticipate that the client will need which medication? A. Naproxen sodium. B. Hydrocortisone. C. Ciprofloxacin. D. Chloroquine.

Answer: A Nonsteroidal anti-inflammatory drugs (NSAIDs), such as naproxen sodium, are recommended for treatment of sprains.

A nurse is teaching a client about the best way to manage their diagnosis of dumping syndrome. Which statements should the nurse include in her client teaching? (Select all that apply) A. Avoid dairy products and sweets. B. Limit food consumption to two meals a day. C. Diet should consist of high fat and protein content. D. Ensure to drink at least one glass of water with meals. Incorrect E. Whole grains, fresh fruit and vegetables are encouraged.

Answer: A C The only way to manage dumping syndrome is through nutritional changes. Clients should be encouraged to eat several small meals throughout the day. Their diet should consists of relatively high fat and protein foods and be low in roughage and carbohydrates. They should avoid all milk products, sweets and sugars. Liquid should only be drunk in between meals to minimize the dumping of food contents in their stomach into the small intestine.

A client, who had a laparoscopy cholecystectomy two days ago, calls the clinic's triage nurse in the morning, complaining that they have been awake all night feel feeling restless and anxious, like something isn't right with them and are afraid of dying. Which is the most appropriate nurse's response? A. Questioned the client, if they are experiencing epigastric pain when eating. B. Advise the client to return to the clinic now to be seen by their gastric surgeon. Incorrect C. Explained to the client, it may be discomfort from the gas used during the procedure. D. Instruct the client to monitor their temperature every four hours and report if temp >100.5°F (37.8°C).

Answer: B Clients status post invasive procedures are at risk of hemorrhaging. As a result of the hemorrhaging, the client could go into shock. A feeling of impending doom, accompanied with restlessness and anxiety could be a signs that the client may be hemorrhaging internally. The nurse needs to instruct the client to return to the clinic now to be seen by their gastric surgeon.

On assessment, a client has diminished breath sounds and bilateral rhonchi. Radial pulses are thready and weak. The client's skin is pale and cool to the touch with capillary refill +4. The client's oral mucosa and conjunctivae are cyanotic. The nurse also notes shortness of breath and increased respiratory rate when the client ambulates. The nurse recognizes that the client is most likely exhibiting signs and symptoms of which condition? A. Aortic aneurysm. B. Left-sided heart failure. C. Respiratory distress syndrome. D. Pulmonary artery stenosis

Answer: B Fluid in the lungs as indicated by the rhonchi and diminished breath sounds is considered a prominent sign of left-sided heart failure. The heart failure causes the heart not to pump effectively leading to poor tissue perfusion and circulation as evidenced by fluid build-up in the lungs, paleness and cyanosis, and weak, thready pulses. Left-sided heart failure leads to shortness of breath due to ineffective perfusion in the lungs for O2/CO2 gas exchange.

A client is admitted with several bruises and has been diagnosed with ascites and a deficiency in clotting. Which intervention helps promote clotting? A. Verify that heparin has been ordered. B. Administer a vitamin K injection. C. Offer the client high protein foods to eat. D. Provide the client a soft bristle toothbrush.

Answer: B Vitamin K is necessary for the liver to produce the protein prothrombin which is one of the factors for the clotting process. Deficiency of vitamin K can result in bruising and bleeding.

A client is suspected to have an adrenal gland dysfunction. Which diagnostic laboratory test would be used to measure hormone levels and their metabolites to determine a gland's endocrine function? A. Stimulation test. B. Suppression test. C. 24-hour urine test D. Venous Sampling Test.

Answer: C A urine test can measure hormone levels and their metabolites. A 24-hour urine collection, rather than a single blood or urine, better reflects a specific gland function.

When counseling a client with a pressure ulcer, which type of diet should the nurse recommend to promote wound healing? A. Low sodium. B. High fiber. C. High protein. D. Low fat.

Answer: C High protein consumption should be encouraged to aid in wound healing and prevention of further skin compromise.

When teaching a client with anemia about foods that are high in iron, which food should the nurse include? A. Brussel sprouts. B. Oranges. C. Liver. D. Iceberg lettuce.

Answer: C When counseling a client with anemia, the nurse should explain the importance of a diet designed to prevent iron deficiency. Liver, as well as other sources of protein, are high in iron.

Question 2 of 10 Client is prescribed angiotensin-converting enzyme (ACE) inhibitors and angiotensin-receptor blockers (ARBs) improve the function of their diagnosed heart failure. What should the nurse assess first prior to administering these medications to a client? A. Apical pulse. B. Heart sounds. C. Blood pressure. D. Intake and output.

Answer: C ACE inhibitors and ARBs can cause severe hypotension. The nurse should assess the client's blood pressure prior to administering the medication.

The nurse is assessing a client who reports a history of hypertension, cirrhosis due to alcoholism, high cholesterol, and human papillomavirus. Which medical condition increases the risk of cancer of the reproductive organs? A. Uncontrolled hypertension. B. End-stage liver disease. C. Sexually transmitted diseases. D. Chronic hyperlipidemia.

Answer: C History of prior sexually transmitted diseases increases the risk of cancer of cervical, ovarian and endometrial tissues in females and testicular cancer in males.

A client is scheduled for an echocardiogram the next morning. What instructions should the nurse communicate to the client about this test? A. A large bore IV will be inserted. B. Pain medication will be administered. C. No food or drink will be allowed after midnight. D. No special preparation is necessary

Answer: D An echocardiogram is a noninvasive test that does not require pre-procedure interventions or preparation.

A client diagnosed with kidney stones is experiencing a urine output decrease of less than 0.5ml/kg per hour; increase BUN and creatinine levels; decrease glomerular filtration rate; flank pain and wheezes and crackles in their lungs, along with 2+ pitting edema in their extremities. Which complication is the client most likely developing? A. Cystitis. B. Urolithiasis. C. Pyelonephritis. D. Acute kidney injury.

Answer: D The client has already been diagnosed with urolithiasis which are the diagnosed kidney stones. The client is presenting signs and symptoms of acute kidney injury as a result of the kidney stones causing obstruction(s). Decrease in urine output less than 0.5m/kg per hour; abnormal or sharp increase of BUN and creatinine levels; decrease in their GFR and signs and symptoms of fluid overload as evidence of pulmonary edema and peripheral edema and flank pain.

The nurse is examining a client for possible anemia. The client complains of fatigue and weight loss and appears pallor in color with slight jaundiced and has a beefy red tongue. The client also complains about poor balance and their fingers and toes feeling tingling and numb.The client's signs and symptoms are indicative of which type of anemia? A. Aplastic anemia. B. Iron deficiency anemia. C. Folic acid deficiency anemia. D. Vitamin B12 deficiency anemia.

Answer: D The classic sign of vitamin B12 deficiency anemia is a beefy red tongue and paresthesia and poor balance. Some clients may also expereince pallor, fatigue and reduced exercise intolerance seen in all anemias.

A client diagnosed with a skull hematoma and fractured (L) clavicle status post a six foot fall from a ladder is admitted to the unit. Which sign should the practical nurse (PN) report immediately?

Answers questions, but is confused. Confusion status post a head injury is usually the first sign of increased intracranial pressure.

A client experiences an ABO incompatibility reaction after receiving multiple blood transfusions. An ABO incompatibility reaction is an example of which type of hypersensitivity?

Antibody-Cytotonic

A client status post-48 hours femoral rod placement surgery, suddenly complains of chest pain and becomes short of breath, pale, and diaphoretic. The practical nurse (PN) immediately assesses their vital signs and obtains 100/80 mm Hg blood pressure, 110 beats/min heart rate, and 36 breaths/min respiratory rate. What nursing action should the PN to do next?

Apply oxygen at 2 L per nasal cannula. The PN should immediately provide oxygen while performing further assessment. Pulmonary embolism and pneumothorax are risks associated with major surgery.

A client comes to the clinic and reports the presence of a painful lesion in the genital area; they described it as a blister 2 days earlier that is now crusty. Which intervention should the practical nurse (PN) implement first?

Ask the client if they have had unprotected sex. These are typical signs and symptoms of herpes simplex virus 2 (HSV2), a sexually transmitted disease (STD), so the PN should ask the client if they had unprotected sex and if the client has exposed others to the disease.

An elderly female client grimaces and demonstrates guarding behavior but denies experiencing pain when asked by the nurse to rate her pain on a numeric scale. Which action should the nurse take next?

Ask the client to describe how she is feeling

When assessing a client's pain the nurse first determines the location and intensity of the pain. To gather data about the quality of the pain, what action should the nurse take?

Ask the client to describe the pain experienced

The health care provider informed a client diagnosed with stage 4 liver cancer that the cancer has spread to their spine. The client states to the practical nurse, "I have a cancer, but it is not malignant." What is the best initial nursing action?

Ask the client to explain his understanding of the term malignancy. The best initial action is to assess the client's knowledge of the term malignancy when used to describe cancer. The client appears to have inaccurate knowledge. Stage 4 cancer means the cancer has spread (metastasized) from where it has started to another body part.

A client is status post-bowel resection and has a nasogastric tube (NGT) attached to low intermittent suction. The client complains to the practical nurse of abdominal distention and nausea. What action should the PN take first?

Assess the NGT drainage in the collection container. The immediate priority is to determine if the tube is functioning correctly, which the PN can do first by assessing the amount and characteristic of the drainage from the nasogastric tube. Based on the findings of the drainage will determine the PN next nursing intervention.

A client has an IV infusing through a catheter inserted at the right antecubital area. The infusion pump is alarming and indicates an occlusion. Which action should the nurse take?

Assist the client to straighten the right arm

A nurse is caring for a client who is postoperative and requesting something to drink. The nurse reads the client's postoperative prescriptions, which include, "Clear liquids, advance diet as tolerated." Which of the following actions should the nurse take first? 1) Offer the client apple juice. 2) Elevate the client's head of bed. 3) Auscultate the client's abdomen. 4) Order a lunch tray for the client.

Auscultate the client's abdomen

A client status post-AV (arteriovenous) fistula placement in the right forearm is transferred from the PACU to the unit. Which nursing measure is essential in promoting safe, effective care for the client?

Avoid BPs or needle sticks in right arm. To prevent damage to the fistula from trauma or injury so that it can mature and be used as a future access site for hemodialysis, all blood pressures, blood draws, and IVs should be avoided to the affected (right) arm.

A nurse is reinforcing teaching with a client who is postoperative after having an ileostomy established. Which of the following instructions should the nurse include in the teaching? 1) Empty the pouch immediately after meals. 2) Change the entire appliance once a day. 3) Limit fluid intake. 4) Avoid medications in capsule or enteric form.

Avoid medications in capsule or enteric forms

The nurse is assessing a client's laboratory values following administration of chemotherapy. Which lab value leads the nurse to suspect that the client is experiencing tumor lysis syndrome (TLS)? A) Serum PTT of 10 seconds. B) Serum calcium of 5 mg/dl. C) Oxygen saturation of 90%. D) Hemoglobin of 10 g/dl.

B) Serum calcium of 5 mg/dl. TLS results in hyperkalemia, hypocalcemia, hyperuricemia, and hyperphosphatemia. A serum calcium level of 5 (B), which is low, is an indicator of possible tumor lysis syndrome. (A, C, and D) are not particularly related to TLS.

50. The nurse is teaching a patient how to self-administer ipratropium (Atrovent) via a metered dose inhaler. Which of the following instructions given by the nurse is most appropriate to help the patient learn proper inhalation technique? A. "Avoid shaking the inhaler before use." B. "Breathe out slowly before positioning the inhaler." C. "After taking a puff, hold the breath for 30 seconds before exhaling." D. "Using a spacer should be avoided for this type of medication."

B. "Breathe out slowly before positioning the inhaler." It is important to breathe out slowly before positioning the inhaler. This allows the patient to take a deeper breath while inhaling the medication thus enhancing the effectiveness of the dose.

Which instruction should the nurse teach a female client about the prevention of toxic shock syndrome? A."Get immunization against human papillomavirus (HPV)." B."Change your tampon frequently." C."Empty your bladder after intercourse." D."Obtain a yearly flu vaccination."

B. "Change your tampon frequently." Rationale: Certain strains of Staphylococcus aureus produce a toxin that can enter the bloodstream through the vaginal mucosa. Changing the tampon frequently reduces the exposure to these toxins, which are the primary cause of toxic shock syndrome. Option A helps prevent cervical cancer, not toxic shock syndrome. Option C can lessen the incidence of urinary tract infection. Option D can help prevent some individuals from contracting the flu and pneumonia, but no relationship to toxic shock syndrome has been proven.

the healthcare provider is working with a client who was recently diagnosed with asthma. Which statement by the client indicates to the nurse that further teaching is needed? A. "It is normal for my heart rate to increase slightly when I use my albuterol inhaler." B. "I should always use my beclomethasone inhaler first, then follow it with my albuterol inhaler." C. "I should wait several minutes between puffs of my inhalers." D. "My albuterol can be used between regular doses if I experience difficulty breathing."

B. "I should always use my beclomethasone inhaler first, then follow it with my albuterol inhaler."

A patient has been receiving oxygen per nasal cannula while hospitalized for COPD. The patient asks the nurse whether oxygen use will be needed at home. Which of the following would be the most appropriate response by the nurse? A. "Long-term home oxygen therapy should be used to prevent respiratory failure." B. "Oxygen will be needed when your oxygen saturation drops to 88% and you have symptoms of hypoxia. C. "Long-term home oxygen therapy should be used to prevent heart problems related to emphysema." D. "Oxygen will not be needed until or unless you are in the terminal stages of this disease."

B. "Oxygen will be needed when your oxygen saturation drops to 88% and you have symptoms of hypoxia.Long-term oxygen therapy in the home should be considered when the oxygen saturation is 88% or less and the patient has signs of tissue hypoxia, such as cor pulmonale, erythrocytosis, or impaired mental status.

7. The nurse notes a physician's order written at 10:00 AM for 2 units of packed red blood cells to be administered to a patient who is anemic secondary to chronic blood loss. If the transfusion is picked up at 11:30, the nurse should plan to hang the unit no later than which of the following times? A. 11:45 AM B. 12:00 noon C. 12:30 PM D. 3:30 PM

B. 12:00 noon The nurse must hang the unit of packed red blood cells within 30 minutes of signing them out from the blood bank

Seconal 0.1 gram PRN at bedtime is prescribed for rest. The scored tablets are labeled grain 1.5 per tablet. How many tablets should the nurse plan to administer? A. 1/2 tablet B. 1 tablet C. 1 1/2 tablet D. 2 tablets

B. 15 gr = 1 g, 0.1 x 15 = 1.5 grains

40. The patient has an order for albuterol 5 mg via nebulizer. Available is a solution containing 2 mg/ml. How many milliliters should the nurse use to prepare the patient's dose? A. 0.2 B. 2.5 C. 3.75 D. 5.0

B. 2.5

The patient has an order for albuterol 5 mg via nebulizer. Available is a solution containing 2 mg/ml. How many milliliters should the nurse use to prepare the patient's dose? A. 0.2 B. 2.5 C. 3.75 D. 5.0

B. 2.5

2. The nurse is caring for a patient with COPD and pneumonia who has an order for arterial blood gases to be drawn. Which of the following is the minimum length of time the nurse should plan to hold pressure on the puncture site? A. 2 minutes B. 5 minutes C. 10 minutes D. 15 minutes

B. 5 minutes Following obtaining an arterial blood gas, the nurse should hold pressure on the puncture site for 5 minutes by the clock to be sure that bleeding has stopped. An artery is an elastic vessel under higher pressure than veins, and significant blood loss or hematoma formation could occur if the time is insufficient.

Which client situation requires the most immediate intervention by the nurse? A. One day day after surgery, a client complains of unpleasant flatulence. B. A bedfast client experiences an episode of urinary incontinence. C. An ambulatory client reports the onset of stress incontinence. D. A client receiving regular dosage of an opioid develops constipation

B. A bedfast client experiences an episode of urinary incontinence.

A 7-year-old child admitted to the hospital with acute glomerulonephritis (AGN). When obtaining the nursing history, which finding should the nurse expect to obtain? A. A recent DPT immunization B. A recent strep throat infection C. High blood cholesterol levels on routine screening D. Increased thirst and urination

B. A recent strep throat infection

A female resident of a long-term care facility is being admitted to the medical department. The client has a fractured humerus and methicillin-resistant staphylococcus aureus (MRSA). Which room should the charge nurse assign this client? A. A private room, and institute protective environment measures B. A semi-private room with a another client who also has MRSA C. A private isolation room with a vented negative airflow system D. A semi-private room with a client who has hepatitis B (HBV)

B. A semi-private room with a another client who also has MRSA

When assigning clients on a medical-surgical floor to an RN and a PN, it is best for the charge nurse to assign which client to the PN? A. A young adult with bacterial meningitis with recent seizures B. An older adult client with pneumonia and viral meningitis C. A female client in isolation with meningococcal meningitis D. A male client 1 day postoperative after drainage of a brain abscess

B. An older adult client with pneumonia and viral meningitis Rationale: The most stable client is option B. Options A, C, and D are all at high risk for increased intracranial pressure and require the expertise of the RN for assessment and management of care.

4. The nurse is caring for a patient who is to receive a transfusion of two units of packed red blood cells. After obtaining the first unit from the blood bank, the nurse would ask which of the following health team members in the nurses' station to assist in checking the unit before administration? A. Unit secretary B. Another registered nurse C. A physician's assistant D. A phlebotomist

B. Another registered nurseBefore hanging a transfusion, the registered nurse must check the unit with another RN or with a licensed practical (vocational) nurse, depending on agency policy.

The nurse is caring for a patient who is to receive a transfusion of two units of packed red blood cells. After obtaining the first unit from the blood bank, the nurse would ask which of the following health team members in the nurses' station to assist in checking the unit before administration? A. Unit secretary B. Another registered nurse C. A physician's assistant D. A phlebotomist

B. Another registered nurseBefore hanging a transfusion, the registered nurse must check the unit with another RN or with a licensed practical (vocational) nurse, depending on agency policy.

A 43-year-old homeless, malnourished female client with a history of alcoholism is transferred to the ICU. She is placed on telemetry, and the rhythm strip shown is obtained. The nurse palpates a heart rate of 160 beats/min, and the client's blood pressure is 90/54. Based on these finding, which IV medication should the nurse administer? A. Amiodarone (Cordarone) B. Magnesium sulfate C. Lidocaine (Xylocaine) D. Procainamide (Pronestyl)

B. Because the client has chronic alcoholism, she is likely to have hypomagnesium. (B) is the recommended drug for torsades de pointes (AHA, 2005), which is a form of polymorphic ventricular tachycardia (VT), usually associated with a prolonged QT interval that occurs with hypomagnesemia. (A and D) increase the QT interval, which can cause the torsades to worsen. (C) is the antiarrhythmic of choice in most cases of drug-induced monomorphic VT, not torsades.

At 1000 the healthcare provider prescribes an increase dose in the dosage of a client's loop diuretic from 40 mg to 80 mg a day. The nurse has already administered today's 40 mg dose of the loop diuretic at 0600. Which action should the nurse implement? A. Schedule the second 40 mg of the diuretic for administration at 1800 that evening B. Clarify the start date of the new dose with the healthcare provider C. Administer an additional 40 mg of the diuretic after checking the serum potassium level D. Implement the increase in the prescribed dosage the following morning

B. Clarify the start date of the new dose with the healthcare provider

The nurse is giving medication to an older client who has a percutaneous esophageal gastrostomy (PEG) tube in place. Which medication drug form should the nurse question? A. Digoxin (Lanoxin) tablet B. Enteric-coated aspirin (Ecotrin) C. Furosidemide (Lasix) tablet D. Megestrol (Megace) suspension

B. Enteric-coated aspirin (Ecotrin)

The nurse caring for a client with dysphagia is attempting to insert a nasogastric tube (NGT), but the client will not swallow and is not gagging. What action should the nurse implement to facilitate the NGT passage into the esophagus? A. Offer the client sips of water or ice and coax to swallow B. Flex the client's head with chin to the chest and insert C. Elevate the bed 90 degrees and hyperextend the head D. Push the NGT beyond the oropharynx gently yet swiftly

B. Flex the client's head with chin to the chest and insert

An adult male is admitted to the psychiatric unit from the emergency department because he is in the manic stage of bipolar disorder. He has lost 10 pounds in the last two weeks and has not bathed in a week because he has been "trying to start a new business" and "too busy to eat." He is alert and oriented to time, place, and person, but not situation. Which nursing problem has the greatest priority? A. Hygiene self-care deficit B. Imbalanced nutrition C. Disturbed sleep pattern D. Self neglect

B. Imbalanced nutrition

As a means of relieving a client's pain associated with osteoarthritis, the nurse plans to provide local rest. To implement this intervention, which action should the nurse take? A. Elevate the affected joint with an icepack on it B. Immobilize the affected joint with a splint C. Administer a prescribed local topical salicylate D. Maintain bedrest with bathroom privileges

B. Immobilize the affected joint with a splint

In monitoring a client's respiratory stauts, which symptom is characteristic of early acute (adult) respiratory distress syndrome (ARDS)? A. Coarse breath sounds B. Increased respiratory rate C. Intercostal retractions D. Pleural friction rubs

B. Increased respiratory rate

In planning strategies to reduce a client's risk for complications following orthopedic surgery, the nurse recognizes which pathology as the underlying cause of osteomyelitis? A. Metastatic process B. Infectious process C. Autoimmune disorder D. Inflammatory disorder

B. Infectious process Osteomyelitis is serve infection of the bone, bone marrow, and surrounding tissue

An adult male who returned from a vacation in Mexico three weeks ago calls the clinic complaining of abdominal pain, weight loss, and diarrhea. What action should the nurse take? A. Encourage the client to go to the emergency room B. Instruct the client to bring in a stool sample C. Ask the client if he is experiencing dyspnea D. Tell the client to eat toast and Gatorade sports drink

B. Instruct the client to bring in a stool sample

When assessing a client who had a supratentorial craniotomy, what action should the nurse implement when determining the client's Glasgow coma scale (GCS) rating? A. Inject cold water into the client's ear. B. Instruct the client to raise an arm C. Determine the intracranial pressure D. Check the patellar and radial reflexes

B. Instruct the client to raise an arm

A male client has just undergone a laryngectomy and has a cuffed tracheostomy tube in place. When initiating bolus tube feedings postoperatively, when should the nurse inflate the cuff? A. Immediately after feeding B. Just prior to tube feeding C. Continuous inflation is required D. Inflation is not required

B. Just prior to tube feeding Rationale: The cuff should be inflated before the feeding to block the trachea and prevent food from entering (B) if oral feedings are started while a cuffed tracheostomy tube is in place. It should remain inflated throughout the feeding to prevent aspiration of food into the respiratory system. (A and D) place the client at risk for aspiration. (C) places the client at risk for tracheal wall necrosis.

Which of the following is a factor significant in the development of anemia in men? A. Condom use B. Large hemorrhoids C. A diet high in cholesterol D. Smoking one pack of cigarettes daily

B. Large hemorrhoids Gastrointestinal (GI) tract bleeding is a common etiologic factor in men and may result from peptic ulcers, hiatal hernia, gastritis, cancer, hemorrhoids, diverticula, ulcerative colitis, or salicylate poisoning.

The nurse makes a supervisory home visit to observes an unlicensed assistive personnel (UAP) who is providing personal care for a client with Alzheimer's Disease. The nurse observes that whenever the client gets upset, the UAP changes the subject. What action should the nurse take in response to this observation? A. Affirm that the UAP is using an effective strategy to reduce the client's anxiety B. Meet with the UAP later to role model more assertive communication techniques C. Assume care of the client to ensure that the effective communication is maintained D. Tell the UAP to offer more choices during the personal care to prevent anxiety

B. Meet with the UAP later to role model more assertive communication techniques

The nurse learns that a client in a semi-private room has a postoperative wound that is colonized with a multi-drug resistant organism. What action should the nurse implement? A. Maintain standard precautions and place the roommate in protective isolation B. Move the client to a private room and implement contact precautions for the client C. Initiate airborne precautions for both clients, leaving the in the semi-private room D. Begin droplet precautions for both clients before moving them to private rooms

B. Move the client to a private room and implement contact precautions for the client

A male client presents to the clinic with large draining ulcers on his lower legs that are characteristic of Kapok's sarcoma lesions. He is accompanied by two family members. What actions should the nurse take? A. Ask the family members to wear gloves when touching the client B. Send family to the waiting area while the client's history is taken C. Obtain a blood sample to determine of the client is HIV positive D. Complete a head to toe assessment to identify other signs of HIV

B. Send family to the waiting area while the client's history is taken

37. The nurse is scheduled to give a dose of salmeterol by metered dose inhaler (MDI). The nurse would administer the right drug by selecting the inhaler with which of the following trade names? A. Vanceril B. Serevent C. AeroBid D. Atrovent

B. Serevent The trade or brand name for salmeterol, an adrenergic bronchodilator, is Serevent.

The nurse is scheduled to give a dose of salmeterol by metered dose inhaler (MDI). The nurse would administer the right drug by selecting the inhaler with which of the following trade names? A. Vanceril B. Serevent C. AeroBid D. Atrovent

B. Serevent The trade or brand name for salmeterol, an adrenergic bronchodilator, is Serevent.

The nurse is assigned to care for a patient in the emergency department admitted with an exacerbation of asthma. The patient has received a β-adrenergic bronchodilator and supplemental oxygen. If the patient's condition does not improve, the nurse should anticipate which of the following is likely to be the next step in treatment? A. Pulmonary function testing B. Systemic corticosteroids C. Biofeedback therapy D. Intravenous fluids

B. Systemic corticosteroids Systemic corticosteroids speed the resolution of asthma exacerbations and are indicated if the initial response to the β-adrenergic bronchodilator is insufficient.

A male client with chronic renal failure (CRF) is admitted to the intensive care unit after missing his last three appointments at the dialysis center. His arterial blood gas (ABG) results are: pH 7.32; PaCO2 32 mmHG; HCO3 18 mEq/L. Which assessment finding should the nurse expect this client to exhibit? A. Diaphoresis B. Tachypnea C. Hypotension D. Bradycardia

B. Tachypnea

A hospitalized client is receiving nasogastric tube feedings via a small-bore tube and a continuous pump infusion. He begins to cough and produces a moderate amount of white sputum. Which action should the nurse take first? A. Auscultate the client's breath sounds. B. Turn off the continuous feeding pump. C. Check placement of the nasogastric tube. D. Measure the amount of residual feeding.

B. Turn off the continuous feeding pump. Rationale: A productive cough may indicate that the feeding has been aspirated. The nurse should first stop the feeding (B) to prevent further aspiration. (A, C, and D) should all be performed before restarting the tube feeding if no evidence of aspiration is present and the tube is in place.

The nurse observes ventricular fibrillation on telemetry and upon entering the clients bathroom finds the client unconscious on the floor. What intervention should the nurse implement first? A. Administer an antidysrhythmic medication. B. Start cardiopulmonary resuscitation. C. Defibrillate the client at 200 joules. D. Assess the client's pulse oximetry.

B. Ventricular fibrillation is a life-threatening dysrhythmia and CPR should be started immediately. A & C are appropriate but B is the priority. D does not address the seriousness of the situation.

When administering medications, when is the last opportunity for the nurse to discover a near-miss medication error? A. During medication administration documentation B. When determining the client'd identity at the bedside C. After the dose is dispensed by the computer-controlled system D. As the prescription is verified with the client's medical record

B. When determining the client'd identity at the bedside

The nurse prepares to suction a client using nasotracheal suctioning. Which ongoing assessments should the nurse plan to complete while performing the procedure? A. amount of wall suction B. color of the lips C. breathing pattern D. breath sounds E. appearance of secretions

B. color of the lips C. breathing pattern E. appearance of secretions

A family member was taught to suction a client's tracheostomy prior to the client's discharge from the hospital. Which observation by the nurse indicates that the family member is capable of correctly performing the suctioning technique? A. Turns on the continuous wall suction to -190 mm Hg B. Inserts the catheter until resistance or coughing occurs C. Withdraws the catheter while maintaining suctioning D. Re-clears the tracheostomy after suctioning the mouth

B. indicates correct technique for performing suctioning. Suction pressure should be between -80 and -120 (A). The catheter should be withdrawn 1-2 cm at a time with intermittent suction (C). (D) introduces pathogens.

A postoperative client receives a Schedule II opioid analgesic for pain. Which assessment finding requires the most immediate intervention by the nurse? A. Hypoactive bowel sounds with abdominal distention B. Client reports continued pain of 8 on a 10-point scale C. Respiratory rate of 12 breaths/min, with O2 saturation of 85% D. Client reports nausea after receiving the medication

C.Respiratory rate of 12 breaths/min, with O2 saturation of 85% Rationale: Administration of a Schedule II opioid analgesic can result in respiratory depression, which requires immediate intervention by the nurse to prevent respiratory arrest. Options A, B, and D require action by the nurse but are of less priority than option C.

A client who is receiving NS at 75 mL/h has dry, sticky mucous membranes and inelastic skin turgor. Which action should the nurse implement?

Continue the NS at 75 mL/h and encourage oral fluids intake

Which of the following nursing interventions is most appropriate to enhance oxygenation in a patient with unilateral malignant lung disease? A. Positioning patient on right side. B. Maintaining adequate fluid intake C. Performing postural drainage every 4 hours D. Positioning patient with "good lung down"

D. Positioning patient with "good lung down" Therapeutic positioning identifies the best position for the patient assuring stable oxygenation status. Research indicates that positioning the patient with the unaffected lung (good lung) dependent best promotes oxygenation in patients with unilateral lung disease. For bilateral lung disease, the right lung down has best ventilation and perfusion. Increasing fluid intake and performing postural drainage will facilitate airway clearance, but positioning is most appropriate to enhance oxygenation.

21. To prevent atelectasis in an 82-year-old patient with a hip fracture, a nurse should A. supply oxygen. B. suction the upper airway. C. ambulate the patient frequently. D. assist the patient with aggressive coughing and deep breathing.

D. assist the patient with aggressive coughing and deep breathing. Decreased mobility after surgery in older adults creates the possibility of fluid buildup and retention in lung tissue. One of the primary goals of nursing intervention is to prevent atelectasis in a high-risk patient. Aggressive coughing and deep breathing can prevent atelectasis in the postoperative patient.

An older client is admitted with a diagnosis of bacterial pneumonia. Which symptom should the nurse report to the health care provider after assessing the client? A. Leukocytosis and febrile B. Polycythemia and crackles C. Pharyngitis and sputum production D. Confusion and tachycardia

D.Confusion and tachycardia Rationale: The onset of pneumonia in the older client may be signaled by general deterioration, confusion, increased heart rate, and/or increased respiratory rate. Options A, B, and C are often absent in the older client with bacterial pneumonia.

A nurse is checking for paradoxical blood pressure on a client who has constrictive pericarditis. Which of the following findings should the nurse expect? 1) Apical pulse rate different than the radial pulse rate 2) Increase in heart rate by 20% when standing 3) Drop in systolic BP by 20 mm Hg when moving from a lying to a sitting position 4) Drop in systolic BP more than 10 mm Hg on inspiration

Drop in systolic BP more than 10 mm Hg on inspiration

The home health nurse visits a client who has a serum sodium level of 123 mEq/L. To explore possible etiologies for this value, what question should the nurse ask the client?

How much water and ice chips do you have each day?

The community health nurse who works in a gulf coast town that caters to tourists is presenting a class to residents on disaster management. Which content is most important for the nurse to be included in this class?

Hurricane preparedness for families and businesses

A nurse is caring for a client who had a gastrectomy 5 days ago. Which of the following interventions will assist in the prevention of dumping syndrome?

Instruct the client to lie in a supine position after meals

A client diagnosed with osteoarthritis. Which intervention should the practical nurse implement to help relieve joint pain and stiffness?

Instruct the client to take an analgesic before walking daily. Adequate pain management is important for the success of an exercise program. Keeping the joints active decreases pain, so taking an analgesic and walking daily is likely to help decrease joint pain and stiffness.

A nurse is assisting with the care of a client following a transurethral resection of the prostate (TURP) and has an indwelling urinary catheter.Which of the following actions should the nurse take? 1) Weigh the client weekly. 2) Irrigate the catheter as prescribed. 3) Instruct the client to report an urge to urinate. 4) Instruct the client to bear down as if to have a bowel movement every hour

Irrigate the catheter as prescribed

A client has a serum potassium level of 3 mEq/L. Which findings should the practical nurse report to the charge nurse? (Select all that apply.)

Muscle cramps Abnormal heart rhythms A normal potassium level ranges from 3.5 to 5 mEq/L (mmol/L). Signs and symptoms of low potassium include muscle cramps and dysrhythmias.

A nurse is reinforcing teaching with a client about how to prevent the onset of manifestations of Raynaud's phenomenon. Which of the following statements should the nurse identify as an indication that the client needs further teaching? 1) "I will keep my house at a cool temperature." 2) "I will try to anticipate and avoid stressful situations." 3) "I will complete the smoking cessation program I started." 4) "I will wear gloves when removing food from the freezer."

"I will keep my house at a cool temperature"

A nurse is reinforcing teaching about rifampin with a female client who has active tuberculosis. Which of the following statements should the nurse include in the teaching? 1) "You should wear glasses instead of contacts while taking this medication." 2) "The medication causes amenorrhea if taken along with an oral contraceptive." 3) "A yellow tint to the skin is an expected reaction to the medication." 4) "Lifelong treatment with this medication is necessary."

"You should wear glasses instead of contact while taking this medication"

A client receiving cholestyramine (Questran) for hyperlipidemia should be evaluated for what vitamin deficiency? A) K. B) B12. C) B6. D) C.

A) K. Clients should be monitored for an increased prothrombin time and prolonged bleeding times which would alert the nurse to a vitamin K deficiency (A). These drugs reduce absorption of the fat soluble (lipid) vitamins A, D, E, and K. (B, C, and D) are not fat soluble vitamins.

A client is admitted to the hospital with a diagnosis of severe acute diverticulitis. Which assessment finding should the nurse expect this client to exhibit? A) Lower left quadrant pain and a low-grade fever. B) Severe pain at McBurney's point and nausea. C) Abdominal pain and intermittent tenesmus. D) Exacerbations of severe diarrhea.

A) Lower left quadrant pain and a low-grade fever. Left lower quadrant pain occurs with diverticulitis because the sigmoid colon is the most common area for diverticula, and the inflammation of diverticula causes a low-grade fever (A). (B) would be indicative of appendicitis. (C and D) are symptoms exhibited with ulcerative colitis.

A splint is prescribed for nighttime use by a client with rheumatoid arthritis. Which statement by the nurse provides the most accurate explanation for use of the splints? A) Prevention of deformities. B) Avoidance of joint trauma. C) Relief of joint inflammation. D) Improvement in joint strength.

A) Prevention of deformities. Splints may be used at night by clients with rheumatoid arthritis to prevent deformities (A) caused by muscle spasms and contractures. Splints are not used for (B). (C) is usually treated with medications, particularly those classified as non-steroidal antiinflammatory drugs (NSAIDs). For (D), a prescribed exercise program is indicated.

In preparing to administer intravenous albumin to a client following surgery, what is the priority nursing intervention? (Select all that apply.) A) Set the infusion pump to infuse the albumin within four hours. B) Compare the client's blood type with the label on the albumin. C) Assign a UAP to monitor blood pressure q15 minutes. D) Administer through a large gauge catheter. E) Monitor hemoglobin and hematocrit levels. F) Assess for increased bleeding after administration.

A) Set the infusion pump to infuse the albumin within four hours. D) Administer through a large gauge catheter. E) Monitor hemoglobin and hematocrit levels. F) Assess for increased bleeding after administration. (A, D, E, and F) are the correct selections. Albumin should be infused within four hours because it does not contain any preservatives. Any fluid remaining after four hours should be discarded (A). Albumin administration does not require blood typing (B). Vital signs should be monitored periodically to assess for fluid volume overload, but every 15 minutes is not necessary (C). This frequency is often used during the first hour of a blood transfusion. A large gauge catheter (D) allows for fast infusion rate, which may be necessary. Hemodilution may decrease hemoglobin and hematocrit levels (E), while increased blood volume and blood pressure may cause bleeding (F).

Small bowel obstruction is a condition characterized by which finding? A) Severe fluid and electrolyte imbalances. B) Metabolic acidosis. C) Ribbon-like stools. D) Intermittent lower abdominal cramping.

A) Severe fluid and electrolyte imbalances. Among the findings characteristic of a small bowel obstruction is the presence of severe fluid and electrolyte imbalances (A). (B, C, and D) are findings associated with large bowel obstruction.

A female client receiving IV vasopressin (Pitressin) for esophageal varice rupture reports to the nurse that she feels substernal tightness and pressure across her chest. Which PRN protocol should the nurse initiate? A) Start an IV nitroglycerin infusion. B) Nasogastric lavage with cool saline. C) Increase the vasopressin infusion. D) Prepare for endotracheal intubation.

A) Start an IV nitroglycerin infusion. Vasopressin is used to promote vasoconstriction, thereby reducing bleeding. Vasoconstriction of the coronary arteries can lead to angina and myocardial infarction, and should be counteracted by IV nitroglycerin per prescribed protocol (A). (B) will not resolve the cardiac problem. (C) will worsen the problem. Endotracheal intubation may be needed if respiratory distress occurs (D).

A patient is having inspiratory stridor (crowing respiration) and the nurse suspects he is experiencing a laryngospasm. Which of the following would be most appropriate to implement for a patient experiencing a laryngospasm? A. Administer 100% oxygen. B. Position the patient in high Fowler's position. C. Insert a 16-gauge (large-bore) IV needle. D. Activate the emergency response team (code blue team) to the patient's room.

A. Administer 100% oxygen.A nurse should immediately administer 100% oxygen to the patient until the airway is fully reestablished, the larynx relaxes, and the spasms stop. Activating the emergency response team is not an immediate nursing action at this time because the nurse can administer the oxygen without the assistance of others. Positioning the patient in high Fowler's will not address the patient's need for immediate reoxygenation because of the patient's compromised respiratory state. Insertion of an IV device is not the first priority response but should be implemented after the nurse has assessed that the airway is stable.

4. When caring for a patient who is 3 hours postoperative laryngectomy, the nurse's highest priority assessment would be: A. Airway patency B. Patient comfort C. Incisional drainage D. Blood pressure and heart rate

A. Airway patency Remember ABCs with prioritization. Airway patency is always the highest priority and is essential for a patient undergoing surgery surrounding the upper respiratory system.

When caring for a patient who is 3 hours postoperative laryngectomy, the nurse's highest priority assessment would be: A. Airway patency B. Patient comfort C. Incisional drainage D. Blood pressure and heart rate

A. Airway patency Remember ABCs with prioritization. Airway patency is always the highest priority and is essential for a patient undergoing surgery surrounding the upper respiratory system.

The patient has an order for each of the following inhalers. Which of the following should the nurse offer to the patient at the onset of an asthma attack? A. Albuterol (Proventil) B. Beclomethasone (Beclovent) C. Ipratropium bromide (Atrovent) D. Salmeterol (Serevent)

A. Albuterol (Proventil) Albuterol is a short-acting bronchodilator that should initially be given when the patient experiences an asthma attack.

The nurse is performing hourly neurological check for a client with a head injury. Which new assessment finding warrants the most immediate intervention by the nurse? A. A unilateral pupil that is dilated and nonreactive to light. B. Client cries out when awakened by a verbal stimulus. C. Client demonstrates a loss of memory to the events leading up to the injury. D. Onset of nausea, headache, and vertigo.

A. Any changes in pupil size and reactivity is an indication of increasing ICP and should be reported immediately. (B) is normal for being awakened. (C & D) are common manifestations of head injury and less of an immediacy than (A).

Following a patient's bone marrow aspiration, which of the following nursing interventions should a nurse anticipate? A. Application of firm pressure to the site B. Positioning the patient in a prone position C. Positioning the patient in a supine position D. Application of a warm, moist compress to the site

A. Application of firm pressure to the site After a bone marrow aspiration procedure, a nurse should apply pressure to the aspiration site until bleeding stops. Application of a warm, moist compress will not alter the potential for bleeding. Positioning the patient to assume a supine or prone position also will not address the need to control bleeding from the aspiration site.

The nurse plans to obtain a urine specimen for culture from a client's indwelling catheter. The nurse enters the room with the syringe and notes that there is 100 ml of urine in the drainage bag, but no urine is in the tubing. What action should the nurse take? A. Clamp the tubing until urine is observed in the tubing B. Obtain sterile normal saline to irrigate the catheter C. Separate the tubing from the catheter and withdraw a urine specimen D. Remove the urine specimen from the drainage bag

A. Clamp the tubing until urine is observed in the tubing

One year after diagnosed with Pneumocystis carinii pneumonia, a client is admitted with respiratory failure. Respirations are shallow with periods of apnea. After the healthcare provider delivers a grim prognosis to the client's family, which intervention should the nurse implement first? A. Clarify client's end of life wishes B. Review client's CD4 cell count C. Obtain arterial blood gases D. Assist with insertion of an airway

A. Clarify client's end of life wishes

Furosemide is prescribed for a 4-year-old who has ventricular septal defect. Which outcome indicates to the nurse that this pharmacological intervention was effective? A. Daily weight decrease of 2 pounds B. Blood urea nitrogen (BUN) increase from 8 to 12 mg/dl C. urine specific gravity change from 1.021 to 1.031 D. urinary output decrease of 5 ml/hour

A. Daily weight decrease of 2 pounds

A client who is 3 weeks postpartum is brought to the mental health unit by her husband for admission because she has been verbalizing that baby is evil. After an assessment interview, the nurse determines the client thinks that the baby is going to bring harm to the other children. How should the nurse document the client's altered thought process? A. Delusional thoughts B. Visual hallucinations C. Ideas of reference D. Nihilistic ideas

A. Delusional thoughts

A resident in a long-term care facility is diagnosed with hepatitis B. Which intervention should the nurse implement with the staff caring for this client? A. Determine if all employees have had the hepatitis B vaccine series. B. Explain that this type of hepatitis can be transmitted when feeding the client. C. Assure the employees that they cannot contract hepatitis B when providing direct care. D. Tell the employees that wearing gloves and a gown are required when providing care.

A. Determine if all employees have had the hepatitis B vaccine series. Rationale: Hepatitis B vaccine should be administered to all health care providers (A). Hepatitis A (not hepatitis B) can be transmitted by fecal-oral contamination (B). There is a chance that staff could contract hepatitis B if exposed to the client's blood and/or body fluids; therefore, (C) is incorrect. There is no need to wear gloves and gowns except with blood or body fluid contact (D).

A male client with chronic asthma tells the nurse that he is having more episodes of bronchoconstriction and increased mucous production. Which action should the nurse implement? A. Determine if rescue inhaler is being used first during an acute episodes. B. Inquire about effectiveness of corticosteroid inhaler during pollen days C. Auscultate lungs for adventitious sounds consistent with fluid accumulation D. Ask if an environmental trigger is associated with the asthmatic episodes

A. Determine if rescue inhaler is being used first during an acute episodes.

Which content about self-care should the nurse include in the teaching plan of a female client who has genital herpes? (Select all that apply.) A. Encourage annual physical and Pap smear. B. Take antiviral medication as prescribed. C. Use condoms to avoid transmission to others. D. Warm sitz baths may relieve itching. E. Use Nystatin suppositories to control itching. F. Use a douche with weak vinegar solution to decrease itching.

A. Encourage annual physical and Pap smear. B. Take antiviral medication as prescribed. C. Use condoms to avoid transmission to others. D. Warm sitz baths may relieve itching. Rationale: The nurse should include (A, B, C, and D) in the teaching plan of a female client with genital herpes. (E) is specific for Candida infections, and (F) is used to treat Trichomonas.

35. The nurse who has administered a first dose of oral prednisone (Deltasone) to the patient with asthma writes on the care plan to begin monitoring which of the following patient parameters? A. Intake and output B. Bowel sounds C. Apical pulse D. Deep tendon reflexes

A. Intake and output Corticosteroids such as prednisone can lead to fluid retention. For this reason, it is important to monitor the patient's intake and output.

Which medication should the nurse anticipate administering to a client who is diagnosed with myxedema coma? A. Intravenous administration of thyroid hormones B. Oral administration of hypnotic agents C. Intravenous bolus of hydrocortisone D. Subcutaneous administration of vitamin K

A. Intravenous administration of thyroid hormones

Which activity is best for the nurse to initiate with a depressed client? A. Make cut-out cookies B. Play chess C. Play volleyball D. Watch television

A. Make cut-out cookies

A client is admitted to the hospital with a diagnosis of severe acute diverticulitis. Which nursing intervention has the highest priority? A. Place the client on NPO status. B. Assess the client's temperature. C. Obtain a stool specimen. D. Administer IV fluids.

A. Place the client on NPO status. Rationale: A client with acute severe diverticulitis is at risk for peritonitis and intestinal obstruction and should be made NPO (A) to reduce risk of intestinal rupture. (B, C, and D) are important but are less of a priority than (A), which is implemented to prevent a severe complication.

The nurse reviews pursed lip breathing with a patient newly diagnosed with emphysema. The nurse reinforces that this technique will assist respiration by which of the following mechanisms? A. Preventing bronchial collapse and air trapping in the lungs during exhalation B. Increasing the respiratory rate and giving the patient control of respiratory patterns C. Loosening secretions so that they may be coughed up more easily D. Promoting maximal inhalation for better oxygenation of the lungs

A. Preventing bronchial collapse and air trapping in the lungs during exhalation The focus of pursed lip breathing is to slow down the exhalation phase of respiration, which decreases bronchial collapse and subsequent air trapping in the lungs during exhalation.

Which consideration is most important when the nurse is assigning a room for a client being admitted with progressive systemic sclerosis (scleroderma)? A. Provide a room that can be kept warm. B. Make sure that the room can be kept dark. C. Keep the client close to the nursing unit. D. Select a room that is visible from the nurses' desk.

A. Provide a room that can be kept warm. Rationale: Abnormal blood flow in response to cold (Raynaud's phenomenon) is precipitated (A) in clients with scleroderma. (B) is not a significant factor. Stress can also precipitate the severe pain of Raynaud's phenomenon, so a quiet environment is preferred to (C), which is often very noisy. (D) is not necessary.

10. If a nurse is caring for an 80-year-old patient with a temperature of 100.4° F, crackles at the right lung base, pain with deep inspiration, and dyspnea, which of the following orders is the nurse's priority? A. Sputum specimen for culture and sensitivity B. Codeine 15 mg orally every 6 hours as needed C. Incentive spirometer every 2 hours while awake D. Amoxicillin (Amoxil) 500 mg orally 4 times a day

A. Sputum specimen for culture and sensitivity The patient presents with signs of a respiratory infection. To initiate the most effective therapy, the health care prescriber must know the pathogen causing the infection. Therefore, the sputum specimen is the nurse's priority. If the antibiotic is administered before the specimen is obtained, the results of the culture might not be as accurate and could impair the effectiveness of therapy. After the specimen is obtained, the nurse can administer codeine for coughing and begin the incentive spirometry to mobilize secretions and improve the patient's ability to expectorate the secretions.

19. The nurse is caring for a postoperative patient with sudden onset of respiratory distress. The physician orders a STAT ventilation-perfusion scan. Which of the following explanations should the nurse provide to the patient about the procedure? A. This test involves injection of a radioisotope to outline the blood vessels in the lungs, followed by inhalation of a radioisotope gas. B. This test will use special technology to examine cross sections of the chest with use of a contrast dye. C. This test will use magnetic fields to produce images of the lungs and chest. D. This test involves injecting contrast dye into a blood vessel to outline the blood vessels of the lungs.

A. This test involves injection of a radioisotope to outline the blood vessels in the lungs, followed by inhalation of a radioisotope gas.A ventilation-perfusion scan has two parts. In the perfusion portion, a radioisotope is injected into the blood and the pulmonary vasculature is outlined. In the ventilation part, the patient inhales a radioactive gas that outlines the alveoli.

27. A patient with acute exacerbation of COPD needs to receive precise amounts of oxygen. Which of the following types of equipment should the nurse prepare to use? A. Venturi mask B. Partial non-rebreather mask C. Oxygen tent D. Nasal cannula

A. Venturi mask The Venturi mask delivers precise concentrations of oxygen and should be selected whenever this is a priority concern. The other methods are less precise in terms of amount of oxygen delivered.

A patient with acute exacerbation of COPD needs to receive precise amounts of oxygen. Which of the following types of equipment should the nurse prepare to use? A. Venturi mask B. Partial non-rebreather mask C. Oxygen tent D. Nasal cannula

A. Venturi mask The Venturi mask delivers precise concentrations of oxygen and should be selected whenever this is a priority concern. The other methods are less precise in terms of amount of oxygen delivered.

12. A nurse is providing care to an adult female patient and observes that the Hb laboratory analysis result is 9 g/dl. Based on this finding, the nurse should expect to observe A. dyspnea. B. bradycardia. C. warm, dry skin. D. activity tolerance without complaint of fatigue.

A. dyspnea.Hb levels are used to determine the severity of anemia. Patients with moderate anemia (Hb 6 to 10 g/dL) may suffer from dyspnea, palpitations, diaphoresis with exertion, and chronic fatigue. Patients who are anemic usually have cool skin related to compensatory mechanism of mild vasoconstriction. Patients who are anemic experience tachycardia because of increased demands placed on the heart to meet overall metabolic requirements. Activity tolerance without complaint is not correct because patients with anemic conditions fatigue readily.

20. Using light pressure with the index and middle fingers, the nurse cannot palpate any of the patient's superficial lymph nodes. The nurse A. records this finding as normal. B. should reassess the lymph nodes using deeper pressure. C. asks the patient about any history of any radiation therapy. D. notifies the health care provider that x-rays of the nodes will be necessary.

A. records this finding as normal. Superficial lymph nodes are evaluated by light palpation, but they are not normally palpable. It may be normal to find small (<1.0 cm), mobile, firm, nontender nodes. Deep lymph nodes are detected radiographically.

Using light pressure with the index and middle fingers, the nurse cannot palpate any of the patient's superficial lymph nodes. The nurse A. records this finding as normal. B. should reassess the lymph nodes using deeper pressure. C. asks the patient about any history of any radiation therapy. D. notifies the health care provider that x-rays of the nodes will be necessary.

A. records this finding as normal. Superficial lymph nodes are evaluated by light palpation, but they are not normally palpable. It may be normal to find small (<1.0 cm), mobile, firm, nontender nodes. Deep lymph nodes are detected radiographically.

17. While caring for a patient with respiratory disease, a nurse observes that the oxygen saturation drops from 94% to 85% when the patient ambulates. The nurse should determine that A. supplemental oxygen should be used when the patient exercises. B. ABG determinations should be done to verify the oxygen saturation reading. C. this finding is a normal response to activity and that the patient should continue to be monitored. D. the oximetry probe should be moved from the finger to the earlobe for an accurate oxygen saturation measurement during activity.

A. supplemental oxygen should be used when the patient exercises.An oxygen saturation lower than 90% indicates inadequate oxygenation. If the drop is related to activity of some type, supplemental oxygen is indicated.

The nurse is counseling a healthy 30-year-old female client regarding osteoporosis prevention. Which activity would be most beneficial in achieving the client's goal of osteoporosis prevention? A. Cross-country skiing B. Scuba diving C. Horseback riding D. Kayaking

A.Cross-country skiing Rationale: Weight-bearing exercise is an important measure to reduce the risk of osteoporosis. Of the activities listed, cross-country skiing includes the most weight-bearing, whereas options B, C, and D involve less.

The nurse is triaging a client with a history of alcoholism who is lying in the fetal position. The client's chief complaint is a sudden, sharp abdominal pain which began in their epigastric and radiates over the entire abdominal area. Their abdomen is tender, rigid and board like; bowel sounds are diminished and their vital signs are T- 102.2° F (39° C); P- 120; RR- 24 and BP- 88/48. Which condition should the nurse suspect the client is experiencing? A. Peritonitis. B. Gastric ulcer. C. Alcohol poisoning. D. Pyloric obstruction.

Answer: A A history of alcohol abuse is a risk factor for the development of a gastric ulcer. Peritonitis can be a complication of perforation of an ulcer. Perforation of an ulcer presents as sudden, sharp pains in the mid- epigastric area which proceeds to radiate over the entire abdominal area. The pain experience by a client is dependent upon the type and amount of contents spilled into the peritoneum cavity. The position of comfort for these clients is to assume the fetal position. The abdominal assessment usually reveal diminished or absent bowel sounds and appears to be tender, rigid and board like. A client can become seriously ill within a few hours and become septic and go into hypovolemic shock. This is a surgical emergency.

Which condition is a leading cause of renal failure? A. Hypertension. B. Crohn's disease. C. Iron deficiency anemia. D. Chronic obstructive pulmonary disease.

Answer: A Although renal disease is a multifactorial disease, there are several conditions that are recognized as main causes of renal failure. Hypertension is the second-leading cause of renal failure in the United States. (Diabetes being number 1 leading cause)

The nurse is providing dietary instructions to a 68-year-old client who is at high risk for development of coronary heart disease (CHD). Which information should the nurse include? A) Limit dietary selection of cholesterol to 300 mg per day. B) Increase intake of soluble fiber to 10 to 25 grams per day. C) Decrease plant stanols and sterols to less than 2 grams/day. D) Ensure saturated fat is less than 30% of total caloric intake.

B) Increase intake of soluble fiber to 10 to 25 grams per day. To reduce risk factors associated with coronary heart disease, the daily intake of soluble fiber (B) should be increased to between 10 and 25 gm. Cholesterol intake (A) should be limited to 180 mg/day or less. Intake of plant stanols and sterols is recommended at 2 g/day (C). Saturated fat (D) intake should be limited to 7% of total daily calories.

The nurse is assessing a client who has a history of Parkinson's disease for the past 5 years. What symptoms should this client most likely exhibit? A) Loss of short-term memory, facial tics and grimaces, and constant writhing movements. B) Shuffling gait, masklike facial expression, and tremors of the head. C) Extreme muscular weakness, easy fatigability, and ptosis. D) Numbness of the extremities, loss of balance, and visual disturbances.

B) Shuffling gait, masklike facial expression, and tremors of the head. (B) are common clinical features of Parkinsonism. (A) are symptoms of chorea, (C) of myasthenia gravis, and (D) of multiple sclerosis.

While the nurse is conducting a daily assessment of an older woman who resides in a long-term facility, the client begins to cry and tell the nurse that her family has stopped calling and visiting. What action should the nurse take first? A. Determine the client's orientation to time and space B. Ask the client when a family member last visited her C. Review the client's record regarding social interactions D. Reassure the client of her family's love for her

B. Ask the client when a family member last visited her

Which change in laboratory values indicates to the nurse that a client with rheumatoid arthritis may be experiencing an adverse effect of methotrexate (Mexate) therapy? A.Increase in rheumatoid factor B.Decrease in hemoglobin level C.Increase in blood glucose level D.Decrease in erythrocyte sedimentation rate (ESR; sed rate)

B. Decrease in hemoglobin level Rationale: Methotrexate is an immunosuppressant. A common side effect is bone marrow depression, which would be reflected by a decrease in the hemoglobin level. Option A indicates disease progression but is not a side effect of the medication. Option C is not related to methotrexate. Option D indicates that inflammation associated with the disease has diminished.

Which assessment is most important for the nurse to include in the daily plan of care for a client with a burned extremity? A. Range of motion B. Distal pulse intensity C. Extremity sensation D. Presence of exudate

B. Distal pulse intensity

The nurse observes that a client with COPD is exhibiting fingernail clubbing. What action should the nurse take? A. Administer a PRN dose of albuterol (Proventil) via inhaler. B. Document the assessment finding in the nurses' notes C. Assist the client to a tripod position in the bed. D. Increase the client's oxygen from 1 to 2 liters per minute.

B. Document the assessment finding in the nurses' notes

If a patient states, "It's hard for me to breathe and I feel short-winded all the time," what is the most appropriate terminology to be applied in documenting this assessment by a nurse? A. Apnea B. Dyspnea C. Tachypnea D. Respiratory fatigue

B. Dyspnea Dyspnea is a subjective description reflective of the patient's statement indicating difficulty in breathing. Apnea refers to absence of breath or breathing. Tachypnea refers to an increased rate of breathing, usually greater than 20 breaths per minute. Respiratory fatigue is subjective and usually refers to the patient exhibiting signs and symptoms associated with a comprehensive respiratory assessment including laborious breathing, use of accessory muscles, and slowing of respirations.

Two months after treatment with radioactive iodine (I131) (RAI) for hyperthyroidism, a male client reports the onset of extreme fatigue, depression, and hair loss. What action should the nurse take? A. Confirm that the client should continue to take his anti-thyroid medication B. Explain to the client that thyroid replacement hormones may be needed. C. Advise the client to increase his caloric intake and eat high protein foods D. Assure the client that these are time-limited side effects of the treatment

B. Explain to the client that thyroid replacement hormones may be needed.

A male client with chronic alcohol use is admitted with signs of early cirrhosis. Which nursing action should the nurse delegate to the unlicensed assistive personnel (UAP)? A. Assist client with preferred meal selections B. Gather oral hygiene products for client's use C. Evaluate client for asterixis D. Assess the client for ascites

B. Gather oral hygiene products for client's use

During a well-baby, 6-month visit, a mother tells the nurse that her infant has had fewer ear infections than her 10-year-old daughter. The nurse should explain that which vaccine is likely to have made the difference in the siblings incidence of otitis media? A. Pneumococcal vaccine B. Hemophilus Influenza type B (H1B) vaccine C. Palrvizumab vaccine for RSV D. Varicella Virus Vaccine Live

B. Hemophilus Influenza type B (H1B) vaccine

A client diagnosed with chronic kidney disease (CDK) 2 years ago is regularly treated at a community hemodialysis facility. In assessing the client before his scheduled dialysis treatment, which electrolyte imbalance should the nurse anticipate? A. Hypophosphatemia B. Hypocalcemia C. Hyponatremia D. Hypokalemia

B. Hypocalcemia develops in CKD due to chronic hyperphosphatemia not (A). (C & D) incorrect you would find hypernatremia and hyperkalemia

9. Which of the following nursing interventions is of the highest priority in helping a patient expectorate thick secretions related to pneumonia? A. Humidify the oxygen as able B. Increase fluid intake to 3L/day if tolerated. C. Administer cough suppressant q4hr. D. Teach patient to splint the affected area.

B. Increase fluid intake to 3L/day if tolerated. Although several interventions may help the patient expectorate mucus, the highest priority should be on increasing fluid intake, which will liquefy the secretions so that the patient can expectorate them more easily. Humidifying the oxygen is also helpful, but is not the primary intervention. Teaching the patient to splint the affected area may also be helpful, but does not liquefy the secretions so that they can be removed.

A client with congestive heart failure and atrial fibrillation develops ventricular ectopy with a pattern of 8 ectopic beats/min. Which action should the nurse take based on this observation? A. Assess for bilateral jugular vein distention. B. Increase oxygen flow via nasal cannula. C. Administer PRN furosemide (Lasix). D. Auscultate for a pleural friction rub.

B. Increase oxygen flow via nasal cannula. Rationale: This client should have the oxygen flow immediately increased to promote oxygenation of the myocardium (B). Ventricular ectopy, characterized by multiple PVCs, is often caused by myocardial ischemia exacerbated by hypokalemia. The nurse would expect the client in congestive heart failure to have some degree of (A), which does not exacerbate the ectopy. (C) could create a more severe hypokalemia, which could increase the ectopy. The client is not exhibiting signs of (D).

A 25-year-old client was admitted yesterday after a motor vehicle collision. Neurodiagnostic studies showed a basal skull fracture in the middle fossa. Assessment on admission revealed both halo and Battle signs. Which new symptom indicates that the client is likely to be experiencing a common life-threatening complication associated with basal skull fracture? A. Bilateral jugular vein distention. B. Oral temperature of 102 degrees F. C. Intermittent focal motor seizures. D. Intractable pain in the cervical region.

B. Increased temp indicates meningitis. (C & D) these symptoms may be exhibited but are not life threatening. (A) JVD is not a typical complication of basal skull fractures.

Sublingual nitroglycerin is administer to a male client with unstable angina who complains of crushing chest pain. Five minutes later the client becomes nauseated and his blood pressure drops to 60/40. Which intervention should the nurse implement. A. Administer a second dose of nitroglycerin B. Infuse a rapid IV normal saline bolus C. Begin external chest compressions D. give a PRN antiemetic medication

B. Infuse a rapid IV normal saline bolus When chest pain is treated with a vasodilator, such as nitroglycerin, and the blood pressure falls to a critical level, a right ventricular infraction may have occurred which requires immediate infusion of IV fluid

A family member was taught to suction a client's tracheostomy prior to the client's discharge from the hospital. Which observation by the nurse indicates that the family member is capable of correctly performing the suctioning technique? A.Turns on the continuous wall suction to 190 mm Hg. B.Inserts the catheter until resistance or coughing occurs. C.Withdraws the catheter while maintaining suctioning. D.Reclears the tracheostomy after suctioning the mouth.

B. Inserts the catheter until resistance or coughing occurs. Rationale: Option B indicates correct technique for performing suctioning. Suction pressure should be between 80 and 120 mm Hg, not 190 mm Hg. The catheter should be withdrawn 1 to 2 cm at a time with intermittent, not continuous, suction. Option D introduces pathogens unnecessarily into the tracheobronchial tree.

A 58-year-old client, who has no health problems, asks the nurse about taking the pneumococcal vaccine (Pneumovax). Which statement give by the nurse would offer the client accurate information about this vaccine? A. "The vaccine is given annually before the flue season to those over 50 years of age." B. "The immunization is administered once to older adults or persons with a history of chronic illness." C. "The vaccine is for all ages and is given primarily to those person traveling overseas to infected areas." D. "The vaccine will prevent the occurrence of pneumococcal pneumonia for up to 5 years."

B. It is usually recommended that persons over 65 years of age and those with a history of chronic illness should receive the vaccine once in a lifetime. (A) the influenza vaccine is given annually. (C) travel is not the main rationale for the vaccine. (D) The vaccine is usually given once in a lifetime.

A male client has just undergone a laryngectomy and has a cuffed tracheostomy tube in place. When initiating bolus tube feedings postoperatively, when should the nurse inflate the cuff? A.Immediately after feeding B.Just prior to tube feeding C.Continuous inflation is required D.Inflation is not required

B. Just prior to tube feeding Rationale: The cuff should be inflated before the feeding to block the trachea and prevent food from entering if oral feedings are started while a cuffed tracheostomy tube is in place. It should remain inflated throughout the feeding to prevent aspiration of food into the respiratory system. Options A and D place the client at risk for aspiration. Option C places the client at risk for tracheal wall necrosis.

A client with Alzheimer's Disease falls in the bathroom. The nurse notifies the charge nurse and completes a fall follow-up assessment. What assessment finding warrants immediate intervention by the nurse? A. Urinary incontinence B. Left forearm hematoma C. Disorientation to surroundings D. Dislodged intravenous site

B. Left forearm hematoma

A 43-year-old homeless, malnourished female client with a history of alcoholism is transferred to the ICU. She is placed on telemetry, and the rhythm strip shown is obtained. The nurse palpates a heart rate of 160 beats/min, and the client's blood pressure is 90/54 mm Hg. Based on these findings, which IV medication should the nurse administer? A. Amiodarone (Cordarone) B. Magnesium sulfate C. Lidocaine (Xylocaine) D. Procainamide (Pronestyl)

B. Magnesium sulfate Rationale: Because the client has chronic alcoholism, she is likely to have hypomagnesemia. (B) is the recommended drug for torsades de pointes, which is a form of polymorphic ventricular tachycardia (VT) usually associated with a prolonged QT interval that occurs with hypomagnesemia. (A and D) increase the QT interval, which can cause the torsades to worsen. (C) is the antiarrhythmic of choice in most cases of drug-induced monomorphic VT, not torsades.

Which abnormal laboratory finding indicates that a client with diabetes needs further evaluation for diabetic nephropathy? A. Hypokalemia B. Microalbuminuria C. Elevated serum lipid levels D. Ketonuria

B. Microalbuminuria Rationale: Microalbuminuria (B) is the earliest sign of diabetic nephropathy and indicates the need for follow-up evaluation. Hyperkalemia, not (A), is associated with end-stage renal disease caused by diabetic nephropathy. (C) may be elevated in end-stage renal disease. (D) may signal the onset of diabetic ketoacidosis (DKA).

Which abnormal laboratory finding indicates that a client with diabetes needs further evaluation for diabetic nephropathy? A.Hypokalemia B.Microalbuminuria C.Elevated serum lipid levels D.Ketonuria

B. Microalbuminuria Rationale: Microalbuminuria is the earliest sign of diabetic nephropathy and indicates the need for follow-up evaluation. Hyperkalemia, not option A, is associated with end-stage renal disease caused by diabetic nephropathy. Option C may be elevated in end-stage renal disease. Option D may signal the onset of diabetic ketoacidosis (DKA).

A male client with pneumonia is diaphoretic and confused. The cardiac monitor indicates tachycardia with frequent premature ventricular beats. Atrial blood gas (ABG) results are: pH 7.24, PaCO2 65 mmHg, HCO3 24 mEq/L. Which intervention is most important for the nurse to include in the client's plan of care? A. Obtain a 12 lead electrocardiogram (ECG) daily B. Observe frequently for signs of hypoventilation C. Assess lungs for increasing pulmonary secretions D. Maintain patent IV catheter for antibiotic therapy

B. Observe frequently for signs of hypoventilation Hypoventilation is a critical sign that of pending respiratory failure. The ABG results indicate respiratory acidosis as evidenced by a decrease of pH, elevated PaCO2 and normal HCO3. Management of severe respiratory acidosis is correction of the underlying cause for hypoventilation, which may require mechanical ventilation

A client with a peripherally inserted central venous catheter (PICC) line has a fever. What client assessment is most important for the nurse to perform? A. Gently palpate the neck for tenderness B. Observe the antecubital fossa for inflammation C. Check for drainage in the subclavian area D. Measure for any increase in abdominal girth

B. Observe the antecubital fossa for inflammation Peripherally inserted central catheters are threaded into the basilic vein at the antecubital area. If the client develops a fever, the nurse should assess for signs of an infection at the insertion site of the catheter

The nurse is assessing an infant on admission to the newborn nursery and finds that both brachial pulses are bounding, but bilateral femoral pulses are only slightly palpable. Which assessment should the nurse implement next? A. Assess heart sounds for a murmur B. Obtain blood pressures in all extremities C. Listen to the lung fields for fine crackles D. Elevate the legs for evidence of edema

B. Obtain blood pressures in all extremities

The nurse manager is concerned about the number of falls that have occurred on the unit in the last month. Which action is most likely to decrease the number of falls? A. Place all clients on the unit regardless of age at risk for falls B. Obtain the evidence based practice guidelines for fall prevention C. Determine if pain medication is related to those who fell D. Inquire about what other units are doing to prevent falls

B. Obtain the evidence based practice guidelines for fall prevention

The nurse is using a straight urinary catheter kit to collect a sterile urine specimen firm a female client. After positioning and prepping the client, rank the actions in the sequence they should be implemented. A. Don sterile gloves and prepare the sterile field. B. Open the sterile catheter kit close to the client's perineum C. Cleanse the urinary meatus using the solution, swabs, and forceps provided D. Place distal end of the catheter in sterile specimen cup and insert catheter into meatus

B. Open the sterile catheter kit close to the client's perineum A. Don sterile gloves and prepare the sterile field. C. Cleanse the urinary meatus using the solution, swabs, and forceps provided D. Place distal end of the catheter in sterile specimen cup and insert catheter into meatus

In assessing a pressure ulcer on a client's hip, which action should the nurse include? A. Determine the degree of elasticity surrounding the lesion B. Photograph the lesion with a ruler placed next to the lesion C. Stage the depth of the ulcer using the Braden numeric scale D. Use a gloved finger to palpate for tunneling around the lesion

B. Photograph the lesion with a ruler placed next to the lesion

The nurse is planning care for a client with diabetes mellitus who has gangrene of the toes to the midfoot. Which goal should be included in this client's plan of care? A. Restore skin integrity. B. Prevent infection. C. Promote healing. D. Improve nutrition.

B. Prevent infection. Rationale: The prevention of infection is a priority goal for this client (B). Gangrene is the result of necrosis (tissue death). If infection develops, there is insufficient circulation to fight the infection and the infection can result in osteomyelitis or sepsis. Because tissue death has already occurred, (A and C) are unattainable goals. (D) is important but of less priority than (B).

The nurse teaches a client with type 2 diabetes nutritional strategies to decrease obesity. Which food item(s) chosen by the client indicates understanding of the teaching? (Select all that apply.) A. White bread B. Salmon C. Broccoli D. Whole milk E. Banana

B. Salmon C. Broccoli E. Banana Rationale: (B, C, and E) provide fresh fruits, lean meats and fish, vegetables, whole grains, and low-fat dairy products. All are recommended by the American Diabetes Association (ADA) and are a part of the My Plate guidelines recommended by the U.S. Department of Agriculture (USDA). Whole milk (D) is high in fat and is not recommended by ADA. White bread is milled, a process that removes the essential nutrients. It should be avoided for weight loss and is a poor choice for the client with diabetes (A).

26. The nurse is assigned to care for a patient in the emergency department admitted with an exacerbation of asthma. The patient has received a β-adrenergic bronchodilator and supplemental oxygen. If the patient's condition does not improve, the nurse should anticipate which of the following is likely to be the next step in treatment? A. Pulmonary function testing B. Systemic corticosteroids C. Biofeedback therapy D. Intravenous fluids

B. Systemic corticosteroids Systemic corticosteroids speed the resolution of asthma exacerbations and are indicated if the initial response to the β-adrenergic bronchodilator is insufficient.

The nurse assesses a postoperative client. Oxygen is being administered at 2 L/min and a saline lock is in place. Assessment shows cool, pale, moist skin. The client is very restless and has scant urine in the urinary drainage bag. What intervention should the nurse implement first. A. Measure urine specific gravity. B. Obtain IV fluids for infusion protocol. C. Prepare for insertion of a central venous catheter. D. Auscultate the client's breath sounds.

B. The client is at risk for hypovolemic shock and is exhibiting early signs. Start IV to restore tissue perfusion. (A, C, D) are all important but less of a priority.

13. If a patient with an uncuffed tracheostomy tube coughs violently during suctioning and dislodges the tracheostomy tube, a nurse should first A. call the physician. B. attempt to reinsert the tracheostomy tube. C. position the patient in a lateral position with the neck extended. D. cover the stoma with a sterile dressing and ventilate the patient with a manual bag-mask until the physician arrives.

B. attempt to reinsert the tracheostomy tube.Retention sutures may be grasped (if present) and the tracheostomy opening spread, or a hemostat may be used to spread the opening. The obturator is inserted into the replacement tube (one size smaller than the original tube), lubricated with saline solution, and inserted into the stoma at a 45-degree angle to the neck. If the attempt is successful, the obturator tube should immediately be removed.

If a patient with an uncuffed tracheostomy tube coughs violently during suctioning and dislodges the tracheostomy tube, a nurse should first A. call the physician. B. attempt to reinsert the tracheostomy tube. C. position the patient in a lateral position with the neck extended. D. cover the stoma with a sterile dressing and ventilate the patient with a manual bag-mask until the physician arrives.

B. attempt to reinsert the tracheostomy tube.Retention sutures may be grasped (if present) and the tracheostomy opening spread, or a hemostat may be used to spread the opening. The obturator is inserted into the replacement tube (one size smaller than the original tube), lubricated with saline solution, and inserted into the stoma at a 45-degree angle to the neck. If the attempt is successful, the obturator tube should immediately be removed.

19. A patient with hemophilia is hospitalized with acute knee pain and swelling. An appropriate nursing intervention for the patient includes A. wrapping the knee with an elastic bandage. B. placing the patient on bed rest and applying ice to the joint. C. gently performing range-of-motion (ROM) exercises to the knee to prevent adhesions. D. administering nonsteroidal anti-inflammatory drugs (NSAIDs) as needed for pain.

B. placing the patient on bed rest and applying ice to the joint. During an acute bleeding episode in a joint, it is important to totally rest the involved joint and slow bleeding with application of ice. Drugs that decrease platelet aggregation, such as aspirin or NSAIDs, should not be used for pain. As soon as bleeding stops, mobilization of the affected area is encouraged with range-of-motion (ROM) exercises and physical therapy.

A patient with hemophilia is hospitalized with acute knee pain and swelling. An appropriate nursing intervention for the patient includes A. wrapping the knee with an elastic bandage. B. placing the patient on bed rest and applying ice to the joint. C. gently performing range-of-motion (ROM) exercises to the knee to prevent adhesions. D. administering nonsteroidal anti-inflammatory drugs (NSAIDs) as needed for pain.

B. placing the patient on bed rest and applying ice to the joint. During an acute bleeding episode in a joint, it is important to totally rest the involved joint and slow bleeding with application of ice. Drugs that decrease platelet aggregation, such as aspirin or NSAIDs, should not be used for pain. As soon as bleeding stops, mobilization of the affected area is encouraged with range-of-motion (ROM) exercises and physical therapy.

A client who recently underwent a tracheostomy is being prepared for discharge to home. Which instructions is most important for the nurse to include in the discharge plan? A. explain how to use communication tools B. teach tracheal suctioning techniques C. encourage self-care and independence D. demonstrate how to clean tracheostomy site

B. teach tracheal suctioning techniques

A client with alcohol-related liver disease is admitted to the unit. Which prescription should the nurse call the health care provider about for reverification for this client? A. Vitamin K1, 5 mg IM daily B. High-calorie, low-sodium diet C. Fluid restriction to 1500 mL/day D. Nembutal sodium at bedtime for rest

B.High-calorie, low-sodium diet Rationale: Sedatives such as pentobarbital are contraindicated for clients with liver damage and can have dangerous consequences. Option A is often prescribed because the normal clotting mechanism is damaged. Option B is needed to help restore energy to the debilitated client. Sodium is often restricted because of edema. Fluids are restricted to decrease ascites, which often accompanies cirrhosis, particularly in the later stages of the disease.

A nurse is caring for a client who is receiving hemodialysis. Which of the following client measurements should the nurse compare before and after dialysis treatment to determine fluid losses? 1) Neck vein distention 2) Blood pressure 3) Body weight 4) Abdominal girth

Body weight

What discharge instruction is most important for a client after a kidney transplant? A) Weigh weekly. B) Report symptoms of secondary Candidiasis. C) Use daily reminders to take immunosuppressants. D) Stop cigarette smoking.

C) Use daily reminders to take immunosuppressants. After renal transplantation, acute rejection is a risk for several months, so immunosuppressive therapy, such as corticosteroids and azathioprine (Imuran), is essential in preventing rejection, so the priority instruction includes measures, such as daily reminders (C), to ensure the client takes these medications regularly. Daily weights, not weekly (A), provides a better indicator of weight gain related to rejection. Although fungal infections related to the immunosuppression should be reported (B), it is more important to ensure medication compliance to prevent rejection. Although smoking (D) increases the risk of atherosclerotic vascular disease which is common in clients with an organ transplant, (C) remains the priority.

The nurse is planning care for a client with newly diagnosed diabetes mellitus that requires insulin. Which assessment should the nurse identify before beginning the teaching session? A) Present knowledge related to the skill of injection. B) Intelligence and developmental level of the client. C) Willingness of the client to learn the injection sites. D) Financial resources available for the equipment.

C) Willingness of the client to learn the injection sites. If a client is incapable or does not want to learn, it is unlikely that learning will occur, so motivation is the first factor the nurse should assess before teaching (C). To determine learning needs, the nurse should assess (A), but this is not the most important factor for the nurse to assess. (B and D) are factors to consider, but not as vital as (C).

The nurse should be correct in withholding a dose of digoxin in a client with congestive heart failure without specific instruction from the healthcare provider if the client's A) serum digoxin level is 1.5. B) blood pressure is 104/68. C) serum potassium level is 3. D) apical pulse is 68/min.

C) serum potassium level is 3. Hypokalemia (C) can precipitate digitalis toxicity in persons receiving digoxin which will increase the chance of dangerous dysrhythmias (normal potassium level is 3.5 to 5.5 mEq/L). The therapeutic range for digoxin is 0.8 to 2 ng/ml (toxic levels= >2 ng/ml); (A) is within this range. (B) would not warrant the nurse withholding the digoxin. The nurse should withhold the digoxin if the apical pulse is less than 60/min (D).

Which of the following statements made by a nurse would indicate proper teaching principles regarding feeding and tracheostomies? A. "Follow each spoon of food consumed with a drink of fluid." B. "Thin your foods to a liquid consistency whenever possible." C. "Tilt your chin forward toward the chest when swallowing your food." D. "Make sure your cuff is overinflated before eating if you have swallowing problems."

C. "Tilt your chin forward toward the chest when swallowing your food." A nurse should instruct a patient to tilt the chin toward the chest, which will close the glottis and allow food to enter the normal passageway. Ideally, foods should be of a thick consistency to enable effective swallowing and reduce the risk of aspiration. Overinflation of the cuff causes swallowing difficulties. Fluids should be consumed in small amounts after swallowing to prevent the risk of aspiration.

A 58-year-old client with chronic kidney disease (CKD) is receiving aluminum hydroxide (Amphojel). He tells the nurse that since he does not have indigestion there is no need for him to take the antacid with his meals. Which response is best for the nurse to provide? A. "CKD stresses your body to over-secrete gastric juices, and antacids help neutralize them." B. "OK, I will let your healthcare provider know that you do not need the antacid." C. "Your serum phosphate levels are up, and aluminum antacids prevent absorption of phosphates in foods." D. "I will hold the antacids for now, and if you get indigestion, I will bring it back."

C. "Your serum phosphate levels are up, and aluminum antacids prevent absorption of phosphates in foods."

6. When preparing to administer an ordered blood transfusion, the nurse selects which of the following intravenous solutions to use when priming the blood tubing? A. 5% dextrose in water B. Lactated Ringer's C. 0.9% sodium chloride D. 0.45% sodium chloride

C. 0.9% sodium chloride The blood set should be primed before the transfusion with 0.9% sodium chloride, also known as normal saline. It is also used to flush the blood tubing after the infusion is complete to ensure the patient receives blood that is left in the tubing when the bag is empty.

When preparing to administer an ordered blood transfusion, the nurse selects which of the following intravenous solutions to use when priming the blood tubing? A. 5% dextrose in water B. Lactated Ringer's C. 0.9% sodium chloride D. 0.45% sodium chloride

C. 0.9% sodium chloride The blood set should be primed before the transfusion with 0.9% sodium chloride, also known as normal saline. It is also used to flush the blood tubing after the infusion is complete to ensure the patient receives blood that is left in the tubing when the bag is empty.

A client with a serum sodium level of 125 mEq/ml or mmol/L (SI)should benefit most from the administration of which intervention solution? A. 10% Dextrose in 0.45% sodium chloride B. 5% Dextrose in 0.2% sodium chloride C. 0.9% sodium chloride solution (normal saline) D. 0.45% sodium chloride solution (half normal saline)

C. 0.9% sodium chloride solution (normal saline)

What symptom is characteristic of ureteral colic in the clients diagnosed with renal calculi? A. Symptoms of irritation associated with urinary tract infection B. Intense, deep ache in the costovertebral region C. Acute, excruciating, wave-like pain radiating to the genitalia D. Chills, fever, and dysuria

C. Acute, excruciating, wave-like pain radiating to the genitalia

A female nurse who took drugs from the unit for personal use was temporary released from duty. After completion of mandatory counseling, the nurse has asked administration to allow her to return to work. When the nurse administrator approaches the charge nurse with the impaired nurse's request, what action is best for the charge nurse to take? A. Since treatment is completed, assign the nurse to routine RN responsibilities B. Ask to meet with the impaired nurse's therapist before allowing her back on the unit C. Allow the impaired nurse to return to work and monitor medication administration D. Meet with staff to assess the feelings about the impaired nurse's return to work.

C. Allow the impaired nurse to return to work and monitor medication administration

The nurse is obtaining a blood sample via venipuncture from a preschool-aged child. Which intervention should the nurse implement? A. Explain in very simple terms why the blood is needed B. Encourage the child to talk about this experience C. Apply a large colored band-aid to the puncture site D. Place the labeled specimen in a paper cup for transport

C. Apply a large colored band-aid to the puncture site

A client is ready for discharge following the creation of an ileostomy. Which instruction should the nurse include in discharge teaching? A. Replace the stoma appliance every day. B. Use warm tap water to irrigate the ileostomy. C. Change the bag when the seal is broken. D. Measure and record the ileostomy output

C. Change the bag when the seal is broken. Rationale: A seal must be maintained to prevent leakage of irritating liquid stool onto the skin (C). (A) is excessive and can cause skin irritation and breakdown. Ileostomies produce liquid fecal drainage, so (B) is not necessary. (D) is not needed.

An adult female client is admitted to the psychiatric unit because of a complex hand washing ritual she preforms daily that takes two hours or longer to complete. She worries about staying clean and refuses to sit on any of the chairs in the day area. This client's hand washing is an example of which clinical behavior? A. Obsession B. Addiction C. Compulsion D. Phobia

C. Compulsion

A male client tells the nurse that he is concerned that he may have a stomach ulcer, because he is experiencing heartburn and a dull gnawing pain that is relieved when he eats. What is the best response by the nurse? A. Advice the client the needs to seek immediate medical evaluation and treatment of these symptoms. B. Assure the client that the symptoms may on reflect reflux, since ulcer pain is not relieved with food. C. Encourage the client to obtain a complete a physical exam since the symptoms are consistent with an ulcer D. Instruct the client that these mild symptoms can generally be resolved with changes in the diet.

C. Encourage the client to obtain a complete a physical exam since the symptoms are consistent with an ulcer

The nurse is completing an admission interview for a client with Parkinson disease. Which question will provide additional information about manifestations that the client is likely to experience? A. "Have you ever experienced any paralysis of your arms or legs?" B. "Do you have frequent blackout spells?" C."Have you ever been frozen in one spot, unable to move?" D. "Do you have headaches, especially ones with throbbing pain?"

C. Have you ever been frozen in one spot, unable to move?" Rationale: Clients with Parkinson disease frequently experience difficulty in initiating, maintaining, and performing motor activities. They may even experience being rooted to the spot and unable to move. Parkinson disease does not typically cause option A, B, or D.

23. Which of the following positions is most appropriate for the nurse to place a patient experiencing an asthma exacerbation? A. Supine B. Lithotomy C. High-Fowler's D. Reverse Trendelenburg

C. High-Fowler'sThe patient experiencing an asthma attack should be placed in high-Fowler's position to allow for optimal chest expansion and enlist the aid of gravity during inspiration.

The nurse and a social worker are talking when a male client with psychosis angrily shouts at the nurse, "Stop talking about me." The nurse should document the client is exhibiting which symptom? A. Auditory hallucinations B. Visual hallucinations C. Ideas of reference D. Thought broadcasting

C. Ideas of reference

The nurse know that normal lab values expected for an adult may vary in an older client. Which data would the nurse expect to find when reviewing laboratory values of an 80-year-old man who is in good overall health. A. Complet blood count reveals increased WBC and decreased RBC counts. B. Chemistries reveal an increased serum bilirubin with slightly increased liver enzymes. C. Urinalysis reveals slight protein in the urine and bacteriuria with pyuria. D. Serum electrolytes reveal a decreased sodium level with an increased potassium level.

C. In older adults the protein found in urine is slightly risen as a result of kidney changes or subclinical UTIs and the client frequently experiences asymptomatic bacteriuria and pyuria as a result of incomplete bladder emptying. (A, B, D) are not normal findings.

8. Which of the following clinical manifestations would the nurse expect to find during assessment of a patient admitted with pneumococcal pneumonia? A. Hyperresonance on percussion B. Fine crackles in all lobes on auscultation C. Increased vocal fremitus on palpation D. Vesicular breath sounds in all lobes

C. Increased vocal fremitus on palpation. A typical physical examination finding for a patient with pneumonia is increased vocal fremitus on palpation. Other signs of pulmonary consolidation include dullness to percussion, bronchial breath sounds, and crackles in the affected area.

Which of the following clinical manifestations would the nurse expect to find during assessment of a patient admitted with pneumococcal pneumonia? A. Hyperresonance on percussion B. Fine crackles in all lobes on auscultation C. Increased vocal fremitus on palpation D. Vesicular breath sounds in all lobes

C. Increased vocal fremitus on palpation. A typical physical examination finding for a patient with pneumonia is increased vocal fremitus on palpation. Other signs of pulmonary consolidation include dullness to percussion, bronchial breath sounds, and crackles in the affected area.

A community health nurse is concerned about the incidence of asthma among preschool aged children in a metropolitan area. Which interventions reflects a primary prevention strategy the nurse might initiate to combat the chronic illness? A. Partner with a major pharmaceutical company to provide nebulizers at a reduced cost to inner-city children B. Refer parents of preschoolers with asthma to a support group sponsored by the American Lung Association C. Inform the city council of the need to strengthen the city's air pollution ordinances D. Offer free asthma screening to children at a health fair sponsored by a local hospital

C. Inform the city council of the need to strengthen the city's air pollution ordinances

Which intervention regarding immediate postoperative care should the nurse plan to include in the preoperative teaching of a client scheduled for an incisional rotator cuff repair? A. Ice will be applied to the incision prior to exercising to help decrease pain B. A cast will be used to hold the joint securely in place until it is healed C. It will be necessary to wear a sling to keep the joint still D. A special machine will be used to keep the joint moving

C. It will be necessary to wear a sling to keep the joint still

What is the correct location for placement of the hands for manual chest compressions during cardiopulmonary resuscitation (CPR) on the adult client? A. Just above the xiphoid process, on the upper third of the sternum B. Below the xiphoid process, midway between the sternum and the umbilicus C. Just above the xiphoid process, on the lower third of the sternum D. Below the xiphoid process, midway between the sternum and the first rib

C. Just above the xiphoid process, on the lower third of the sternum Rationale: The correct placement of the hands for chest compressions in CPR is just above the notch where the ribs meet the sternum on the lower part of the sternum (C). (A) is too high. (B) would not compress the heart. (D) would likely cause damage to both structures, possibly causing a puncture of the heart, and would not render effective compressions.

When assessing a patient's respiratory status, which of the following nonrespiratory data are most important for the nurse to obtain? A. Height and weight B. Neck circumference C. Occupation and hobbies D. Usual daily fluid intake

C. Occupation and hobbiesMany respiratory problems occur as a result of chronic exposure to inhalation irritants. Common occupational sources of inhalation irritants include mines, granaries, farms, lawn care companies, paint, plastics and rubber manufacture, and building remodeling. Hobbies associated with inhalation irritants include woodworking, metal finishing, furniture refinishing, painting, and ceramics. Daily fluids, height, and weight are more related to respiratory problems secondary to cardiac issues.

30. The nurse determines that a patient is experiencing common adverse effects from the inhaled corticosteroid beclomethasone (Beclovent) after noting which of the following? A. Adrenocortical dysfunction and hyperglycemia B. Elevation of blood glucose and calcium levels C. Oropharyngeal candidiasis and hoarseness D. Hypertension and pulmonary edema

C. Oropharyngeal candidiasis and hoarseness Oropharyngeal candidiasis and hoarseness are common adverse effects from the use of inhaled corticosteroids because the medication can lead to overgrowth of organisms and local irritation if the patient does not rinse the mouth following each dose.

14. A 71-year-old patient is admitted with acute respiratory distress related to cor pulmonale. Which of the following nursing interventions is most appropriate during admission of this patient? A. Delay any physical assessment of the patient and review with the family the patient's history of respiratory problems. B. Perform a comprehensive health history with the patient to review prior respiratory problems. C. Perform a physical assessment of the respiratory system and ask specific questions related to this episode of respiratory distress. D. Complete a full physical examination to determine the effect of the respiratory distress on other body functions.

C. Perform a physical assessment of the respiratory system and ask specific questions related to this episode of respiratory distress.Because the patient is having respiratory difficulty, the nurse should ask specific questions about this episode and perform a physical assessment of this system. Further history taking and physical examination of other body systems can proceed once the patient's acute respiratory distress is being managed.

Following a devastating hurricane, a client is admitted for dehydration as the result of vomiting and diarrhea that occurred after ingesting contaminated water. The client expresses feelings of fear and anger about the destruction of homes, the loss of property due to the storm, and the looting that occurred following the storm. According to Maslow's hierarchy of need, what priority need should be addressed first? A. Seld-actualization B. Love and belonging C. Physiological needs D. Safety and security

C. Physiological needs

10. During discharge teaching for a 65-year-old patient with emphysema and pneumonia, which of the following vaccines should the nurse recommend the patient receive? A. S. aureus B. H. influenzae C. Pneumococcal D. Bacille Calmette-Guérin (BCG)

C. Pneumococcal The pneumococcal vaccine is important for patients with a history of heart or lung disease, recovering from a severe illness, age 65 or over, or living in a long-term care facility.

During discharge teaching for a 65-year-old patient with emphysema and pneumonia, which of the following vaccines should the nurse recommend the patient receive? A. S. aureus B. H. influenzae C. Pneumococcal D. Bacille Calmette-Guérin (BCG)

C. Pneumococcal The pneumococcal vaccine is important for patients with a history of heart or lung disease, recovering from a severe illness, age 65 or over, or living in a long-term care facility.

In assessing a client diagnosed with primary aldosteronism, the nurse expects the laboratory test results to indicate a decreased serum level of which substance? A.Sodium B.Phosphate C.Potassium D.Glucose

C. Potassium Rationale: Clients with primary aldosteronism exhibit a profound decline in serum levels of potassium; hypokalemia; hypertension is the most prominent and universal sign. The serum sodium level is normal or elevated, depending on the amount of water resorbed with the sodium. Option B is influenced by parathyroid hormone (PTH). Option D is not affected by primary aldosteronism.

26. Which of the following conditions is manifested by unexplained shortness of breath and a high mortality rate? A. Bleeding ulcer B. Transient ischemia C. Pulmonary embolism D. MI

C. Pulmonary embolism A high mortality rate is associated with a pulmonary embolism. A pulmonary embolism is an obstruction of the pulmonary artery caused by an embolus. It presents with hypoxia, anxiety, restlessness, and shortness of breath. Bleeding ulcers, MI, and transient ischemia are not associated with such a high mortality rate.

What action should the nurse implement to reduce a client's risk for nosocomial infection? A. Apply a face shield or googles before irrigating an infected wound B. Wear sterile gloves to administer an intravenous medication through a saline loc C. Replace continuous tube feeding bag and tubing at least daily D. Obtain a prescription to irrigate a urinary catheter with sterile saline daily

C. Replace continuous tube feeding bag and tubing at least daily

A client receives a prescription for an intramuscular pain medication. The nurse uses the Z-track method to administer the injection Which rationale supports the nurse's use of this method? A. Prevents injury to the underlying bones, nerves, and blood vessels B. Minimize client's discomfort a the injection site C. Seals needle track to avoid medication leakage through the tissue D. Ensures medication reaches the intramuscular site

C. Seals needle track to avoid medication leakage through the tissue

What is the correct procedure for performing an ophthalmoscopic examination on a client's right eye? A. Instruct the client to look at the examiner's nose and not move his/her eyes during the exam. B. Set ophthalmoscope on the plus 2 to 3 lens and hold it in front of the examiner's right eye. C. From a distance of 8 to 12 inches and slightly to the side, shine the light into the client's pupil. D. For optimum visualization, keep the ophthalmoscope at least 3 inches for the client's eye

C. The client should focus on a distant object in order to promote pupil dilation. The ophthalmoscope should be set on the 0 lens to begin (creates no correction) and should be held in front of the examiner's left eye when examining the client's right eye and kept 1" from the client's eye for optimum visualization. (A, B, D) are incorrect procedures.

Which data would the nurse expect to find when reviewing laboratory values of an 80-year-old man who is in good health overall? A. Complete blood count reveals increased white blood cell (WBC) and decreased red blood cell (RBC) counts. B. Chemistries reveal an increased serum bilirubin level with slightly increased liver enzyme levels. C. Urinalysis reveals slight protein in the urine and bacteriuria, with pyuria. D. Serum electrolytes reveal a decreased sodium level and increased potassium level.

C. Urinalysis reveals slight protein in the urine and bacteriuria, with pyuria. Rationale: In older adults, the protein found in urine slightly rises, probably as a result of kidney changes or subclinical urinary tract infections, and clients frequently experience asymptomatic bacteriuria and pyuria as a result of incomplete bladder emptying (C). Laboratory findings in (A, B, and D) are not considered to be normal findings in an older adult.

An elderly client diagnosed with chronic obstructive pulmonary disease for years ago presents in the ED with shortness of breath, cyanosis, and greenish colored sputum. His initial arterial blood bases on room air are: pH 7.39, pO2 46, pCO2 53, and HCO3 32. Which description best characterizes this client's presentation problem?

Chronic respiratory acidosis

A nurse is contribution to the plan of care for a client who is 12 hr postoperative following a right radical mastectomy with closed suction drains present. The nurse should expect that the client will be unable to perform which of the following activities with her right arm? 1) Combing her hair 2) Eating her breakfast 3) Buttoning her blouse 4) Tying her shoes

Combing her hair

client who had a radical neck dissection returns to the surgical unit with two Jackson Pratt drains in the right side of the incision. One JP bulb is open and has minimal drainage. Which action should the nurse take to increase drainage into the JP?

Compress the bulb with the tab open and then reinsert the tab into its opening

An older client is admitted to a long term care facility. Upon admission the client is oriented but fatigued, is incontinent of urine, and has a stage II pressure ulcer on the left heel. Which additional finding requires further assessment?

Confusion to time, place, and environment

An elderly client is admitted with a diagnosis of bacterial pneumonia. The nurse's assessment of the client is most likely to reveal which sign/symptom? A) Leukocytosis and febrile. B) Polycythemia and crackles. C) Pharyngitis and sputum production. D) Confusion and tachycardia.

D) Confusion and tachycardia. The onset of pneumonia in the elderly may be signaled by general deterioration, confusion, increased heart rate or increased respiratory rate (D). (A, B, and C) are often absent in the elderly with bacterial pneumonia.

17. The nurse evaluates that teaching for the patient with iron deficiency anemia has been effective when the patient states A. "I will need to take the iron supplements the rest of my life." B. "I will increase my dietary intake of milk and milk products." C. "I should increase my activity to increase my aerobic capacity." D. "I should take the iron for several months after my blood is normal."

D. "I should take the iron for several months after my blood is normal." To replace the body's iron stores, iron supplements should be continued for 2 to 3 months after the Hb level returns to normal, but if the cause of the iron deficiency is corrected, the supplements do not need to be taken for a lifetime. Milk and milk products are poor sources of dietary iron. Activity should be gradually increased as Hb levels return to normal because aerobic capacity can be increased when adequate Hb is available.

The nurse who works in labor and delivery is reassigned to the cardiac care unit for the because of a low census in labor and delivery. Which assignment is best for the charge nurse to give this nurse? A. Transfer a client to another hit B. Monitor the central telemetry C. Perform the admission of a new client D. Assis cardiac nurse with their assignments

D. Assis cardiac nurse with their assignments

A female who was admitted for alcohol detoxification is nauseated and describes feeling like roaches are crawling all over her. She is tremulous, and her blood pressure is 146/92; her pulse rate is 94 beats/minute; and her temperature is 100.8F. Which PRN medication should the nurse administer first? A. Ondansetron (Zofran) B. Acetaminophen (Tylenol) C. Ramiprl (Altace) D. Lorazepam (Ativan)

D. Lorazepam (Ativan)

A client who is admitted to the hospital is suspected of having meningitis. The nurse should plan to prepare the client for which diagnostic test? A. CT scan of brain B. Electroencephalogram (EEG) C. Synovial fluid analysis D. Lumbar puncture

D. Lumbar puncture

15. When planning appropriate nursing interventions for a patient with metastatic lung cancer and a 60-pack-year history of cigarette smoking, the nurse recognizes that the smoking has most likely decreased the patient's underlying respiratory defenses because of impairment of which of the following? A. Reflex bronchoconstriction B. Ability to filter particles from the air C. Cough reflex D. Mucociliary clearance

D. Mucociliary clearance Smoking decreases the ciliary action in the tracheobronchial tree, resulting in impaired clearance of respiratory secretions, chronic cough, and frequent respiratory infections.

The nurse is assessing a 9-year-old boy who is experiencing an acute asthma attack. When auscultating this child's breaths sounds, which finding is the nurse most likely to obtain? A. Diminished breath sounds heard throughout all areas B. Fine crackles upon inspiration C. Louder breath sounds over the lower lung fields D. Musical sounds upon expiration

D. Musical sounds upon expiration

A female client who received a nephrotoxic drug is admitted with acute renal failure and asks the nurse if she will need dialysis for the rest of her life. Which pathophysiologic consequence should the nurse explain that supports the need for temporary dialysis until acute tubular necrosis subsides? A. Azotemia B. Oliguria C.Hyperkalemia D.Nephron obstruction

D. Nephron obstruction Rationale: CKD is characterized by progressive and irreversible destruction of nephrons, frequently caused by hypertension and diabetes mellitus. Nephrotoxins cause acute tubular necrosis, a reversible acute renal failure, which creates renal tubular obstruction from endothelial cells that are sloughed or become edematous. The obstruction of urine flow will resolve with the return of an adequate glomerular filtration rate, and when it does, dialysis will no longer be needed. Options A, B, and C are manifestations seen in the acute and chronic forms of kidney disease.

A client with alcohol-related liver disease is admitted to the unit. Which prescription should the nurse call the health care provider about for reverification for this client? A. Vitamin K1 (AquaMEPHYTON), 5 mg IM daily B. High-calorie, low-sodium diet C. Fluid restriction to 1500 mL/day D. Pentobarbital (Nembutal sodium) at bedtime for rest

D. Pentobarbital (Nembutal sodium) at bedtime for rest Rationale: Sedatives such as Nembutal (D) are contraindicated for clients with liver damage and can have dangerous consequences. (A) is often prescribed because the normal clotting mechanism is damaged. (B) is needed to help restore energy to the debilitated client. Sodium is often restricted because of edema. Fluids are restricted (C) to decrease ascites, which often accompanies cirrhosis, particularly in the later stages of the disease.

13. Which of the following nursing interventions is most appropriate to enhance oxygenation in a patient with unilateral malignant lung disease? A. Positioning patient on right side. B. Maintaining adequate fluid intake C. Performing postural drainage every 4 hours D. Positioning patient with "good lung down"

D. Positioning patient with "good lung down" Therapeutic positioning identifies the best position for the patient assuring stable oxygenation status. Research indicates that positioning the patient with the unaffected lung (good lung) dependent best promotes oxygenation in patients with unilateral lung disease. For bilateral lung disease, the right lung down has best ventilation and perfusion. Increasing fluid intake and performing postural drainage will facilitate airway clearance, but positioning is most appropriate to enhance oxygenation.

Anticoagulant therapy is used in the treatment of thromboembolic disease because anticoagulants can A. dissolve the thrombi. B. decrease blood viscosity. C. prevent absorption of vitamin K. D. inhibit the synthesis of clotting factors.

D. inhibit the synthesis of clotting factors. Anticoagulant therapy is based on the premise that the initiation or extension of thrombi can be prevented by inhibiting the synthesis of clotting factors or by accelerating their inactivation. The anticoagulants heparin and warfarin do not induce thrombolysis but effectively prevent clot extension.

The nurse notes that the client's drainage has decreased from 50 to 5 mL/hr 12 hours after chest tube insertion for hemothorax. What is the best initial action for the nurse to take? A. Document this expected decrease in drainage. B. Clamp the chest tube while assessing for air leaks. C. Milk the tube to remove any excessive blood clot buildup. D. Assess for kinks or dependent loops in the tubing.

D.Assess for kinks or dependent loops in the tubing. Rationale: The least invasive nursing action should be performed first to determine why the drainage has diminished. Option A is completed after assessing for any problems causing the decrease in drainage. Option B is no longer considered standard protocol because the increase in pressure may be harmful to the client. Option C is an appropriate nursing action after the tube has been assessed for kinks or dependent loops.

Which nursing action is necessary for the client with a flail chest? A.Withhold prescribed analgesic medications. B. Percuss the fractured rib area with light taps. C.Avoid implementing pulmonary suctioning. D.Encourage coughing and deep breathing.

D.Encourage coughing and deep breathing. Rationale: Treatment of flail chest is focused on preventing atelectasis and related complications of compromised ventilation by encouraging coughing and deep breathing. This condition is typically diagnosed in clients with three or more rib fractures, resulting in paradoxic movement of a segment of the chest wall. Option C should not be avoided because suctioning is necessary to maintain pulmonary toilet in clients who require mechanical ventilation. Option A should not be withheld. Option B should not be applied because the fractures are clearly visible on the chest radiograph.

The nurse receives the client's next scheduled bag of TPN labeled with the additive NPH insulin. Which action should the nurse implement? A. Hang the solution at the current rate. B. Refrigerate the solution until needed. C. Prepare the solution with new tubing. D. Return the solution to the pharmacy.

D.Return the solution to the pharmacy. Rationale: Only regular insulin is administered by the IV route, so the TPN solution containing NPH insulin should be returned to the pharmacy. Options A, B, and C are not indicated because the solution should not be administered.

A client diagnosed with Guillain-Barré syndrome is hospitalized. Which finding is most important for the practical nurse to report to the primary health care provider?

Decrease in cognitive status of the client. A decline in cognitive status in a client is indicative of symptoms of hypoxia that are the result of the respiratory muscles being affected and an indication that the client may require the assistance of mechanical ventilation.

A client diagnosed with viral influenza is prescribed vitamin C 1000 mg PO daily and acetaminophen 650 mg PO every 4 hours PRN. The client complains to the practical nurse of abdominal cramping and increasing episodes of diarrhea. Which prescription change should the nurse anticipate?

Decrease the dose of vitamin C. Diarrhea is an adverse effect of high doses of vitamin C, so the nurse should anticipate a reduction in the dose of vitamin C.

A nurse is collecting data from a client who is postoperative following a transurethral resection of the prostate (TURP). After the nurse discontinues the client's urinary catheter, which of the following findings should the nurse report to the provider?

Decreased UO

A nurse is collecting data from a client who has a possible cataract. Which of the following manifestations should the nurse expect the client to report? 1) Decreased color perception 2) Loss of peripheral vision 3) Bright flashes of light 4) Eyestrain

Decreased color perception

A nurse is caring for a client who is 12 hours postoperative following a transurethral resection of the prostate (TURP) and has a 3-way urinary catheter with continuous irrigation.The Nurse Notes there has not been any urinary output in the last hour. Which of the following actions should the nurse perform first? 1) Notify the provider. 2) Administer a prescribed analgesic. 3) Offer oral fluids. 4) Determine the patency of the tubing.

Determine the patency of the tubing (If select all that apply choose notify provider too)

A nurse is assessing a client who has Graves' disease. Which of the following findings should the nurse expect the client to display?

Difficulty sleeping -Rationale: a client who has graves' disease has difficulty sleeping and anxiety due to the overproduction of thyroid hormone

An elderly client with a history of cardiac disease is admitted to the hospital. Since admission, the client has been confused and complaining about muscle cramps and has vomited twice. The client's vital signs are BP 130/70, P-47, and R-18. Which medication in the client's history should the practical nurse (PN) be most concerned?

Digitalis Elderly clients are particularly susceptible to the accumulation and toxicity of cardiac glycosides, such as digoxin or digitoxin (digitalis derivatives). Toxicity can cause anorexia, nausea, vomiting, diarrhea, headache, muscle cramps, and fatigue.

A healthcare worker with no known exposure to tuberculosis has received a Mantoux tuberculosis skin test. The nurse's assessment of the test after 62 hours indicates 5 mm of erythema without induration. Which is the best initial nursing action?

Document negative results in the client's medical record.

The practical nurse (PN) is providing care to a client who is experiencing slight scrotal edema following indirect herniorrhaphy. Which postoperative action is not recommended for this client?

Encourage deep breathing and coughing. A client should be discouraged from coughing following a hernia repair. The coughing will create too much intra-abdominal pressure putting increase pressure on the abdominal wall and could cause a dehiscence and/or evisceration of the surgical site. All other interventions are recommended for postop care of a hernia.

Which intervention should the nurse include in the plan of care for a client who has a chest tube due to a hemothorax?

Encourage the client to breath deeply and cough at frequent intervals.

A nurse is caring for a client who has COPD. Which of the following actions should the nurse take? 1) Encourage the client to drink 8 glasses of water a day. 2) Instruct the client to cough every 4 hr. 3) Provide the client with a low protein diet. 4) Advise the client to lie down after eating

Encourage the client to drink 8 glasses of water a day

A nurse is caring for a client who is receiving chemotherapy for treatment of ovarian cancer and experiencing nausea. Which of the following actions should the nurse take? 1) Advise the client to lie down after meals. 2) Instruct the client to restrict food intake prior to treatment. 3) Provide the client with an antiemetic 2 hr prior to the chemotherapy. 4) Encourage the client to drink a carbonated beverage 1 hr before meals.

Encourage the client to drink a carbonated beverage 1 hour before meals

A hospitalized client is receiving continuous nasogastric tube feedings at 90 mL/hour via a small-bore tube and an enteral infusion pump. Upon entering the client's room, which action should the practical nurse (PN) take first?

Ensure the client's head of bed is raised at least 30°. The 30° is the minimum degree elevation of the head of the bed for a client receiving continuous tube feedings to prevent aspiration.

A nurse is caring for a client who is experiencing an acute exacerbation of ulcerative colitis. The nurse should recognize that which of the following actions is the priority? 1) Review stress factors that can cause disease exacerbation. 2) Evaluate fluid and electrolyte levels. 3) Provide emotional support. 4) Promote physical mobility.

Evaluate fluid and electrolyte levels

A nurse is checking laboratory values to determine if a client who has diabetes is adhering to the treatment plan. Which of the following tests should the nurse use to make this determination?

Glycosylated hemoglobin levels (HbA1c)

A client diagnosed with duodenal ulcers is admitted to the hospital. The client was administered ranitidine hydrochloride 150 mg PO at bedtime. Which finding would indicate a therapeutic response of the medication?

No complaints of abdominal pain or heartburn verbalized. Lack of abdominal pain within 4 hours after meals indicates decreased duodenal irritation, a positive outcome in the treatment of duodenal ulcer.

A client residing in a memory care nursing facility with a diagnosis of diabetes approaches the nurse crying, saying "I just don't feel good." What action should the practical nurse take first?

Obtain a fingerstick blood glucose test. An early sign of hypoglycemia is increase confusion and/or irritability, sometimes described as "feeling bad." Based on the history of diabetes, the PN should first obtain objective data of a fingerstick blood glucose level to provide information to guide further nursing actions.

A nurse is caring for a client who is receiving peritoneal dialysis. The nurse should monitor the client for which of the following adverse effects?

Peritonitis

A nurse is contributing to the plan of care for a client who is postoperative following peritoneal lavage for peritonitis. The client has a nasogastric tube to low-intermittent suction and closed suction drains in place. Which of the following interventions should the nurse include in the plan? 1) Irrigate the nasogastric tube with tap water. 2) Mark abdominal girth once daily. 3) Ambulate the client twice daily. 4) Place the client in a high Fowler's position.

Place the client in a high Fowler's position

The chest x ray for a client who is admitted for pneumonia shows pleural effusion with decreased air flow in the entire left upper; lobe. After auscultating the left upper lobe, which breath sounds that verify the x ray findings should the nurse document?

Pleural friction rub

A nurse is caring for a client who has diabetes insipidus. For which of the following findings should the nurse monitor?

Polyuria -Rationale: DI is characterized by increased thirst and increased urination

A nurse is collecting data from a client who has acute gastroenteritis. Which of the following data collection findings should the nurse identify as the priority? 1) Weight loss of 3% of total body weight. 2) Blood glucose 150 mg/dL. 3) Potassium 2.5 mEq/L 4) Urine specific gravity 1.035

Potassium 2.5 mEq/L

134.) A nurse reinforces instructions to a client who is taking levothyroxine (Synthroid). The nurse tells the client to take the medication: 1. With food 2. At lunchtime 3. On an empty stomach 4. At bedtime with a snack

Rationale: Oral doses of levothyroxine (Synthroid) should be taken on an empty stomach to enhance absorption. Dosing should be done in the morning before breakfast. **Note that options 1, 2, and 4 are comparable or alike in that these options address administering the medication with food.**

Which interventions should the practical nurse implement to decrease the possibility of the client developing hypercalcemia? (Select all that apply.)

Remind the client to ambulate around the room at least three times daily. Increase fluid intake. Hypercalcemia can result from immobility. Ambulation of the client helps to prevent calcium from leaking out of bones into the serum. Increasing fluid volume PO or IV helps to decrease calcium levels in the blood.

A nurse in a provider's office is collecting data for a 45-year-old client who is having manifestations associated with perimenopause. Which of the following findings should the nurse expect? 1) Report of urinary retention 2) Elevated blood pressure above 140/90 3) Report of dryness with vaginal intercourse 4) Elevated body temperature above 37.8° C (100° F)

Report of dryness with vaginal intercourse

A client diagnosed with congestive heart failure has developed increasing pedal edema and pulmonary edema. What dietary modification is most important for the practical nurse (PN) to reinforce with this client?

Restrict salt and fluid intake. Salt and fluid restrictions are the first dietary modifications for a client who is retaining fluid as manifested by edema and pulmonary edema.

government office worker is seen in the emergency room after opening and envelope containing a powder like substance which is being tested for anthrax. Which discharge instruction should the nurse provide the client concerning inhalation anthrax?

Return to the emergency room if flu-like symptoms develop within 42 days.

A nurse is caring for a client who is scheduled to undergo thoracentesis. In which of the following positions should the nurse place the client for the procedure? 1) Prone with arms raised over the head. 2) Sitting, leaning forward over the bedside table. 3) High Fowler's position 4) Side-lying with knees drawn up to the chest.

Sitting, leaning forward over the bedside table

A client diagnosed with hypertension is prescribed a low-sodium diet. Which food choices selected from the hospital's menu demonstrates to the PN that the client understands their dietary restrictions?

Skim milk, turkey salad, roll, vanilla ice cream The client's selection of skim milk, turkey salad, roll, and vanilla ice cream, although containing some sodium, are considered low-sodium foods.

A nurse in the emergency department is assessing a client who has a history of hypothyroidism. Which of the following assessment findings should indicate to the nurse that the client is at risk for myxedema coma?

Slow speech

The parent of an adolescent tells the clinic nurse: my child has athlete's feet. I have been applying triple antibiotic ointment for two days, but there has been no improvement. Which instruction should the nurse provide?

Stop using the ointment and encourage complete drying of feet and wearing clean socks

A nurse is reviewing the plan of care for a client who has cellulitis of the leg. Which of the following interventions should the nurse recommend? 1) Apply topical antifungal agents. 2) Apply fresh ice packs every 4 hr. 3) Wash daily with an antibacterial soap. 4) Keep draining lesions uncovered to air dry.

Wash daily with an antibacterial soap

A nurse is providing teaching to a client who has type 2 diabetes mellitus about pathophysiology of the disease. Which of the following statements by the client indicates an understanding of the teaching?

"My cells are resistant to effects of insulin."

105.) A nurse is collecting data from a client and the client's spouse reports that the client is taking donepezil hydrochloride (Aricept). Which disorder would the nurse suspect that this client may have based on the use of this medication? 1. Dementia 2. Schizophrenia 3. Seizure disorder 4. Obsessive-compulsive disorder

1. Dementia Rationale: Donepezil hydrochloride is a cholinergic agent used in the treatment of mild to moderate dementia of the Alzheimer type. It enhances cholinergic functions by increasing the concentration of acetylcholine. It slows the progression of Alzheimer's disease. Options 2, 3, and 4 are incorrect.

112.) A hospitalized client is started on phenelzine sulfate (Nardil) for the treatment of depression. The nurse instructs the client to avoid consuming which foods while taking this medication? Select all that apply. 1. Figs 2. Yogurt 3. Crackers 4. Aged cheese 5 Tossed salad 6. Oatmeal cookies

1. Figs 2. Yogurt 4. Aged cheese Rationale: Phenelzine sulfate (Nardil) is a monoamine oxidase inhibitor(MAOI). The client should avoid taking in foods that are high in tyramine. Use of these foods could trigger a potentially fatal hypertensive crisis. Some foods to avoid include yogurt, aged cheeses, smoked or processed meats, red wines, and fruits such as avocados, raisins, and figs.

167.) A nurse prepares to reinforce instructions to a client who is taking allopurinol (Zyloprim). The nurse plans to include which of the following in the instructions? 1. Instruct the client to drink 3000 mL of fluid per day. 2. Instruct the client to take the medication on an empty stomach. 3. Inform the client that the effect of the medication will occur immediately. 4. Instruct the client that, if swelling of the lips occurs, this is a normal expected response.

1. Instruct the client to drink 3000 mL of fluid per day. Rationale: Allopurinol (Zyloprim) is an antigout medication used to decrease uric acid levels. Clients taking allopurinol are encouraged to drink 3000 mL of fluid a day. A full therapeutic effect may take 1 week or longer. Allopurinol is to be given with or immediately following meals or milk to prevent gastrointestinal irritation. If the client develops a rash, irritation of the eyes, or swelling of the lips or mouth, he or she should contact the health care provider because this may indicate hypersensitivity.

113.) A nurse is reinforcing discharge instructions to a client receiving sulfisoxazole. Which of the following would be included in the plan of care for instructions? 1. Maintain a high fluid intake. 2. Discontinue the medication when feeling better. 3. If the urine turns dark brown, call the health care provider immediately. 4. Decrease the dosage when symptoms are improving to prevent an allergic response.

1. Maintain a high fluid intake. Rationale: Each dose of sulfisoxazole should be administered with a full glass of water, and the client should maintain a high fluid intake. The medication is more soluble in alkaline urine. The client should not be instructed to taper or discontinue the dose. Some forms of sulfisoxazole cause the urine to turn dark brown or red. This does not indicate the need to notify the health care provider.

188.) The nurse should anticipate that the most likely medication to be prescribed prophylactically for a child with spina bifida (myelomeningocele) who has a neurogenic bladder would be: 1. Prednisone 2. Sulfisoxazole 3. Furosemide (Lasix) 4. Intravenous immune globulin (IVIG)

2. Sulfisoxazole Rationale: A neurogenic bladder prevents the bladder from completely emptying because of the decrease in muscle tone. The most likely medication to be prescribed to prevent urinary tract infection would be an antibiotic. A common prescribed medication is sulfisoxazole. Prednisone relieves allergic reactions and inflammation rather than preventing infection. Furosemide promotes diuresis and decreases edema caused by congestive heart failure. IVIG assists with antibody production in immunocompromised clients.

106.) Fluoxetine (Prozac) is prescribed for the client. The nurse reinforces instructions to the client regarding the administration of the medication. Which statement by the client indicates an understanding about administration of the medication? 1. "I should take the medication with my evening meal." 2. "I should take the medication at noon with an antacid." 3. "I should take the medication in the morning when I first arise." 4. "I should take the medication right before bedtime with a snack."

3. "I should take the medication in the morning when I first arise." Rationale: Fluoxetine hydrochloride is administered in the early morning without consideration to meals. **Eliminate options 1, 2, and 4 because they are comparable or alike and indicate taking the medication with an antacid or food.**

174.) A client with portosystemic encephalopathy is receiving oral lactulose (Chronulac) daily. The nurse assesses which of the following to determine medication effectiveness? 1. Lung sounds 2. Blood pressure 3. Blood ammonia level 4. Serum potassium level

3. Blood ammonia level Rationale: Lactulose is a hyperosmotic laxative and ammonia detoxicant. It is used to prevent or treat portosystemic encephalopathy, including hepatic precoma and coma. It also is used to treat constipation. The medication retains ammonia in the colon (decreases the blood ammonia concentration), producing an osmotic effect. It promotes increased peristalsis and bowel evacuation, expelling ammonia from the colon.

181.) A client is taking ticlopidine hydrochloride (Ticlid). The nurse tells the client to avoid which of the following while taking this medication? 1. Vitamin C 2. Vitamin D 3. Acetaminophen (Tylenol) 4. Acetylsalicylic acid (aspirin)

4. Acetylsalicylic acid (aspirin) Rationale: Ticlopidine hydrochloride is a platelet aggregation inhibitor. It is used to decrease the risk of thrombotic strokes in clients with precursor symptoms. Because it is an antiplatelet agent, other medications that precipitate or aggravate bleeding should be avoided during its use. Therefore, aspirin or any aspirin-containing product should be avoided.

39.) The client with a gastric ulcer has a prescription for sucralfate (Carafate), 1 g by mouth four times daily. The nurse schedules the medication for which times? 1. With meals and at bedtime 2. Every 6 hours around the clock 3. One hour after meals and at bedtime 4. One hour before meals and at bedtime

4. One hour before meals and at bedtime Rationale: Sucralfate is a gastric protectant. The medication should be scheduled for administration 1 hour before meals and at bedtime. The medication is timed to allow it to form a protective coating over the ulcer before food intake stimulates gastric acid production and mechanical irritation. The other options are incorrect.

Which menu selection by a male client indicates to the nurse that he understands the dietary management of Crohn's disease? A. Grilled chicken sandwich and pasta B. Tossed green salad and breadsticks C. Cheese enchiladas and beans D. Hamburger and French fries

A. Grilled chicken sandwich and pasta

A nurse is collecting data from a client who was bitten by a tick one week ago. Which of the following client manifestations should the nurse identify as an indication of the development of Lyme disease? 1) An expanding circular rash 2) Swollen, painful joints 3) Decreased level of consciousness 4) Necrosis at the site of the bite

An expanding circular rash

A nurse is caring for a client who has a diagnosis of renal calculi and reports severe flank pain. Which of the following is the priority nursing intervention?

Relieve the client's pain

A client demonstrated pupillary constriction when a pen light was shine in their eyes. Which nursing action should the practical nurse perform?

Record the finding on the assessment notes. Pupillary constriction to light is a normal finding and should be documented in the assessment notes.

A nurse is providing discharge instructions to a client who has active tuberculosis (TB). Which of the following should be included in the discharge instructions?

Sputum specimens are necessary every 2-4 weeks until there are three negative cultures

The nurse is developing a plan of care for an adult client with cardiovascular disease who reports blurred vision. Which outcome should the nurse include in the plan of care for this client?

The client's daily blood pressure will be less than 140/80 mmHg this month

A nurse is collecting data on a client who has a surgical wound healing by secondary intention. Which of the following findings should the nurse report to the charge nurse? 1) The wound is tender to touch. 2) The wound has pink, shiny tissue with a granular appearance. 3) The wound has serosanguineous drainage. 4) The wound has a halo of erythema on the surrounding skin

The wound has a halo of erythema on the surrounding skin

A nurse notes a small section of bowel protruding from the abdominal incision of a client who is postoperative. After calling for assistance, which of the following actions should the nurse take first? 1) Cover the client's wound with a moist, sterile dressing. 2) Have the client lie supine with knees flexed. 3) Check the client's vital signs. 4) Inform the client about the need to return to surgery

cover the client's wound with a moist, sterile dressing

An older African American man comes into the hypertension screening booth at a community fair. The nurse finds BP 170/94 mmHg. The client tells the nurse that he has never been treated for high blood pressure. Which action should the nurse take?

Emphasize the need for another blood pressure measurement within a week

Which food is most important for the nurse to encourage a male client with osteomalacia to include in his daily diet?

fortified milk and cereals

A client is admitted with syncopal episodes related to a third degree heart block. After the placement of a transcutaneous pacemaker, the nurse observes several episodes of the pacemaker's failure to sense. What action should the nurse take? A. Turn off the pacemaker B. Adjust the miliamperes (mA) C. Increase the pacemaker rate D. Increase the sensitively

D. Increase the sensitively

A nurse is caring for a client who has heart failure and respiratory arrest. Which of the following actions should the nurse take first? 1) Establish IV access. 2) Feel for a carotid pulse. 3) Establish an open airway. 4) Auscultate for breath sounds.

Feel for a carotid pulse

The home health practical nurse is visiting with a client who has a history of second degree heart block and pacemaker placement six months ago. Which symptom compliant by the client would be indicative of pacemaker failure?

Feelings of dizziness Feelings of dizziness may occur as the result of a decreased heart rate, leading to decreased cardiac output as a result of pacemaker failure.

In assessing a client with ulcers on the lower extremity, which findings indicate that the ulcers are likely to be of venous, rather than arterial, origin?

irregular ulcer shapes and severe edema

A nurse is caring for a client who has a seizure disorder and reports experiencing an aura. The nurse should recognize the client is experiencing which of the following conditions? 1) A continuous seizure state in which seizures occur in rapid succession 2) A sensory warning that a seizure is imminent 3) A period of sleepiness following the seizure during which arousal is difficult 4) A brief loss of consciousness accompanied by staring

A sensory warning that a seizure is imminent

The practical nurse (PN) is interviewing a male client diagnosed with hypertension. Which finding places the client at the greatest risk for development of a cerebral vascular accident (CVA)?

A waistline greater than 40 inches (101.6 cm) Males with waist sizes larger than 40 inches (101.6 cm) and females with waist sizes larger than 35 inches (88.9 cm) are at greater risk of cardiac disease increasing the risk of CVA. The more abdominal fat an individual has the more the risk goes up.

A nurse is collecting data from a client who has skeletal traction. Which of the following findings should the nurse identify as an indication of infection at the pin sites? 1) Serosanguineous drainage 2) Mild erythema 3) Warmth 4) Fever

Fever

A client diagnosed with emphysema that is oxygen-dependent lives alone at home and manages self-care with no difficulty. Which finding should prompt the home health practical nurse to consult the registered nurse case manager?

A weight loss of five pounds since the last monthly home visit A weight loss of five pounds in 1 month is a concern. Clients with COPD need additional calorie intake because they are using up a lot from the energy they are using to breath. The practical nurse needs to consult with the registered nurse case manager for a Nutrition Consult.

A client with macular degeneration reports increasing difficulty seeing things. This client is most likely experiencing which type of vision problem? A. Increasing loss of central vision. B. Decreased range of peripheral vision. C. Dry eyes with blurry vision despite glasses. D. Occasional episodes of diplopia.

Answer: A Macular degeneration affects the central area of vision. Age-related macular degeneration affects the macula, a small area at the center of the retina which allows clear, sharp vision.

If a health care provider is planning to transfuse a patient with a unit of packed red blood cells, which of the following solutions should the health care provider hang with the transfusion? A. 5% dextrose in water B. 0.9% sodium chloride C. 5% dextrose in 0.9% sodium chloride D. 5% dextrose in lactated Ringer's solution

B. 0.9% sodium chloride The only solution appropriate for administration with whole blood or blood products is 0.9% sodium chloride. The other options are not appropriate for use with blood products.

A client is status post-48 hours a colon resection. Which finding requires the most immediate intervention by the practical nurse (PN)?

Fever of 102° F (38.9° C) and chills A sudden increase in temperature is an indicator of peritonitis and chills, along with abdominal pain and tenderness. The PN should immediately notify the charge nurse, who should notify the health care provider.

12.) A nurse is caring for a client who is receiving an intravenous (IV) infusion of an antineoplastic medication. During the infusion, the client complains of pain at the insertion site. During an inspection of the site, the nurse notes redness and swelling and that the rate of infusion of the medication has slowed. The nurse should take which appropriate action? 1. Notify the registered nurse. 2. Administer pain medication to reduce the discomfort. 3. Apply ice and maintain the infusion rate, as prescribed. 4. Elevate the extremity of the IV site, and slow the infusion.

1. Notify the registered nurse. Rationale: When antineoplastic medications (Chemotheraputic Agents) are administered via IV, great care must be taken to prevent the medication from escaping into the tissues surrounding the injection site, because pain, tissue damage, and necrosis can result. The nurse monitors for signs of extravasation, such as redness or swelling at the insertion site and a decreased infusion rate. If extravasation occurs, the registered nurse needs to be notified; he or she will then contact the health care provider.

A nurse is collecting data on a client who has hyperthyroidism. Which of the following manifestations should the nurse expect the client to report? 1) Frequent mood changes 2) Constipation 3) Sensitivity to cold 4) Weight gain

Frequent mood changes

6.) The burn client is receiving treatments of topical mafenide acetate (Sulfamylon) to the site of injury. The nurse monitors the client, knowing that which of the following indicates that a systemic effect has occurred? 1.Hyperventilation 2.Elevated blood pressure 3.Local pain at the burn site 4.Local rash at the burn site

1.Hyperventilation Rationale: Mafenide acetate is a carbonic anhydrase inhibitor and can suppress renal excretion of acid, thereby causing acidosis. Clients receiving this treatment should be monitored for signs of an acid-base imbalance (hyperventilation). If this occurs, the medication should be discontinued for 1 to 2 days. Options 3 and 4 describe local rather than systemic effects. An elevated blood pressure may be expected from the pain that occurs with a burn injury.

A client is receiving an IV solution labeled Heparin Sodium 20,000 units in 5% Dextrose Injection 500 ml at 25 ml/hour. How many units of heparin is the client receiving each hour?

1000 units/hour

A nurse is reinforcing teaching with a client about cancer prevention and plans to address the importance of foods high in antioxidants. Which of the following foods should the nurse include in the teaching? 1) Cottage cheese 2) Fresh berries 3) Bran cereal 4) Skim milk

Fresh berries

A nurse is caring for a client who has multiple myeloma and has a WBC count of 2,200/mm3. Which of the following food items brought by the family should the nurse prohibit from being given to the client? 1) Baked chicken 2) Bagels 3) A factory-sealed box of chocolates 4) Fresh fruit basket

Fresh fruit basket

198.) Coal tar has been prescribed for a client with a diagnosis of psoriasis, and the nurse provides instructions to the client about the medication. Which statement by the client indicates a need for further instructions? 1. "The medication can cause phototoxicity." 2. "The medication has an unpleasant odor." 3. "The medication can stain the skin and hair." 4. "The medication can cause systemic effects."

4. "The medication can cause systemic effects." Rationale: Coal tar is used to treat psoriasis and other chronic disorders of the skin. It suppresses DNA synthesis, mitotic activity, and cell proliferation. It has an unpleasant odor, can frequently stain the skin and hair, and can cause phototoxicity. Systemic toxicity does not occur. **The name of the medication will assist in eliminating options 2 and 3**

A nurse is caring for a client who has a temperature of 39.7° C (103.5° F) and has a prescription for a hypothermia blanket. The nurse should monitor the client for which of the following adverse effects of the hypothermia blanket? 1) Shivering 2) Infection 3) Burns 4) Hypervolemia

shivering

133.) A nurse is monitoring a client receiving desmopressin acetate (DDAVP) for adverse effects to the medication. Which of the following indicates the presence of an adverse effect? 1. Insomnia 2. Drowsiness 3. Weight loss 4. Increased urination

2. Drowsiness Rationale: Water intoxication (overhydration) or hyponatremia is an adverse effect to desmopressin. Early signs include drowsiness, listlessness, and headache. Decreased urination, rapid weight gain, confusion, seizures, and coma also may occur in overhydration. **Recall that this medication is used to treat diabetes insipidus to eliminate weight loss and increased urination.**

a males client was transferred yesterday from the Emergency department to the telemetry unit because he had ST depression and resolved chest pain. When his EKG monitor alarms for ventricular tachycardia (VT), what action should the nurse take first? A. determine the clients responsiveness and respirations B. bring the crash cart to the room to defibrillate the client C. immediately initiate chest compressions D. notify the emergency response team

A. determine the clients responsiveness and respirations

238.) Ribavirin (Virazole) is prescribed for the hospitalized child with respiratory syncytial virus (RSV). The nurse prepares to administer this medication via which of the following routes? 1. Orally 2. Via face mask 3. Intravenously 4. Intramuscularly

2. Via face mask Rationale: Ribavirin is an antiviral respiratory medication used mainly in hospitalized children with severe RSV and in high-risk children. Administration is via hood, face mask, or oxygen tent. The medication is most effective if administered within the first 3 days of the infection.

57. When admitting a patient with the diagnosis of asthma exacerbation, the nurse will assess for which of the following potential triggers? (Select all that apply.) A. Exercise B. Allergies C. Emotional stress D. Decreased humidity

A,B,C Although the exact mechanism of asthma is unknown, there are several triggers that may precipitate an attack. These include allergens, exercise, air pollutants, respiratory infections, drug and food additives, psychologic factors, and GERD.

207.) A client is suspected of having myasthenia gravis, and the health care provider administers edrophonium (Enlon) to determine the diagnosis. After administration of this medication, which of the following would indicate the presence of myasthenia gravis? 1. Joint pain 2. A decrease in muscle strength 3. An increase in muscle strength 4. Feelings of faintness, dizziness, hypotension, and signs of flushing in the client

3. An increase in muscle strength Rationale: Edrophonium is a short-acting acetylcholinesterase inhibitor used as a diagnostic agent. When a client with suspected myasthenia gravis is given the medication intravenously, an increase in muscle strength would be seen in 1 to 3 minutes. If no response occurs, another dose is given over the next 2 minutes, and muscle strength is again tested. If no increase in muscle strength occurs with this higher dose, the muscle weakness is not caused by myasthenia gravis. Clients receiving injections of this medication commonly demonstrate a drop of blood pressure, feel faint and dizzy, and are flushed.

139.) Prednisone is prescribed for a client with diabetes mellitus who is taking Humulin neutral protamine Hagedorn (NPH) insulin daily. Which of the following prescription changes does the nurse anticipate during therapy with the prednisone? 1. An additional dose of prednisone daily 2. A decreased amount of daily Humulin NPH insulin 3. An increased amount of daily Humulin NPH insulin 4. The addition of an oral hypoglycemic medication daily

3. An increased amount of daily Humulin NPH insulin Rationale: Glucocorticoids can elevate blood glucose levels. Clients with diabetes mellitus may need their dosages of insulin or oral hypoglycemic medications increased during glucocorticoid therapy. Therefore the other options are incorrect.

96.) The nurse is caring for a postrenal transplant client taking cyclosporine (Sandimmune, Gengraf, Neoral). The nurse notes an increase in one of the client's vital signs, and the client is complaining of a headache. What is the vital sign that is most likely increased? 1. Pulse 2. Respirations 3. Blood pressure 4. Pulse oximetry

3. Blood pressure Rationale: Hypertension can occur in a client taking cyclosporine (Sandimmune, Gengraf, Neoral), and because this client is also complaining of a headache, the blood pressure is the vital sign to be monitoring most closely. Other adverse effects include infection, nephrotoxicity, and hirsutism. Options 1, 2, and 4 are unrelated to the use of this medication.

During admission of a patient diagnosed with non-small cell carcinoma of the lung, the nurse questions the patient related to a history of which of the following risk factors for this type of cancer? (Select all that apply.) A. Asbestos exposure B. Cigarette smoking C. Exposure to uranium D. Chronic interstitial fibrosis

A,B,C Non-small carcinoma is associated with cigarette smoking and exposure to environmental carcinogens, including asbestos and uranium. Chronic interstitial fibrosis is associated with the development of adenocarcinoma of the lung.

230.) A client is placed on chloral hydrate (Somnote) for short-term treatment. Which nursing action indicates an understanding of the major side effect of this medication? 1. Monitoring neurological signs every 2 hours 2. Monitoring the blood pressure every 4 hours 3. Instructing the client to call for ambulation assistance 4. Lowering the bed and clearing a path to the bathroom at bedtime

3. Instructing the client to call for ambulation assistance Rationale: Chloral hydrate (a sedative-hypnotic) causes sedation and impairment of motor coordination; therefore, safety measures need to be implemented. The client is instructed to call for assistance with ambulation. Options 1 and 2 are not specifically associated with the use of this medication. Although option 4 is an appropriate nursing intervention, it is most important to instruct the client to call for assistance with ambulation.

210.) Dantrolene (Dantrium) is prescribed for a client with a spinal cord injury for discomfort resulting from spasticity. The nurse tells the client about the importance of follow-up and the need for which blood study? 1. Creatinine level 2. Sedimentation rate 3. Liver function studies 4. White blood cell count

3. Liver function studies Rationale: Dantrolene can cause liver damage, and the nurse should monitor liver function studies. Baseline liver function studies are done before therapy starts, and regular liver function studies are performed throughout therapy. Dantrolene is discontinued if no relief of spasticity is achieved in 6 weeks.

58.) Isosorbide mononitrate (Imdur) is prescribed for a client with angina pectoris. The client tells the nurse that the medication is causing a chronic headache. The nurse appropriately suggests that the client: 1. Cut the dose in half. 2. Discontinue the medication. 3. Take the medication with food. 4. Contact the health care provider (HCP).

3. Take the medication with food. Rationale: Isosorbide mononitrate is an antianginal medication. Headache is a frequent side effect of isosorbide mononitrate and usually disappears during continued therapy. If a headache occurs during therapy, the client should be instructed to take the medication with food or meals. It is not necessary to contact the HCP unless the headaches persist with therapy. It is not appropriate to instruct the client to discontinue therapy or adjust the dosages.

80.) A nurse is caring for a client who is taking phenytoin (Dilantin) for control of seizures. During data collection, the nurse notes that the client is taking birth control pills. Which of the following information should the nurse provide to the client? 1. Pregnancy should be avoided while taking phenytoin (Dilantin). 2. The client may stop taking the phenytoin (Dilantin) if it is causing severe gastrointestinal effects. 3. The potential for decreased effectiveness of the birth control pills exists while taking phenytoin (Dilantin). 4. The increased risk of thrombophlebitis exists while taking phenytoin (Dilantin) and birth control pills together.

3. The potential for decreased effectiveness of the birth control pills exists while taking phenytoin (Dilantin). Rationale: Phenytoin (Dilantin) enhances the rate of estrogen metabolism, which can decrease the effectiveness of some birth control pills. Options 1, 2, are 4 are not accurate.

129.) Megestrol acetate (Megace), an antineoplastic medication, is prescribed for the client with metastatic endometrial carcinoma. The nurse reviews the client's history and contacts the registered nurse if which diagnosis is documented in the client's history? 1. Gout 2. Asthma 3. Thrombophlebitis 4. Myocardial infarction

3. Thrombophlebitis Rationale: Megestrol acetate (Megace) suppresses the release of luteinizing hormone from the anterior pituitary by inhibiting pituitary function and regressing tumor size. Megestrol is used with caution if the client has a history of thrombophlebitis. **megestrol acetate is a hormonal antagonist enzyme and that a side effect is thrombotic disorders**

Which content about self-care should the nurse include in the teaching plan of a client who has genital herpes? (Select all that apply.) A. Encourage annual physical and Pap smear. B. Take antiviral medication as prescribed. C. Use condoms to avoid transmission to others. D. Warm sitz baths may relieve itching. E. Use Nystatin suppositories to control itching. F. Douche with weak vinegar solutions to decrease itching.

A,B,C,D. (E) is specific for Candida infections and (F) is used to treat Trichomonas.

94.) The client with acquired immunodeficiency syndrome has begun therapy with zidovudine (Retrovir, Azidothymidine, AZT, ZDV). The nurse carefully monitors which of the following laboratory results during treatment with this medication? 1. Blood culture 2. Blood glucose level 3. Blood urea nitrogen 4. Complete blood count

4. Complete blood count Rationale: A common side effect of therapy with zidovudine is leukopenia and anemia. The nurse monitors the complete blood count results for these changes. Options 1, 2, and 3 are unrelated to the use of this medication.

An unconscious client is admitted to the intensive care unit and is placed on a ventilator. The ventilator alarms continuously and the client's oxygen saturation level is 62%. What action should the nurse take first? A. Call respiratory therapy B. Begin manual ventilation immediately C. Monitor oxygen saturation levels q5 minutes D. Silence the alarm and call the technician

A. Call respiratory therapy

The nurse is counseling a healthy 30-year-old female client regarding osteoporosis prevention. Which activity would be most beneficial in achieving the client's goal of osteoporosis prevention? A. Cross-country skiing B. Scuba diving C. Horseback riding D. Kayaking

A. Cross-country skiing Rationale: Weight-bearing exercise is an important measure to reduce the risk of osteoporosis. Of the activities listed, cross-country skiing (A) includes the most weight-bearing, whereas (B, C, and D) involve less.

The school nurse is planning to begin an obesity screening program in a school system. It is best to begin the screening program with which group? A. Kindergarden B. Third grade C. High school D. Onset of puberty

A. Kindergarden

A client with an electrical burn is admitted to the emergency department on a backboard with a cervical collar. Which intervention should the nurse implement? A. Place the client on cardiac telemetry B. Obtain STAT arterial blood gases C. Flush the burned area with sterile normal saline D. Elevate the client's head of bed to 45 degrees

A. Place the client on cardiac telemetry

The nurse is caring for a client with a chest tube to water seal drainage that was inserted 10 days ago because of a ruptured bullae and pneumothorax. Which finding should the nurse report to the health care provider before the chest tube is removed? A. Tidaling of water in water seal chamber B. Bilateral muffled breath sounds at bases C. Temperature of 101° F D. Absence of chest tube drainage for 2 days

A. Tidaling of water in water seal chamber Rationale: Tidaling (rising and falling of water with respirations) in the water seal chamber should be reported to the health care provider before the chest tube is removed (A) to rule out an unresolved pneumothorax or persistent air leak, which is characteristic of a ruptured bullae caused by abnormally wide changes in negative intrathoracic pressure. (B) may indicate hypoventilation from chest tube discomfort and usually improves when the chest tube is removed. (C) usually indicates an infection, which may not be related to the chest tube. (D) is an expected finding.

A client experiences residual effects following an acute attack of Meniere's disease and receives a new prescription for an antihistamine. Which assessment finding indicates that the medication is effective?

Ambulates easily without vertigo

A nurse is assisting with the care of a client who has multiple injuries following a motor vehicle crash. The nurse should monitor for which of the following manifestations of a pneumothorax? 1) Inspiratory stridor 2) Expiratory wheeze 3) Absence of breath sounds 4) Coarse crackles

Absence of breath sounds

During a clinic visit, a male client with heart failure (HF) reports that he has gained 4 pounds (1.8 kg) in the last 3 days. Which action should the nurse implement? A. Recommend controlled portions at mealtimes B. Assess for bilateral pitting pedal edema C. Auscultate all lung fields for fine crackles D. Encourage a reduced intake of table salts

C. Auscultate all lung fields for fine crackles

The client has been diagnosed with iron deficiency anemia. The nurse should anticipate that the client will need which medication? A. Hyoscyamine. B. Ferrous sulfate. C. Fiber supplements. D. Lactulose.

Answer C

The nurse who is performing blood sugar and cholesterol screenings at a community health fair determines that a female client's blood sugar is 59 mg/dl at 10:00 a.m. Which nursing intervention is most important for the nurse to implement? A. Check the client's cholesterol B. Ask the client how she's feeling C. Take the client's blood pressure D. Encourage the client to rest

B. Ask the client how she's feeling

In monitoring a client receiving propylthiouracil (PTU) for hyperthyroidism, an increase in which finding indicates that the medication is producing the desired effect? A. Urinary output B. Body weight C. Pulse rate D. Blood pressure

B. Body weight

A nurse is collecting data from a client who is 1 week postoperative following a living donor kidney transplant. Which of the following findings should the nurse indicate the client is experiencing acute kidney rejection?

BP 160/90

The nurse is assessing a client who has left ventricular heart failure. Which assessment finding is this client most likely to exhibit?

Bilateral basilar crackles

A nurse is reinforcing teaching with a client who has type 2 diabetes mellitus. The nurse determines that teaching has been effective when the client identifies which of the following manifestations of hypoglycemia? (Select all that apply.) 1) Polyuria 2) Blurry vision 3) Tachycardia 4) Polydipsia 5) Sweating

Blurry vision Tachycardia Sweating

A nurse is collecting data from a client who has Cushing's syndrome. Which of the following manifestations should the nurse expect? 1) Bruising 2) Weight loss 3) Hyperpigmentation 4) Double vision

Bruising

-While changing the dressing of a client with a leg ulcer, the nurse observes a red, tender, and swollen wound at the site of the lesion. Before reporting this finding to the healtcare provider, the nurse should note which of the client's laboratory values?

Neutrophil count

During the change of shift report, the charge nurse reviews the infusions being received by clients on the oncology unit. The client receiving which infusion should be assessed first? A. Continuous IV infusion of magnesium B. One-time infusion of albumin C. Continuous epidural infusion of morphine D. Intermittent infusion of IV vancomycin

C. Continuous epidural infusion of morphine Rationale: All four of these clients have the potential to have significant complications. The client with the morphine epidural infusion (C) is at highest risk for respiratory depression and should be assessed first. (A) can cause hypotension. The client receiving (B) is at lowest risk for serious complications. Although (D) can cause nephrotoxicity and phlebitis, these problems are not as immediately life threatening as (C).

Which assessment finding has the highest priority when planning nursing care for a client with peptic ulcer disease (PUD)? A. Epigastric pain after eating B. Dizziness when rising form a sitting position C. Dark tarry liquid stool D. Weight loss of 10 pounds in the past month

C. Dark tarry liquid stool

The nurse is triaging victims of a tornado at an emergency shelter. An adult woman who has been wandering and crying comes to the nurse. What action should the nurse take? A. Check the client's temperature, blood sugar, and urine output B. Transport the client for laboratory and electrocardiogram (EKG) C. Delegate care of the crying client to an unlicensed assistant D. Send the client to the shelter's nutrient center to obtain water and food

C. Delegate care of the crying client to an unlicensed assistant

The nurse is assessing a 4-year-old with eczema. Her skin is dry and scaly, and the mother reports that she frequently scratches her skin to the point of causing bleeding. Which guideline is indicated for care for this child? A. Apply baby lotion to her skin twice daily B. Allow her to wear only 100% cotton clothing C. Keep her nails trimmed short D. Bathe her daily with bath oil

C. Keep her nails trimmed short

The nurse observes a newly-employed unlicensed assistive personnel (UAP) taking an elderly client's blood pressure. The nurse says "You need to start over. The blood pressure reading you obtained was falsely high for this client." What is the most likely explanation for the erroneous reading? A. The UAP was standing above the sphygmomanometer while taking the reading B. The sphygmomanometer was two feet away from the UAP during the procedure C. The size of the cuff used was too small for this adult client's arm D. The client's arm was elevated above the level of the heart

C. The size of the cuff used was too small for this adult client's arm

A nurse caring for a client who is postoperative following a parathyroidectomy to treat hyperparathyroidism. Which of the following laboratory values should the nurse expect to decrease as a therapeutic effect of the procedure?

Calcium -Rationale: parathyroid hormone regulates calcium, phosphorous, magnesium balance in the blood and bone

Based on the interpretation of this strip, what action should be implemented next?

Call a code.

The practical nurse (PN) is assigned a client diagnosed with a hemothorax who had a chest tube inserted 36 hours ago; upon entering the room, the PN observes the client resting comfortably in the semi-Fowler position; respirations appear even and unlabored; the water in the water-seal chamber is bubbling; and there is serous drainage noted in the collection chamber. What is the best initial action for the PN to take?

Change the client to a high-Fowler position. The client should be placed in the high-Fowler position to facilitate the draining of the fluid from the hemothorax. Clients with pneumothorax may be placed in a semi-Fowler position.

A 57-year-old male client is scheduled to have a stress-thallium test the following morning and is NPO after midnight. At 0130, he is agitated because he cannot eat and is demanding food. Which response is best for the nurse to provide to this client? A) I'm sorry sir, you have a prescription for nothing by mouth from midnight tonight. B) I will let you have one cracker, but that is all you can have for the rest of tonight. C) What did the healthcare provider tell you about the test you are having tomorrow? D) The test you are having tomorrow requires that you have nothing by mouth tonight.

D) The test you are having tomorrow requires that you have nothing by mouth tonight. (D) is the most therapeutic statement because the nurse is responding to the client's question. (A) is not an explanation and the nurse should teach the client why eating is prohibited after midnight, rather than enforcing this requirement without an explanation for it. (B) may result in an inaccurate test result, or may cause the test to be cancelled, which could also delay diagnosis and treatment. (C) defers the responsibility for answering the client's question to the healthcare provider, when the nurse could address the situation through client teaching.

The nurse is planning to initiate a socialization group for older residents of a long-term facility. Which information is most useful to the nurse when planning activities for the group? A) The length of time each group member has resided at the nursing home. B) A brief description of each resident's family life. C) The age of each group member. D) The usual activity patterns of each member of the group.

D) The usual activity patterns of each member of the group. An older person's level of activity (D) is a determining factor in adjustment to aging as described by the Activity Theory of Aging. All information described in the options might be useful to the nurse, but the most useful information initially would be an assessment of each individual's adjustment to the aging process.

The nurse determines that the patient understood medication instructions about the use of a spacer device when taking inhaled medications after hearing the patient state which of the following as the primary benefit? A. "Now I will not need to breathe in as deeply when taking the inhaler medications." B. "This device will make it so much easier and faster to take my inhaled medications." C. "I will pay less for medication because it will last longer." D. "More of the medication will get down into my lungs to help my breathing."

D. "More of the medication will get down into my lungs to help my breathing." A spacer assists more medication to reach the lungs, with less being deposited in the mouth and the back of the throat.

Assessment of the fetal heart rate is an important finding when caring for a laboring client. Deceleration in fetal heart rate that are of most concern occur at what time during the contraction cycle? A. Before a contraction B. During a contraction C, Between contractions D. After a contraction

D. After a contraction

A terminally ill client on a palliative care unit has an advanced directive stipulating comfort measure only. The client has not taken oral fluids in the last 36 hours and is not receiving intravenous fluids. The clients blood pressure is 64/38 and urinary output is 50 ml for the last 12 hours. What is the priority nursing intervention? A. Assess for the presence of dependent edema B. Prepare to initiate intravenous fluids C. Gently massage the client's bladder D. Determine the client's level of discomfort

D. Determine the client's level of discomfort

A 77-year-old female client is admitted to the hospital with confusion and anorexia of several days' duration. She has symptoms of nausea and vomiting and is currently complaining of a headache. The client's pulse rate is 43 beats/min. The nurse is most concerned about the client's history related to which medication? A. Warfarin (Coumadin) B. Ibuprofen (Motrin) C. Nitroglycerin (Nitrostat) D. Digoxin (Lanoxin)

D. Digoxin Rationale: Older persons are particularly susceptible to the buildup of cardiac glycosides, such as digoxin or digitoxin (medications derived from digitalis), to a toxic level in their systems. Toxicity can cause anorexia, nausea, vomiting, diarrhea, headache, and fatigue. Options A, B, and C are unlikely to result in the symptoms described.

The nurse-manager of a pediatric units needs to assign a room for a 6-month-old diagnosed with respiratory syncytial virus (RSV). Which room assignment is the best for the nurse-manager to make? A. Private room furtherest form the nurses station B. Double room with a 6-month-old on droplet precautions C. Private room with negative air pressure D. Double room with a 4-month-old who has RSV

D. Double room with a 4-month-old who has RSV

The nurse should instruct the parents of an 11-year-old with Type 1 diabetes mellitus to carefully watch their child for symptoms of diabetic ketoacidosis at risk for becoming ketoacidotic? A. While adjusting the amount of the insulin dosage B. When changing to a new brand of insulin C. After skipping two or more meals consecutively D. During the course of an acute illness

D. During the course of an acute illness

If a patient with arthritis develops iron-deficiency anemia, a nurse should ask about the patient's use of A. alcoholic beverages. B. stool softeners and laxatives. C. caffeinated foods and beverages. D. NSAIDs.

D. NSAIDs. NSAIDs decrease the level of vitamin C, which aids in the absorption of iron. These drugs also compete with folate and vitamin K and may cause gastritis. Excessive alcoholic beverage consumption can cause stomach irritation; alcohol would not be directly related to iron-deficiency anemia unless bleeding ulcers or gastritis were to occur. NSAID consumption, not stool softeners and laxative use, would be suspect for iron-deficiency anemia. Caffeinated foods and beverages can cause gastric irritation and discomfort but are not associated with iron-deficiency anemia.

12. After admitting a patient to the medical unit with a diagnosis of pneumonia, the nurse will verify that which of the following physician orders have been completed before administering a dose of cefotetan (Cefotan) to the patient? A. Serum laboratory studies ordered for AM B. Pulmonary function evaluation C. Orthostatic blood pressures D. Sputum culture and sensitivity

D. Sputum culture and sensitivityThe nurse should ensure that the sputum for culture and sensitivity was sent to the laboratory before administering the cefotetan. It is important that the organisms are correctly identified (by the culture) before their numbers are affected by the antibiotic; the test will also determine whether the proper antibiotic has been ordered (sensitivity testing). Although antibiotic administration should not be unduly delayed while waiting for the patient to expectorate sputum, all of the other options will not be affected by the administration of antibiotics.

25. The nurse identifies the nursing diagnosis of activity intolerance for a patient with asthma. The nurse assesses for which of the following etiologic factor for this nursing diagnosis in patients with asthma? A. Anxiety and restlessness B. Effects of medications C. Fear of suffocation D. Work of breathing

D. Work of breathingWhen the patient does not have sufficient gas exchange to engage in activity, the etiologic factor is often the work of breathing. When patients with asthma do not have effective respirations, they use all available energy to breathe and have little left over for purposeful activity.

A client with cirrhosis develops increasing pedal edema and ascites. Which dietary modification is most important for the nurse to teach this client? A. Avoid high-carbohydrate foods. B.Decrease intake of fat-soluble vitamins. C.Decrease caloric intake. D. Restrict salt and fluid intake.

D.Restrict salt and fluid intake. Rationale: Salt and fluid restrictions are the first dietary modifications for a client who is retaining fluid as manifested by edema and ascites. Options A, B, and C will not affect fluid retention.

A nurse is reinforcing pre-operative teaching for a client who is scheduled for surgery and isto take hydroxyzine preoperatively. Which of the following effects of the medication should the nurse include in the teaching? (Select all that apply.) 1) Decreasing anxiety 2) Controlling emesis 3) Relaxing skeletal muscles 4) Preventing surgical site infections 5) Reducing the amount of narcotics needed for pain relief

Decreasing anxiety Controlling emesis Reducing the amount of narcotics needed for pain relief

When assessing a client with a serum potassium level of 2.5 mEq/L which intervention is most important for the nurse to implement?

Determine apical pulse rate and rhythm

A nurse is planning to change an abdominal dressing for a client who has an incision with a drain. Which of the following actions should the nurse plan to take? 1) Remove the entire dressing at once. 2) Loosen the dressing by pulling the tape away from the wound. 3) Don clean gloves to remove the dressing. 4) Open sterile supplies before removing the dressing.

Don clean gloves to remove the dressing

The nurse is teaching a client who was recently diagnosed with gout how to manage the disease. Which information should the nurse include in this client's teaching plan?

Fluid intake should increase to at least 3 liters per day

A client diagnosed with prostate cancer is prescribed radioactive seed implantation (brachytherapy). What is the most important nursing action for the practical nurse (PN) to do?

Follow radiation exposure precautions. Clients being treated for prostate cancer with brachytherapy (radioactive seeds implant) should be placed on radiation exposure precautions. The PN needs to follow the institution's protocols put in place regarding the amount of time and distance needed to prevent excessive exposure that would pose a hazard to others.

A nurse is reinforcing teaching with the family of a client who has primary dementia. Which of the following manifestations of dementia should the nurse include in the teaching? 1) Temporary, reversible loss of brain function 2) Forgetfulness gradually progressing to disorientation 3) Sleeping more during the day than nighttime 4) Hyper vigilant behaviors

Forgetfulness gradually progressing to disorientation

A client diagnosed with diabetes mellitus complains of vomiting and feeling confused to the practical nurse. Which of the following symptoms are possible signs of diabetic ketoacidosis (DKA)? (Select all that apply.)

Fruity breath odor Rapid, weak pulse Extreme thirst Urinary frequency Diabetic ketoacidosis is caused by a profound deficiency of insulin. Some common characteristics include a sweet, fruity breath odor; a rapid weak pulse; extreme thirst; urinary frequency; and sunken-appearing eyeballs.

The nurse is evaluating teaching about drug therapy to treat gout. Which statement by the client demonstrates an understanding of the use of allopurinol to treat gout?

I need to take this drug every day to keep from having any flare-ups.

An adult male client is admitted for Pneumocystis jiroveci pneumonia secondary to AIDS. While hospitalized, the client receives IV pentamidine therapy. In preparing this client for discharge, which important aspect regarding this medication therapy should the nurse explain?

IV pentamidine may offer protection to other AIDS related conditions, such as Kaposi's sarcoma

A client whose history includes IV drug abuse is admitted to the intensive care unit with Kaposi's sarcoma associated with AIDS. Which intervention is most important for the nurse to include in the client's plan of care?

Identify signs of opportunistic infections

While completing a health assessment for a client with migraine headaches, the nurse assesses bilateral weakness in the client's hand grips. The client reports joint pain and trouble twisting a doorknob due to the weakness. Which action should the nurse take in response to these findings?

Implement fall precautions to reduce the client's risk for injury

A male client with AIDS and Pneumocystis jirovenci pneumonia has a CD4 T cell count of 200 cells/microliter. The client asks the nurse why he keeps getting these massive infections. Which pathophysiologic mechanism should the nurse describe in response to this client's question?

Inadequate numbers of T lymphocytes are available to initiate cellular immunity and macrophages.

the nurse is providing wound care to a client with a stage 3 pressure ulcer that has a large amount of eschar. The wound care prescription states: clean the wound and then apply collagenase. Collagenase is a debriding agent. The prescription does not specify a cleansing method. Which technique should the nurse use to clean the pressure ulcer?

Irrigate the wound with sterile NS

A nurse is assisting with caring for a client who has a new concussion following a motor-vehicle crash. The nurse should monitor the client for which of the following manifestations of increased intracranial pressure? 1) Polyuria 2) Battle's sign 3) Nuchal rigidity 4) Lethargy

Lethargy

A client experiences an ABO incompatibility reaction after multiple blood transfusions. Which finding should the nurse report immediately to the health care provider?

Lower back pain and hypotension

The practical nurse receives shift report on their assigned clients. Based on the change of shift report which situation has the highest priority?

No output in a hemovac from the abdominal incision of a client who is post-op day 1. The PN should first evaluate the client who has no hemovac output from the abdominal surgical site to determine if the hemovac needs to be compressed, drainage tube kinked, or if the drain is displaced from the wound.

When assessing a client with degenerative joint disease, the nurse observes Heberden's nodes, large prominences on the clients fingers that are reddened. The client reports that the nodes are painful. Which action should the nurse take?

Notify the healthcare provider of the finding immediately

A client with alcohol-related liver disease is admitted to the unit. Which prescription should the nurse call the health care provider about for reverification for this client? A. Vitamin K1 (AquaMEPHYTON), 5 mg IM daily B. High-calorie, low-sodium diet C. Fluid restriction to 1500 mL/day D. Pentobarbital (Nembutal sodium) at bedtime for rest

Rationale: D Sedatives such as Nembutal (D) are contraindicated for clients with liver damage and can have dangerous consequences. (A) is often prescribed because the normal clotting mechanism is damaged. (B) is needed to help restore energy to the debilitated client. Sodium is often restricted because of edema. Fluids are restricted (C) to decrease ascites, which often accompanies cirrhosis, particularly in the later stages of the disease.

A nurse is assisting with planning care for a client who is recovering from a left-hemispheric stroke. Which of the following interventions should the nurse include in the plan? 1) Control impulsive behavior. 2) Compensate for left visual field deficits. 3) Re-establish communication. 4) Improve left-side motor function.

Re-establish communication

A nurse is contributing to the plan of care for a client who has a terminal illness. Which of the following interventions should the nurse identify as the priority? 1) Promote the client's expression of feelings about loss of self-care ability. 2) Encourage the client to recall positive life events. 3) Schedule pain medication on a routine basis. 4) Suggest ways the client can continue interacting with social contacts

Schedule pain medication on a routine basis

An 82-year-old female client with Type 2 diabetes and degenerative arthritis complains to the nurse that she has a hard time cutting her toenails. Which should the nurse recommend?

Seek routine nail care with a podiatrist.

The healthcare provider prescribes a normal saline bolus for a frail elderly client who is dehydrated. The client's vital signs are BP 90/60 mmHg, heart rate 122, and respiratory rate 20. When preparing to establish IV access using a peripheral over the needle IV catheter, which intervention is most importan for the nurse to implement?

Select a suitable catheter gauge for the vein

.A nurse is assisting in the care of a client who is receiving a transfusion of packed red blood cells. The client develops itching and hives. Which of the following actions should the nurse take first? 1) Obtain vital signs. 2) Stop the transfusion. 3) Notify the registered nurse. 4) Administer diphenhydramine.

Stop the transfusion

A 55 year old client with osteoarthritis asks which form of exercise would be most beneficial. What is the best response by the nurse?

Swimming is an excellent exercise for you.

A nurse is reinforcing teaching with a client who is newly diagnosed with myasthenia gravis and is to start taking neostigmine. Which of the following instructions should the nurse include in the teaching? 1) Take the medication 45 minutes before eating. 2) Expect diaphoresis as a side effect of the neostigmine. 3) If a medication dose is missed, wait until the next scheduled dose to take the medication. 4) Treat nasal rhinitis with an over-the-counter antihistamine.

Take the medication 45 minutes before eating

A nurse in a provider's office is reinforcing teaching with a client who has anemia and has been taking ferrous gluconate for several weeks. Which of the following instructions should the nurse include? 1) Take this medication between meals. 2) Limit intake of Vitamin C while taking this medication. 3) Take this medication with milk. 4) Limit intake of whole grains while taking this medication.

Take this medication between meals

After initializing a steroid nebulizer treatment for an asthmatic client in respiratory distress, which intervention is most important for the nurse implement?

Teach proper use of a rescue inhaler

A client diagnosed with ovarian cancer has just been informed that their cancer has metastasized to their liver. The client is quietly crying in their room. Which statement is appropriate for the practical nurse to say?

Tell me about what you are feeling right now. The most appropriate response and therapeutic action is for the nurse to be an active listener and to encourage the client to explore their feelings.

A nurse is caring for a client who has had a cerebrovascular accident. Which of the following findings indicates that homonymous hemianopsia is present?

The client has to turn her head to see the entire visual field

A client diagnosed with lymphoma is receiving chemotherapy. The client's hemoglobin is currently 6 g/dL. The practical nurse (PN) assigns an unlicensed assistive personnel (UAP) to provide personal hygiene for this client. What instruction should the PN provide to the UAP?

The client will be weak and unsteady and tire easily. A hemoglobin of 6 g/dL indicates anemia (normal for a female is 12 to 16 g/dL, for a male is 14 to 18 g/dL), which is a common adverse effect of chemotherapy. The UAP should be given instructions about how this will cause weakness and unsteadiness in the client and they will tire easily.

A nurse is reviewing data for a client who has a head injury. Which of the following findings should indicate to the nurse that the client might have diabetes insipidus? 1) Serum sodium 145 mEq/L 2) Urine specific gravity 1.028 3) Urine output 650 mL/hr 4) Blood glucose 198 mg/dL

Urine output 650 mL/hr

A nurse is preparing a 24-hour urine specimen for a client who is suspected to have pheochromocytoma. Which of the following laboratory tests from the 24-hour urine specimen should the nurse use to determine the client's condition?

Vanillylmandelic acid (VMA)

On admission to the ICU for sepsis the client's temperature is 104 and blood pressure is 68/42. Other hemodynamic findings are cardiac output of 10.7L/min, systemic vascular resistance of 480 dynes/sec/cm, and white blood cell count 28,000. Which classification of prescribed medication should the nurse evaluate for client stabilization?

Vasoconstrictor

A nurse is caring for a client who is being evaluated for endometrial cancer. Which of the following findings should the nurse expect the client to report? 1) Hot flashes 2) Recurrent urinary tract infections 3) Blood in the stool 4) Abnormal vaginal bleeding

abnormal vaginal bleeding

Two days following abdominal surgery a client begins to report cramping abdominal pain, and the nurse's inspection of the abdomen indicates slight distention. Which action should the nurse implement first?

auscultate the client's abdomen

A client with pheochromocytoma reports the onset of a severe headache. The nurse observes that the client is very diaphoretic. Which assessment data should the nurse obtain next?

blood pressure

A nurse is assisting in the plan of care for a client who had a removal of the pituitary gland. Which of the following actions should the nurse include in the plan? 1) Position the client supine while in bed. 2) Change the nasal drip pad as needed. 3) Encourage frequent brushing of teeth. 4) Encourage the client to cough every 2 hr following surgery.

change the nasal drip pad as needed

A nurse is reinforcing discharge teaching with a client about how to care for a newly created ileal conduit. Which of the following instructions should the nurse include in the teaching? 1) Change the ostomy pouch daily. 2) Empty the ostomy pouch when it is 2/3 full. 3) Trim the opening of the ostomy seal to be 1/2 in. wider than the stoma. 4) Apply lotion to the peristomal skin when changing the ostomy pouch.

change the ostomy pouch daily

An older male client with long-standing lung disease is admitted to the medical unit for treatment of pulmonary infection. In assessing for signs of increasing hypoxia, which actions should the nurse include? Select all that apply.

check for changes in mentation observe color of skin and mucous assess breathing patterns

A nurse is caring for a client who has heart failure and has been taking digoxin 0.25 mg daily. The client refuses breakfast and reports nausea. Which of the following actions should the nurse take first? 1) Suggest that the client rests before eating the meal. 2) Request a dietary consult. 3) Check the client's vital signs. 4) Request an order for an antiemetic.

check the client's vital signs

A 70-year-old male client with type 2 diabetes mellitus (DM) is hospitalized with an infected ulcer on his great right toe. Which instruction should the nurse emphasize during discharge teaching?

check the insides and linings of all enclosed shoes before putting the shoes on

After a transurethral resection of the prostate (TURP), a client has bloody urine output with large clots. The nurse implements the postoperative prescription to irrigate the indwelling catheter PRN to maintain the catheter's patency. Which action should the nurse implement?

clamp the catheter for 30 minutes prior to irrigating with saline

The nurse assists a male client with Parkinson's disease (PD) to ambulate in the hallway. The client appears to "freeze" and then carefully lifts one leg and steps forward. He tells the nurse that he is pretending to step over a crack on the floor. How should the nurse respond?

confirm that this is an effective technique to help with ambulation

A male client with a history of asthma reports having episodes of bronchoconstriction and increased mucous production while exercising. Which action should the nurse implement?

determine if the client is using an inhaler before exercising

A client with type 2 diabetes mellitus (DM) is admitted to the hospital for uncontrolled DM. Insulin therapy is initiated with an initial dose of Humulin N insulin at 0800. At 1600, the client complains of diaphoresis, rapid heartbeat, and feeling shaky. What should the nurse do first?

determine the clients current glucose level

A nurse is collecting data from a client who has emphysema. Which of the following findings should the nurse expect? (Select all that apply.) 1) Dyspnea 2) Barrel chest 3) Clubbing of the fingers 4) Shallow respirations 5) Bradycardia

dyspnea barrel chest clubbing of the fingers shallow respirations

A client who is experiencing respiratory distress is admitted with respiratory acidosis. Which pathophysiological process supports the client's respiratory acidosis/

electrolytes

A client with unstable asthma had an emergent cardiac catheterization. Which complication should the nurse monitor for in the initial 24 hours after the procedure?

thrombus formation

A nurse is collecting data from a client who has alcohol use disorder and is experiencing metabolic acidosis. Which of the following manifestations should the nurse expect? 1) Cool, clammy skin 2) Hyperventilation 3) Increased blood pressure 4) Bradycardia

hyperventilation

What information should the nurse include in the teaching plan of a client diagnosed with gastroesophageal reflux disease (GERD)?

minimize symptoms by wearing loose, comfortable clothing

A client with an acute exacerbation of rheumatoid arthritis (RA) has localized pain and inflammation of the fingers and feet; swelling, redness, and restricted joint motion; and reports feeling fatigued. Which nursing diagnosis has the highest priority for this client?

pain related to joint inflammation

A client with chronic kidney disease (CDK) arrives at the clinic reporting shortness of breath on exertion and extreme weakness. Vital signs are temperature 100.4 F (38 C), heart rate 110 beats/minute, respirations 28 breaths/minute, and blood pressure 175/98 mmHg. The client usually receives dialysis three times a week but missed the last treatment. STAT blood specimens are sent to the laboratory for analysis. Which laboratory results should the nurse report to the healthcare provider immediately?

potassium of 6.5

A nurse is caring for a client who has a spinal cord injury at T-4. The nurse should recognize that the client is at risk for autonomic dysreflexia. Which of the following interventions should the nurse take to prevent autonomic dysreflexia? 1) Monitor for elevated blood pressure. 2) Provide analgesia for headaches. 3) Prevent bladder distention. 4) Elevate the client's head.

prevent bladder distension

An older adult with heart failure is hospitalized during an acute exacerbation. To reduce cardiac workload, which intervention should the nurse include in the client's plan of care?

provide a bedside commode for toileting

A nurse is caring for a client who is postoperative following a tracheostomy, and has copious and tenacious secretions. Which of the following is an acceptable method for the nurse to use to thin this client's secretions? 1) Provide humidified oxygen 2) Perform chest physiotherapy prior to suctioning 3) Prelubricate the suction catheter tip with sterile saline when suctioning the airway 4) Hyperventilate the client with 100% oxygen before suctioning the airway..

provide humidified oxygen

A nurse is caring for a client who is 3 days postoperative following a cholecystectomy. The nurse suspects the client's wound is infected because the drainage from the dressing is yellow and thick. Which of the following findings should the nurse report as the type of drainage found? 1) Sanguineous 2) Serous 3) Serosanguineous 4) Purulent

purulent

The nurse determines that a client who arrives in the preoperative holding area before surgery is allergic to bananas. Which action should the nurse implement prior to taking the client into the operative area?

replace latex containing devices in the OR with alternate synthetic materials

A nurse is reinforcing teaching with a client who has HIV and is being discharged to home. Which of the following instructions should the nurse include in the teaching? 1) Take temperature once a day 2) Wash the armpits and genitals with a gentle cleanser daily 3) Change the litter boxes while wearing gloves 4) Wash dishes in warm water.

take temperature once a day

A nurse is caring for a client who asks why she is being prescribed aspirin 325 mg daily following a myocardial infarction. The nurse should instruct the client that aspirin is prescribed for clients who have coronary artery disease for which of the following effects? 1) To provide analgesia 2) To reduce inflammation 3) To prevent blood clotting 4) To prevent fever

to prevent blood clotting

A woman who works as a data entry clerk is concerned as to how recent diagnosis of Raynaud's syndrome is going to affect her job performance. Which instruction should the nurse provide this client?

use a space heater to keep the workspace warm

A nurse is caring for a client scheduled for coronary artery bypass grafting who reports he is no longer certain he wants to have the procedure. Which of the following responses should the nurse make? 1) "Why have you changed your mind about the surgery?" 2) "Bypass surgery must be very frightening for you." 3) "Your provider would not have scheduled the surgery unless you needed it." 4) "I will call your doctor and have him discuss your surgery with you.

"Bypass surgery must be very frightening for you"

An elderly client diagnosed with dementia was admitted from a long-term facility to the hospital two days ago. The client's children express concern that their parent's confusion has gotten worse since being admitted. How should the practical nurse (PN) respond?

"Confusion in an older person is expected with a relocation to new surroundings." Relocation often results in confusion among elderly clients and is stressful for clients of all ages.

Based on a client's recent history, a nurse suspects that a client is beginning menopause. Which of the following questions should the nurse ask the client to help confirm the client is experiencing manifestations of menopause? 1) "Do you sleep well at night?" 2) "Have you been experiencing chills?" 3) "Have you experienced increased hair growth?" 4) "When did you begin your menses?"

"Do you sleep well at night?"

A nurse is reinforcing teaching with a client prior to a cystoscopy. Which of the following statements should the nurse make?

"Expect to have pink-tinged urine after this procedure"

.A nurse is caring for a client who is scheduled for surgical repair of a femur fracture and has a prescription for lorazepam preoperatively. Which of the following statements by the client should indicate to the nurse that the medication has been effective? 1) "My mouth is very dry." 2) "I feel very sleepy." 3) "I am not hungry any longer." 4) "My leg feels numb."

"I feel very sleepy"

A practical nurse (PN) reinforced client teaching regarding the transmission of the HIV virus. Which statement by the client demonstrates an understanding of the reinforced teaching?

"I may still contract HIV even though I am 62 years old." More than 10% of all AIDS cases in the United States are among those older than 50 years of age.

A nurse is caring for a client who just had cataract surgery. Which of the following comments from the client should the nurse report to the provider? 1) "The bright light in this room is really bothering me." 2) "My eye really itches, but I'm trying not to rub it." 3) "It's really hard to see with a patch on one eye." 4) "I need something for the horrible pain in my eye."

"I need something for the horrible pain in my eye"

A nurse is reinforcing teaching with a client who has been newly diagnosed with chronic open angle glaucoma. Which of the following statements by the client indicated an understanding of the teaching? 1) "When my vision improves, I will be able to stop taking the eye drops" 2) "If I forget to take my eye drops, I should wait until the next time they are due" 3) "I should call the clinic before taking any OTC medications" 4) "Every two years I will need to have my vision checked by an eye doctor"

"I should call the clinic before taking any OTC medications"

A nurse is caring for a client who has recurrent kidney stones and a history of diabetes mellitus. The client is scheduled for an intravenous pyelogram (IVP). The nurse should collect additional data about which of the following statements made by the client? 1) "I took a laxative yesterday." 2) "I took my metformin before breakfast." 3) "I haven't had anything to eat or drink since last night." 4) "The last time I voided it was painful."

"I took my metformin before breakfast"

A nurse is reinforcing discharge instructions with a client who has hepatitis A. Which of the following statements by the client indicates an understanding of the teaching? 1) "I will not eat fried foods." 2) "I will abstain from sexualintercourse." 3) "I will refrain from international travel." 4) "I will not order a salad in a restaurant."

"I will abstain from sexual intercourse"

A nurse is reinforcing teaching about cyclosporine for a client who is postoperative following a renal transplant. Which of the following statements by the client indicates an understanding of the teaching? 1) "I will take this medication until my BUN returns to normal." 2) "This medication will help my new kidney make adequate urine." 3) "I will need to take this medication for the rest of my life." 4) "This medication will boost my immune system."

"I will need to take this medication for the rest of my life"

A nurse is evaluating discharge instructions for a client following a right cataract extraction. Which of the following client statements indicates the teaching is effective? 1) "I will take a stool softener until my eye is healed." 2) "I will expect to have moderately severe pain for 1-2 days." 3) "I will refrain from cooking for 1 week." 4) "I will bend at the waist to tie my shoes."

"I will take a stool softener until my eye is healed"

A nurse is reinforcing discharge teaching with a client who had a total abdominal hysterectomy and a vaginalrepair. Which of the following statements by the client indicates a need for further teaching? 1) "I should increase my intake of protein and vitamin C." 2) "I will no longer have menstrual periods." 3) "Once I am able to resume sexual activity, I can use a water-based lubricant ifI experience discomfort." 4) "I will take a tub bath instead of a shower."

"I will take a tube bath instead of a shower"

A nurse is reinforcing teaching with a client who has iron deficiency anemia and is to start taking ferrous sulfate twice a day. Which of the following statements by the client indicate an understanding of the teaching? 1) "I will take the medication with orange juice." 2) "I should expect to have loose stools while taking this medication." 3) "I will have clay colored stools while taking this medication." 4) "I should take the medication with milk."

"I will take the medication with orange juice"

A nurse is reinforcing teaching with a client who has a history of urinary tract infections (UTIs). Which of the following statements should indicate to the nurse the need for additional instructions?

"I will vaginal douche daily"

The practical nurse (PN) is assigned a client with a medical history of diabetes and gangrene who had an (R) below the knee amputation. At the time of rewrapping and inspecting the stump, the client refuses to look at their stump. The practical nurse (PN) tells the client that the incision is healing well, but the client refuses to talk about it. What is the best response to this client's silence?

"It is okay if you don't want to talk about your surgery. I will be available when you are ready." Informing the client that it is okay they don't want to talk about their surgery and stating that the PN is available when they need them, displays sensitivity and understanding without judging the client.

A nurse is caring for a client who has hemiplegia following a stroke. The client's adult son is distressed over his mother's crying and condition. Which of the following responses should the nurse make? 1) "If you just sit quietly with your mother, I'm sure she will calm down." 2) "I'll talk with your mother and see if I can comfort her." 3) "It must be hard to see your mother so ill and upset." 4) "Your mother's crying seems to bother you more than it does her."

"It must be hard to see your mother so ill and upset"

A nurse is caring for a client with severe burns to both lower extremities. The client is scheduled for an escharotomy and wants to know what the procedure involves. Which of the following statements is appropriate for the nurse to make? 1) "An escharotomy surgically removes dead tissue." 2) "A cannula will be inserted into the bone to infuse fluids and antibiotics." 3) "A piece of skin will be removed and grafted over the burned area." 4) "Large incisions will be made in the burned tissue to improve circulation."

"Large incisions will be made in the burned tissue to improve circulation"

A nurse is caring for a client scheduled for a bone marrow biopsy. The client expresses fear about the procedure and asks the nurse if the biopsy will hurt. Which of the following responses should the nurse make? 1) "You must be very worried about what the biopsy will show." 2) "You'll be asleep for the whole biopsy procedure and won't be aware of what's happening." 3) "Your provider scheduled this, so she will want to know you still have questions about the procedure." 4) "The biopsy can be uncomfortable, but we will try to keep you as comfortable as possible."

"The biopsy can be uncomfortable, but we will try to keep you as comfortable as possible"

A nurse is reinforcing teaching with a client who reports right shoulder pain following a laparoscopic cholecystectomy. Which of the following statements should the nurse make? 1) "The pain results from lying in one position too long during surgery." 2) "The pain occurs as a residual pain from cholecystitis." 3) "The pain will dissipate if you ambulate frequently." 4) "The pain is caused from the nitrous dioxide injected into the abdomen.

"The pain will dissipate if you ambulate frequently"

A nurse is reinforcing teaching about a tonometry examination with a client who has manifestations of glaucoma. Which of the following statements should the nurse include in the teaching? 1) "Tonometry is performed to evaluate peripheral vision." 2) "This test will diagnose the type of your glaucoma." 3) "Tonometry will allow inspection of the optic disc for signs of degeneration." 4) "This test will measure the intraocular pressure of the eye."

"This test will measure the intraocular pressure of the eye"

A nurse is providing teaching to a client who has type 1 diabetes mellitus about exercise. Which of the following statements should the nurse include in the teaching?

"Wear a medical alert identification tag when you exercise."

A nurse is reinforcing teaching with a client who has acute pyelonephritis. Which of the following instructions should the nurse include in the teaching?

"You should complete the entire cycle of antibiotic therapy"

A nurse is reinforcing teaching to a client prior to a renal biopsy. Which of the following statements should the nurse make?

"You will need to be on bedrest following this procedure"

After reviewing the admission assessment of a client with chronic pain, which interventions should the nurse include in this client's plan of care. SATA

- Determine client's subjective measure of pain using a numerical pain scale - Assist the client to ambulate as much as possible during waking hours - Implement a 24 hours schedule of routine administration of prescribed analgesic - Provide comfort measures such as topical warm application and tactile massage

After administering an antihypertensive medication to an older client, which actions should the nurse implement? SATA

-Instruct the client to call the nurse before getting out of bed -Assess the client's blood pressure before getting out of bed -Verify that the client understands hot to use the call button

An older adult with chronic obstructive pulmonary disease was recently admitted to the hospital with heart failure. Which actions should the nurse take in providing care? SATA

-Monitor electrolytes values -Maintain pulse oximeter -Encourage increased fluid intake

An older client who is agitated, dyspneic, orthopneic, and using accessory muscles to breath is admitted for further treatment. Initial assessment includes a heart rate of 128 and irregular respirations 38 per minute, blood pressure 168/100 mmHg, wheezes and crackles in all lung fields. An hour after the administration of furosemide 60 mg IV, which assessment should the nurse obtain to determine the client's response to treatment? SATA

-Urinary output -Lung sounds -Oxygen saturation

A school-aged child who weighs 42 pounds receives a post-tonsillectomy prescription for promethazine (Phenergan) 0.5 mg/kg IM to prevent postoperative nausea. The medication is available in 25 mg/ml ampules. How many ml should the nurse administer?

0.4 ml

The healthcare provider prescribes epoetin alfa (Procrit) 8,200 units subcutaneously for a client with chronic kidney disease (CKD). The 2 ml multidose vial is labeled, "Each 1 ml of solution contains 10,000 units of epoetin alfa." How many ml should the nurse administer?

0.8

A client receives a prescription for vitamin K 2 mg daily. The pharmacy delivers a vial labeled vitamin K 5 mg/single use vial. The instructions read: reconstitute with 2.7 mL of sterile water to yield 2 mg/mL. How many mL should the nurse administer? ( Enter the numerical value only

1

Following a disaster, which person should the nurse triage with a red tag, indicating that immediate attention is required?

A young adult with a history of asthma who is speaking in short sentences

173.) A nurse reviews the medication history of a client admitted to the hospital and notes that the client is taking leflunomide (Arava). During data collection, the nurse asks which question to determine medication effectiveness? 1. "Do you have any joint pain?" 2. "Are you having any diarrhea?" 3. "Do you have frequent headaches?" 4. "Are you experiencing heartburn?"

1. "Do you have any joint pain?" Rationale: Leflunomide is an immunosuppressive agent and has an anti-inflammatory action. The medication provides symptomatic relief of rheumatoid arthritis. Diarrhea can occur as a side effect of the medication. The other options are unrelated to medication effectiveness.

127.) The nurse provides medication instructions to an older hypertensive client who is taking 20 mg of lisinopril (Prinivil, Zestril) orally daily. The nurse evaluates the need for further teaching when the client states which of the following? 1. "I can skip a dose once a week." 2. "I need to change my position slowly." 3. "I take the pill after breakfast each day." 4. "If I get a bad headache, I should call my doctor immediately."

1. "I can skip a dose once a week." Rationale: Lisinopril is an antihypertensive angiotensin-converting enzyme (ACE) inhibitor. The usual dosage range is 20 to 40 mg per day. Adverse effects include headache, dizziness, fatigue, orthostatic hypotension, tachycardia, and angioedema. Specific client teaching points include taking one pill a day, not stopping the medication without consulting the health care provider (HCP), and monitoring for side effects and adverse reactions. The client should notify the HCP if side effects occur.

10.) The clinic nurse is performing an admission assessment on a client. The nurse notes that the client is taking azelaic acid (Azelex). Because of the medication prescription, the nurse would suspect that the client is being treated for: 1. Acne 2. Eczema 3. Hair loss 4. Herpes simplex

1. Acne Rationale: Azelaic acid is a topical medication used to treat mild to moderate acne. The acid appears to work by suppressing the growth of Propionibacterium acnes and decreasing the proliferation of keratinocytes. Options 2, 3, and 4 are incorrect.

145.) A nurse has a prescription to give a client albuterol (Proventil HFA) (two puffs) and beclomethasone dipropionate (Qvar) (nasal inhalation, two puffs), by metered-dose inhaler. The nurse administers the medication by giving the: 1. Albuterol first and then the beclomethasone dipropionate 2. Beclomethasone dipropionate first and then the albuterol 3. Alternating a single puff of each, beginning with the albuterol 4. Alternating a single puff of each, beginning with the beclomethasone dipropionate

1. Albuterol first and then the beclomethasone dipropionate Rationale: Albuterol is a bronchodilator. Beclomethasone dipropionate is a glucocorticoid. Bronchodilators are always administered before glucocorticoids when both are to be given on the same time schedule. This allows for widening of the air passages by the bronchodilator, which then makes the glucocorticoid more effective.

193.) Sodium hypochlorite (Dakin's solution) is prescribed for a client with a leg wound containing purulent drainage. The nurse is assisting in developing a plan of care for the client and includes which of the following in the plan? 1. Ensure that the solution is freshly prepared before use. 2. Soak a sterile dressing with solution and pack into the wound. 3. Allow the solution to remain in the wound following irrigation. 4. Apply the solution to the wound and on normal skin tissue surrounding the wound.

1. Ensure that the solution is freshly prepared before use. Rationale: Dakin solution is a chloride solution that is used for irrigating and cleaning necrotic or purulent wounds. It can be used for packing necrotic wounds. It cannot be used to pack purulent wounds because the solution is inactivated by copious pus. It should not come into contact with healing or normal tissue, and it should be rinsed off immediately if used for irrigation. Solutions are unstable and the nurse must ensure that the solution has been prepared fresh before use. **Eliminate options 2 and 3 first because they are comparable or alike. It makes sense to ensure that the solution is freshly prepared; therefore, select option 1**

109.) A client taking buspirone (BuSpar) for 1 month returns to the clinic for a follow-up visit. Which of the following would indicate medication effectiveness? 1. No rapid heartbeats or anxiety 2. No paranoid thought processes 3. No thought broadcasting or delusions 4. No reports of alcohol withdrawal symptoms

1. No rapid heartbeats or anxiety Rationale: Buspirone hydrochloride is not recommended for the treatment of drug or alcohol withdrawal, paranoid thought disorders, or schizophrenia (thought broadcasting or delusions). Buspirone hydrochloride is most often indicated for the treatment of anxiety and aggression.

90.) A nurse is reviewing the record of a client who has been prescribed baclofen (Lioresal). Which of the following disorders, if noted in the client's history, would alert the nurse to contact the health care provider? 1. Seizure disorders 2. Hyperthyroidism 3. Diabetes mellitus 4. Coronary artery disease

1. Seizure disorders Rationale: Clients with seizure disorders may have a lowered seizure threshold when baclofen is administered. Concurrent therapy may require an increase in the anticonvulsive medication. The disorders in options 2, 3, and 4 are not a concern when the client is taking baclofen.

79.) Ibuprofen (Advil) is prescribed for a client. The nurse tells the client to take the medication: 1. With 8 oz of milk 2. In the morning after arising 3. 60 minutes before breakfast 4. At bedtime on an empty stomach

1. With 8 oz of milk Rationale: Ibuprofen is a nonsteroidal anti-inflammatory drug (NSAID). NSAIDs should be given with milk or food to prevent gastrointestinal irritation. Options 2, 3, and 4 are incorrect.

The healthcare provider prescribes Morphine Sulfate Oral Solution 38 mg PO q4 hours for a client who is opioid-tolerant. The available 30 ml bottle is labeled, 100 mg/5 ml (20 mg/ml), and is packed with a calibrated oral syringe to provide accurate dose measurements. How many ml should the nurse administer?

1.9 ml

The health care provider prescribes ampicillin 375 mg every 12 hours to infuse over 90 minutes. The pharmacy delivers the drug in a 150 mL IV bag. How many mL/hour should the nurse program the infusion pump? Enter a whole number only

100 ML/hour__

The healthcare provider prescribes diltiazem (Cardizem) for a child with hypertension who weighs 66 pounds. Based on the recommended dose of 3.5 mg/kg/day, how many mg should the child receive per day?

105 mg/day

A client status post-cholecystectomy three days is being prepared to be discharged home. Which client finding is the best indication to the practical nurse that postoperative nursing interventions have prevented respiratory complications?

Has a 95% pulse oximeter value on room air Pulse oximetry of 95% on room air, indicates adequate oxygenation.

As the nurse is turning a client with a chest tube, the chest tube becomes dislodged from the pleural space. Which action should the nurse take first?

Have the client exhale forcefully, and tape 3 sides of a sterile gauze over insertion site

78.) A client is taking phenytoin (Dilantin) for seizure control and a sample for a serum drug level is drawn. Which of the following indicates a therapeutic serum drug range? 1. 5 to 10 mcg/mL 2. 10 to 20 mcg/mL 3. 20 to 30 mcg/mL 4. 30 to 40 mcg/mL

2. 10 to 20 mcg/mL Rationale: The therapeutic serum drug level range for phenytoin (Dilantin) is 10 to 20 mcg/mL. ** A helpful hint may be to remember that the theophylline therapeutic range and the acetaminophen (Tylenol) therapeutic range are the same as the phenytoin (Dilantin) therapeutic range.**

A nurse is caring for a client who is postoperative following a bilateral adrenalectomy. the nurse should expect to administer glucocorticoids following the procedure to enhance which of the following therapeutic effects?

Compensate for decrease in cortisol levels -Rationale: glucocorticoids are used to prevent an adrenal crisis caused by a sudden drop in cortisol levels

What is the depth of the compression the practical nurse (PN) should do when performing the manual chest compressions during cardiopulmonary resuscitation (CPR) on an adult client?

2 inches (5 cm) to 2.4 inches (6 cm) According to the American Heart Association 2015 guidelines, the depth of compressions on an adult during CPR should be at least 2 inches (5 cm) to 2.4 inches (6 cm).

A client receives a prescription for bacitracin 20,000 units every 12 hours IM. The medication is available in a vial that contains 50,000 units and includes reconstitution instructions: "Use 4.8 ml diluent to yield a total volume of 5 ml." How many ml should the nurse administer?

2 ml

176.) A nurse notes that a client is taking lansoprazole (Prevacid). On data collection, the nurse asks which question to determine medication effectiveness? 1. "Has your appetite increased?" 2. "Are you experiencing any heartburn?" 3. "Do you have any problems with vision?" 4. "Do you experience any leg pain when walking?"

2. "Are you experiencing any heartburn?" Rationale: Lansoprazole is a gastric acid pump inhibitor used to treat gastric and duodenal ulcers, erosive esophagitis, and hypersecretory conditions. It also is used to treat gastroesophageal reflux disease (GERD). It is not used to treat visual problems, problems with appetite, or leg pain. **NOTE: "-zole" refers to gastric acid pump inhibitors**

183.) A client who received a kidney transplant is taking azathioprine (Imuran), and the nurse provides instructions about the medication. Which statement by the client indicates a need for further instructions? 1. "I need to watch for signs of infection." 2. "I need to discontinue the medication after 14 days of use." 3. "I can take the medication with meals to minimize nausea." 4. "I need to call the health care provider (HCP) if more than one dose is missed."

2. "I need to discontinue the medication after 14 days of use." Rationale: Azathioprine is an immunosuppressant medication that is taken for life. Because of the effects of the medication, the client must watch for signs of infection, which are reported immediately to the HCP. The client should also call the HCP if more than one dose is missed. The medication may be taken with meals to minimize nausea.

194.) A nurse provides instructions to a client regarding the use of tretinoin (Retin-A). Which statement by the client indicates the need for further instructions? 1. "Optimal results will be seen after 6 weeks." 2. "I should apply a very thin layer to my skin." 3. "I should wash my hands thoroughly after applying the medication." 4. "I should cleanse my skin thoroughly before applying the medication."

2. "I should apply a very thin layer to my skin." Rationale: Tretinoin is applied liberally to the skin. The hands are washed thoroughly immediately after applying. Therapeutic results should be seen after 2 to 3 weeks but may not be optimal until after 6 weeks. The skin needs to be cleansed thoroughly before applying the medication.

227.) When teaching a client who is being started on imipramine hydrochloride (Tofranil), the nurse would inform the client that the desired effects of the medication may: 1. Start during the first week of administration 2. Not occur for 2 to 3 weeks of administration 3. Start during the second week of administration 4. Not occur until after a month of administration

2. Not occur for 2 to 3 weeks of administration Rationale: The therapeutic effects of administration of imipramine hydrochloride may not occur for 2 to 3 weeks after the antidepressant therapy has been initiated. Therefore options 1, 3, and 4 are incorrect.

82.) A client is receiving meperidine hydrochloride (Demerol) for pain. Which of the following are side effects of this medication. Select all that apply. 1. Diarrhea 2. Tremors 3. Drowsiness 4. Hypotension 5. Urinary frequency 6. Increased respiratory rate

2. Tremors 3. Drowsiness 4. Hypotension Rationale: Meperidine hydrochloride is an opioid analgesic. Side effects include respiratory depression, drowsiness, hypotension, constipation, urinary retention, nausea, vomiting, and tremors.

135.) A nurse reinforces medication instructions to a client who is taking levothyroxine (Synthroid). The nurse instructs the client to notify the health care provider (HCP) if which of the following occurs? 1. Fatigue 2. Tremors 3. Cold intolerance 4. Excessively dry skin

2. Tremors Rationale: Excessive doses of levothyroxine (Synthroid) can produce signs and symptoms of hyperthyroidism. These include tachycardia, chest pain, tremors, nervousness, insomnia, hyperthermia, heat intolerance, and sweating. The client should be instructed to notify the HCP if these occur. Options 1, 3, and 4 are signs of hypothyroidism.

A young adult client, admitted to the ED following a motor vehicle collision, is transfused with 4 units of PRBCs. The client's pretransfusion hematocrit is 17%. Which hematocrit value should the nurse expect the client to have after all of the PRBCs have been transfused?

29%

35.) The client has a PRN prescription for loperamide hydrochloride (Imodium). The nurse understands that this medication is used for which condition? 1. Constipation 2. Abdominal pain 3. An episode of diarrhea 4. Hematest-positive nasogastric tube drainage

3. An episode of diarrhea Rationale: Loperamide is an antidiarrheal agent. It is used to manage acute and also chronic diarrhea in conditions such as inflammatory bowel disease. Loperamide also can be used to reduce the volume of drainage from an ileostomy. It is not used for the conditions in options 1, 2, and 4.

229.) A client who is taking lithium carbonate (Lithobid) is scheduled for surgery. The nurse informs the client that: 1. The medication will be discontinued a week before the surgery and resumed 1 week postoperatively. 2. The medication is to be taken until the day of surgery and resumed by injection immediately postoperatively. 3. The medication will be discontinued 1 to 2 days before the surgery and resumed as soon as full oral intake is allowed. 4. The medication will be discontinued several days before surgery and resumed by injection in the immediate postoperative period.

3. The medication will be discontinued 1 to 2 days before the surgery and resumed as soon as full oral intake is allowed. Rationale: The client who is on lithium carbonate must be off the medication for 1 to 2 days before a scheduled surgical procedure and can resume the medication when full oral intake is prescribed after the surgery. **lithium carbonate is an oral medication and is not given as an injection**

22.) A nurse is caring for a client after thyroidectomy and notes that calcium gluconate is prescribed for the client. The nurse determines that this medication has been prescribed to: 1. Treat thyroid storm. 2. Prevent cardiac irritability. 3. Treat hypocalcemic tetany. 4. Stimulate the release of parathyroid hormone.

3. Treat hypocalcemic tetany. Rationale: Hypocalcemia can develop after thyroidectomy if the parathyroid glands are accidentally removed or injured during surgery. Manifestations develop 1 to 7 days after surgery. If the client develops numbness and tingling around the mouth, fingertips, or toes or muscle spasms or twitching, the health care provider is notified immediately. Calcium gluconate should be kept at the bedside.

27.) Sildenafil (Viagra) is prescribed to treat a client with erectile dysfunction. A nurse reviews the client's medical record and would question the prescription if which of the following is noted in the client's history? 1. Neuralgia 2. Insomnia 3. Use of nitroglycerin 4. Use of multivitamins

3. Use of nitroglycerin Rationale: Sildenafil (Viagra) enhances the vasodilating effect of nitric oxide in the corpus cavernosum of the penis, thus sustaining an erection. Because of the effect of the medication, it is contraindicated with concurrent use of organic nitrates and nitroglycerin. Sildenafil is not contraindicated with the use of vitamins. Neuralgia and insomnia are side effects of the medication.

A client with a psychotic disorder is receiving haloperidol (Haldol) 3 mg IM q30 minutes x 3 hours for agitation control. The medication is available in 5 mg/ml. How many ml will the client receive over the next 3 hours?

3.6 ml

Sodium nitroprusside (Nipride) at 0.8 mcg/kg/minute is prescribed for a client who weighs 65 kg. The available IV solution is labeled Nipride 50 mg in 500 ml D5W. The nurse should program the infusion pump to deliver how many ml/hour?

31 ml/hour

118.) A nurse is caring for an older client with a diagnosis of myasthenia gravis and has reinforced self-care instructions. Which statement by the client indicates that further teaching is necessary? 1. "I rest each afternoon after my walk." 2. "I cough and deep breathe many times during the day." 3. "If I get abdominal cramps and diarrhea, I should call my doctor." 4. "I can change the time of my medication on the mornings that I feel strong."

4. "I can change the time of my medication on the mornings that I feel strong." Rationale: The client with myasthenia gravis should be taught that timing of anticholinesterase medication is critical. It is important to instruct the client to administer the medication on time to maintain a chemical balance at the neuromuscular junction. If not given on time, the client may become too weak to swallow. Options 1, 2, and 3 include the necessary information that the client needs to understand to maintain health with this neurological degenerative disease.

54.) A nurse reinforces discharge instructions to a postoperative client who is taking warfarin sodium (Coumadin). Which statement, if made by the client, reflects the need for further teaching? 1. "I will take my pills every day at the same time." 2. "I will be certain to avoid alcohol consumption." 3. "I have already called my family to pick up a Medic-Alert bracelet." 4. "I will take Ecotrin (enteric-coated aspirin) for my headaches because it is coated."

4. "I will take Ecotrin (enteric-coated aspirin) for my headaches because it is coated." Rationale: Ecotrin is an aspirin-containing product and should be avoided. Alcohol consumption should be avoided by a client taking warfarin sodium. Taking prescribed medication at the same time each day increases client compliance. The Medic-Alert bracelet provides health care personnel emergency information.

226.) A client receiving lithium carbonate (Lithobid) complains of loose, watery stools and difficulty walking. The nurse would expect the serum lithium level to be which of the following? 1. 0.7 mEq/L 2. 1.0 mEq/L 3. 1.2 mEq/L 4. 1.7 mEq/L

4. 1.7 mEq/L Rationale: The therapeutic serum level of lithium ranges from 0.6 to 1.2 mEq/L. Serum lithium levels above the therapeutic level will produce signs of toxicity.

185.) Which of the following herbal therapies would be prescribed for its use as an antispasmodic? Select all that apply. 1.Aloe 2.Kava 3.Ginger 4.Chamomile 5.Peppermint oil

4.Chamomile 5.Peppermint oil Rationale: Chamomile has a mild sedative effect and acts as an antispasmodic and anti-inflammatory. Peppermint oil acts as an antispasmodic and is used for irritable bowel syndrome. Topical aloe promotes wound healing. Aloe taken orally acts as a laxative. Kava has an anxiolytic, sedative, and analgesic effect. Ginger is effective in relieving nausea.

The mother reports to the nurse that the thick honey-colored crusts on her child's legs began as flat red spots. This is highly indicative of what condition? A. Impetigo B. Eczema C. Ringworm D. Psoriasis

A. Impetigo

The nurse is triaging clients who have been injured during a tornado. Which client requires immediate action?

A middle aged female with a broken humerus who is unable to follow commands and is crying.

A client is admitted to the hospital for a Crohn's disease flare up and severe dehydration. Which findings should the practical nurse report to the registered nurse and/or health care provider immediately?

A rigid hard abdomen and elevated white blood cell count A hard, rigid abdomen and elevated white blood cell (WBC) count are indicative of peritonitis, which is a medical emergency and should be reported to the RN and health care provider immediately.

A male client who has never smoked but has had COPD for the past 5 years is now being assessed for cancer of the lung. The nurse knows that he is most likely to develop which type of lung cancer? A) Adenocarcinoma. B) Oat-cell carcinoma. C) Malignant melanoma. D) Squamous-cell carcinoma.

A) Adenocarcinoma. Adenocarcinoma is the only lung cancer not related to cigarette smoking (A). It has been found to be directly related to lung scarring and fibrosis from preexisting pulmonary disease such as TB or COPD. Both (B and D) are malignant lung cancers related to cigarette smoking. (C) is a skin cancer and is related to exposure to sunlight, not to lung problems.

During assessment of a client with amyotrophic lateral sclerosis (ALS), which finding should the nurse identify when planning care for this client? A) Muscle weakness. B) Urinary frequency. C) Abnormal involuntary movements. D) A decline in cognitive function.

A) Muscle weakness. Amyotrophic lateral sclerosis (ALS) is characterized by a degeneration of motor neurons in the brainstem and spinal cord and are manifested by muscle weakness (A) and wasting. ALS does not manifest (B and C). In ALS, the client remains cognitively intact, not (D), while the physical status deteriorates.

When admitting a patient with the diagnosis of asthma exacerbation, the nurse will assess for which of the following potential triggers? (Select all that apply.) A. Exercise B. Allergies C. Emotional stress D. Decreased humidity

A,B,C Although the exact mechanism of asthma is unknown, there are several triggers that may precipitate an attack. These include allergens, exercise, air pollutants, respiratory infections, drug and food additives, psychologic factors, and GERD.

55. During admission of a patient diagnosed with non-small cell carcinoma of the lung, the nurse questions the patient related to a history of which of the following risk factors for this type of cancer? (Select all that apply.) A. Asbestos exposure B. Cigarette smoking C. Exposure to uranium D. Chronic interstitial fibrosis

A,B,C Non-small carcinoma is associated with cigarette smoking and exposure to environmental carcinogens, including asbestos and uranium. Chronic interstitial fibrosis is associated with the development of adenocarcinoma of the lung.

52. When assessing a patient's sleep-rest pattern related to respiratory health, the nurse would ask if the patient: (Select all that apply.) A. Has trouble falling asleep B. Awakens abruptly during the night C. Sleeps more than 8 hours per night D. Has to sleep with the head elevated

A,B,D The patient with sleep apnea may have insomnia and/or abrupt awakenings. Patients with cardiovascular disease (e.g., heart failure that may affect respiratory health) may need to sleep with the head elevated on several pillows (orthopnea). Sleeping more than 8 hours per night is not indicative of impaired respiratory health.

When assessing a patient's sleep-rest pattern related to respiratory health, the nurse would ask if the patient: (Select all that apply.) A. Has trouble falling asleep B. Awakens abruptly during the night C. Sleeps more than 8 hours per night D. Has to sleep with the head elevated

A,B,D The patient with sleep apnea may have insomnia and/or abrupt awakenings. Patients with cardiovascular disease (e.g., heart failure that may affect respiratory health) may need to sleep with the head elevated on several pillows (orthopnea). Sleeping more than 8 hours per night is not indicative of impaired respiratory health.

56. When admitting a 45-year-old female with a diagnosis of pulmonary embolism, the nurse will assess the patient for which of the following risk factors? (Select all that apply.) A. Obesity B. Pneumonia C. Hypertension D. Cigarette smoking

A,C,D Research has demonstrated an increased risk of pulmonary embolism in women associated with obesity, heavy cigarette smoking, and hypertension. Other risk factors include immobilization, surgery within the last 3 months, stroke, history of DVT, and malignancy.

When admitting a 45-year-old female with a diagnosis of pulmonary embolism, the nurse will assess the patient for which of the following risk factors? (Select all that apply.) A. Obesity B. Pneumonia C. Hypertension D. Cigarette smoking

A,C,D Research has demonstrated an increased risk of pulmonary embolism in women associated with obesity, heavy cigarette smoking, and hypertension. Other risk factors include immobilization, surgery within the last 3 months, stroke, history of DVT, and malignancy.

54. To promote airway clearance in a patient with pneumonia, the nurse instructs the patient to do which of the following? (Select all that apply.) A. Splint the chest when coughing B. Maintain a semi-Fowler's position C. Maintain adequate fluid intake D. Instruct patient to cough at end of exhalation

A,C,D The nurse should instruct the patient to splint the chest while coughing. This will reduce discomfort and allow for a more effective cough. Maintaining adequate fluid intake liquefies secretions, allowing easier expectoration. Coughing at the end of exhalation promotes a more effective cough. The patient should be positioned in an upright sitting position (high-Fowler's) with head slightly flexed.

During morning rounds, the nurse finds a client who has no spontaneous respirations and does not respond to shaking. The nurse activates the "Code-Blue" system. While waiting for the code team to arrive, what action should the nurse implement? A. Check the pulse for 10 seconds and begin chest compressions B. Check the airway for a foreign body and remove it if visualized C. Assess blood pressure and pupillary response to light D. Seal the face mask of an Ambu bag over the client's mouth and nose

A. Check the pulse for 10 seconds and begin chest compressions

5. When initially teaching a patient the supraglottic swallow following a radical neck dissection, with which of the following foods should the nurse begin? A. Cola B. Applesauce C. French fries D. White grape juice

A. ColaWhen learning the supraglottic swallow, it may be helpful to start with carbonated beverages because the effervescence provides clues about the liquid's position. Thin, watery fluids should be avoided because they are difficult to swallow and increase the risk of aspiration. Nonpourable pureed foods, such as applesauce, would decrease the risk of aspiration, but carbonated beverages are the better choice to start with.

The register nurse (RN) is observing a newly hired practical nurse (PN) give a newborn a vitamin K (AquaMEPHYTON) injection. The PN uses a filter need to draw 0.25 ml of AquaMEPHYTON into the syringe, cleanses the thigh with alcohol in a circular motion, and prepares to inject the needle at a 90 degree angle in the left vastus lateralis. What action should the RN take? A. Direct the PN to change the filter needle to a 1/2 inch needle B. Confirm that the correct injection technique is being used C. Suggest injecting the medication into the dorsogluteal muscle D. Recommend administration of the injection at a 45 degree angle

A. Direct the PN to change the filter needle to a 1/2 inch needle

The nurse is preparing to administer 1.6 ml of medication IM to a 4-month-old infant. Which intervention should the nurse implement? A. Divide the medication into two injections with volumes under 1 ml B. Inject into the center one-third of the medical aspect of the thigh C. Administer into the ventrogluteal muscle with child prone D. Apply a topical anesthetic ointment prior to the injection

A. Divide the medication into two injections with volumes under 1 ml

To prevent infection by autocontamination during the acute phase of recovery from multiple burns, which intervention is most important for the nurse to implement? A. Dress each wound separately B. Avoid sharing equipment between multiple clients C. Use gown, mask, and gloves with dressing change D. Implement protective isolation

A. Dress each wound separately

The husband of an older woman, diagnosed with pernicious anemia, calls the clinic to report that his wife still has memory loss and some confusion since she received the first dose of nasal cyanocobalamin two days ago. He tells the nurse that he is worried that she may be getting Alzheimer's disease. What action should the nurse take? A. Explain that memory loss and confusion are common with Vitamin B12 deficiency B. Ask if the client is experiencing any change in bowel habits C. Determine if the client is taking iron and folic aid supplements D. Encourage the husband to bring the client to the clinic for a complete blood count

A. Explain that memory loss and confusion are common with Vitamin B12 deficiency

In caring for a patient with a PCA infusion of morphine sulfate through the right cephalic vein, the nurse assess that client is lethargic, with a blood pressure of 90/60, pulse rate of 118 beats/minute, and a respiratory rate of 8 breaths/minute. What assessment should the nurse perform next? A. Observe the amount and dose of morphine in the PCA pump syringe B. Note the appearance and patency of the client's peripheral IV site C. Palpate the volume of the client's right radial pulse D. Auscultate the client's breath sounds bilaterally

A. Observe the amount and dose of morphine in the PCA pump syringe

During assessment of a client in the intensive care unit, the nurse notes that the client's breath sounds are clear on auscultation, but jugular vein distention and muffled heart sounds are present. Which intervention should the nurse implement? A. Prepare the client for a pericardial tap. B. Administer intravenous furosemide (Lasix). C. Assist the client to cough and breathe deeply. D. Instruct the client to restrict the oral fluid intake.

A. Prepare the client for a pericardial tap. Rationale: The client is exhibiting symptoms of cardiac tamponade, a collection of fluid in the pericardial sac that results in a reduction in cardiac output, which is a potentially fatal complication of pericarditis. Treatment for tamponade is a pericardial tap (A). Lasix IV is not indicated for treatment of pericarditis (B). Because the client's breath sounds are clear, (C) is not a priority. Fluids are frequently increased (D) in the initial treatment of tamponade to compensate for the decrease in cardiac output, but this is not the same priority as (A).

The nurse observes an unlicensed assistive personnel (UAP) applying an alcohol-based hand rub while leaving a client's room after taking vital signs. What action should the nurse take? A. Remind the UAP to continue rubbing the hands together until they dry B. Supervise the UAP in the next client's room to evaluate hand hygiene C. Instruct the UAP to return to the clients room to perform hand washing D. Advise the UAP to wear gloves when obtaining vital signs for all clients

A. Remind the UAP to continue rubbing the hands together until they dry

A resident in a long-term care facility is diagnosed with hepatitis B. Which intervention should the nurse implement with the staff caring for this client? A.Determine if all employees have had the hepatitis B vaccine series. B.Explain that this type of hepatitis can be transmitted when feeding the client. C.Assure the employees that they cannot contract hepatitis B when providing direct care. D.Tell the employees that wearing gloves and a gown are required when providing care.

A.Determine if all employees have had the hepatitis B vaccine series Rationale: Hepatitis B vaccine should be administered to all health care providers. Hepatitis A (not hepatitis B) can be transmitted by fecal-oral contamination. There is a chance that staff could contract hepatitis B if exposed to the client's blood and/or body fluids; therefore, option C is incorrect. There is no need to wear gloves and gowns except with blood or body fluid contact.

Which nursing diagnosis should be selected for a client who is receiving thrombolytic infusions for treatment of an acute myocardial infarction?

Risk for injury related to effects of thrombolysis.

After three days of persistent epigastric pain, a female client presents to the clinic. She has been taking oral antacids without relief. Her vital signs are heart rate 122, respirations 16, O2 saturation 96%, and blood pressure 116/70. The nurse obtains an ECG. Which assessment finding is most critical?

ST elevation in three leads

A nurse is reinforcing teaching about breast self-examination (BSE) with a client who has a regular menstrual cycle. The nurse should instruct the client to perform BSE at which of the following times? 1) On the same day every month 2) Prior to the beginning of menses 3) Three to seven days after menses stops 4) On the second day of menstruation

Three to seven day after menses stops

A postoperative client has a large amount of serosanguineous drainage on the surgical dressing and the nurse notes that the operative report indicates that the client has a Penrose drain near the incision. What intervention should the nurse implement when changing the client's dressing?

Apply sterile gloves before removing the soiled dressing

A client who has asthma (a reactive airway disease) asks the nurse about the role of leukotrienes in the process. Leukotrienes have which immune response?

Tighten airway and produce mucus

An adult client with osteoarthritis of both knees tells the nurse the desire to continue daily walks in the park with friends. How should the nurse respond?

Advise less weight bearing to prevent joint destruction

A nurse is caring for an older adult client who has dysphagia and left-sided weakness following a stroke. Which of the following actions should the nurse take? 1) Instruct the client to tilt her head back when she swallows. 2) Place food on the left side of the client's mouth. 3) Add thickener to fluids. 4) Serve food at room temperature.

Add thickener to fluids

A nurse is caring for a client who is receiving peritoneal dialysis. The client is reporting anorexia and constipation. Which of the following should the nurse include in the client's plan of care?

Administer enema prior to dialysis

A client with a history of chronic pain requests a nonopioid analgesic. The client is alert but has difficulty describing the exact nature and location of the pain to the nurse. What action should the nurse implement next?

Administer the analgesic as requested

A client who is scheduled for surgery at 11—has a preoperative intravenous antibiotic scheduled to be administered at 1000. At 0800 the nurse learns that the surgery time has been changed to 1000. Which action should the nurse take?

Administer the prescribed antibiotic intravenously at 0900

9. If a health care provider is planning to transfuse a patient with a unit of packed red blood cells, which of the following solutions should the health care provider hang with the transfusion? A. 5% dextrose in water B. 0.9% sodium chloride C. 5% dextrose in 0.9% sodium chloride D. 5% dextrose in lactated Ringer's solution

B. 0.9% sodium chloride The only solution appropriate for administration with whole blood or blood products is 0.9% sodium chloride. The other options are not appropriate for use with blood products.

Seconal, 0.1 g PRN at bedtime, is prescribed for rest. The scored tablets are labeled grain 1.5 per tablet. How many tablets should the nurse plan to administer? A.½ tablet B. 1 tablet C. 1½ tablets D. 2 tablets

B. 1 tablet Rationale: 15 gr = 1 g. Converting the prescribed dose of 0.1 g to grains requires multiplying 0.1 × 15 = 1.5 grains. The tablets come in 1.5 grains; therefore, the nurse should plan to administer one tablet (B). (A, C, and D) are incorrect.

The nurse is presenting a class at the community center about the prevention of colorectal cancer. Which statements should the nurse include in their teaching? A. Decrease the consumption of fat, refined carbohydrates and low in animal fat. B.After the age of 50, a colonoscopy should be done every 10 years. C. Increase the consumption vegetables such as broccoli, cabbage and sprouts. D. Exercise a minimum of three to four times a week. E. Fecal occult blood testing should be done yearly, starting at the age of 30.

Answer A B C D The nurse should include in the class on colorectal cancer: decrease the amount of fat, refined carbohydrates and animal fat in their diets. Clients should be told to consumed more baked and broiled foods, high fiber and plants that come from a stem or stalk such as broccoli, cabbage, cauliflower and sprouts. These types of plants have been shown to help protect the intestinal mucosa from colon cancer. The class should also include the importance of not being sedentary, avoidance of smoking and excessive alcohol. After the age of 50, the participants should be told that they should have a colonoscopy every 10 years; or a double-contrast barium enema or sigmoidoscopy with fecal occult blood testing every five years.

A nurse is assigned the care of a client who is presently experiencing hypovolemic shock. The client's MAP has decreased by 20 mmHg from its baseline, tissue ischemia and anoxia of non-vital organs is occurring, pulses are weak, urine output is absent and the client's skin is cool and moist. The client appears to be confused and extremely anxious. Which stage of hypovolemic shock do these clinical signs and symptoms indicate? A. Initial stage. B. Refractory Stage. C. Progressive stage. D. Non-progressive stage.

Answer C The client is exhibiting the clinical signs and symptoms of the progressive stage of hypovolemic shock which if allowed to progress will lead into the refractory stage and eventually death. The clinical signs and symptoms of hypovolemic progresses in four stages, if the condition causing the shock and/or interventions is not done and/or successful. If the interventions and the cause of the shock are corrected within 1-2 hours after onset, then the effects are temporary and irreversible.

Which abnormal urine test result is considered a reliable indicator for a diagnosis of diabetes insipidus? A. Low specific gravity. B. Elevated pH. C. Increased protein. D. Decreased ketones.

Answer: A Central diabetes insipidus is a condition as a result of injury cause to the pituitary gland or hypothalamus which interferes in the production, storage and release of the anti-diuretic hormone (ADH). This condition is characterized by excessive thirst and excretion of large amounts of dilute urine. This large volume of dilute urine results in a low specific gravity of urine.

A nurse is providing teaching to a client who has Addison's disease about healthy snack foods. Which of the following food choices by the client indicates an understanding of the teaching?

Turkey and cheese sandwich -Rationale: high in protein, carbohydrates, and sodium. A client who has Addison's requires a diet low in potassium, and high in protein, carbs, and sodium.

The nurse is assigned a client whose condition has been caused by an infection which as spread throughout the body; their current clinical findings consists of temperature 101°F (38.3°C); HR 94bpm; RR 22bpm; WBC 14,000/mm3; SBP 87mmHg; and a MAP 66mmHg. The client's urine output has decreased to less than 0.5ml/kg/hr, despite RL at 100ml/hr. The client clinical signs indicative of which condition? A. Organ failure (severe sepsis). B. Systemic infection (early sepsis). C. Systemic inflammatory response syndrome. D. Multiple organ dysfunction syndrome (MODS).

Answer: C The client is experiencing a systemic inflammatory response syndrome as their sepsis becomes progressively worse. Sepsis in accompanied with increases temperature of >101°F (38.3°C) or <96.8°F(36°C); HR greater than 90bpm; RR greater than 20bpm; WBC count (>12,000/mm3 or <4000/mm3); SBP <90mmHg; MAP <70mmHg; urine output < 0.5ml/kg/hr despite fluid resuscitation.

A client from a nursing home is admitted with diagnoses of diabetes mellitus, chronic pancreatitis and alcoholism. The healthcare provider has prescribed the client pancrelipase (Creon, Pancrease). How should the nurse document the effectiveness of this prescribed medication? A. The absence or presence of delirium tremors. B. The character and quality of abdominal pain. C. Glucometer readings before and after each meal. D. The number, frequency and consistency of stools per day.

Answer: D Pancrelipase is a pancreatic enzyme to aid in the digestion of carbohydrates, protein and fat due to pancreatic insufficiency from the chronic pancreatitis. To evaluate the effectiveness, the nurse should record the number, frequency and consistency of the client's daily stools. If the medication is being effective the stools should become less frequent and have less steatorrhea.

A client diagnosed with diabetic neuropathy has a nonhealing ulcer on the lateral aspect of their (R) foot. Which question should the practical nurse (PN) ask to gather objective data for the development of a diabetic foot care teaching plan?

Ask to look at the pair of shoes they wear. Asking to look at the client's shoes will give the most objective data. The PN needs to inspect the client's shoes' inside for any area such as a seam that may be rubbing against the client foot causing trauma to their feet. Well-fitted shoes are very important to prevent the development of foot ulcers for the diabetic client.

Which nursing intervention promotes achievement of the goal "optimal mobility" for a client who had a total hip replacement 8 hours ago?

Assist the client to turn while an abductor pillow is between the legs.

A client diagnosed with chronic obstructive pulmonary disease complains to the practical nurse of extreme fatigue after coughing. Which self-care measures can help minimize the client's dyspnea? (Select all that apply.)

Assume a sitting position with shoulders relaxed and knees flexed. Support forearms with a pillow and place both feet flat on the floor. Slightly drop the head, bend forward, and slowly exhale with pursed lips. Resume sitting up straight, using diaphragmatic breathing to inhale slowly and deeply. Effective coughing can help the client to cough secretions, therefore improving gas exchange and minimize fatigue. The client should assume the sitting position with shoulders relaxed and knees flexed. Their forearms should be supported with a pillow and both feet place flat on the floor. The client should slightly drop their head, bent forward, and slowly exhale through pursed lips using slow and deep diaphragmatic breathing to help facilitate effective coughing. The client should repeat the previous steps two or three times. The client should initiate the cough reflex, not wait for it. The client should also take a deep abdominal breath before initiating a cough.

A nurse is caring for a client who has Alzheimer's disease. The nurse discovers the client entering the room of another client, who becomes upset and frightened. Which of the following actions should the nurse take? 1) Attempt to determine what the client was looking for. 2) Explain the client's Alzheimer's diagnosis to the frightened client. 3) Reprimand the client for invading the other client's privacy. 4) Ask the client to apologize for his behavior.

Attempt to determine what the client was looking for

A nurse is caring for a client who has second- and third-degree burns and a prescription for a high-calorie, high-protein diet. Which of the following menu choices should the nurse recommend? 1) ½ cup whole-grain pasta with tomato sauce and pears 2) Turkey and cheese sandwich with scalloped potatoes 3) ½ cup black beans with a brownie 4) Roast beef with romaine lettuce salad

Turkey and cheese sandwich with scalloped potatoes

In assessing cancer risk, the nurse identifies which woman as being at greatest risk of developing breast cancer? A) A 35-year-old multipara who never breastfed. B) A 50-year-old whose mother had unilateral breast cancer. C) A 55-year-old whose mother-in-law had bilateral breast cancer. D) A 20-year-old whose menarche occurred at age 9.

B) A 50-year-old whose mother had unilateral breast cancer. The most predictive risk factors for development of breast cancer are over 40 years of age and a positive family history (occurrence in the immediate family, i.e., mother or sister). Other risk factors include nulliparity, no history of breastfeeding, early menarche and late menopause. Although all of the women described have one of the risk factors for developing breast cancer, (B) has the greater risk over (A, C, and D).

A female client requests information about using the calendar method of contraception. Which assessment is most important for the nurse to obtain? A) Amount of weight gain or weight loss during the previous year. B) An accurate menstrual cycle diary for the past 6 to 12 months. C) Skin pigmentation and hair texture for evidence of hormonal changes. D) Previous birth-control methods and beliefs about the calendar method.

B) An accurate menstrual cycle diary for the past 6 to 12 months. The fertile period, which occurs 2 weeks prior to the onset of menses, is determined using an accurate record of the number of days of the menstrual cycles for the past 6 months, so it is most important to emphasize to the client that accuracy and compliancy of a menstrual diary (B) is the basis of the calendar method. (A and C) may be partially related to hormonal fluctuations but are not indicators for using the calendar method. (D) may demonstrate client understanding and compliancy but is not the most important aspect.

Which symptoms should the nurse expect a client to exhibit who is known to have a pheochromocytoma? A) Numbness, tingling, and cramps in the extremities. B) Headache, diaphoresis, and palpitations. C) Cyanosis, fever, and classic signs of shock. D) Nausea, vomiting, and muscular weakness.

B) Headache, diaphoresis, and palpitations. (B) is the typical triad of symptoms of tumors of the adrenal medulla (symptoms depend on the relative proportions of epinephrine and norepinephrine secretion). (A) lists the signs of latent tetany, exhibited by clients diagnosed with hypoparathyroidism. (C) lists the signs of an Addisonian (adrenal) crisis. (D) lists the signs of hyperparathyroidism.

Healthcare workers must protect themselves against becoming infected with HIV. The Center for Disease Control has issued guidelines for healthcare workers in relation to protection from HIV. These guidelines include which recommendation? A) Place HIV positive clients in strict isolation and limit visitors. B) Wear gloves when coming in contact with the blood or body fluids of any client. C) Conduct mandatory HIV testing of those who work with AIDS clients. D) Freeze HIV blood specimens at -70° F to kill the virus.

B) Wear gloves when coming in contact with the blood or body fluids of any client. The CDC guidelines recommend that healthcare workers use gloves when coming in contact with blood or body fluids from ANY client (B) since HIV is infectious before the client becomes aware of symptoms. (A) is not recommended, nor is it necessary. (C) is very controversial, difficult to enforce, and is not recommended by CDC. (D) does not guarantee to kill the virus. Additionally, the purpose of the blood specimen will determine how it is stored and handled.

A client is admitted to the hospital with a medical diagnosis of pneumococcal pneumonia. The nurse knows that the prognosis for gram-negative pneumonias (such as E. coli, Klebsiella, Pseudomonas, and Proteus) is very poor because A) they occur in the lower lobe alveoli which are more sensitive to infection. B) gram-negative organisms are more resistant to antibiotic therapy. C) they occur in healthy young adults who have recently been debilitated by an upper respiratory infection. D) gram-negative pneumonias usually affect infants and small children.

B) gram-negative organisms are more resistant to antibiotic therapy. The gram-negative organisms are resistant to drug therapy (B) which makes recovery very difficult. Gram-negative pneumonias affect all lobes of the lung (A). The mean age for contracting this type of pneumonia is 50 years (C and D), and it usually strikes debilitated persons such as alcoholics, diabetics, and those with chronic lung diseases.

The nurse notes that a client who is scheduled for surgery the next morning has an elevated blood urea nitrogen (BUN) level. Which condition is most likely to have contributed to this finding? A. Myocardial infarction 2 months ago B.Anorexia and vomiting for the past 2 days C.Recently diagnosed type 2 diabetes mellitus D.Skeletal traction for a right hip fracture

B. Anorexia and vomiting for the past 2 days Rationale: The blood urea nitrogen (BUN) level indicates the effectiveness of the kidneys in filtering waste from the blood. Dehydration, which could be caused by vomiting, would cause an increased BUN level. Option A would affect serum enzyme levels, not the BUN level. Option C would primarily affect the blood glucose level; renal failure that could increase the BUN level would be unlikely in a client newly diagnosed with type 2 diabetes. Effects of option D might affect the complete blood count (CBC) but would not directly increase the BUN level.

The nurse is teaching a patient how to self-administer ipratropium (Atrovent) via a metered dose inhaler. Which of the following instructions given by the nurse is most appropriate to help the patient learn proper inhalation technique? A. "Avoid shaking the inhaler before use." B. "Breathe out slowly before positioning the inhaler." C. "After taking a puff, hold the breath for 30 seconds before exhaling." D. "Using a spacer should be avoided for this type of medication."

B. "Breathe out slowly before positioning the inhaler." It is important to breathe out slowly before positioning the inhaler. This allows the patient to take a deeper breath while inhaling the medication thus enhancing the effectiveness of the dose.

The nurse is conducting health assessments. Which assessment finding increases a 56-year-old woman's risk for developing osteoporosis? A. Body mass index (BMI) of 31 B. 20 pack-year history of cigarette smoking C. Birth control pill usage until age 45 D. Diabetes mellitus in family history

B. 20 pack-year history of cigarette smoking

Which client situation requires the most immediate intervention by the nurse? A. One day after surgery, a client complains of unpleasant flatulence B. A bedfast client experiences an episode of urinary incontinence C. An ambulatory client reports the onset of stress incontinence D. A client receiving regular dose of an opioid develops constipation

B. A bedfast client experiences an episode of urinary incontinence

A newly graduated and licensed registered nurse (RN) is in the second day of orientation to the hospital unit. The education director tells the charge nurse that the new graduate should be assigned to care for one client. Which client is best for the nurse to assign to this new graduate? A. A client returning from surgery following a bowel resection B. A client who is 4-days post myocardial infraction C. A 6-day postoperative client who is receiving a blood transfusion D. A client with AIDS who is in the final stages of dying

B. A client who is 4-days post myocardial infraction

The nurse is reviewing routine medications taken by a client with chronic angle closure glaucoma. Which medication prescription should the nurse question? A. Antianginal with a therapeutic effect of vasodilation B. Anticholinergic with a side effect of pupillary dilation C. Antihistamine with a side effect of sedation D. Corticosteroid with a side effect of hyperglycemia

B. Anticholinergic with a side effect of pupillary dilation Rationale: Clients with angle-closure glaucoma should not take medications that dilate the pupil (B) because this can precipitate acute and severely increased intraocular pressure. (A, C, and D) do not cause increased intracranial pressure, which is the primary concern with angle-closure glaucoma.

The nurse is reviewing routine medications taken by a client with chronic angle-closure glaucoma. Which medication prescription should the nurse question? A. Antianginal with a therapeutic effect of vasodilation B.Anticholinergic with a side effect of pupillary dilation C. Antihistamine with a side effect of sedation D. Corticosteroid with a side effect of hyperglycemia

B. Anticholinergic with a side effect of pupillary dilation Rationale: Clients with angle-closure glaucoma should not take medications that dilate the pupil because this can precipitate acute and severely increased intraocular pressure. Options A, C, and D do not cause increased intracranial pressure, which is the primary concern with angle-closure glaucoma.

The nurse is reviewing the routine medications taken by a client with chronic angle closure glaucoma. Which medication prescription should the nurse question? A. An antianginal with a therapeutic effect of vasodilation. B. An anticholinergic with a side effect of pupillary dilation. C. An antihistamine with a side effect of sedation. D. A corticosteroid with a side effect of hyperglycemia.

B. Clients with angle closure glaucoma should not take medications that dilate the pupil (B) because this can precipitate acute and severely increased intraocular pressure. (A, C, D) do not cause increased intraocular pressure, which is the primary concern.

The nurse is caring for a client with a fractured right elbow. Which assessment finding has the highest priority and requires immediate intervention? A.Ecchymosis over the right elbow area B.Deep unrelenting pain in the right arm C.An edematous right elbow D.The presence of crepitus in the right elbow

B. Deep unrelenting pain in the right arm Rationale: Compartment syndrome is a condition involving increased pressure and constriction of the nerves and vessels within an anatomic compartment, causing pain uncontrolled by opioids and neurovascular compromise. Option A is an expected finding. Option C related to compartment syndrome cannot be seen, and any visible edema is an expected finding related to the injury. Option D is an expected finding.

The new mother tells the nurse that she does not want her newborn to receive any immunizations. It is the hospital's policy to routinely administer immunizations to all newborns. What intervention should the nurse implement? A. Advise the mother to sign out of the hospital AMA if wishing to refuse the immunizations B. Document that the mother has refused immunization permission for the newborn C. Administer the immunizations after first explaining the hospital policy to the mother D. Report the immunization status of the infant to the office of child protective services

B. Document that the mother has refused immunization permission for the newborn

The nurse is triaging clients in an urgent car clinic. The client with which symptoms should be referred to the health care provider immediately? A. High fever, skin rash, and a productive cough B. Headache, photophobia, and nuchal rigidity C. Nausea, vomiting, and poor skin turgor D. Malaise, fever, and stiff, swollen joints

B. Headache, photophobia, and nuchal rigidity

A family member was taught to suction a client's tracheostomy prior to the client's discharge from the hospital. Which observation by the nurse indicates that the family member is capable of correctly performing the suctioning technique? A. Turns on the continuous wall suction to −190 mm Hg B. Inserts the catheter until resistance or coughing occurs C. Withdraws the catheter while maintaining suctioning D. Reclears the tracheostomy after suctioning the mouth

B. Inserts the catheter until resistance or coughing occurs Rationale: (B) indicates correct technique for performing suctioning. Suction pressure should be between −80 and −120 mm Hg, not −190 mm Hg (A). The catheter should be withdrawn 1 to 2 cm at a time with intermittent, not continuous, suction (C). (D) introduces pathogens unnecessarily into the tracheobronchial tree.

The nurse assesses a postoperative client whose skin is cool, pale, and moist. The client is very restless and has scant urine output. Oxygen is being administered at 2 L/min, and a saline lock is in place. Which intervention should the nurse implement first? A. Measure the urine specific gravity. B. Obtain IV fluids for infusion per protocol. C. Prepare for insertion of a central venous catheter. D. Auscultate the client's breath sounds

B. Obtain IV fluids for infusion per protocol. Rationale: The client is at risk for hypovolemic shock because of the postoperative status and is exhibiting early signs of shock. A priority intervention is the initiation of IV fluids to restore tissue perfusion. Options A, C, and D are all important interventions but are of lower priority than option B.

An older male client comes to the geriatric screening clinic complaining of pain in his left calf. The nurse notices a reddened area on the calf of his right leg that is warm to touch and the nurse suspects that the client may have thrombophlebitis. Which addition assessment is most important for the nurse to perform? A. Measure calf circumference. B. Auscultate the client's breath sounds. C. Observe for ecchymosis and petechiae. D. Obtain the client's blood pressure.

B. Since the client may have a pulmonary embolus secondary to the thrombophlebitis. A. Would support the nurses assessment. C. Least helpful since bruising is not associated with thrombophlebitis. D. Less important then auscultation.

The nurse observes ventricular fibrillation on telemetry and, on entering the client's bathroom, finds the client unconscious on the floor. Which intervention should the nurse implement first? A. Administer an antidysrhythmic medication. B. Start cardiopulmonary resuscitation. C. Defibrillate the client at 200 J. D. Assess the client's pulse oximetry.

B. Start cardiopulmonary resuscitation. Rationale: Ventricular fibrillation is a life-threatening dysrhythmia, and CPR should be started immediately (B). (A and C) are appropriate, but CPR is the priority action. The client is dying, and (D) does not address the seriousness of this situation.

A female client with a nasogastric tube attached to low suction states that she is nauseated. The nurse assesses that there has been no drainage through the nasogastric tube in the last 2 hours. Which action should the nurse take first? A. Irrigate the nasogastric tube with sterile normal saline. B. Reposition the client on her side. C. Advance the nasogastric tube 5 cm. D. Administer an intravenous antiemetic as prescribed.

B. The priority is to determined if the tube is functioning correctly, which would relieve the client's nausea. The least invasive intervention is to reposition the client (B), should be attempted first, followed by (A & C) if these are unsuccessful then (D).

A nurse call the nurse to report that at 0900 she administered a PO dose of digoxin to her 4-month-old infant, but at 0920 the baby vomited the medicine. What instruction should the nurse provide to his mother? A. give another dose B. withhold this dose C. administer a half dose now D. mix the next dose with food

B. withhold this dose

A nurse is reviewing the laboratory values of a client who has diabetic ketoacidosis. The nurse should understand that which of the following laboratory values is consistent with diabetic ketoacidosis?

Bicarbonate level 12 mEq/L -Rationale: DKA patients have bicarbonate levels less than 15

A 70-year-old client status post-hip replacement is transferred to a rehabilitation facility. Which scale should the practical nurse (PN) identify as the best tool to predict the client's risk for developing skin breakdown?

Braden Scale The Braden Scale is made up of six subscales: sensory perception, moisture, activity, mobility, nutrition, and friction and shear. A hospitalized adult with a score 16 or below or an older adult with a score of 18 or below is at an increased risk for skin breakdown.

A Nurse is reinforcing discharge teaching on actions that improve gas exchange to a client diagnosed with emphysema. Which of the following instructions should be included in the teaching? 1) Rest in a supine position. 2) Consume a low-protein diet. 3) Breathe in through her nose and out through pursed lips. 4) Limit fluid intake throughout the day.

Breath in through her nose and out through pursed lips

A nurse is assessing a client who has Addison's disease. Which of the following skin manifestations should the nurse expect to find?

Bronze pigmentation of skin -Rationale: hormone deficiency caused by damage to the outer layer of the adrenal gland.

A nurse is caring for a client who has Cushing's syndrome. Which of the following clinical manifestations should the nurse expect to observe? (Select all that apply.) 1) Buffalo hump 2) Purple striations 3) Moon face 4) Tremors 5) Obese extremities

Buffalo hump Purple striations Moon face

The preeclamptic client who delivered 24 hours ago remains in the labor and delivery recovery room. She continues to receive magnesium sulfate at 3 grams per hour. Her total input is limited to 125 ml per hour, and her urinary output for the last hour was 850 ml. What intervention should the nurse implement? A. Discontinue the magnesium sulfate immediately B. Decrease the client's IV rate to 50 ml per hour C. Continue with the plan of care for this client D. Change the client's diet to NPO status

C. Continue with the plan of care for this client

A client taking a thiazide diuretic for the past six months has a serum potassium level of 3. The nurse anticipates which change in prescription for the client? A) The dosage of the diuretic will be decreased. B) The diuretic will be discontinued. C) A potassium supplement will be prescribed. D) The dosage of the diuretic will be increased.

C) A potassium supplement will be prescribed. This client's potassium level is too low (normal is 3.5 to 5). Taking a thiazide diuretic often results in a loss of potassium, so a potassium supplement needs to be prescribed to restore a normal serum potassium level (C). (A, B, and D) are not recommended actions for restoring a normal serum potassium level.

Which finding should the nurse identify as most significant for a client diagnosed with polycystic kidney disease (PKD)? A) Hematuria. B) 2 pounds weight gain. C) 3+ bacteria in urine. D) Steady, dull flank pain.

C) 3+ bacteria in urine. Urinary tract infections (UTI) for a client with PKD require prompt antibiotic therapy to prevent renal damage and scarring which may cause further progression of the disease, so bacteria in the urine (C) is the most significant finding at this time. (A) is an expected finding from the rupture of the cysts. (B) does not provide a time frame to determine if the weight gain is a significant fluid fluctuation, which is determined within a 24-hour time frame. Although kidney pain can also be abrupt, episodic, and colicky related to bleeding into the cysts, (D) is more likely an early symptom in PKD.

After the fourth dose of gentamicin sulfate (Garamycin) IV, the nurse plans to draw blood samples to determine peak and trough levels. When are the best times to draw these samples? A) 15 minutes before and 15 minutes after the next dose. B) One hour before and one hour after the next dose. C) 5 minutes before and 30 minutes after the next dose. D) 30 minutes before and 30 minutes after the next dose.

C) 5 minutes before and 30 minutes after the next dose. Peak drug serum levels are achieved 30 minutes after IV administration of aminoglycosides. The best time to draw a trough is the closest time to the next administration (C). (A, B, and D) are not as good a time to draw the trough as (C). (B and D) are not the best times to draw the peak of an aminoglycoside that has been administered IV.

A female client taking oral contraceptives reports to the nurse that she is experiencing calf pain. What action should the nurse implement? A) Determine if the client has also experienced breast tenderness and weight gain. B) Encourage the client to begin a regular, daily program of walking and exercise. C) Advise the client to notify the healthcare provider for immediate medical attention. D) Tell the client to stop taking the medication for a week to see if symptoms subside.

C) Advise the client to notify the healthcare provider for immediate medical attention. Calf pain is indicative of thrombophlebitis, a serious, life-threatening complication associated with the use of oral contraceptives which requires further assessment and possibly immediate medical intervention (C). (A) are symptoms of oral contraceptive use, but are of less immediacy than (C). (B) may cause an embolism if thrombophlebitis is present. By not seeking immediate attention, (D) is potentially dangerous to the client.

A client has been taking oral corticosteroids for the past five days because of seasonal allergies. Which assessment finding is of most concern to the nurse? A) White blood count of 10,000 mm3. B) Serum glucose of 115 mg/dl. C) Purulent sputum. D) Excessive hunger.

C) Purulent sputum. Steroids cause immunosuppression, and a purulent sputum (C) is an indication of infection, so this symptom is of greatest concern. Oral steroids may increase (A) and often cause (D). (B) may remain normal, borderline, or increase while taking oral steroids.

The nurse is completing an admission interview for a client with Parkinson's disease. Which question will provide additional information about manifestations that the client is likely to experience? A. "Have you ever experienced any paralysis of your arms or legs?" B. "Do you have frequent blackout spells?" C. "Have you ever been frozen in one spot, unable to move?" D. "Do you have headaches, especially ones with throbbing pain?"

C. "Have you ever been frozen in one spot, unable to move?" Rationale: Clients with Parkinson's disease frequently experience difficulty in initiating, maintaining, and performing motor activities. They may even experience being rooted to the spot and unable to move (C). Parkinson's disease does not typically cause (A, B, or D).

A home health nurse is visiting a client with a history of heart failure (HF). When interviewing the client, which question provides the most useful information for the nurse? A. "Have you been weighing yourself once a month?" B. "Have you had any headaches lately?" C. "How many pillows do you sleep on at night?" D. "How much caffeine are you drinking?"

C. "How many pillows do you sleep on at night?"

The nurse is evaluating the health teaching of a female client with condyloma acuminata. Which statement by the client indicates that teaching has been effective? A. "These warts are caused by a fungus" B. "Early treatment is very effective" C. "I need to have regular pap smears" D. "I will clean my hot tub better"

C. "I need to have regular pap smears"

51. Which of the following statements made by a patient with COPD indicates a need for further education regarding the use of an ipratropium inhaler? A. "I should rinse my mouth following the two puffs to get rid of the bad taste." B. "I should wait at least 1 to 2 minutes between each puff of the inhaler." C. "If my breathing gets worse, I should keep taking extra puffs of the inhaler until I can breathe more easily." D. "Because this medication is not fast-acting, I cannot use it in an emergency if my breathing gets worse.

C. "If my breathing gets worse, I should keep taking extra puffs of the inhaler until I can breathe more easily." The patient should not take extra puffs of the inhaler at will to make breathing easier. Excessive treatment could trigger paradoxical bronchospasm, which would worsen the patient's respiratory status.

The home health nurse is assessing a male client being treated for Parkinson disease with levodopa-carbidopa (Sinemet). The nurse observes that he does not demonstrate any apparent emotions when speaking and rarely blinks. Which intervention should the nurse implement? A. Perform a complete cranial nerve assessment. B. Instruct the client that he may be experiencing medication toxicity. C. Document the presence of these assessment findings. D. Advise the client to seek immediate medical evaluation.

C. A mask-like expression and infrequent blinking are common clinical features of Parkinsonism. The nurse should document the findings. (A & D) are not necessary. Signs of toxicity (B) are dyskinesia, hallucinations, and psychosis.

A male client with bipolar disorder has difficulty concentrating and plans to attend group for the first time. He tells the nurse that he will try to stay for the music relaxation group. After 20 minutes in the group, he becomes restless and begins to leave. What should the nurse do? A. Ask the client to stay until the end B. Encourage the client to go to another group C. Allow the client to leave the group D. Offer the client an antianxiety medication

C. Allow the client to leave the group

A client admitted with an acute myocardial infraction receives a cardiac diet with sodium restriction and complains that his hamburger is flavorless. Which condiment should the nurse offer? A. Pickle relish B. Steak sauce C. Fresh horseradish D. Tomato ketchup

C. Fresh horseradish

An infant has a medical diagnosis of tracheosophageal fistula (TEF). Which nursing intervention is indicated for this infant prior to surgical repair? A. Administer isotonic enemas as prescribed B. Evaluate the infant's tolerance for small volume of formula C. Maintain suction equipment available at all times D. Prepare the child for a barium enema to correct the condition

C. Maintain suction equipment available at all times

The nurse identifies which recent event as placing a client at high risk for cardiogenic shock? A. Gunshot wounds to the chest and abdomen B. Traumatic amputation of the leg at the groin C. Myocardial infraction in the right ventricle D. Multiple bee stings around the head and neck

C. Myocardial infraction in the right ventricle

When reviewing the results of a 83-year-old patient's blood tests, which of the following findings would be of most concern to the nurse? A. Platelets of 150,000/µl B. Serum iron of 50 mcg/dl C. Partial thromboplastin time (PTT) of 60 seconds D. Erythrocyte sedimentation rate (ESR) of 35 mm in 1 hour

C. Partial thromboplastin time (PTT) of 60 seconds In aging, the partial thromboplastin time (PTT) is normally decreased, so an abnormally high PTT of 60 seconds is an indication that bleeding could readily occur. Platelets are unaffected by aging, and 150,000 is a normal count. Serum iron levels are decreased and the erythrocyte sedimentation rate (ESR) is significantly increased with aging, as are reflected in these values.

The nurse administers the opioid antagonist naloxone HCL (Narcan) to a young adult client who overdosed on hydromorphone (Dilaudid). Which assessment data indicates that the naloxone is effective? A. Babinski reflex changes from negative to positive B. Pupil size increases from 4 mm to 6 mm C. Respiration increases from 8/minute to 12/minute D. Blood pressure decrease from 140/94 to 120/78

C. Respiration increases from 8/minute to 12/minute

A postoperative client receives a Schedule II opioid analgesic for pain. Which assessment finding requires the most immediate intervention by the nurse? A. Hypoactive bowel sounds with abdominal distention B. Client reports continued pain of 8 on a 10-point scale C. Respiratory rate of 12 breaths/min, with O2 saturation of 85% D. Client reports nausea after receiving the medication

C. Respiratory rate of 12 breaths/min, with O2 saturation of 85% Rationale: Administration of a Schedule II opioid analgesic can result in respiratory depression (C), which requires immediate intervention by the nurse to prevent respiratory arrest. (A, B, and D) require action by the nurse but are of less priority than (C).

The nurse includes frequent oral care in the plan of care for a client scheduled for an esophagogastrostomy for esophageal cancer. This intervention is included in the client's plan of care to address which nursing diagnosis? A. Fluid volume deficit B. Self-care deficit C. Risk for infection D. Impaired nutrition

C. Risk for infection Rationale: The primary reason for performing frequent mouth care preoperatively is to reduce the risk of postoperative infection (C) because these clients may be regurgitating retained food particles, blood, or pus from the tumor. Meticulous oral care should be provided several times a day before surgery. Although oral care will be of benefit to the client who may also be experiencing (A, B, or D), these problems are not the primary reason for the provision of frequent oral care.

Which data would the nurse expect to find when reviewing laboratory values of an 80-year-old man who is in good health overall? A.Complete blood count reveals increased white blood cell (WBC) and decreased red blood cell (RBC) counts. B.Chemistries reveal an increased serum bilirubin level with slightly increased liver enzyme levels. C.Urinalysis reveals slight protein in the urine and bacteriuria, with pyuria. D.Serum electrolytes reveal a decreased sodium level and increased potassium level.

C. Urinalysis reveals slight protein in the urine and bacteriuria, with pyuria. Rationale: In older adults, the protein found in urine slightly rises, probably as a result of kidney changes or subclinical urinary tract infections, and clients frequently experience asymptomatic bacteriuria and pyuria as a result of incomplete bladder emptying. Laboratory findings in options A, B, and D are not considered to be normal findings in an older adult.

The nurse is preparing a client for a scheduled cesarean section. In which order should the nurse perform these actions? A. Prepping the site with iodophor (Betadine) B. Inserting an indwelling catheter C. Verifying the consent form is signed D. Performing a time-out procedure

C. Verifying the consent form is signed B. Inserting an indwelling catheter A. Prepping the site with iodophor (Betadine) D. Performing a time-out procedure

11. If a patient has pernicious anemia, the nurse should provide information regarding A. frequent bouts of dyspnea. B. risks relative to dehydration. C. deficiency of intrinsic factor. D. lack of any effective treatment for this condition.

C. deficiency of intrinsic factor. Pernicious anemia is a type of anemia caused by failure of absorption of vitamin B12 (cobalamin). The most common cause is lack of intrinsic factor, a glucoprotein produced by the parietal cells of the gastric lining.

A client with a new diagnosis of Raynaud's disease lives alone. Which instruction should the nurse include in the client's discharge teaching plan? A. have a caregiver for 8 hours daily. B. develop a walking exercise routine C. keep room temperature at 80F D. Wear TED stockings at night

C. keep room temperature at 80F keeping the environment warm may minimize vasoconstriction, which decreases blood flow and causes the pain associated with Raynaud's disease.

A client is ready for discharge following the creation of an ileostomy. Which instruction should the nurse include in discharge teaching? A. Replace the stoma appliance every day. B.Use warm tap water to irrigate the ileostomy. C.Change the bag when the seal is broken. D.Measure and record the ileostomy output.

C.Change the bag when the seal is broken. Rationale: A seal must be maintained to prevent leakage of irritating liquid stool onto the skin. Option A is excessive and can cause skin irritation and breakdown. Ileostomies produce liquid fecal drainage, so option B is not necessary. Option D is not needed.

A client in the emergency department is bleeding profusely from a gunshot wound to the abdomen. What action should the nurse immediately take to promote maintenance of the client's blood pressure above a systolic pressure of 90 mm Hg? A. Place the client in a 45-degree Trendelenburg position to promote cerebral blood flow. B. Turn the client prone to place pressure on the abdominal wound to help staunch the bleeding. C. Maintain the client in a supine position to reduce diaphragmatic pressure and visualize the wound. D. Put the client on the right side to apply pressure to the liver and spleen to stop hemorrhaging.

C.Maintain the client in a supine position to reduce diaphragmatic pressure and visualize the wound. Rationale: Placing the client in a supine position reduces diaphragmatic pressure, thereby enhancing oxygenation, and allows for visualization of the abdominal wound. Option A compromises diaphragmatic expansion and inhibits pressoreceptor activity. Option B places the client at risk of evisceration of the abdominal wound and increased bleeding. Option D will not stop internal bleeding in the liver and spleen caused by the gunshot wound.

A client with pernicious anemia takes supplemental folate and self administer monthly vitamin B12 injections. The client reports feeling increasingly fatigued. Which laboratory value should the nurse review?

CBC (Complete blood count)

A nurse is collecting data from a client who has AIDS.When checking the client's mouth, the nurse notes a white, creamy covering on the tongue and buccal membranes. The nurse should recognize this is a manifestation of which of the following conditions? 1) Xerostomia 2) Gingivitis 3) Candidiasis 4) Halitosis

Cadidiasis

A nurse is reinforcing dietary teaching with a client who has late-stage chronic kidney disease (CKD). Which of the following nutrients should the nurse instruct the client to increase in her diet?

Calcium

During report an older female client, who had a right total hip replacement three days ago, is described as intolerant of pain. When assessing the client, the nurse finds that the client can't straighten her right foot which is pointing outward from the body. Which action should the nurse take?

Call the healthcare provider immediately and report that the client's prosthesis is most likely dislocated.

A nurse is preparing to provide morning hygiene care for a client who has Alzheimer's disease. The client becomes agitated and combative when the nurse approaches him. Which of the following actions should the nurse plan to take? 1) Turn the water on and ask the client to test the temperature. 2) Obtain assistance to place mitten restraints on the client. 3) Firmly tell the client that good hygiene is important. 4) Calmly ask the client if he would like to listen to some music

Calmly ask the client if he would like to listen to some music

An older adult male is brought to the ED by his daughter because he has been experiencing chest pain for the last 4 hours. He is diagnosed with ST elevation myocardial infarction (STEMI) and was admitted 3 weeks ago for gastrointestinal bleeding. Until the client can be transferred to the percutaneous coronary intervention unit, which assessment is most important for the nurse to monitor?

Cardiac rate and rhythm

An adult client is admitted with flank pain and is diagnosed with acute pyelonephritis. What is the priority nursing action?

administer IV antibiotics as prescribed

A client diagnosed with bacterial meningitis is admitted to the unit and is prescribed neuro checks every 2 hours. Which manifestation would the practical nurse monitor for that would provide the first indication of altered neurological function?

Change in level of consciousness A decrease or change in the level of consciousness is usually the first indication of neurological deterioration.

A nurse is caring for a client who is receiving peritoneal dialysis. The nurse notes that dialysate output is less than input, and the client's abdomen is distended. Which of the following actions should the nurse take?

Change the client's position

A nurse is caring for a client immediately following a cardiac catheterization with a femoral artery approach. Which of the following actions should the nurse take? 1) Check pedal pulses every 15 min. 2) Perform passive range-of-motion for the affected extremity. 3) Remind the client not to turn from side to side. 4) Keep the client in high-Fowler's position for 6 hr.

Check pedal pulses every 15 min

Pathology in which part of the ear results in sensorineural hearing loss?

Cochlear apparatus

An older female client has normal saline infusion at 45 mL/hour. She complains of pain at the insertion site of the IV catheter. There is no redness or edema around the IV site. Which action should the nurse take?

Consult with the health care provider about the best location to start a new IV

A client with partial thickness and full thickness burns over 50% of the body is admitted to the ED. The healthcare provider prescribes hydromorphone 4 mg IM every 4 hours for pain. The client is complaining of 9 on 10 scale. Which intervention should the nurse implement first?

Contact the provider and question the prescription

While performing assisted range of motion exercises for a client with osteoarthritis, the nurse notes joint crepitus. Which action should the nurse take?

Continue the range of motion exercises

A client diagnosed with acute respiratory distress syndrome (ARDS) is receiving oxygen 8 L/min via nasal cannula and has these arterial blood gases: pH 7.35, pCO2 45, pO2 75, HCO3 24. Based on these ABGs, which intervention should the nurse implement?

Continue to monitor the client

A client who suffered an electrical injury with the entrance site on the left hand and the exit site on the left foot is admitted to the burn unit. Which intervention is most important for the nurse to include in this client's plan of care?

Continuous cardiac monitoring

While changing a client's postoperative dressing, the nurse observes purulent drainage at the wound. Before reporting this finding to the healthcare provider, the nurse should note which of the client's laboratory values?

Culture for sensitive organisms

After receiving lactulose, a client with hepatic encephalopathy has several loose stools. What action should the nurse implement? A. Send a stool specimen to the lab B. Measure abdominal girth C. Encourage increased fiber in diet D. Monitor mental status

D. Monitor mental status Lactulose is administered to clients with hepatic encephalopathy to lower serum ammonia levels, which should improve the client's mental status

What is the correct procedure for performing an ophthalmoscopic examination on a client's right retina? A) Instruct the client to look at examiner's nose and not move his/her eyes during the exam. B) Set ophthalmoscope on the plus 2 to 3 lens and hold it in front of the examiner's right eye. C) From a distance of 8 to 12 inches and slightly to the side, shine the light into the client's pupil. D) For optimum visualization, keep the ophthalmoscope at least 3 inches from the client's eye.

D) For optimum visualization, keep the ophthalmoscope at least 3 inches from the client's eye. The client should focus on a distant object in order to promote pupil dilation. The ophthalmoscope should be set on the 0 lens to begin (creates no correction at the beginning of the exam), and should be held in front of the examiner's left eye when examining the client's right eye. For optimum visualization, the ophthalmoscope should be kept within one to three inches of the client's eye (D). (A and B) describe incorrect methods for conducting an ophthalmoscopic examination. (C) should illicit a red reflex as the light travels through the crystalline lens to the retina.

A client with a completed ischemic stroke has a blood pressure of 180/90 mm Hg. Which action should the nurse implement? A) Position the head of the bed (HOB) flat. B) Withhold intravenous fluids. C) Administer a bolus of IV fluids. D) Give an antihypertensive medications.

D) Give an antihypertensive medications. Most ischemic strokes occur during sleep when baseline blood pressure declines or blood viscosity increases due to minimal fluid intake. Completed strokes usually produce neurologic deficits within an hour, the client's current elevated blood pressure requires antihypertensive medication (D). Positioning the HOB flat (A) decreases venous drainage and contributes to cerebral edema post stroke. Increased blood viscosity during sleep may be related to reduced fluids, so (B) is not indicated. Increasing the vascular fluid volume increases the blood pressure, so (C) is not indicated.

A client with gastroesophageal reflux disease (GERD) has been experiencing severe reflux during sleep. Which recommendation by the nurse is most effective to assist the client? A) Losing weight. B) Decreasing caffeine intake. C) Avoiding large meals. D) Raising the head of the bed on blocks.

D) Raising the head of the bed on blocks. Raising the head of the bed on blocks (D) (reverse Trendelenburg position) to reduce reflux and subsequent aspiration is the most effective recommendation for a client experiencing severe gastroesophageal reflux during sleep. (A, B and C) may be effective recommendations but raising the head of the bed is more effective for relief during sleep.

Dysrhythmias are a concern for any client. However, the presence of a dysrhythmia is more serious in an elderly person because A) elderly persons usually live alone and cannot summon help when symptoms appear. B) elderly persons are more likely to eat high-fat diets which make them susceptible to heart disease. C) cardiac symptoms, such as confusion, are more difficult to recognize in the elderly. D) elderly persons are intolerant of decreased cardiac output which may result in dizziness and falls.

D) elderly persons are intolerant of decreased cardiac output which may result in dizziness and falls. Cardiac output is decreased with aging (D). Because of loss of contractility and elasticity, blood flow is decreased and tachycardia is poorly tolerated. Therefore, if an elderly person experiences dysrhythmia (tachycardia or bradycardia), further compromising their cardiac output, they are more likely to experience syncope, falls, transient ischemic attacks, and possibly dementia. Most elderly persons do not eat high-fat diets (B) and most are not confused (C). Although many elderly persons do live alone, inability to summon help (A) cannot be assumed.

The charge nurse in the Labor and Delivery Unit makes assignments for a nurse and unlicensed assistive personnel (UAP). A client in labor is admitted with contractions occurring every 3 to 5 minutes. Which task should be assigned to the UAP? A. Teach patterned breathing B. Apply external fetal monitor C. Measure the fundal height D. Collet a urine specimen

D. Collet a urine specimen

The nurse witnesses a baseball player receive a blunt trauma to the back of the head with a softball. What assessment data should the nurse collect immediately? A. Reactivity of deep tendon reflexes, comparing upper with lower extremities B. Vital sign readings, excluding blood pressure if needed equipment is unavailable C. Memory of events that occurred before and after the blow to the head D. Ability to open the eyes spontaneously before any tactile stimuli are given

D. Ability to open the eyes spontaneously before any tactile stimuli are given Rationale: The level of consciousness (LOC) should be established immediately when a head injury has occurred. Spontaneous eye opening (D) is a simple measure of alertness that indicates that arousal mechanisms are intact. (A) is not the best indicator of LOC. Although (B) is important, vital signs are not the best indicators of LOC and can be evaluated after the client's LOC has been determined. (C) can be assessed after LOC has been established by assessing eye opening.

The nurse assesses the perineum of a client who is complaining of perineal pain 6 hours after a normal delivery, and finds small perineal (vulvar) hematomas. Based on this assessment finding, which treatment should the nurse implement? A. Cleanse the area with warm water B. Prepare for surgical excision C. Spray topical analgesic to the perineum D. Apply ice packs on the perineum

D. Apply ice packs on the perineum

The nurse is preparing a 45-year-old client for discharge from a cancer center following ileostomy surgery for colon cancer. Which discharge goal should the nurse include in this client's discharge plan? A. Reduce the daily intake of animal fat to 10% of the diet within 6 weeks. B. Exhibit regular, soft-formed stool within 1 month. C. Demonstrate the irrigation procedure correctly within 1 week. D. Attend an ostomy support group within 2 weeks.

D. Attend an ostomy support group within 2 weeks. Rationale: Attending a support group (D) will be beneficial to the client and should be encouraged because adaptation to the ostomy can be difficult. This goal is attainable and is measurable. (A) is not specifically related to ileostomy care. The client with an ileostomy will not be able to accomplish (B). (C) is not necessary.

A 3-year-old boy is brought to the emergency department after the mother found the child in the back yard holding a piece of a toy in his hand and in respiratory distress. The child is dusky with a loud, inspiratory stridor and weak attempts to cough. Which actions should the nurse implement? A. Obtain a pulse oximetry reading and arterial blood gases B. Request a start chest x-ray and prepare medications for an asthmatic episode C. Determine if the child ingested a toxic substance and if vomiting occurred D. Auscultate all pulmonary lung fields and attempt a Heimlich maneuver

D. Auscultate all pulmonary lung fields and attempt a Heimlich maneuver

When planning appropriate nursing interventions for a patient with metastatic lung cancer and a 60-pack-year history of cigarette smoking, the nurse recognizes that the smoking has most likely decreased the patient's underlying respiratory defenses because of impairment of which of the following? A. Reflex bronchoconstriction B. Ability to filter particles from the air C. Cough reflex D. Mucociliary clearance

D. Mucociliary clearance Smoking decreases the ciliary action in the tracheobronchial tree, resulting in impaired clearance of respiratory secretions, chronic cough, and frequent respiratory infections.

When developing a discharge teaching plan for a client after the insertion of a permanent pacemaker, the nurse writes a goal of "The client will verbalize symptoms of pacemaker failure." Which behavior indicates that the goal has been met? A.The client demonstrates the procedures to change the rate of the pacemaker using a magnet. B.The client carries a card in his wallet stating the type and serial number of the pacemaker. C. The client tells the nurse that it is important to report redness and tenderness at the insertion site. D.The client states that changes in the pulse and feelings of dizziness are significant changes.

D. The client states that changes in the pulse and feelings of dizziness are significant changes. Rationale: Changes in pulse rate and/or rhythm may indicate pacer failure. Feelings of dizziness may be caused by a decreased heart rate, leading to decreased cardiac output. The rate of a pacemaker is not changed by a client, although the client may be familiar with this procedure as explained by his health care provider. Option B is an important step in preparing the client for discharge but does not demonstrate knowledge of the symptoms of pacer failure. Option C indicates symptoms of possible incisional infection or irritation but does not indicate pacer failure.

The nurse in the outpatient department is caring for a client who had a right femoral cardiac catheterization two hours ago. What assessment finding requires immediate intervention? A. The client's right foot is warm to touch B. The client's blood pressure is 110/70 and pulse 88 C. The client's pulse oximeter reading is 98% D. The client wants assistance walking to the bathroom

D. The client wants assistance walking to the bathroom

An adult is brought to the emergency department by ambulance following a motorcycle accident. He was not wearing a helmet and presents with periorbital bruising and bloody drainage from both ears. Which assessment finding warrants immediate intervention by the nurse? A. rebound abdominal tenderness B. diminished bilateral breath sounds C. rub pain with deep inspiration D. nausea with projectile vomiting

D. nausea with projectile vomiting

The most appropriate position to assist a patient with chronic obstructive pulmonary disease (COPD) who is having difficulty breathing would be a A. high Fowler's position without a pillow behind the head. B. semi-Fowler's position with a single pillow behind the head. C. right side-lying position with the head of the bed at 45 degrees' elevation. D. sitting upright and forward position with arms supported on an over-the-bed table.

D. sitting upright and forward position with arms supported on an over-the-bed table.Sitting upright and leaning forward with arms supported on an over-the-bed table would be of most help to this patient, because it allows for expansion of the thoracic cage in all four directions (front, back, and two sides).

15. When administering oxygen to a patient with COPD with the potential for carbon dioxide narcosis, the nurse should A. never administer oxygen at a rate of more than 2 L/min. B. monitor the patient's use of oxygen to detect oxygen dependency. C. monitor the patient for symptoms of oxygen toxicity, such as paresthesias. D. use ABGs as a guide to determine what FIO2 level meets the patient's needs.

D. use ABGs as a guide to determine what FIO2 level meets the patient's needs. It is critical to start oxygen at low flow rates and then use ABGs as a guide to determine what FIO2 level is sufficient and can be tolerated.

Which statement reflects the highest priority nursing diagnosis for an older client recently admitted to the hospital for a new-onset cardiac dysrhythmia? A.Diarrhea related to medication side effects B.Anxiety related to fear of recurrent anginal episodes C.Altered nutrition related to high serum lipid levels D.Risk for injury related to syncope and confusion

D.Risk for injury related to syncope and confusion Rationale: The loss of cardiac function in aging decreases cardiac output, so dysrhythmias, particularly tachycardias, are poorly tolerated. With onset of a tachycardic or bradycardic dysrhythmia, cardiac output is compromised further, placing the client at risk of syncope and falling, as well as confusion. Option A is of high priority but less so than maintaining client safety. Clients may experience option B as a result of a newly diagnosed cardiac condition, but this nursing diagnosis does not have the priority of option D. Option C also does not have the priority of option D.

A nurse is caring for a client who has Parkinson's disease and is taking selegiline 5 mg by mouth twice daily. Which of the following therapeutic outcomes should the nurse monitor for with a client who is taking this medication? 1) Improved speech patterns 2) Increased bladder function. 3) Decreased tremors 4) Diminished drooling

Decreased tremors

A nurse is caring for a client newly diagnosed with ovarian cancer. Which of the following reactions from the client should the nurse initially expect? 1) Denial 2) Bargaining 3) Acceptance 4) Anger

Denial

A client is admitted to the emergency department with flu like symptoms. Which information should the nurse obtain to rule out exposure to anthrax spores?

Determine the client's occupation

A postoperative client reports incisional pain. The client has two prescriptions for PRN analgesia that accompanied the client from the postanesthesia unit. Before selecting which medication to administer, which action should the nurse implement?

Determine which prescription will have the quickest onset of action

Which common physiological change that occurs with aging is likely to influence an older adult's nutritional status?

Diminished sense of smell

The nurse is preparing discharge instructions for an older female client who is going home after a hip replacement. The client tells the nurse that she can hardly wait to get home and see her cat. Which instruction regarding the cat is most critical to the client's safety at home?

Do not bend over and pick up the cat. Teach him to jump up so that you can reach him

A client diagnosed with a fracture of the (L) radius has a plaster cast applied. Which instruction should the practical nurse give the client to dry the cast over the next 24 hours?

Do not cover and keep the cast surfaces exposed to circulating air. The nurse should instruct the client to keep the cast exposed to circulating air and avoid covering it with material that might keep it moist.

The nurse measures the PR interval on a client telemetry strips as 0.16 seconds. Which action should the nurse take in response to this finding?

Document the finding

During a home visit, the nurse should evaluate the effectiveness of a client's treatment for chronic obstructive pulmonary disease by assessing for which primary symptom?

Dyspnea

A nurse is collecting data from a client who has an exacerbation of gout. Which of the following findings should the nurse expect? (Select all that apply.) 1) Edema 2) Erythema 3) Tophi 4) Tight skin 5) Symmetrical joint pain

Edema Erythema Tophi TIght skin

A client is diagnosed with acute myocardial infarction (MI). Which diagnostic laboratory value should the practical nurse (PN) anticipate to be the first to elevate to establish a diagnosis of an acute myocardial infarction (MI)?

Elevated troponin Tissue damage in the myocardium causes the release of cardiac enzymes into the blood system. According the American College of Cardiology (ACS) and the European Society of cardiology (ESC), an elevation of the troponin will occur within 2 to 3 hours of an MI and is use to establish the diagnosis. It takes the CK-MB level 6 to 9 hours or longer to elevate.

A nurse is caring for a client who is postoperative open reduction and internal fixation with placement of a wound drain to repair a hip fracture. Which of the following actions should the nurse take? 1) Empty the suction device every 4 hr. 2) Monitor circulation on the affected extremity every 2 hr for the first 12 hr. 3) Position the client's hip so that it is internally rotated. 4) Encourage foot exercises every 4 hr.

Empty the suction device every 4 hours

A nurse is assisting with the care of a client immediately following a lumbar puncture. Which of the following actions should the nurse take? (Select all that apply.) 1) Encourage fluid intake. 2) Monitor the puncture site for hematoma. 3) Insert a urinary catheter. 4) Elevate the client's head of bed. 5) Apply a cervical collar to the client.

Encourage fluid intake Monitor the puncture site for hematoma

A nurse is assisting with the care of a client who has a femur fracture and is in skeletal traction. Which of the following actions should the nurse take? 1) Loosen the knots on the ropes if the client is experiencing pain. 2) Ensure the client's weights are hanging freely from the bed. 3) Check the client's bony prominences every 12 hr. 4) Cleanse the client's pin sites with povidone-iodine.

Ensure the client's weights are hanging freely from the bed

A nurse is collecting data from a client who is having an acute asthma exacerbation. When auscultating the client's chest, the nurse should expect to hear which of the following sounds? 1) Expiratory wheeze 2) Pleural friction rub 3) Fine rales 4) Rhonchi

Expiratory wheeze

After intubating a client correct placement of the endotracheal tube (ETT) is confirmed with a chest x ray. Which intervention should the nurse implement to ensure that the ETT placement is maintained?

Firmly secure the ETT in place

A nurse is assisting with the care of a client who is postoperative and has a closed-wound drainage system in place. Which of the following actions should the nurse take? 1) Fully recollapse the reservoir after emptying it. 2) Empty the reservoir once per day. 3) Replace the drainage plug after releasing hand pressure on the device. 4) Irrigate the tubing with sterile normal saline solution at least once every 8 hr.

Fully recollapse the resevoir after emptying it

A client's telemetry monitor indicates ventricular fibrillation (VF). After delivering one counter shock, the nurse resumes chest compression. After another minute of compressions, the client's rhythm converts to supraventricular tachycardia (SVT) on the monitor. At this point, what is the priority intervention for the nurse?

Give IV dose of adenosine rapidly over 1-2 seconds

-A client with partial thickness burns to the lower extremities is scheduled for whirlpool therapy to debride the burned area. Which intervention should the nurse implement before transporting the client to the physical therapy department?

Give a prescribed narcotic analgesic agent.

A nurse is collecting data on a client's wound. The nurse observes that the wound surface is covered with soft,red tissue that bleeds easily.The nurse should recognize this is a manifestation of which of the following? 1) Decreased perfusion 2) Infection 3) Granulation tissue 4) An inflammatory response

Granulation tissue

Which foods should the practical nurse encourage a client to eat to increase their potassium intake? (Select all that apply.)

Green beans Milk Flounder Sweet potatoes Cantaloupe Flounder, sweet potatoes, milk, green beans, and cantaloupe are all potassium-rich foods.

A nurse is collecting data from a client who has scleroderma. Which of the following findings should the nurse expect? 1) A dry raised rash 2) Excessive salivation 3) Periorbital edema 4) Hardened skin

Hardened skin

A client diagnosed with a brain tumor is receiving radiation beam treatments to the right frontal area. The practical nurse (PN) should observe this client for which problem during the early post-therapy days?

Headache Radiotherapy is a local treatment, and most side effects are site-specific, such as inflammation of surrounding brain tissue, swelling, headache, and fatigue.

A nurse is caring for an older adult client who has colon cancer. The client asks the nurse several questions about his treatment plan. Which of the following actions should the nurse take? 1) Tell the client to have a family member call the provider to ask what options he plans to recommend. 2) Assure the client that the provider will tell him what is planned. 3) Help the client write down questions to ask his provider. 4) Provide the client with a pamphlet of information about cancer.

Help the client write down questions to ask his provider

.A nurse is assisting with teaching a client who has a history of smoking about recognizing early manifestations of laryngeal cancer. The nurse should instruct the client to monitor and report which of the following manifestations of laryngeal cancer? 1) Aphagia 2) Hoarseness 3) Tinnitus 4) Epistaxis

Hoarseness

A client diagnosed with a seizure disorder is prescribed phenobarbital. While attempting to administer the medication to the client, the practical nurse has difficulty trying to arouse the sleepy client. What nursing action should the practical nurse implement?

Hold the medication and notify the health care provider. The client is exhibiting signs of antiepileptic drug (AED) toxicity, and a serum phenobarbital level needs to be obtained to determine if the client is experiencing drug toxicity.

A nurse is collecting data from a client who is 6 days post craniotomy for removal of an intracerebral aneurysm. The nurse should monitor the client for which of the following manifestations of increased intracranial pressure? 1) Decreased pedal pulses 2) Hypertension 3) Peripheral edema 4) Diarrhea

Hypertension

In an effort to respond to an increasing myocardial workload, the cells of the left ventricle increase in size. Which term best describes the left ventricle's adaptive change?

Hypertrophy

A nurse is monitoring a client who has syndrome of inappropriate antidiuretic secretion (SIADH). Which of the following findings should the nurse expect?

Hyponatremia -Rationale: the client who has SIADH will have hyponatremia caused by the excessive release of ADH. As a result of the excessive secretion the client retains water

A nurse is assisting with the care of a client who has diabetes insipidus. The nurse should monitor the client for which of the following manifestations? 1) Hypotension 2) Polyphagia 3) Hyperglycemia 4) Bradycardia

Hypotension

A nurse is caring for a client who is postoperative and has a history Addison's disease. For which of the following manifestations should the nurse monitor? 1) Hypernatremia 2) Hypotension 3) Bradycardia 4) Hypokalemia

Hypotension

A nurse is reviewing the laboratory results of a client who is taking cyclosporine following a kidney transplant. Which of the following laboratory findings should the nurse identify as the most important to report to the provider? 1) Increase in serum glucose 2) Increase in serum creatinine 3) Decrease in white blood cell count 4) Decrease in platelets

Increase in serum creatinine

A nurse is assessing a client who has manifestations of acromegaly. Which of the following findings should the nurse expect?

Increased head size -Rationale: enlarged head size due to excessive production of growth hormones after the closing of the epiphyses.

A nurse is caring for a client who has type 2 diabetes mellitus and is displaying manifestations of hyperglycemia. Which of the following findings should indicate to the nurse that the client has hyperglycemia?

Increased urination -Rationale: increased urination/polyuria, is a manifestation of hyperglycemia due to a deficiency of insulin, which can lead to osmotic diuresis

A nurse is caring for a client after a radical neck dissection. To which of the following should the nurse give priority in the immediate postoperative period? 1) Malnourishment related to NPO status and dysphagia 2) Impaired verbal communication related to the tracheostomy 3) High risk for infection related to surgical incisions 4) Ineffective airway clearance related to thick, copious secretions

Ineffective airway clearance related to think, copious secretions

The practical nurse (PN) is reinforcing colostomy care teaching to a client who is three days following placement of their colostomy. The client asked the PN why is it necessary to measure the colostomy's stoma each time when changing into a new appliance/wafer. What is the best response by the PN?

Inform the client that the stoma will become smaller when the initial swelling diminishes. Post-operative swelling causes enlargement of the stoma. The PN needs to reinforce to the client that the stoma will become smaller when the swelling is diminished. This is necessary in order to prevent irritation to the surrounding skin from the colostomy's drainage (effluent). The purpose of the colostomy appliance is to provide a protective barrier surrounding the stomal skin, along with containing effluent from the colostomy.

A client is admitted with a deep and productive cough, hemoptysis, and a low grade fever. The client's Mantoux skint test has a 15 mm induration. Which intervention should the nurse implement first?

Initiate airborne particulate isolation precautions

A nurse is reinforcing teaching with a client who has a new prescription for epoetin alfa. The nurse should reinforce to the client to take which of the following dietary supplements with this medication? 1) Vitamin D 2) Vitamin A 3) Iron 4) Niacin

Iron

In assessing a client with ulcers on the lower extremity, which finding indicate that the ulcers are likely to be venous, rather than arterial, origin?

Irregular ulcer shapes and severe edema

An adult male client is admitted for Pneumocystis jiroveci pneumonia secondary to AIDS. While hospitalized, the client receives IV pentamidine therapy. In preparing this client for discharge, which important aspect regarding his medication therapy the nurse explain?

It will be necessary to continue prophylactic doses of IV or aerosol pentamidine every month

An adult client who received partial thickness and full thickness burns over 40% of the body in a house fire is admitted to the inpatient burn unit. Which fluid should the nurse prepare to administer during the acute phase of the client's burn recovery?

Lactate Ringer's

A client has a prescription for a fentanyl transdermal patch to be applied every 72 hours. The patch was applied 48 hours ago and the client now reports experiencing breakthrough pain. What action should the nurse implement?

Leave the patch in place and administer a prescribed PRN analgesic

A nurse is reinforcing teaching with a client who has chronic kidney disease (CKD). Which of the following instructions should the nurse include?

Limit fluid intake

A nurse is contributing to the plan of care for an older adult client who is postoperative following a right hip arthroplasty. Which of the following interventions should the nurse include in the plan? 1) Perform the client's personal care activities for her. 2) Limit the client's fluid intake. 3) Monitor the Homan's sign. 4) Maintain abduction of the right hip

Maintain abduction of the right hip

An adult male client who was recently diagnose with glaucoma tells the nurse that he fells like he is driving through a tunnel. The client expresses great concern about going blind. Which nursing instruction is most important to provide this client?

Maintain prescribed eye drop regimen

A nurse is contributing to the plan of care for a client who has an intestinal obstruction and is receiving continuous gastrointestinal decompression using a nasogastric tube. Which of the following interventions should the nurse include in the plan of care? 1) Measure abdominal girth daily. 2) Use sterile water to irrigate the nasogastric tube.. 3) Maintain the client in Fowler's position. 4) Moisten the client's lips with lemon-glycerin swabs.

Maintain the client in Fowler's position

A client is admitted from the Emergency Department with a diagnosis of (L) tibia fracture and the (L) leg has a splint in place. The client was medicated approximately 2 hours ago with a prescribed analgesic. The client is now complaining of excruciating leg pain and demanding "stronger pain medications." What initial action is most important for the practical nurse (PN) to take?

Measure the pulse volume and capillary refill distal to the fracture. The PN needs to measure the pulse volume and capillary refill distal to the fracture. Pain and diminished pulse volume are signs of compartment syndrome, which can progress to complete loss of the peripheral pulse in the extremity. Compartment syndrome occurs when external pressure (usually from a cast) or internal pressure after an injury resulting in inflammation and edema building up in the fascia space of the muscle which results in the pressure in this space building up and stopping the flow of blood to the tissues in the compartment. This is a medical emergency requiring a fasciotomy to relieve the pressure.

A nurse is planning a community health screening for a group of clients who are at risk for type 2 diabetes mellitus. Which of the following clients should the nurse include in the screening?

Men and women who are obese

Methotrexate is prescribed for a client with rheumatoid arthritis who is also taking aspirin. What is the best explanation for the nurse to provide as to why a second medication has been added?

Methotrexate slows the disease progression while aspirin controls the symptoms.

A client diagnosed with diabetes complains to the practical nurse of decreased tactile sensation in their feet and feelings like their feet are on fire sometimes. Which abnormal laboratory finding should the practical nurse (PN) identify that indicates that a client with diabetes needs further evaluation for diabetic nephropathy?

Microalbuminuria Microalbuminuria is the earliest sign of diabetic nephropathy and indicates the need for follow-up evaluation.

The nurse is reviewing blood pressure readings for a group of clients on a medical unit. Which client is at the highest risk for complications related to hypertension?

Middle-aged African-American male who has a serum creatinine level of 2.9 Mg/dL

A nurse is caring for a client who is in the oliguric phase of acute kidney injury. Which of the following actions should the nurse take? 1) Provide a diet high in protein. 2) Provide ibuprofen for retroperitoneal discomfort. 3) Monitor intake and output hourly 4) Encourage the client to consume at least 2 L of fluid daily.

Monitor intake and output hourly

A nurse is contributing to the plan of care for a client who is admitted with a deep vein thrombosis (DVT) of the left leg. Which of the following interventions should the nurse include in the plan? 1) Apply ice to the extremity 2) Monitor platelet levels 3) Restrict oral fluids 4) Administer vasodilating medications

Monitor platelet levels

A nurse is planning care for a client who is experiencing the Somogyi effect and takes intermittent-acting insulin. Which of the following actions should the nurse include in the plan?

Monitor the client's nighttime blood glucose levels

While caring for a client with full thickness burns covering 40% of the body, the nurse observes purulent drainage from the wounds. Before reporting this finding to the HCP, the nurse should evaluate which laboratory value?

Neutrophil count

While assessing the skin on a older adult, the nurse notices hyperpigmented freckles on the client's hands and arms. Which additional nursing assessment are indicated?

No additional assessment needed

A client who experienced partial thickness burns with over 50% BSA 2 weeks ago has several open wounds and develops watery diarrhea. The client's blood pressure is 82/40 mmHg and temperature 96 F. Which action is most important for the nurse to take?

Notify the rapid response team

A client with a productive cough has obtained a sputum specimen for culture as instructed. What is the best initial nursing action?

Observe the color, consistency, and amount of sputum

A client diagnosed with epilepsy is admitted to the unit. What intervention should the practical nurse (PN) implement if the client experiences a seizure?

Observe the length and activity of the seizure The PN should observe the client as they have their seizure. The length of time and movement by the client needs to be observed and then documented once the client is stable. The client should be placed on their side to help prevent aspiration.

The nurse provides dietary instructions about iron rich foods to a client with iron deficiency anemia. Which food selection made by the client indicates a need for additional instructions?

Oranges

The practical nurse has been assigned a client with a history of chronic obstructive pulmonary disease (COPD) who has been admitted to the hospital with a medical diagnosis of pneumonia. Which intervention poses the greatest risk of respiratory depression for a client with a history of COPD should the practical nurse determine is most significant in the development of this client's COPD?

Oxygen administration via nasal cannula 4 L/minute. Clients with COPD drive to breathe is a hypoxic state. Their body becomes use to the high CO2 levels and too much oxygen could cause the client to decrease their respiratory drive to breath. Oxygen administration in clients with COPD needs to be carefully monitored.

A client diagnosed with diabetes has a prescription of 5 units of regular insulin and 15 units of NPH insulin. In which order should the practical nurse prepare to administer the insulin? List in order the nursing actions to be performed.

Perform hand hygiene according to facility policy. Inspect insulin vials for type and expiration dates. Inject 15 units of air into NPH insulin vial. Inject 5 units of air into regular insulin vial. Withdraw 5 units of regular insulin from vial. Withdraw 15 units of NPH insulin from vial. The first nursing action is to perform hand hygiene. The next action is to inspect vials for type and expiration dates and then add 15 units of air to NPH insulin vial. Next add 5 units of air into regular insulin vial, then withdraw 5 units of regular insulin from vial, and withdraw 15 units of NPH insulin from vial. Note that it is important to fill the syringe with regular insulin (shorter acting insulin) first to prevent contamination of the NPH insulin (intermediate-acting insulin).

A nurse is collecting data from a client who has right-sided heart failure. Which of the following findings should the nurse expect? 1) Frothy sputum 2) Dyspnea 3) Orthopnea 4) Peripheral edema

Peripheral edema

A nurse is caring for a client who is difficult to arouse and very sleepy for several hours following a generalized tonic-clonic seizure. Which of the following descriptions should the nurse use when documenting this finding in the medical record? 1) Aura phase 2) Presence of automatisms 3) Postictal phase 4) Presence of absence seizures

Postictal phase

which nursing problem has the highest priority when planning care for a client with Meniere's disease?

Potential for injury related to vertigo

A client in the operating room received succinylcholine. The client is experiencing muscle rigidity and has an extremely high temperature. Which action should the nurse implement?

Prepare ice packs for placement in the client's axillary area

A client who had colon surgery 3 days ago is anxious and requesting assistance to reposition. While the nurse is turning the client, the wound dehiscenses and eviscerates. The nurse moistens an available sterile dressing and places it over the wound. Which intervention should the nurse implement next?

Prepare the client to return to the operating room

A nurse is assisting with the care of a client who is postoperative following surgical repair of a fractured mandible. The client's jaw is wired shut to repair and stabilize the fracture. The nurse should recognize which of the following is the priority action? 1) Relieve the client's pain. 2) Check the client's pressure points for redness. 3) Provide oral hygiene. 4) Prevent aspiration.

Prevent aspiration

A client diagnosed with rheumatoid arthritis is prescribed splints for night time use. Which statement by the client demonstrates to the practical nurse (PN) an accurate understanding of the use of the splints?

Prevention of deformities Splints may be used at night by clients with rheumatoid arthritis to prevent deformities caused by muscle spasms and contractures.

A nurse is contributing to the plan of care for a client who has a spinal cord injury at level C8 who is admitted for comprehensive rehabilitation. Which ofthe following long-term goals is appropriate with regard to the client's mobility? 1) Walk with leg braces and crutches. 2) Drive an electric wheelchair with a hand-control device. 3) Drive an electric wheelchair equipped with a chin-control device. 4) Propel a wheelchair equipped with knobs on the wheels.

Propel a wheelchair equipped with knobs on the wheels

A community hit by a hurricane has suffered mass destruction and flooding. Sewage facilities are non-functioning, and the area is contaminated with human excreta. The nurse is developing a plan of care for clients diagnosed with cholera after an outbreak. Which intervention has the highest priority?

Provide fluid and electrolyte replacement.

Which instruction should the nurse include in the discharge teaching plan for an adult client with hypernatremia?

Review food labels for sodium content.

182.) A client with angina pectoris is experiencing chest pain that radiates down the left arm. The nurse administers a sublingual nitroglycerin tablet to the client. The client's pain is unrelieved, and the nurse determines that the client needs another nitroglycerin tablet. Which of the following vital signs is most important for the nurse to check before administering the medication? 1. Temperature 2. Respirations 3. Blood pressure 4. Radial pulse rate

Rationale: Nitroglycerin acts directly on the smooth muscle of the blood vessels, causing relaxation and dilation. As a result, hypotension can occur. The nurse would check the client's blood pressure before administering the second nitroglycerin tablet. Although the respirations and apical pulse may be checked, these vital signs are not affected as a result of this medication. The temperature also is not associated with the administration of this medication.

206.) A client is receiving baclofen (Lioresal) for muscle spasms caused by a spinal cord injury. The nurse monitors the client, knowing that which of the following is a side effect of this medication? 1. Muscle pain 2. Hypertension 3. Slurred speech 4. Photosensitivity

Rationale: Side effects of baclofen include drowsiness, dizziness, weakness, and nausea. Occasional side effects include headache, paresthesia of the hands and feet, constipation or diarrhea, anorexia, hypotension, confusion, and nasal congestion. Paradoxical central nervous system excitement and restlessness can occur, along with slurred speech, tremor, dry mouth, nocturia, and impotence. **Option 3 is most closely associated with a neurological disorder**

During the shift report, the charge nurse informs a nurse that she has been assigned to another unit for the day. The nurse begins to sigh deeply and tosses about her belongings as she prepares to leave, making it known that she is very unhappy about being floated to the other unit. What is the best immediate action for the charge nurse to take? A. Continue with the shift report and talk to the nurse about the incident at a later time. B. Ask the nurse to call the house supervisor to see if she must be reassigned. C. Stop the shift report and remind the nurse that all staff are floated equally. D. Inform the nurse that her behavior is disruptive to the rest of the staff

Rationale: A Continuing with the shift report (A) is the best immediate action because it allows the nurse who was floated some cooling off time. At a later time (after the nurse has cooled off) the charge nurse should discuss the conduct of the nurse in private. (B) encourages the nurse to shirk the float assignment. (C) is disruptive. Reprimanding the nurse in front of the staff would increase the nurse's hostility, so the nurse should be counseled in private (D).

A nurse is caring for a client during the immediate postoperative period following thoracic surgery. When administering an opioid analgesic for pain, the nurse should explain that the medication should have which of the following effects? 1) Reducing anxiety 2) Increasing blood pressure 3) Increasing coughing 4) Increasing the client's respiratory rate

Reducing anxiety

A nurse is collecting data from a client who is postoperative following a extracorporeal shockwave lithotripsy (ESWL). The nurse should identify that which of the following findings is the priority?

Report of palpitations/dysrhythmias

A nurse is caring for a client who has myasthenia gravis (MG). Which of the following is a complication of MG for which the nurse should monitor? 1) Respiratory difficulty 2) Confusion 3) Increased intracranial pressure 4) Joint pain

Respiratory difficulty

During a clinic visit, a client reports to the practical nurse (PN) that they felt a solid mass in their breast during self-examination, but it was not painful. What instruction should the PN reinforce with the client?

Schedule an appointment with the health care provider for evaluation. A painless breast mass is an abnormal finding, and the PN should instruct the client to obtain prompt medical evaluation.

.A nurse is collecting data from a client who has systemic lupus erythematosus (SLE). Which of the following laboratory values should the nurse review to determine the client's renal function? 1) Antinuclear antibody 2) C-reactive protein 3) Erythrocyte sedimentation rate 4) Serum creatinine

Serum creatinine

A client with severe Parkinson disease diagnosed with anorexia, dysphagia, drooling, generalized weakness, and slurred speech is admitted to the unit. Which nursing action should the practical nurse implement first for this client?

Set up a suction and Yankauer at client's bedside. Dysphagia and drooling predispose this client to aspiration. A suction machine and Yankauer should be set up and near the client to be use to help prevent aspiration pneumonia. Aspiration is the primary concern in this situation.

The practical nurse is preparing a room for a client being admitted from the Emergency Department with a diagnosis of new onset of seizures. Which intervention should the nurse implement first?

Set-up and check for functioning of a suction apparatus and oxygen delivery system at bedside Maintaining the airway during a seizure is a priority for safety. The practical nurse needs to ensure there is a functioning suction apparatus to ensure airway clearance and an oxygen delivery system at bedside in the event of a seizure.

Which of the following actions demonstrate to the practical nurse that the client understands the correct procedure administration of a metered dose inhaler (MDI)? (Select all that apply.)

Shake the inhaler. Attach the canister of medication to the mouthpiece. Use a spacer attachment and place the mouthpiece in the mouth. Close the lips around the mouthpiece. After inhaling the medication, hold the breath 10 seconds. The correct sequence of MDI administration includes shaking the inhaler, attaching the canister to the mouthpiece, attaching the spacer, the client should then let their breath out through the mouth to empty the lungs and place the mouthpiece in the mouth, closing the lips and mouth around the mouthpiece, and inhaling medication and holding the breath for 10 seconds.

A nurse is providing teaching to a client who has type 1 diabetes mellitus about hypoglycemia. Which of the following manifestations should the nurse include in the teaching?

Shakiness -Rationale: an early manifestation of hypoglycemia is shakiness

two hours after thoracic surgery and chest tube insertion, a client is transferred from the post anesthesia care unit to the surgical unit. Which assessment finding requires immediate intervention by the nurse?

Subjective verbal report of feeling pressure in the chest

A female client with rheumatoid arthritis (RA) comes to the clinic with swelling of her wrists after she braced herself against a fall in the bathroom. What pathophysiological mechanism should the nurse explain to the client regarding to RA?

T cell generate cytokine production and antigen antibody reactions that trigger inflammatory responses

A nurse is caring for a client who had a thyroidectomy to treat hyperthyroidism caused by an adenoma. Which of the following findings should the nurse report to the provider? (select all that apply)

Tachycardia and hypertension Laryngeal stridor and hoarseness A positive Trousseau's sign

A nurse is checking the suction control chamber of a client's chest tube and notes that there is no bubbling in the suction control chamber. Which of the following actions should the nurse take? 1) Notify the provider. 2) Verify that the suction regulator is on. 3) Continue to monitor the client because this is an expected finding. 4) Milk the chest tube to dislodge any clots in the tubing that may be occluding it.

Verify that the suction regulator is on

A plan of care for a 56-year-old client who has been diagnosed with osteopenia has been developed. The plan is focused on preventing further bone resorption and increasing bone mass. Which outcome statement should be included in the plan of care?

The client will decrease the number of cigarettes smoked by 50% within 2 weeks. A desired outcome statement should be client-centered with a measurable outcome, and the client decreasing the number of cigarettes smoked by 50% within 2 weeks is both client-centered and measurable. Cigarette smoking has a negative effect on bone resorption, so the client should be advised to stop smoking.

A client diagnosed with ulcerative colitis (UC) asks the practical nurse why they are prescribed a low-residue diet. Which is the most appropriate response?

To reduce the amount and frequency of stool The purpose of a low-residue diet is to reduce the amount and frequency of stooling to promote healing of the bowels by consuming foods that do not irritate the intestinal lining and prolong intestinal transit time to encourage optimal absorption of nutrients.

The practical nurse (PN) is reviewing the health histories of the following individuals. Which factor does have a potential for development of throat cancer?

Tobacco use The most common risk factors for throat cancer are tobacco use, alcohol abuse, human papillomavirus (HPV), a diet lacking in fruits and vegetable and gastroesophageal reflux disease (GERD). Foods seasoned with herbs and spices have shown to have some health benefits in decreasing the risk of developing cancer.

In order to provide culturally competent care, what action must the practical nurse do first?

Understand one's own world views in addition to the client's. The nurse should understand their own values and views to prevent those beliefs being imparted to others, in addition to understanding the client's cultural views when providing cultural competent care. Treating every client the same or assuming that all clients share the same values does not exhibit cultural competence or sensitivity.

An older male reports to the clinic nurse that he sometimes feels chest pressure and becomes breathless. He further describes an event where he had to sit for about one hour before he felt better. He is asymptomatic at rest, but is concerned that he has had a heart attack. The clinical picture is consistent with which cardiac ischemic event?

Unstable angina

Which educational materials should the practical nurse select for reinforcement of teaching for secondary prevention? (Select all that apply.)

Video that teaches client to do breast self-examinations Pamphlets describing how to do testicular self-examinations Postcard reminders for clients to get papanicolaou (Pap) smears and mammograms Secondary prevention deals with early diagnosis to treat disease in the beginning of its development. Breast self-examinations, testicular self-examinations, mammograms, and Pap smears are considered secondary prevention methods.

A nurse is reinforcing teaching about pernicious anemia with a client following a total gastrectomy. Which of the following dietary supplements should the nurse include in the teaching as the treatment for pernicious anemia? 1) Vitamin B12 2) Vitamin C 3) Iron 4) Folate

Vitamin B12

A client with a liver abscess undergoes surgical evacuation and drainage of the abscess. Which laboratory value is most important for the nurse to monitor following the procedure?

WBC count

The Centers for Disease Control and Prevention (CDC) has issued guidelines for health care workers in relation to protection from HIV. The practical nurse (PN) who suspects they may be pregnant is assigned a client who is HIV+. What action should the PN implement?

Wear gloves when coming in contact with the blood or body fluids of a client. The CDC guidelines for standard precautions recommend that health care workers use gloves when coming in contact with blood or body fluids from any client because HIV is infectious before the client becomes aware of symptoms. Pregnancy of a nurse should not inhibit the nurse for taking care of a HIV+ client as long as standard precautions are observed.

The family suspects that AIDS dementia is occurring in their son who is HIV positive. Which symptom confirms their suspicions?

a change has recently occurred in his handwriting

A nurse is assisting in the care of a client who is 2 hours postoperative following a wedge resection of the left lung and has a chest tube to suction. Which of the following is the priority finding the nurse should report to the provider? 1) Abdomen is distended 2) Chest tube drainage of 70 mL in the last hour 3) Subcutaneous emphysema is noted to the left chest wall 4) Pain level of 6 on a 0 to 10 scale

abdomen is distended

A nurse is reinforcing discharge teaching to a client following arthroscopic surgery. To prevent postoperative complications which of the following actions should be reinforced during the teaching? 1) Administer an opioid analgesic to the client 30 min prior to initiating CPM exercises. 2) Place the client's affected leg into the CPM machine with the machine in the flexed position. 3) Place the client into a high Fowler's position when initiating the CPM exercises. 4) Align the joints of the CPM machine with the knee gatch in the client's bed.

administer an opioid analgesic to the client 30 mins prior to initiating CPM exercises

208.) A client with myasthenia gravis verbalizes complaints of feeling much weaker than normal. The health care provider plans to implement a diagnostic test to determine if the client is experiencing a myasthenic crisis and administers edrophonium (Enlon). Which of the following would indicate that the client is experiencing a myasthenic crisis? 1. Increasing weakness 2. No change in the condition 3. An increase in muscle spasms 4. A temporary improvement in the condition

auto-define "A client with myasthen..." Rationale: Edrophonium (Enlon) is administered to determine whether the client is reacting to an overdose of a medication (cholinergic crisis) or to an increasing severity of the disease (myasthenic crisis). When the edrophonium (Enlon) injection is given and the condition improves temporarily, the client is in myasthenic crisis. This is known as a positive test. Increasing weakness would occur in cholinergic crisis. Options 2 and 3 would not occur in either crisis.

A nurse is reinforcing discharge teaching with a client following a cataract extraction. Which of the following should the nurse include in the teaching? 1) Avoid bending at the waist. 2) Remove the eye shield at bedtime. 3) Limit the use of laxatives if constipated. 4) Seeing flashes of light is an expected finding following extraction.

avoid bending at the waist

A client with ulcerative colitis is admitted to the medical unit during an acute exacerbation. The nurse should instruct the unlicensed assistive personnel (UAP) to report which finding related to the client's bowel movements?

blood in the stool

During preoperative teaching for a male client scheduled for repair of an inguinal hernia, the client tells the nurse that he has had several surgeries and understands the need to perform coughing and deep breathing exercises after surgery. How should the nurse respond?

explain that coughing should be avoided

A client with draining skin lesions of the lower extremity is admitted with possible Methicillin-Resistant Staphylococcus Aureus (MRSA). Which nursing interventions should the nurse include in the plan on care? (Select all that apply.)

institute contact precautions for staff and visitors send wound drainage for culture and sensitivity monitor the clients WBC count

A nurse is collecting data from a client who has open-angle glaucoma. Which of the following findings should the nurse expect? 1) Loss of peripheral vision 2) Headache 3) Halos around lights 4) Discomfort in the eyes

loss of peripheral vision

Following admission, a client with a vascular occlusion of the right lower extremity calls the nurse and reports difficulty sleeping because of cold feet. Which of the following nursing actions should the nurse take to promote the client's comfort? 1) Rub the client's feet briskly for several minutes 2) Obtain a pair of slipper socks for the client. 3) Increase the client's oral fluid intake 4) Place a moist heating pad under the client's feet.

obtain a pair of slipper socks for the client

The nurse is providing discharge instructions to a client who is receiving prednisone (Deltasone) 5 mg PO daily for a rash due to contact with poison ivy. Which symptom should the nurse tell the client to report to the healthcare provider?

rapid weight gain

When explaining dietary guidelines to a client with acute glomerulonephritis (AGN), which instruction should the nurse include in the dietary teaching?

restrict sodium intake

A female client who received partial-thickness and full-thickness burns over 40% of her body in a house fire is admitted to the inpatient burn unit. What fluid should the nurse prepare to administer during the acute phase of the client's burn recovery?

ringers lactate

A nurse is caring for a client who sustained a basal skull fracture. When performing morning hygiene care, the nurse notices a thin stream of clear drainage coming from out of the client's right nostril. Which of the following actions should the nurse take first? 1) Take the client's temperature. 2) Place a dressing under the client's nose. 3) Notify the charge nurse. 4) Test the drainage for glucose.

test the drainage for glucose

A nurse is caring for a client is who is 4 hr postoperative following a transurethral resection of the prostate (TURP). Which of the following is the priority finding for the nurse report to the provider? 1) Emesis of 100 mL 2) Oral temperature of 37.5° C (99.5° F) 3) Thick, red-colored urine 4) Pain level of 4 on a 0 to 10 rating scale

thick, red-colored urine

A nurse is planning care for a client who has Cushing's syndrome due to chronic corticosteroid use. Which of the following actions should the nurse involve in the plan of care?

Check the client's urine specific gravity. -Rationale: to assess for fluid volume overload

A nurse is reinforcing teaching with a client who has peripheral vascular disease (PVD). The nurse should recognize that which of the following statements by the client indicates a need for further teaching? 1) "I will avoid crossing my legs at the knees." 2) "I will use a thermometer to check the temperature of my bath water." 3) "I will not go barefoot." 4) "I will wear stockings with elastic tops."

"I will wear stockings with elastic tops"

A client status post closed-appendectomy is prescribed to begin ambulation the next day. The next day when the practical nurse (PN) goes to assist the client with ambulation, the client yells they are watching the television and they don't feel like getting out of bed. Which response should the PN provide?

"I'll be back in 30 minutes to help you get out of bed and walk around the room." Returning within 30 minutes provides a "cooling off" period, is firm, direct, and nonthreatening, and avoids arguing with the client.

31.) A community health nurse visits a client at home. Prednisone 10 mg orally daily has been prescribed for the client and the nurse reinforces teaching for the client about the medication. Which statement, if made by the client, indicates that further teaching is necessary? 1. "I can take aspirin or my antihistamine if I need it." 2. "I need to take the medication every day at the same time." 3. "I need to avoid coffee, tea, cola, and chocolate in my diet." 4. "If I gain more than 5 pounds a week, I will call my doctor."

1. "I can take aspirin or my antihistamine if I need it." Rationale: Aspirin and other over-the-counter medications should not be taken unless the client consults with the health care provider (HCP). The client needs to take the medication at the same time every day and should be instructed not to stop the medication. A slight weight gain as a result of an improved appetite is expected, but after the dosage is stabilized, a weight gain of 5 lb or more weekly should be reported to the HCP. Caffeine-containing foods and fluids need to be avoided because they may contribute to steroid-ulcer development.

179.) A nurse provides medication instructions to a client who had a kidney transplant about therapy with cyclosporine (Sandimmune). Which statement by the client indicates a need for further instruction? 1. "I need to obtain a yearly influenza vaccine." 2. "I need to have dental checkups every 3 months." 3. "I need to self-monitor my blood pressure at home." 4. "I need to call the health care provider (HCP) if my urine volume decreases or my urine becomes cloudy."

1. "I need to obtain a yearly influenza vaccine." Rationale: Cyclosporine is an immunosuppressant medication. Because of the medication's effects, the client should not receive any vaccinations without first consulting the HCP. The client should report decreased urine output or cloudy urine, which could indicate kidney rejection or infection, respectively. The client must be able to self-monitor blood pressure to check for the side effect of hypertension. The client needs meticulous oral care and dental cleaning every 3 months to help prevent gingival hyperplasia.

77.) Phenytoin (Dilantin), 100 mg orally three times daily, has been prescribed for a client for seizure control. The nurse reinforces instructions regarding the medication to the client. Which statement by the client indicates an understanding of the instructions? 1. "I will use a soft toothbrush to brush my teeth." 2. "It's all right to break the capsules to make it easier for me to swallow them." 3. "If I forget to take my medication, I can wait until the next dose and eliminate that dose." 4. "If my throat becomes sore, it's a normal effect of the medication and it's nothing to be concerned about."

1. "I will use a soft toothbrush to brush my teeth." Rationale: Phenytoin (Dilantin) is an anticonvulsant. Gingival hyperplasia, bleeding, swelling, and tenderness of the gums can occur with the use of this medication. The client needs to be taught good oral hygiene, gum massage, and the need for regular dentist visits. The client should not skip medication doses, because this could precipitate a seizure. Capsules should not be chewed or broken and they must be swallowed. The client needs to be instructed to report a sore throat, fever, glandular swelling, or any skin reaction, because this indicates hematological toxicity.

202.) A nurse is collecting data from a client about medications being taken, and the client tells the nurse that he is taking herbal supplements for the treatment of varicose veins. The nurse understands that the client is most likely taking which of the following? 1. Bilberry 2. Ginseng 3. Feverfew 4. Evening primrose

1. Bilberry Rationale: Bilberry is an herbal supplement that has been used to treat varicose veins. This supplement has also been used to treat cataracts, retinopathy, diabetes mellitus, and peripheral vascular disease. Ginseng has been used to improve memory performance and decrease blood glucose levels in type 2 diabetes mellitus. Feverfew is used to prevent migraine headaches and to treat rheumatoid arthritis. Evening primrose is used to treat eczema and skin irritation.

26.) Glimepiride (Amaryl) is prescribed for a client with diabetes mellitus. A nurse reinforces instructions for the client and tells the client to avoid which of the following while taking this medication? 1. Alcohol 2. Organ meats 3. Whole-grain cereals 4. Carbonated beverages

1. Alcohol Rationale: When alcohol is combined with glimepiride (Amaryl), a disulfiram-like reaction may occur. This syndrome includes flushing, palpitations, and nausea. Alcohol can also potentiate the hypoglycemic effects of the medication. Clients need to be instructed to avoid alcohol consumption while taking this medication. The items in options 2, 3, and 4 do not need to be avoided.

100.) Saquinavir (Invirase) is prescribed for the client who is human immunodeficiency virus seropositive. The nurse reinforces medication instructions and tells the client to: 1. Avoid sun exposure. 2. Eat low-calorie foods. 3. Eat foods that are low in fat. 4. Take the medication on an empty stomach.

1. Avoid sun exposure. Rationale: Saquinavir (Invirase) is an antiretroviral (protease inhibitor) used with other antiretroviral medications to manage human immunodeficiency virus infection. Saquinavir is administered with meals and is best absorbed if the client consumes high-calorie, high-fat meals. Saquinavir can cause photosensitivity, and the nurse should instruct the client to avoid sun exposure.

32.) Desmopressin acetate (DDAVP) is prescribed for the treatment of diabetes insipidus. The nurse monitors the client after medication administration for which therapeutic response? 1. Decreased urinary output 2. Decreased blood pressure 3. Decreased peripheral edema 4. Decreased blood glucose level

1. Decreased urinary output Rationale: Desmopressin promotes renal conservation of water. The hormone carries out this action by acting on the collecting ducts of the kidney to increase their permeability to water, which results in increased water reabsorption. The therapeutic effect of this medication would be manifested by a decreased urine output. Options 2, 3, and 4 are unrelated to the effects of this medication.

33.) The home health care nurse is visiting a client who was recently diagnosed with type 2 diabetes mellitus. The client is prescribed repaglinide (Prandin) and metformin (Glucophage) and asks the nurse to explain these medications. The nurse should reinforce which instructions to the client? Select all that apply. 1. Diarrhea can occur secondary to the metformin. 2. The repaglinide is not taken if a meal is skipped. 3. The repaglinide is taken 30 minutes before eating. 4. Candy or another simple sugar is carried and used to treat mild hypoglycemia episodes. 5. Metformin increases hepatic glucose production to prevent hypoglycemia associated with repaglinide. 6. Muscle pain is an expected side effect of metformin and may be treated with acetaminophen (Tylenol).

1. Diarrhea can occur secondary to the metformin. 2. The repaglinide is not taken if a meal is skipped. 3. The repaglinide is taken 30 minutes before eating. 4. Candy or another simple sugar is carried and used to treat mild hypoglycemia episodes. Rationale: Repaglinide is a rapid-acting oral hypoglycemic agent that stimulates pancreatic insulin secretion that should be taken before meals, and that should be withheld if the client does not eat. Hypoglycemia is a side effect of repaglinide and the client should always be prepared by carrying a simple sugar with her or him at all times. Metformin is an oral hypoglycemic given in combination with repaglinide and works by decreasing hepatic glucose production. A common side effect of metformin is diarrhea. Muscle pain may occur as an adverse effect from metformin but it might signify a more serious condition that warrants health care provider notification, not the use of acetaminophen.

91.) Cyclobenzaprine (Flexeril) is prescribed for a client to treat muscle spasms, and the nurse is reviewing the client's record. Which of the following disorders, if noted in the client's record, would indicate a need to contact the health care provider regarding the administration of this medication? 1. Glaucoma 2. Emphysema 3. Hyperthyroidism 4. Diabetes mellitus

1. Glaucoma Rationale: Because this medication has anticholinergic effects, it should be used with caution in clients with a history of urinary retention, angle-closure glaucoma, and increased intraocular pressure. Cyclobenzaprine hydrochloride should be used only for short-term 2- to 3-week therapy.

158.) A client with chronic renal failure is receiving epoetin alfa (Epogen, Procrit). Which laboratory result would indicate a therapeutic effect of the medication? 1. Hematocrit of 32% 2. Platelet count of 400,000 cells/mm3 3. White blood cell count of 6000 cells/mm3 4. Blood urea nitrogen (BUN) level of 15 mg/dL

1. Hematocrit of 32% Rationale: Epoetin alfa is used to reverse anemia associated with chronic renal failure. A therapeutic effect is seen when the hematocrit is between 30% and 33%. The laboratory tests noted in the other options are unrelated to the use of this medication.

43.) A histamine (H2)-receptor antagonist will be prescribed for a client. The nurse understands that which medications are H2-receptor antagonists? Select all that apply. 1. Nizatidine (Axid) 2. Ranitidine (Zantac) 3. Famotidine (Pepcid) 4. Cimetidine (Tagamet) 5. Esomeprazole (Nexium) 6. Lansoprazole (Prevacid)

1. Nizatidine (Axid) 2. Ranitidine (Zantac) 3. Famotidine (Pepcid) 4. Cimetidine (Tagamet) Rationale: H2-receptor antagonists suppress secretion of gastric acid, alleviate symptoms of heartburn, and assist in preventing complications of peptic ulcer disease. These medications also suppress gastric acid secretions and are used in active ulcer disease, erosive esophagitis, and pathological hypersecretory conditions. The other medications listed are proton pump inhibitors. H2-receptor antagonists medication names end with -dine. Proton pump inhibitors medication names end with -zole.

3.) Salicylic acid is prescribed for a client with a diagnosis of psoriasis. The nurse monitors the client, knowing that which of the following would indicate the presence of systemic toxicity from this medication? 1. Tinnitus 2. Diarrhea 3. Constipation 4. Decreased respirations

1. Tinnitus Rationale: Salicylic acid is absorbed readily through the skin, and systemic toxicity (salicylism) can result. Symptoms include tinnitus, dizziness, hyperpnea, and psychological disturbances. Constipation and diarrhea are not associated with salicylism.

235.) A tricyclic antidepressant is administered to a client daily. The nurse plans to monitor for the common side effects of the medication and includes which of the following in the plan of care? 1. Offer hard candy or gum periodically. 2. Offer a nutritious snack between meals. 3. Monitor the blood pressure every 2 hours. 4. Review the white blood cell (WBC) count results daily.

1. Offer hard candy or gum periodically. Rationale: Dry mouth is a common side effect of tricyclic antidepressants. Frequent mouth rinsing with water, sucking on hard candy, and chewing gum will alleviate this common side effect. It is not necessary to monitor the blood pressure every 2 hours. In addition, it is not necessary to check the WBC daily. Weight gain is a common side effect and frequent snacks will aggravate this problem.

18.) The nurse is reviewing the history and physical examination of a client who will be receiving asparaginase (Elspar), an antineoplastic agent. The nurse consults with the registered nurse regarding the administration of the medication if which of the following is documented in the client's history? 1. Pancreatitis 2. Diabetes mellitus 3. Myocardial infarction 4. Chronic obstructive pulmonary disease

1. Pancreatitis Rationale: Asparaginase (Elspar) is contraindicated if hypersensitivity exists, in pancreatitis, or if the client has a history of pancreatitis. The medication impairs pancreatic function and pancreatic function tests should be performed before therapy begins and when a week or more has elapsed between administration of the doses. The client needs to be monitored for signs of pancreatitis, which include nausea, vomiting, and abdominal pain. The conditions noted in options 2, 3, and 4 are not contraindicated with this medication.

171.) A nurse is preparing to administer furosemide (Lasix) to a client with a diagnosis of heart failure. The most important laboratory test result for the nurse to check before administering this medication is: 1. Potassium level 2. Creatinine level 3. Cholesterol level 4. Blood urea nitrogen

1. Potassium level Rationale: Furosemide is a loop diuretic. The medication causes a decrease in the client's electrolytes, especially potassium, sodium, and chloride. Administering furosemide to a client with low electrolyte levels could precipitate ventricular dysrhythmias. Options 2 and 4 reflect renal function. The cholesterol level is unrelated to the administration of this medication.

30.) A client with diabetes mellitus visits a health care clinic. The client's diabetes mellitus previously had been well controlled with glyburide (DiaBeta) daily, but recently the fasting blood glucose level has been 180 to 200 mg/dL. Which medication, if added to the client's regimen, may have contributed to the hyperglycemia? 1. Prednisone 2. Phenelzine (Nardil) 3. Atenolol (Tenormin) 4. Allopurinol (Zyloprim)

1. Prednisone Rationale: Prednisone may decrease the effect of oral hypoglycemics, insulin, diuretics, and potassium supplements. Option 2, a monoamine oxidase inhibitor, and option 3, a β-blocker, have their own intrinsic hypoglycemic activity. Option 4 decreases urinary excretion of sulfonylurea agents, causing increased levels of the oral agents, which can lead to hypoglycemia.

152.) Intravenous heparin therapy is prescribed for a client. While implementing this prescription, a nurse ensures that which of the following medications is available on the nursing unit? 1. Protamine sulfate 2. Potassium chloride 3. Phytonadione (vitamin K ) 4. Aminocaproic acid (Amicar)

1. Protamine sulfate Rationale: The antidote to heparin is protamine sulfate; it should be readily available for use if excessive bleeding or hemorrhage occurs. Potassium chloride is administered for a potassium deficit. Vitamin K is an antidote for warfarin sodium. Aminocaproic acid is the antidote for thrombolytic therapy.

102.) A client with human immunodeficiency virus is taking nevirapine (Viramune). The nurse should monitor for which adverse effects of the medication? Select all that apply. 1. Rash 2. Hepatotoxicity 3. Hyperglycemia 4. Peripheral neuropathy 5. Reduced bone mineral density

1. Rash 2. Hepatotoxicity Rationale: Nevirapine (Viramune) is a non-nucleoside reverse transcriptase inhibitors (NRTI) that is used to treat HIV infection. It is used in combination with other antiretroviral medications to treat HIV. Adverse effects include rash, Stevens-Johnson syndrome, hepatitis, and increased transaminase levels. Hyperglycemia, peripheral neuropathy, and reduced bone density are not adverse effects of this medication.

72.) Cinoxacin (Cinobac), a urinary antiseptic, is prescribed for the client. The nurse reviews the client's medical record and should contact the health care provider (HCP) regarding which documented finding to verify the prescription? Refer to chart. 1. Renal insufficiency 2. Chest x-ray: normal 3. Blood glucose, 102 mg/dL 4. Folic acid (vitamin B6) 0.5 mg, orally daily

1. Renal insufficiency Rationale: Cinoxacin should be administered with caution in clients with renal impairment. The dosage should be reduced, and failure to do so could result in accumulation of cinoxacin to toxic levels. Therefore the nurse would verify the prescription if the client had a documented history of renal insufficiency. The laboratory and diagnostic test results are normal findings. Folic acid (vitamin B6) may be prescribed for a client with renal insufficiency to prevent anemia.

110.) A client taking lithium carbonate (Lithobid) reports vomiting, abdominal pain, diarrhea, blurred vision, tinnitus, and tremors. The lithium level is checked as a part of the routine follow-up and the level is 3.0 mEq/L. The nurse knows that this level is: 1. Toxic 2. Normal 3. Slightly above normal 4. Excessively below normal

1. Toxic Rationale: The therapeutic serum level of lithium is 0.6 to 1.2 mEq/L. A level of 3 mEq/L indicates toxicity.

53.) Rifabutin (Mycobutin) is prescribed for a client with active Mycobacterium avium complex (MAC) disease and tuberculosis. The nurse monitors for which side effects of the medication? Select all that apply. 1. Signs of hepatitis 2. Flu-like syndrome 3. Low neutrophil count 4. Vitamin B6 deficiency 5. Ocular pain or blurred vision 6. Tingling and numbness of the fingers

1. Signs of hepatitis 2. Flu-like syndrome 3. Low neutrophil count 5. Ocular pain or blurred vision Rationale: Rifabutin (Mycobutin) may be prescribed for a client with active MAC disease and tuberculosis. It inhibits mycobacterial DNA-dependent RNA polymerase and suppresses protein synthesis. Side effects include rash, gastrointestinal disturbances, neutropenia (low neutrophil count), red-orange body secretions, uveitis (blurred vision and eye pain), myositis, arthralgia, hepatitis, chest pain with dyspnea, and flu-like syndrome. Vitamin B6 deficiency and numbness and tingling in the extremities are associated with the use of isoniazid (INH). Ethambutol (Myambutol) also causes peripheral neuritis.

92.) In monitoring a client's response to disease-modifying antirheumatic drugs (DMARDs), which findings would the nurse interpret as acceptable responses? Select all that apply. 1. Symptom control during periods of emotional stress 2. Normal white blood cell counts, platelet, and neutrophil counts 3. Radiological findings that show nonprogression of joint degeneration 4. An increased range of motion in the affected joints 3 months into therapy 5. Inflammation and irritation at the injection site 3 days after injection is given 6. A low-grade temperature upon rising in the morning that remains throughout the day

1. Symptom control during periods of emotional stress 2. Normal white blood cell counts, platelet, and neutrophil counts 3. Radiological findings that show nonprogression of joint degeneration 4. An increased range of motion in the affected joints 3 months into therapy Rationale: Because emotional stress frequently exacerbates the symptoms of rheumatoid arthritis, the absence of symptoms is a positive finding. DMARDs are given to slow progression of joint degeneration. In addition, the improvement in the range of motion after 3 months of therapy with normal blood work is a positive finding. Temperature elevation and inflammation and irritation at the medication injection site could indicate signs of infection.

149.) A client taking fexofenadine (Allegra) is scheduled for allergy skin testing and tells the nurse in the health care provider's office that a dose was taken this morning. The nurse determines that: 1. The client should reschedule the appointment. 2. A lower dose of allergen will need to be injected. 3. A higher dose of allergen will need to be injected. 4. The client should have the skin test read a day later than usual.

1. The client should reschedule the appointment. Rationale: Fexofenadine is an antihistamine, which provides relief of symptoms caused by allergy. Antihistamines should be discontinued for at least 3 days (72 hours) before allergy skin testing to avoid false-negative readings. This client should have the appointment rescheduled for 3 days after discontinuing the medication.

221.) A nurse is reviewing the health care provider's prescriptions for an adult client who has been admitted to the hospital following a back injury. Carisoprodol (Soma) is prescribed for the client to relieve the muscle spasms; the health care provider has prescribed 350 mg to be administered four times a day. When preparing to give this medication, the nurse determines that this dosage is: 1. The normal adult dosage 2. A lower than normal dosage 3. A higher than normal dosage 4. A dosage requiring further clarification

1. The normal adult dosage Rationale: The normal adult dosage for carisoprodol is 350 mg orally three or four times daily.

21.) A nurse is assisting with caring for a client with cancer who is receiving cisplatin. Select the adverse effects that the nurse monitors for that are associated with this medication. Select all that apply. 1. Tinnitus 2. Ototoxicity 3. Hyperkalemia 4. Hypercalcemia 5. Nephrotoxicity 6. Hypomagnesemia

1. Tinnitus 2. Ototoxicity 5. Nephrotoxicity 6. Hypomagnesemia Rationale: Cisplatin is an alkylating medication. Alkylating medications are cell cycle phase-nonspecific medications that affect the synthesis of DNA by causing the cross-linking of DNA to inhibit cell reproduction. Cisplatin may cause ototoxicity, tinnitus, hypokalemia, hypocalcemia, hypomagnesemia, and nephrotoxicity. Amifostine (Ethyol) may be administered before cisplatin to reduce the potential for renal toxicity.

213.) A client is admitted to the hospital with complaints of back spasms. The client states, "I have been taking two or three aspirin every 4 hours for the past week and it hasn't helped my back." Aspirin intoxication is suspected. Which of the following complaints would indicate aspirin intoxication? 1. Tinnitus 2. Constipation 3. Photosensitivity 4. Abdominal cramps

1. Tinnitus Rationale: Mild intoxication with acetylsalicylic acid (aspirin) is called salicylism and is commonly experienced when the daily dosage is higher than 4 g. Tinnitus (ringing in the ears) is the most frequently occurring effect noted with intoxication. Hyperventilation may occur because salicylate stimulates the respiratory center. Fever may result because salicylate interferes with the metabolic pathways involved with oxygen consumption and heat production. Options 2, 3, and 4 are incorrect.

8.) A client with severe acne is seen in the clinic and the health care provider (HCP) prescribes isotretinoin. The nurse reviews the client's medication record and would contact the (HCP) if the client is taking which medication? 1. Vitamin A 2. Digoxin (Lanoxin) 3. Furosemide (Lasix) 4. Phenytoin (Dilantin)

1. Vitamin A Rationale: Isotretinoin is a metabolite of vitamin A and can produce generalized intensification of isotretinoin toxicity. Because of the potential for increased toxicity, vitamin A supplements should be discontinued before isotretinoin therapy. Options 2, 3, and 4 are not contraindicated with the use of isotretinoin.

239.) Which of the following precautions will the nurse specifically take during the administration of ribavirin (Virazole) to a child with respiratory syncytial virus (RSV)? 1. Wearing goggles 2. Wearing a gown 3. Wearing a gown and a mask 4. Handwashing before administration

1. Wearing goggles Rationale: Some caregivers experience headaches, burning nasal passages and eyes, and crystallization of soft contact lenses as a result of administration of ribavirin. Specific to this medication is the use of goggles. A gown is not necessary. A mask may be worn. Handwashing is to be performed before and after any child contact.

24.) A nurse is reinforcing teaching for a client regarding how to mix regular insulin and NPH insulin in the same syringe. Which of the following actions, if performed by the client, indicates the need for further teaching? 1. Withdraws the NPH insulin first 2. Withdraws the regular insulin first 3. Injects air into NPH insulin vial first 4. Injects an amount of air equal to the desired dose of insulin into the vial

1. Withdraws the NPH insulin first Rationale: When preparing a mixture of regular insulin with another insulin preparation, the regular insulin is drawn into the syringe first. This sequence will avoid contaminating the vial of regular insulin with insulin of another type. Options 2, 3, and 4 identify the correct actions for preparing NPH and regular insulin.

195.) A nurse is caring for a client who is taking metoprolol (Lopressor). The nurse measures the client's blood pressure (BP) and apical pulse (AP) immediately before administration. The client's BP is 122/78 mm/Hg and the AP is 58 beats/min. Based on this data, which of the following is the appropriate action? 1. Withhold the medication. 2. Notify the registered nurse immediately. 3. Administer the medication as prescribed. 4. Administer half of the prescribed medication.

1. Withhold the medication. Rationale: Metoprolol (Lopressor) is classified as a beta-adrenergic blocker and is used in the treatment of hypertension, angina, and myocardial infarction. Baseline nursing assessments include measurement of BP and AP immediately before administration. If the systolic BP is below 90 mm/Hg and the AP is below 60 beats/min, the nurse should withhold the medication and document this action. Although the registered nurse should be informed of the client's vital signs, it is not necessary to do so immediately. The medication should not be administered because the data is outside of the prescribed parameters for this medication. The nurse should not administer half of the medication, or alter any dosages at any point in time.

234.) A hospitalized client is started on phenelzine sulfate (Nardil) for the treatment of depression. At lunchtime, a tray is delivered to the client. Which food item on the tray will the nurse remove? 1. Yogurt 2. Crackers 3. Tossed salad 4. Oatmeal cookies

1. Yogurt Rationale: Phenelzine sulfate is a monoamine oxidase inhibitor (MAOI). The client should avoid taking in foods that are high in tyramine. These foods could trigger a potentially fatal hypertensive crisis. Foods to avoid include yogurt, aged cheeses, smoked or processed meats, red wines, and fruits such as avocados, raisins, or figs.

A client with dehydration is prescribed a potassium chloride infusion at 10 mEq/h. Potassium chloride 80 mEq is mixed with 1 liter of NS. The nurse should regulate the infusion pump to deliver how many mL/hour? Round to nearest whole number if required

125 ml/hr

224.) Neuroleptic malignant syndrome is suspected in a client who is taking chlorpromazine. Which medication would the nurse prepare in anticipation of being prescribed to treat this adverse effect related to the use of chlorpromazine? 1. Protamine sulfate 2. Bromocriptine (Parlodel) 3. Phytonadione (vitamin K) 4. Enalapril maleate (Vasotec)

2. Bromocriptine (Parlodel) Rationale: Bromocriptine is an antiparkinsonian prolactin inhibitor used in the treatment of neuroleptic malignant syndrome. Vitamin K is the antidote for warfarin (Coumadin) overdose. Protamine sulfate is the antidote for heparin overdose. Enalapril maleate is an antihypertensive used in the treatment of hypertension.

215.) A client with rheumatoid arthritis is taking acetylsalicylic acid (aspirin) on a daily basis. Which medication dose should the nurse expect the client to be taking? 1. 1 g daily 2. 4 g daily 3. 325 mg daily 4. 1000 mg daily

2. 4 g daily Rationale: Aspirin may be used to treat the client with rheumatoid arthritis. It may also be used to reduce the risk of recurrent transient ischemic attack (TIA) or brain attack (stroke) or reduce the risk of myocardial infarction (MI) in clients with unstable angina or a history of a previous MI. The normal dose for clients being treated with aspirin to decrease thrombosis and MI is 300 to 325 mg/day. Clients being treated to prevent TIAs are usually prescribed 1.3 g/day in two to four divided doses. Clients with rheumatoid arthritis are treated with 3.6 to 5.4 g/day in divided doses. **Eliminate options 1 and 4 because they are alike**

29.) A client is taking Humulin NPH insulin daily every morning. The nurse reinforces instructions for the client and tells the client that the most likely time for a hypoglycemic reaction to occur is: 1. 2 to 4 hours after administration 2. 4 to 12 hours after administration 3. 16 to 18 hours after administration 4. 18 to 24 hours after administration

2. 4 to 12 hours after administration Rationale: Humulin NPH is an intermediate-acting insulin. The onset of action is 1.5 hours, it peaks in 4 to 12 hours, and its duration of action is 24 hours. Hypoglycemic reactions most likely occur during peak time.

64.) Nalidixic acid (NegGram) is prescribed for a client with a urinary tract infection. On review of the client's record, the nurse notes that the client is taking warfarin sodium (Coumadin) daily. Which prescription should the nurse anticipate for this client? 1. Discontinuation of warfarin sodium (Coumadin) 2. A decrease in the warfarin sodium (Coumadin) dosage 3. An increase in the warfarin sodium (Coumadin) dosage 4. A decrease in the usual dose of nalidixic acid (NegGram)

2. A decrease in the warfarin sodium (Coumadin) dosage Rationale: Nalidixic acid can intensify the effects of oral anticoagulants by displacing these agents from binding sites on plasma protein. When an oral anticoagulant is combined with nalidixic acid, a decrease in the anticoagulant dosage may be needed.

88.) Dantrolene sodium (Dantrium) is prescribed for a client experiencing flexor spasms, and the client asks the nurse about the action of the medication. The nurse responds, knowing that the therapeutic action of this medication is which of the following? 1. Depresses spinal reflexes 2. Acts directly on the skeletal muscle to relieve spasticity 3. Acts within the spinal cord to suppress hyperactive reflexes 4. Acts on the central nervous system (CNS) to suppress spasms

2. Acts directly on the skeletal muscle to relieve spasticity Rationale: Dantrium acts directly on skeletal muscle to relieve muscle spasticity. The primary action is the suppression of calcium release from the sarcoplasmic reticulum. This in turn decreases the ability of the skeletal muscle to contract. **Options 1, 3, and 4 are all comparable or alike in that they address CNS suppression and the depression of reflexes. Therefore, eliminate these options.**

74.) A client with myasthenia gravis is receiving pyridostigmine (Mestinon). The nurse monitors for signs and symptoms of cholinergic crisis caused by overdose of the medication. The nurse checks the medication supply to ensure that which medication is available for administration if a cholinergic crisis occurs? 1. Vitamin K 2. Atropine sulfate 3. Protamine sulfate 4. Acetylcysteine (Mucomyst)

2. Atropine sulfate Rationale: The antidote for cholinergic crisis is atropine sulfate. Vitamin K is the antidote for warfarin (Coumadin). Protamine sulfate is the antidote for heparin, and acetylcysteine (Mucomyst) is the antidote for acetaminophen (Tylenol).

86.) A nurse is reinforcing discharge instructions to a client receiving baclofen (Lioresal). Which of the following would the nurse include in the instructions? 1. Restrict fluid intake. 2. Avoid the use of alcohol. 3. Stop the medication if diarrhea occurs. 4. Notify the health care provider if fatigue occurs.

2. Avoid the use of alcohol. Rationale: Baclofen is a central nervous system (CNS) depressant. The client should be cautioned against the use of alcohol and other CNS depressants, because baclofen potentiates the depressant activity of these agents. Constipation rather than diarrhea is an adverse effect of baclofen. It is not necessary to restrict fluids, but the client should be warned that urinary retention can occur. Fatigue is related to a CNS effect that is most intense during the early phase of therapy and diminishes with continued medication use. It is not necessary that the client notify the health care provider if fatigue occurs.

9.) The nurse is applying a topical corticosteroid to a client with eczema. The nurse would monitor for the potential for increased systemic absorption of the medication if the medication were being applied to which of the following body areas? 1. Back 2. Axilla 3. Soles of the feet 4. Palms of the hands

2. Axilla Rationale: Topical corticosteroids can be absorbed into the systemic circulation. Absorption is higher from regions where the skin is especially permeable (scalp, axilla, face, eyelids, neck, perineum, genitalia), and lower from regions in which permeability is poor (back, palms, soles).

199.) A nurse is applying a topical glucocorticoid to a client with eczema. The nurse monitors for systemic absorption of the medication if the medication is being applied to which of the following body areas? 1. Back 2. Axilla 3. Soles of the feet 4. Palms of the hands

2. Axilla Rationale: Topical glucocorticoids can be absorbed into the systemic circulation. Absorption is higher from regions where the skin is especially permeable (scalp, axillae, face, eyelids, neck, perineum, genitalia), and lower from regions where penetrability is poor (back, palms, soles). **Eliminate options 3 and 4 because these body areas are similar in terms of skin characteristics**

123.) A nurse is planning to administer amlodipine (Norvasc) to a client. The nurse plans to check which of the following before giving the medication? 1. Respiratory rate 2. Blood pressure and heart rate 3. Heart rate and respiratory rate 4. Level of consciousness and blood pressure

2. Blood pressure and heart rate Rationale: Amlodipine is a calcium channel blocker. This medication decreases the rate and force of cardiac contraction. Before administering a calcium channel blocking agent, the nurse should check the blood pressure and heart rate, which could both decrease in response to the action of this medication. This action will help to prevent or identify early problems related to decreased cardiac contractility, heart rate, and conduction. **amlodipine is a calcium channel blocker, and this group of medications decreases the rate and force of cardiac contraction. This in turn lowers the pulse rate and blood pressure.**

20.) The client with metastatic breast cancer is receiving tamoxifen. The nurse specifically monitors which laboratory value while the client is taking this medication? 1. Glucose level 2. Calcium level 3. Potassium level 4. Prothrombin time

2. Calcium level Rationale: Tamoxifen may increase calcium, cholesterol, and triglyceride levels. Before the initiation of therapy, a complete blood count, platelet count, and serum calcium levels should be assessed. These blood levels, along with cholesterol and triglyceride levels, should be monitored periodically during therapy. The nurse should assess for hypercalcemia while the client is taking this medication. Signs of hypercalcemia include increased urine volume, excessive thirst, nausea, vomiting, constipation, hypotonicity of muscles, and deep bone and flank pain.

170.) Atenolol hydrochloride (Tenormin) is prescribed for a hospitalized client. The nurse should perform which of the following as a priority action before administering the medication? 1. Listen to the client's lung sounds. 2. Check the client's blood pressure. 3. Check the recent electrolyte levels. 4. Assess the client for muscle weakness.

2. Check the client's blood pressure. Rationale: Atenolol hydrochloride is a beta-blocker used to treat hypertension. Therefore the priority nursing action before administration of the medication is to check the client's blood pressure. The nurse also checks the client's apical heart rate. If the systolic blood pressure is below 90 mm Hg or the apical pulse is 60 beats per minute or lower, the medication is withheld and the registered nurse and/or health care provider is notified. The nurse would check baseline renal and liver function tests. The medication may cause weakness, and the nurse would assist the client with activities if weakness occurs. **Beta-blockers have "-lol" at the end of the medication name**

34.) A client with Crohn's disease is scheduled to receive an infusion of infliximab (Remicade). The nurse assisting in caring for the client should take which action to monitor the effectiveness of treatment? 1. Monitoring the leukocyte count for 2 days after the infusion 2. Checking the frequency and consistency of bowel movements 3. Checking serum liver enzyme levels before and after the infusion 4. Carrying out a Hematest on gastric fluids after the infusion is completed

2. Checking the frequency and consistency of bowel movements Rationale: The principal manifestations of Crohn's disease are diarrhea and abdominal pain. Infliximab (Remicade) is an immunomodulator that reduces the degree of inflammation in the colon, thereby reducing the diarrhea. Options 1, 3, and 4 are unrelated to this medication.

146.) A client has begun therapy with theophylline (Theo-24). The nurse tells the client to limit the intake of which of the following while taking this medication? 1. Oranges and pineapple 2. Coffee, cola, and chocolate 3. Oysters, lobster, and shrimp 4. Cottage cheese, cream cheese, and dairy creamers

2. Coffee, cola, and chocolate Rationale: Theophylline is a xanthine bronchodilator. The nurse teaches the client to limit the intake of xanthine-containing foods while taking this medication. These include coffee, cola, and chocolate.

63.) A client with coronary artery disease complains of substernal chest pain. After checking the client's heart rate and blood pressure, a nurse administers nitroglycerin, 0.4 mg, sublingually. After 5 minutes, the client states, "My chest still hurts." Select the appropriate actions that the nurse should take. Select all that apply. 1. Call a code blue. 2. Contact the registered nurse. 3. Contact the client's family. 4. Assess the client's pain level. 5. Check the client's blood pressure. 6. Administer a second nitroglycerin, 0.4 mg, sublingually.

2. Contact the registered nurse. 4. Assess the client's pain level. 5. Check the client's blood pressure. 6. Administer a second nitroglycerin, 0.4 mg, sublingually. Rationale: The usual guideline for administering nitroglycerin tablets for a hospitalized client with chest pain is to administer one tablet every 5 minutes PRN for chest pain, for a total dose of three tablets. The registered nurse should be notified of the client's condition, who will then notify the health care provider as appropriate. Because the client is still complaining of chest pain, the nurse would administer a second nitroglycerin tablet. The nurse would assess the client's pain level and check the client's blood pressure before administering each nitroglycerin dose. There are no data in the question that indicate the need to call a code blue. In addition, it is not necessary to contact the client's family unless the client has requested this.

84.) Baclofen (Lioresal) is prescribed for the client with multiple sclerosis. The nurse assists in planning care, knowing that the primary therapeutic effect of this medication is which of the following? 1. Increased muscle tone 2. Decreased muscle spasms 3. Increased range of motion 4. Decreased local pain and tenderness

2. Decreased muscle spasms Rationale: Baclofen is a skeletal muscle relaxant and central nervous system depressant and acts at the spinal cord level to decrease the frequency and amplitude of muscle spasms in clients with spinal cord injuries or diseases and in clients with multiple sclerosis. Options 1, 3, and 4 are incorrect.

40.) The client who chronically uses nonsteroidal anti-inflammatory drugs has been taking misoprostol (Cytotec). The nurse determines that the medication is having the intended therapeutic effect if which of the following is noted? 1. Resolved diarrhea 2. Relief of epigastric pain 3. Decreased platelet count 4. Decreased white blood cell count

2. Relief of epigastric pain Rationale: The client who chronically uses nonsteroidal anti-inflammatory drugs (NSAIDs) is prone to gastric mucosal injury. Misoprostol is a gastric protectant and is given specifically to prevent this occurrence. Diarrhea can be a side effect of the medication, but is not an intended effect. Options 3 and 4 are incorrect.

209.) A client with multiple sclerosis is receiving diazepam (Valium), a centrally acting skeletal muscle relaxant. Which of the following would indicate that the client is experiencing a side effect related to this medication? 1. Headache 2. Drowsiness 3. Urinary retention 4. Increased salivation

2. Drowsiness Rationale: Incoordination and drowsiness are common side effects resulting from this medication. Options 1, 3, and 4 are incorrect.

240.) A client with Parkinson's disease has been prescribed benztropine (Cogentin). The nurse monitors for which gastrointestinal (GI) side effect of this medication? 1. Diarrhea 2. Dry mouth 3. Increased appetite 4. Hyperactive bowel sounds

2. Dry mouth Rationale: Common GI side effects of benztropine therapy include constipation and dry mouth. Other GI side effects include nausea and ileus. These effects are the result of the anticholinergic properties of the medication. **Eliminate options 1 and 4 because they are comparable or alike. Recall that the medication is an anticholinergic, which causes dry mouth**

108.) A nurse is performing a follow-up teaching session with a client discharged 1 month ago who is taking fluoxetine (Prozac). What information would be important for the nurse to gather regarding the adverse effects related to the medication? 1. Cardiovascular symptoms 2. Gastrointestinal dysfunctions 3. Problems with mouth dryness 4. Problems with excessive sweating

2. Gastrointestinal dysfunctions Rationale: The most common adverse effects related to fluoxetine include central nervous system (CNS) and gastrointestinal (GI) system dysfunction. This medication affects the GI system by causing nausea and vomiting, cramping, and diarrhea. Options 1, 3, and 4 are not adverse effects of this medication.

136.) A nurse performs an admission assessment on a client who visits a health care clinic for the first time. The client tells the nurse that propylthiouracil (PTU) is taken daily. The nurse continues to collect data from the client, suspecting that the client has a history of: 1. Myxedema 2. Graves' disease 3. Addison's disease 4. Cushing's syndrome

2. Graves' disease Rationale: PTU inhibits thyroid hormone synthesis and is used to treat hyperthyroidism, or Graves' disease. Myxedema indicates hypothyroidism. Cushing's syndrome and Addison's disease are disorders related to adrenal function.

41.) The client has been taking omeprazole (Prilosec) for 4 weeks. The ambulatory care nurse evaluates that the client is receiving optimal intended effect of the medication if the client reports the absence of which symptom? 1. Diarrhea 2. Heartburn 3. Flatulence 4. Constipation

2. Heartburn Rationale: Omeprazole is a proton pump inhibitor classified as an antiulcer agent. The intended effect of the medication is relief of pain from gastric irritation, often called heartburn by clients. Omeprazole is not used to treat the conditions identified in options 1, 3, and 4.

131.) The nurse is reinforcing medication instructions to a client with breast cancer who is receiving cyclophosphamide (Neosar). The nurse tells the client to: 1. Take the medication with food. 2. Increase fluid intake to 2000 to 3000 mL daily. 3. Decrease sodium intake while taking the medication. 4. Increase potassium intake while taking the medication.

2. Increase fluid intake to 2000 to 3000 mL daily. Rationale: Hemorrhagic cystitis is a toxic effect that can occur with the use of cyclophosphamide. The client needs to be instructed to drink copious amounts of fluid during the administration of this medication. Clients also should monitor urine output for hematuria. The medication should be taken on an empty stomach, unless gastrointestinal (GI) upset occurs. Hyperkalemia can result from the use of the medication; therefore the client would not be told to increase potassium intake. The client would not be instructed to alter sodium intake.

101.) Ketoconazole is prescribed for a client with a diagnosis of candidiasis. Select the interventions that the nurse includes when administering this medication. Select all that apply. 1. Restrict fluid intake. 2. Instruct the client to avoid alcohol. 3. Monitor hepatic and liver function studies. 4. Administer the medication with an antacid. 5. Instruct the client to avoid exposure to the sun. 6. Administer the medication on an empty stomach.

2. Instruct the client to avoid alcohol. 3. Monitor hepatic and liver function studies. 5. Instruct the client to avoid exposure to the sun. Rationale: Ketoconazole is an antifungal medication. It is administered with food (not on an empty stomach) and antacids are avoided for 2 hours after taking the medication to ensure absorption. The medication is hepatotoxic and the nurse monitors liver function studies. The client is instructed to avoid exposure to the sun because the medication increases photosensitivity. The client is also instructed to avoid alcohol. There is no reason for the client to restrict fluid intake. In fact, this could be harmful to the client.

162.) Carbamazepine (Tegretol) is prescribed for a client with a diagnosis of psychomotor seizures. The nurse reviews the client's health history, knowing that this medication is contraindicated if which of the following disorders is present? 1. Headaches 2. Liver disease 3. Hypothyroidism 4. Diabetes mellitus

2. Liver disease Rationale: Carbamazepine (Tegretol) is contraindicated in liver disease, and liver function tests are routinely prescribed for baseline purposes and are monitored during therapy. It is also contraindicated if the client has a history of blood dyscrasias. It is not contraindicated in the conditions noted in the incorrect options.

89.) A nurse is reviewing the laboratory studies on a client receiving dantrolene sodium (Dantrium). Which laboratory test would identify an adverse effect associated with the administration of this medication? 1. Creatinine 2. Liver function tests 3. Blood urea nitrogen 4. Hematological function tests

2. Liver function tests Rationale: Dose-related liver damage is the most serious adverse effect of dantrolene. To reduce the risk of liver damage, liver function tests should be performed before treatment and periodically throughout the treatment course. It is administered in the lowest effective dosage for the shortest time necessary. **Eliminate options 1 and 3 because these tests both assess kidney function.**

65.) A nurse is reinforcing discharge instructions to a client receiving sulfisoxazole. Which of the following should be included in the list of instructions? 1. Restrict fluid intake. 2. Maintain a high fluid intake. 3. If the urine turns dark brown, call the health care provider (HCP) immediately. 4. Decrease the dosage when symptoms are improving to prevent an allergic response.

2. Maintain a high fluid intake. Rationale: Each dose of sulfisoxazole should be administered with a full glass of water, and the client should maintain a high fluid intake. The medication is more soluble in alkaline urine. The client should not be instructed to taper or discontinue the dose. Some forms of sulfisoxazole cause urine to turn dark brown or red. This does not indicate the need to notify the HCP.

161.) A nurse is caring for a client with severe back pain, and codeine sulfate has been prescribed for the client. Which of the following would the nurse include in the plan of care while the client is taking this medication? 1. Restrict fluid intake. 2. Monitor bowel activity. 3. Monitor for hypertension. 4. Monitor peripheral pulses.

2. Monitor bowel activity. Rationale: While the client is taking codeine sulfate, an opioid analgesic, the nurse would monitor vital signs and monitor for hypotension. The nurse should also increase fluid intake, palpate the bladder for urinary retention, auscultate bowel sounds, and monitor the pattern of daily bowel activity and stool consistency (codeine can cause constipation). The nurse should monitor respiratory status and initiate breathing and coughing exercises. In addition, the nurse monitors the effectiveness of the pain medication.

204.) A client receives a dose of edrophonium (Enlon). The client shows improvement in muscle strength for a period of time following the injection. The nurse interprets that this finding is compatible with: 1. Multiple sclerosis 2. Myasthenia gravis 3. Muscular dystrophy 4. Amyotrophic lateral sclerosis

2. Myasthenia gravis Rationale: Myasthenia gravis can often be diagnosed based on clinical signs and symptoms. The diagnosis can be confirmed by injecting the client with a dose of edrophonium . This medication inhibits the breakdown of an enzyme in the neuromuscular junction, so more acetylcholine binds to receptors. If the muscle is strengthened for 3 to 5 minutes after this injection, it confirms a diagnosis of myasthenia gravis. Another medication, neostigmine (Prostigmin), also may be used because its effect lasts for 1 to 2 hours, providing a better analysis. For either medication, atropine sulfate should be available as the antidote.

169.) Insulin glargine (Lantus) is prescribed for a client with diabetes mellitus. The nurse tells the client that it is best to take the insulin: 1. 1 hour after each meal 2. Once daily, at the same time each day 3. 15 minutes before breakfast, lunch, and dinner 4. Before each meal, on the basis of the blood glucose level

2. Once daily, at the same time each day Rationale: Insulin glargine is a long-acting recombinant DNA human insulin used to treat type 1 and type 2 diabetes mellitus. It has a 24-hour duration of action and is administered once a day, at the same time each day.

47.) A client has been taking isoniazid (INH) for 2 months. The client complains to a nurse about numbness, paresthesias, and tingling in the extremities. The nurse interprets that the client is experiencing: 1. Hypercalcemia 2. Peripheral neuritis 3. Small blood vessel spasm 4. Impaired peripheral circulation

2. Peripheral neuritis Rationale: A common side effect of the TB drug INH is peripheral neuritis. This is manifested by numbness, tingling, and paresthesias in the extremities. This side effect can be minimized by pyridoxine (vitamin B6) intake. Options 1, 3, and 4 are incorrect.

228.) A client receiving an anxiolytic medication complains that he feels very "faint" when he tries to get out of bed in the morning. The nurse recognizes this complaint as a symptom of: 1. Cardiac dysrhythmias 2. Postural hypotension 3. Psychosomatic symptoms 4. Respiratory insufficiency

2. Postural hypotension Rationale: Anxiolytic medications can cause postural hypotension. The client needs to be taught to rise to a sitting position and get out of bed slowly because of this adverse effect related to the medication. Options 1, 3, and 4 are unrelated to the use of this medication.

50.) A nurse has given a client taking ethambutol (Myambutol) information about the medication. The nurse determines that the client understands the instructions if the client states that he or she will immediately report: 1. Impaired sense of hearing 2. Problems with visual acuity 3. Gastrointestinal (GI) side effects 4. Orange-red discoloration of body secretions

2. Problems with visual acuity Rationale: Ethambutol causes optic neuritis, which decreases visual acuity and the ability to discriminate between the colors red and green. This poses a potential safety hazard when a client is driving a motor vehicle. The client is taught to report this symptom immediately. The client is also taught to take the medication with food if GI upset occurs. Impaired hearing results from antitubercular therapy with streptomycin. Orange-red discoloration of secretions occurs with rifampin (Rifadin).

25.) A home care nurse visits a client recently diagnosed with diabetes mellitus who is taking Humulin NPH insulin daily. The client asks the nurse how to store the unopened vials of insulin. The nurse tells the client to: 1. Freeze the insulin. 2. Refrigerate the insulin. 3. Store the insulin in a dark, dry place. 4. Keep the insulin at room temperature.

2. Refrigerate the insulin. Rationale: Insulin in unopened vials should be stored under refrigeration until needed. Vials should not be frozen. When stored unopened under refrigeration, insulin can be used up to the expiration date on the vial. Options 1, 3, and 4 are incorrect.

186.) A nurse prepares to administer sodium polystyrene sulfonate (Kayexalate) to a client. Before administering the medication, the nurse reviews the action of the medication and understands that it: 1. Releases bicarbonate in exchange for primarily sodium ions 2. Releases sodium ions in exchange for primarily potassium ions 3. Releases potassium ions in exchange for primarily sodium ions 4. Releases sodium ions in exchange for primarily bicarbonate ions

2. Releases sodium ions in exchange for primarily potassium ions Rationale: Sodium polystyrene sulfonate is a cation exchange resin used in the treatment of hyperkalemia. The resin either passes through the intestine or is retained in the colon. It releases sodium ions in exchange for primarily potassium ions. The therapeutic effect occurs 2 to 12 hours after oral administration and longer after rectal administration.

48.) A client is to begin a 6-month course of therapy with isoniazid (INH). A nurse plans to teach the client to: 1. Drink alcohol in small amounts only. 2. Report yellow eyes or skin immediately. 3. Increase intake of Swiss or aged cheeses. 4. Avoid vitamin supplements during therapy.

2. Report yellow eyes or skin immediately. Rationale: INH is hepatotoxic, and therefore the client is taught to report signs and symptoms of hepatitis immediately (which include yellow skin and sclera). For the same reason, alcohol should be avoided during therapy. The client should avoid intake of Swiss cheese, fish such as tuna, and foods containing tyramine because they may cause a reaction characterized by redness and itching of the skin, flushing, sweating, tachycardia, headache, or lightheadedness. The client can avoid developing peripheral neuritis by increasing the intake of pyridoxine (vitamin B6) during the course of INH therapy for TB.

23.) A client who has been newly diagnosed with diabetes mellitus has been stabilized with daily insulin injections. Which information should the nurse teach when carrying out plans for discharge? 1. Keep insulin vials refrigerated at all times. 2. Rotate the insulin injection sites systematically. 3. Increase the amount of insulin before unusual exercise. 4. Monitor the urine acetone level to determine the insulin dosage.

2. Rotate the insulin injection sites systematically. Rationale: Insulin dosages should not be adjusted or increased before unusual exercise. If acetone is found in the urine, it may possibly indicate the need for additional insulin. To minimize the discomfort associated with insulin injections, the insulin should be administered at room temperature. Injection sites should be systematically rotated from one area to another. The client should be instructed to give injections in one area, about 1 inch apart, until the whole area has been used and then to change to another site. This prevents dramatic changes in daily insulin absorption.

216.) A nurse is caring for a client with gout who is taking Colcrys (colchicine). The client has been instructed to restrict the diet to low-purine foods. Which of the following foods should the nurse instruct the client to avoid while taking this medication? 1. Spinach 2. Scallops 3. Potatoes 4. Ice cream

2. Scallops Rationale: Colchicine is a medication used for clients with gout to inhibit the reabsorption of uric acid by the kidney and promote excretion of uric acid in the urine. Uric acid is produced when purine is catabolized. Clients are instructed to modify their diet and limit excessive purine intake. High-purine foods to avoid or limit include organ meats, roe, sardines, scallops, anchovies, broth, mincemeat, herring, shrimp, mackerel, gravy, and yeast.

114.) A postoperative client requests medication for flatulence (gas pains). Which medication from the following PRN list should the nurse administer to this client? 1. Ondansetron (Zofran) 2. Simethicone (Mylicon) 3. Acetaminophen (Tylenol) 4. Magnesium hydroxide (milk of magnesia, MOM)

2. Simethicone (Mylicon) Rationale: Simethicone is an antiflatulent used in the relief of pain caused by excessive gas in the gastrointestinal tract. Ondansetron is used to treat postoperative nausea and vomiting. Acetaminophen is a nonopioid analgesic. Magnesium hydroxide is an antacid and laxative.

172.) A nurse provides dietary instructions to a client who will be taking warfarin sodium (Coumadin). The nurse tells the client to avoid which food item? 1. Grapes 2. Spinach 3. Watermelon 4. Cottage cheese

2. Spinach Rationale: Warfarin sodium is an anticoagulant. Anticoagulant medications act by antagonizing the action of vitamin K, which is needed for clotting. When a client is taking an anticoagulant, foods high in vitamin K often are omitted from the diet. Vitamin K-rich foods include green, leafy vegetables, fish, liver, coffee, and tea.

147.) A client with a prescription to take theophylline (Theo-24) daily has been given medication instructions by the nurse. The nurse determines that the client needs further information about the medication if the client states that he or she will: 1. Drink at least 2 L of fluid per day. 2. Take the daily dose at bedtime. 3. Avoid changing brands of the medication without health care provider (HCP) approval. 4. Avoid over-the-counter (OTC) cough and cold medications unless approved by the HCP.

2. Take the daily dose at bedtime. Rationale: The client taking a single daily dose of theophylline, a xanthine bronchodilator, should take the medication early in the morning. This enables the client to have maximal benefit from the medication during daytime activities. In addition, this medication causes insomnia. The client should take in at least 2 L of fluid per day to decrease viscosity of secretions. The client should check with the physician before changing brands of the medication. The client also checks with the HCP before taking OTC cough, cold, or other respiratory preparations because they could cause interactive effects, increasing the side effects of theophylline and causing dysrhythmias.

45.) A client has a prescription to take guaifenesin (Humibid) every 4 hours, as needed. The nurse determines that the client understands the most effective use of this medication if the client states that he or she will: 1. Watch for irritability as a side effect. 2. Take the tablet with a full glass of water. 3. Take an extra dose if the cough is accompanied by fever. 4. Crush the sustained-release tablet if immediate relief is needed.

2. Take the tablet with a full glass of water. Rationale: Guaifenesin is an expectorant. It should be taken with a full glass of water to decrease viscosity of secretions. Sustained-release preparations should not be broken open, crushed, or chewed. The medication may occasionally cause dizziness, headache, or drowsiness as side effects. The client should contact the health care provider if the cough lasts longer than 1 week or is accompanied by fever, rash, sore throat, or persistent headache.

214.) A health care provider initiates carbidopa/levodopa (Sinemet) therapy for the client with Parkinson's disease. A few days after the client starts the medication, the client complains of nausea and vomiting. The nurse tells the client that: 1. Taking an antiemetic is the best measure to prevent the nausea. 2. Taking the medication with food will help to prevent the nausea. 3. This is an expected side effect of the medication and will decrease over time. 4. The nausea and vomiting will decrease when the dose of levodopa is stabilized.

2. Taking the medication with food will help to prevent the nausea. Rationale: If carbidopa/levodopa is causing nausea and vomiting, the nurse would tell the client that taking the medication with food will prevent the nausea. Additionally, the client should be instructed not to take the medication with a high-protein meal because the high-protein will affect absorption. Antiemetics from the phenothiazine class should not be used because they block the therapeutic action of dopamine. **eliminate options 3 and 4 because they are comparable or alike**

57.) A nurse is monitoring a client who is taking propranolol (Inderal LA). Which data collection finding would indicate a potential serious complication associated with propranolol? 1. The development of complaints of insomnia 2. The development of audible expiratory wheezes 3. A baseline blood pressure of 150/80 mm Hg followed by a blood pressure of 138/72 mm Hg after two doses of the medication 4. A baseline resting heart rate of 88 beats/min followed by a resting heart rate of 72 beats/min after two doses of the medication

2. The development of audible expiratory wheezes Rationale: Audible expiratory wheezes may indicate a serious adverse reaction, bronchospasm. β-Blockers may induce this reaction, particularly in clients with chronic obstructive pulmonary disease or asthma. Normal decreases in blood pressure and heart rate are expected. Insomnia is a frequent mild side effect and should be monitored.

28.) The health care provider (HCP) prescribes exenatide (Byetta) for a client with type 1 diabetes mellitus who takes insulin. The nurse knows that which of the following is the appropriate intervention? 1. The medication is administered within 60 minutes before the morning and evening meal. 2. The medication is withheld and the HCP is called to question the prescription for the client. 3. The client is monitored for gastrointestinal side effects after administration of the medication. 4. The insulin is withdrawn from the Penlet into an insulin syringe to prepare for administration.

2. The medication is withheld and the HCP is called to question the prescription for the client. Rationale: Exenatide (Byetta) is an incretin mimetic used for type 2 diabetes mellitus only. It is not recommended for clients taking insulin. Hence, the nurse should hold the medication and question the HCP regarding this prescription. Although options 1 and 3 are correct statements about the medication, in this situation the medication should not be administered. The medication is packaged in prefilled pens ready for injection without the need for drawing it up into another syringe.

165.) The client has been on treatment for rheumatoid arthritis for 3 weeks. During the administration of etanercept (Enbrel), it is most important for the nurse to assess: 1. The injection site for itching and edema 2. The white blood cell counts and platelet counts 3. Whether the client is experiencing fatigue and joint pain 4. A metallic taste in the mouth and a loss of appetite

2. The white blood cell counts and platelet counts Rationale: Infection and pancytopenia are adverse effects of etanercept (Enbrel). Laboratory studies are performed before and during treatment. The appearance of abnormal white blood cell counts and abnormal platelet counts can alert the nurse to a potential life-threatening infection. Injection site itching is a common occurrence following administration of the medication. In early treatment, residual fatigue and joint pain may still be apparent. A metallic taste and loss of appetite are not common signs of side effects of this medication.

83.) The client has been on treatment for rheumatoid arthritis for 3 weeks. During the administration of etanercept (Enbrel), it is most important for the nurse to check: 1. The injection site for itching and edema 2. The white blood cell counts and platelet counts 3. Whether the client is experiencing fatigue and joint pain 4. A metallic taste in the mouth, with a loss of appetite

2. The white blood cell counts and platelet counts Rationale: Infection and pancytopenia are side effects of etanercept (Enbrel). Laboratory studies are performed before and during drug treatment. The appearance of abnormal white blood cell counts and abnormal platelet counts can alert the nurse to a potentially life-threatening infection. Injection site itching is a common occurrence following administration. A metallic taste with loss of appetite are not common signs of side effects of this medication.

126.) A nurse is caring for a client who has been prescribed furosemide (Lasix) and is monitoring for adverse effects associated with this medication. Which of the following should the nurse recognize as a potential adverse effect Select all that apply. 1. Nausea 2. Tinnitus 3. Hypotension 4. Hypokalemia 5. Photosensitivity 6. Increased urinary frequency

2. Tinnitus 3. Hypotension 4. Hypokalemia Rationale: Furosemide is a loop diuretic; therefore, an expected effect is increased urinary frequency. Nausea is a frequent side effect, not an adverse effect. Photosensitivity is an occasional side effect. Adverse effects include tinnitus (ototoxicity), hypotension, and hypokalemia and occur as a result of sudden volume depletion.

51.) Cycloserine (Seromycin) is added to the medication regimen for a client with tuberculosis. Which of the following would the nurse include in the client-teaching plan regarding this medication? 1. To take the medication before meals 2. To return to the clinic weekly for serum drug-level testing 3. It is not necessary to call the health care provider (HCP) if a skin rash occurs. 4. It is not necessary to restrict alcohol intake with this medication.

2. To return to the clinic weekly for serum drug-level testing Rationale: Cycloserine (Seromycin) is an antitubercular medication that requires weekly serum drug level determinations to monitor for the potential of neurotoxicity. Serum drug levels lower than 30 mcg/mL reduce the incidence of neurotoxicity. The medication must be taken after meals to prevent gastrointestinal irritation. The client must be instructed to notify the HCP if a skin rash or signs of central nervous system toxicity are noted. Alcohol must be avoided because it increases the risk of seizure activity.

7.) Isotretinoin is prescribed for a client with severe acne. Before the administration of this medication, the nurse anticipates that which laboratory test will be prescribed? 1. Platelet count 2. Triglyceride level 3. Complete blood count 4. White blood cell count

2. Triglyceride level Rationale: Isotretinoin can elevate triglyceride levels. Blood triglyceride levels should be measured before treatment and periodically thereafter until the effect on the triglycerides has been evaluated. Options 1, 3, and 4 do not need to be monitored specifically during this treatment.

14.) The client with acute myelocytic leukemia is being treated with busulfan (Myleran). Which laboratory value would the nurse specifically monitor during treatment with this medication? 1. Clotting time 2. Uric acid level 3. Potassium level 4. Blood glucose level

2. Uric acid level Rationale: Busulfan (Myleran) can cause an increase in the uric acid level. Hyperuricemia can produce uric acid nephropathy, renal stones, and acute renal failure. Options 1, 3, and 4 are not specifically related to this medication.

68.) Bethanechol chloride (Urecholine) is prescribed for a client with urinary retention. Which disorder would be a contraindication to the administration of this medication? 1. Gastric atony 2. Urinary strictures 3. Neurogenic atony 4. Gastroesophageal reflux

2. Urinary strictures Rationale: Bethanechol chloride (Urecholine) can be harmful to clients with urinary tract obstruction or weakness of the bladder wall. The medication has the ability to contract the bladder and thereby increase pressure within the urinary tract. Elevation of pressure within the urinary tract could rupture the bladder in clients with these conditions.

55.) A client who is receiving digoxin (Lanoxin) daily has a serum potassium level of 3.0 mEq/L and is complaining of anorexia. A health care provider prescribes a digoxin level to rule out digoxin toxicity. A nurse checks the results, knowing that which of the following is the therapeutic serum level (range) for digoxin? 1. 3 to 5 ng/mL 2. 0.5 to 2 ng/mL 3. 1.2 to 2.8 ng/mL 4. 3.5 to 5.5 ng/mL

2.) 0.5 to 2 ng/mL Rationale: Therapeutic levels for digoxin range from 0.5 to 2 ng/mL. Therefore, options 1, 3, and 4 are incorrect.

128.) A nurse is providing instructions to an adolescent who has a history of seizures and is taking an anticonvulsant medication. Which of the following statements indicates that the client understands the instructions? 1. "I will never be able to drive a car." 2. "My anticonvulsant medication will clear up my skin." 3. "I can't drink alcohol while I am taking my medication." 4. "If I forget my morning medication, I can take two pills at bedtime."

3. "I can't drink alcohol while I am taking my medication." Rationale: Alcohol will lower the seizure threshold and should be avoided. Adolescents can obtain a driver's license in most states when they have been seizure free for 1 year. Anticonvulsants cause acne and oily skin; therefore a dermatologist may need to be consulted. If an anticonvulsant medication is missed, the health care provider should be notified.

197.) Collagenase (Santyl) is prescribed for a client with a severe burn to the hand. The nurse provides instructions to the client regarding the use of the medication. Which statement by the client indicates an accurate understanding of the use of this medication? 1. "I will apply the ointment once a day and leave it open to the air." 2. "I will apply the ointment twice a day and leave it open to the air." 3. "I will apply the ointment once a day and cover it with a sterile dressing." 4. "I will apply the ointment at bedtime and in the morning and cover it with a sterile dressing."

3. "I will apply the ointment once a day and cover it with a sterile dressing." Rationale: Collagenase is used to promote debridement of dermal lesions and severe burns. It is usually applied once daily and covered with a sterile dressing.

164.) A client receives a prescription for methocarbamol (Robaxin), and the nurse reinforces instructions to the client regarding the medication. Which client statement would indicate a need for further instructions? 1. "My urine may turn brown or green." 2. "This medication is prescribed to help relieve my muscle spasms." 3. "If my vision becomes blurred, I don't need to be concerned about it." 4. "I need to call my doctor if I experience nasal congestion from this medication."

3. "If my vision becomes blurred, I don't need to be concerned about it." Rationale: The client needs to be told that the urine may turn brown, black, or green. Other adverse effects include blurred vision, nasal congestion, urticaria, and rash. The client needs to be instructed that, if these adverse effects occur, the health care provider needs to be notified. The medication is used to relieve muscle spasms.

11.) The health care provider has prescribed silver sulfadiazine (Silvadene) for the client with a partial-thickness burn, which has cultured positive for gram-negative bacteria. The nurse is reinforcing information to the client about the medication. Which statement made by the client indicates a lack of understanding about the treatments? 1. "The medication is an antibacterial." 2. "The medication will help heal the burn." 3. "The medication will permanently stain my skin." 4. "The medication should be applied directly to the wound."

3. "The medication will permanently stain my skin." Rationale: Silver sulfadiazine (Silvadene) is an antibacterial that has a broad spectrum of activity against gram-negative bacteria, gram-positive bacteria, and yeast. It is applied directly to the wound to assist in healing. It does not stain the skin.

42.) A client with a peptic ulcer is diagnosed with a Helicobacter pylori infection. The nurse is reinforcing teaching for the client about the medications prescribed, including clarithromycin (Biaxin), esomeprazole (Nexium), and amoxicillin (Amoxil). Which statement by the client indicates the best understanding of the medication regimen? 1. "My ulcer will heal because these medications will kill the bacteria." 2. "These medications are only taken when I have pain from my ulcer." 3. "The medications will kill the bacteria and stop the acid production." 4. "These medications will coat the ulcer and decrease the acid production in my stomach."

3. "The medications will kill the bacteria and stop the acid production." Rationale: Triple therapy for Helicobacter pylori infection usually includes two antibacterial drugs and a proton pump inhibitor. Clarithromycin and amoxicillin are antibacterials. Esomeprazole is a proton pump inhibitor. These medications will kill the bacteria and decrease acid production.

141.) The nurse has reinforced instructions to a client who has been prescribed cholestyramine (Questran). Which statement by the client indicates a need for further instructions? 1. "I will continue taking vitamin supplements." 2. "This medication will help lower my cholesterol." 3. "This medication should only be taken with water." 4. "A high-fiber diet is important while taking this medication."

3. "This medication should only be taken with water." Rationale: Cholestyramine (Questran) is a bile acid sequestrant used to lower the cholesterol level, and client compliance is a problem because of its taste and palatability. The use of flavored products or fruit juices can improve the taste. Some side effects of bile acid sequestrants include constipation and decreased vitamin absorption. **Note the closed-ended word "only" in option 3**

219.) A health care provider instructs a client with rheumatoid arthritis to take ibuprofen (Motrin). The nurse reinforces the instructions, knowing that the normal adult dose for this client is which of the following? 1. 100 mg orally twice a day 2. 200 mg orally twice a day 3. 400 mg orally three times a day 4. 1000 mg orally four times a day

3. 400 mg orally three times a day Rationale: For acute or chronic rheumatoid arthritis or osteoarthritis, the normal oral adult dose is 400 to 800 mg three or four times daily.

191.) A child is brought to the emergency department for treatment of an acute asthma attack. The nurse prepares to administer which of the following medications first? 1. Oral corticosteroids 2. A leukotriene modifier 3. A β2 agonist 4. A nonsteroidal anti-inflammatory

3. A β2 agonist Rationale: In treating an acute asthma attack, a short acting β2 agonist such as albuterol (Proventil HFA) will be given to produce bronchodilation. Options 1, 2, and 4 are long-term control (preventive) medications.

120.) A client is taking lansoprazole (Prevacid) for the chronic management of Zollinger-Ellison syndrome. The nurse advises the client to take which of the following products if needed for a headache? 1. Naprosyn (Aleve) 2. Ibuprofen (Advil) 3. Acetaminophen (Tylenol) 4. Acetylsalicylic acid (aspirin)

3. Acetaminophen (Tylenol) Rationale: Zollinger-Ellison syndrome is a hypersecretory condition of the stomach. The client should avoid taking medications that are irritating to the stomach lining. Irritants would include aspirin and nonsteroidal antiinflammatory drugs (ibuprofen). The client should be advised to take acetaminophen for headache. **Remember that options that are comparable or alike are not likely to be correct. With this in mind, eliminate options 1 and 2 first.**

233.) Diphenhydramine hydrochloride (Benadryl) is used in the treatment of allergic rhinitis for a hospitalized client with a chronic psychotic disorder. The client asks the nurse why the medication is being discontinued before hospital discharge. The nurse responds, knowing that: 1. Allergic symptoms are short in duration. 2. This medication promotes long-term extrapyramidal symptoms. 3. Addictive properties are enhanced in the presence of psychotropic medications. 4. Poor compliance causes this medication to fail to reach its therapeutic blood level.

3. Addictive properties are enhanced in the presence of psychotropic medications. Rationale: The addictive properties of diphenhydramine hydrochloride are enhanced when used with psychotropic medications. Allergic symptoms may not be short term and will occur if allergens are present in the environment. Poor compliance may be a problem with psychotic clients but is not the subject of the question. Diphenhydramine hydrochloride may be used for extrapyramidal symptoms and mild medication-induced movement disorders.

107.) A client receiving a tricyclic antidepressant arrives at the mental health clinic. Which observation indicates that the client is correctly following the medication plan? 1. Reports not going to work for this past week 2. Complains of not being able to "do anything" anymore 3. Arrives at the clinic neat and appropriate in appearance 4. Reports sleeping 12 hours per night and 3 to 4 hours during the day

3. Arrives at the clinic neat and appropriate in appearance Rationale: Depressed individuals will sleep for long periods, are not able to go to work, and feel as if they cannot "do anything." Once they have had some therapeutic effect from their medication, they will report resolution of many of these complaints as well as demonstrate an improvement in their appearance.

153.) A client is diagnosed with pulmonary embolism and is to be treated with streptokinase (Streptase). A nurse would report which priority data collection finding to the registered nurse before initiating this therapy? 1. Adventitious breath sounds 2. Temperature of 99.4° F orally 3. Blood pressure of 198/110 mm Hg 4. Respiratory rate of 28 breaths/min

3. Blood pressure of 198/110 mm Hg Rationale: Thrombolytic therapy is contraindicated in a number of preexisting conditions in which there is a risk of uncontrolled bleeding, similar to the case in anticoagulant therapy. Thrombolytic therapy also is contraindicated in severe uncontrolled hypertension because of the risk of cerebral hemorrhage. Therefore the nurse would report the results of the blood pressure to the registered nurse before initiating therapy. The findings in options 1, 2, and 4 may be present in the client with pulmonary embolism.

69.) A nurse who is administering bethanechol chloride (Urecholine) is monitoring for acute toxicity associated with the medication. The nurse checks the client for which sign of toxicity? 1. Dry skin 2. Dry mouth 3. Bradycardia 4. Signs of dehydration

3. Bradycardia Rationale: Toxicity (overdose) produces manifestations of excessive muscarinic stimulation such as salivation, sweating, involuntary urination and defecation, bradycardia, and severe hypotension. Treatment includes supportive measures and the administration of atropine sulfate subcutaneously or intravenously.

49.) A client has been started on long-term therapy with rifampin (Rifadin). A nurse teaches the client that the medication: 1. Should always be taken with food or antacids 2. Should be double-dosed if one dose is forgotten 3. Causes orange discoloration of sweat, tears, urine, and feces 4. May be discontinued independently if symptoms are gone in 3 months

3. Causes orange discoloration of sweat, tears, urine, and feces Rationale: Rifampin should be taken exactly as directed as part of TB therapy. Doses should not be doubled or skipped. The client should not stop therapy until directed to do so by a health care provider. The medication should be administered on an empty stomach unless it causes gastrointestinal upset, and then it may be taken with food. Antacids, if prescribed, should be taken at least 1 hour before the medication. Rifampin causes orange-red discoloration of body secretions and will permanently stain soft contact lenses.

38.) An older client recently has been taking cimetidine (Tagamet). The nurse monitors the client for which most frequent central nervous system side effect of this medication? 1. Tremors 2. Dizziness 3. Confusion 4. Hallucinations

3. Confusion Rationale: Cimetidine is a histamine 2 (H2)-receptor antagonist. Older clients are especially susceptible to central nervous system side effects of cimetidine. The most frequent of these is confusion. Less common central nervous system side effects include headache, dizziness, drowsiness, and hallucinations.

16.) The clinic nurse is reviewing a teaching plan for the client receiving an antineoplastic medication. When implementing the plan, the nurse tells the client: 1. To take aspirin (acetylsalicylic acid) as needed for headache 2. Drink beverages containing alcohol in moderate amounts each evening 3. Consult with health care providers (HCPs) before receiving immunizations 4. That it is not necessary to consult HCPs before receiving a flu vaccine at the local health fair

3. Consult with health care providers (HCPs) before receiving immunizations Rationale: Because antineoplastic medications lower the resistance of the body, clients must be informed not to receive immunizations without a HCP's approval. Clients also need to avoid contact with individuals who have recently received a live virus vaccine. Clients need to avoid aspirin and aspirin-containing products to minimize the risk of bleeding, and they need to avoid alcohol to minimize the risk of toxicity and side effects.

150.) A client complaining of not feeling well is seen in a clinic. The client is taking several medications for the control of heart disease and hypertension. These medications include a β-blocker, digoxin (Lanoxin), and a diuretic. A tentative diagnosis of digoxin toxicity is made. Which of the following assessment data would support this diagnosis? 1. Dyspnea, edema, and palpitations 2. Chest pain, hypotension, and paresthesia 3. Double vision, loss of appetite, and nausea 4. Constipation, dry mouth, and sleep disorder

3. Double vision, loss of appetite, and nausea Rationale: Double vision, loss of appetite, and nausea are signs of digoxin toxicity. Additional signs of digoxin toxicity include bradycardia, difficulty reading, visual alterations such as green and yellow vision or seeing spots or halos, confusion, vomiting, diarrhea, decreased libido, and impotence. **gastrointestinal (GI) and visual disturbances occur with digoxin toxicity**

148.) A client is taking cetirizine hydrochloride (Zyrtec). The nurse checks for which of the following side effects of this medication? 1. Diarrhea 2. Excitability 3. Drowsiness 4. Excess salivation

3. Drowsiness Rationale: A frequent side effect of cetirizine hydrochloride (Zyrtec), an antihistamine, is drowsiness or sedation. Others include blurred vision, hypertension (and sometimes hypotension), dry mouth, constipation, urinary retention, and sweating.

85.) A nurse is monitoring a client receiving baclofen (Lioresal) for side effects related to the medication. Which of the following would indicate that the client is experiencing a side effect? 1. Polyuria 2. Diarrhea 3. Drowsiness 4. Muscular excitability

3. Drowsiness Rationale: Baclofen is a central nervous system (CNS) depressant and frequently causes drowsiness, dizziness, weakness, and fatigue. It can also cause nausea, constipation, and urinary retention. Clients should be warned about the possible reactions. Options 1, 2, and 4 are not side effects.

138.) A daily dose of prednisone is prescribed for a client. A nurse reinforces instructions to the client regarding administration of the medication and instructs the client that the best time to take this medication is: 1. At noon 2. At bedtime 3. Early morning 4. Anytime, at the same time, each day

3. Early morning Rationale: Corticosteroids (glucocorticoids) should be administered before 9:00 AM. Administration at this time helps minimize adrenal insufficiency and mimics the burst of glucocorticoids released naturally by the adrenal glands each morning. **Note the suffix "-sone," and recall that medication names that end with these letters are corticosteroids.**

71.) After kidney transplantation, cyclosporine (Sand immune) is prescribed for a client. Which laboratory result would indicate an adverse effect from the use of this medication? 1. Decreased creatinine level 2. Decreased hemoglobin level 3. Elevated blood urea nitrogen level 4. Decreased white blood cell count

3. Elevated blood urea nitrogen level Rationale: Nephrotoxicity can occur from the use of cyclosporine (Sandimmune). Nephrotoxicity is evaluated by monitoring for elevated blood urea nitrogen (BUN) and serum creatinine levels. Cyclosporine is an immunosuppressant but does not depress the bone marrow.

244.) A client has a prescription for valproic acid (Depakene) orally once daily. The nurse plans to: 1. Administer the medication with an antacid. 2. Administer the medication with a carbonated beverage. 3. Ensure that the medication is administered at the same time each day. 4. Ensure that the medication is administered 2 hours before breakfast only, when the client's stomach is empty.

3. Ensure that the medication is administered at the same time each day. Rationale: Valproic acid is an anticonvulsant, antimanic, and antimigraine medication. It may be administered with or without food. It should not be taken with an antacid or carbonated beverage because these products will affect medication absorption. The medication is administered at the same time each day to maintain therapeutic serum levels. **Use general pharmacology guidelines to assist in eliminating options 1 and 2. Eliminate option 4 because of the closed-ended word "only."**

154.) A nurse is reinforcing dietary instructions to a client who has been prescribed cyclosporine (Sandimmune). Which food item would the nurse instruct the client to avoid? 1. Red meats 2. Orange juice 3. Grapefruit juice 4. Green, leafy vegetables

3. Grapefruit juice Rationale: A compound present in grapefruit juice inhibits metabolism of cyclosporine. As a result, the consumption of grapefruit juice can raise cyclosporine levels by 50% to 100%, thereby greatly increasing the risk of toxicity. Grapefruit juice needs to be avoided. Red meats, orange juice, and green leafy vegetables are acceptable to consume.

97.) Amikacin (Amikin) is prescribed for a client with a bacterial infection. The client is instructed to contact the health care provider (HCP) immediately if which of the following occurs? 1. Nausea 2. Lethargy 3. Hearing loss 4. Muscle aches

3. Hearing loss Rationale: Amikacin (Amikin) is an aminoglycoside. Adverse effects of aminoglycosides include ototoxicity (hearing problems), confusion, disorientation, gastrointestinal irritation, palpitations, blood pressure changes, nephrotoxicity, and hypersensitivity. The nurse instructs the client to report hearing loss to the HCP immediately. Lethargy and muscle aches are not associated with the use of this medication. It is not necessary to contact the HCP immediately if nausea occurs. If nausea persists or results in vomiting, the HCP should be notified. **(most aminoglycoside medication names end in the letters -cin)**

60.) A nurse is planning to administer hydrochlorothiazide (HydroDIURIL) to a client. The nurse understands that which of the following are concerns related to the administration of this medication? 1. Hypouricemia, hyperkalemia 2. Increased risk of osteoporosis 3. Hypokalemia, hyperglycemia, sulfa allergy 4. Hyperkalemia, hypoglycemia, penicillin allergy

3. Hypokalemia, hyperglycemia, sulfa allergy Rationale: Thiazide diuretics such as hydrochlorothiazide are sulfa-based medications, and a client with a sulfa allergy is at risk for an allergic reaction. Also, clients are at risk for hypokalemia, hyperglycemia, hypercalcemia, hyperlipidemia, and hyperuricemia.

130.) The nurse is analyzing the laboratory results of a client with leukemia who has received a regimen of chemotherapy. Which laboratory value would the nurse specifically note as a result of the massive cell destruction that occurred from the chemotherapy? 1. Anemia 2. Decreased platelets 3. Increased uric acid level 4. Decreased leukocyte count

3. Increased uric acid level Rationale: Hyperuricemia is especially common following treatment for leukemias and lymphomas because chemotherapy results in a massive cell kill. Although options 1, 2, and 4 also may be noted, an increased uric acid level is related specifically to cell destruction.

122.) A client who has begun taking fosinopril (Monopril) is very distressed, telling the nurse that he cannot taste food normally since beginning the medication 2 weeks ago. The nurse provides the best support to the client by: 1. Telling the client not to take the medication with food 2. Suggesting that the client taper the dose until taste returns to normal 3. Informing the client that impaired taste is expected and generally disappears in 2 to 3 months 4. Requesting that the health care provider (HCP) change the prescription to another brand of angiotensin-converting enzyme (ACE) inhibitor

3. Informing the client that impaired taste is expected and generally disappears in 2 to 3 months Rationale: ACE inhibitors, such as fosinopril, cause temporary impairment of taste (dysgeusia). The nurse can tell the client that this effect usually disappears in 2 to 3 months, even with continued therapy, and provide nutritional counseling if appropriate to avoid weight loss. Options 1, 2, and 4 are inappropriate actions. Taking this medication with or without food does not affect absorption and action. The dosage should never be tapered without HCP approval and the medication should never be stopped abruptly.

5.) Mafenide acetate (Sulfamylon) is prescribed for the client with a burn injury. When applying the medication, the client complains of local discomfort and burning. Which of the following is the most appropriate nursing action? 1. Notifying the registered nurse 2. Discontinuing the medication 3. Informing the client that this is normal 4. Applying a thinner film than prescribed to the burn site

3. Informing the client that this is normal Rationale: Mafenide acetate is bacteriostatic for gram-negative and gram-positive organisms and is used to treat burns to reduce bacteria present in avascular tissues. The client should be informed that the medication will cause local discomfort and burning and that this is a normal reaction; therefore options 1, 2, and 4 are incorrect

243.) A hospitalized client is having the dosage of clonazepam (Klonopin) adjusted. The nurse should plan to: 1. Weigh the client daily. 2. Observe for ecchymosis. 3. Institute seizure precautions. 4. Monitor blood glucose levels.

3. Institute seizure precautions. Rationale: Clonazepam is a benzodiazepine used as an anticonvulsant. During initial therapy and during periods of dosage adjustment, the nurse should initiate seizure precautions for the client. Options 1, 2, and 4 are not associated with the use of this medication.

241.) A client with a history of simple partial seizures is taking clorazepate (Tranxene), and asks the nurse if there is a risk of addiction. The nurse's response is based on the understanding that clorazepate: 1. Is not habit forming, either physically or psychologically 2. Leads to physical tolerance, but only after 10 or more years of therapy 3. Leads to physical and psychological dependence with prolonged high-dose therapy 4. Can result in psychological dependence only, because of the nature of the medication

3. Leads to physical and psychological dependence with prolonged high-dose therapy Rationale: Clorazepate is classified as an anticonvulsant, antianxiety agent, and sedative-hypnotic (benzodiazepine). One of the concerns with clorazepate therapy is that the medication can lead to physical or psychological dependence with prolonged therapy at high doses. For this reason, the amount of medication that is readily available to the client at any one time is restricted. **Eliminate options 2 and 4 first because of the closed-ended word "only"**

52.) A client with tuberculosis is being started on antituberculosis therapy with isoniazid (INH). Before giving the client the first dose, a nurse ensures that which of the following baseline studies has been completed? 1. Electrolyte levels 2. Coagulation times 3. Liver enzyme levels 4. Serum creatinine level

3. Liver enzyme levels Rationale: INH therapy can cause an elevation of hepatic enzyme levels and hepatitis. Therefore, liver enzyme levels are monitored when therapy is initiated and during the first 3 months of therapy. They may be monitored longer in the client who is greater than age 50 or abuses alcohol.

59.) A client is diagnosed with an acute myocardial infarction and is receiving tissue plasminogen activator, alteplase (Activase, tPA). Which action is a priority nursing intervention? 1. Monitor for renal failure. 2. Monitor psychosocial status. 3. Monitor for signs of bleeding. 4. Have heparin sodium available.

3. Monitor for signs of bleeding. Rationale: Tissue plasminogen activator is a thrombolytic. Hemorrhage is a complication of any type of thrombolytic medication. The client is monitored for bleeding. Monitoring for renal failure and monitoring the client's psychosocial status are important but are not the most critical interventions. Heparin is given after thrombolytic therapy, but the question is not asking about follow-up medications.

151.) A client is being treated for acute congestive heart failure with intravenously administered bumetanide. The vital signs are as follows: blood pressure, 100/60 mm Hg; pulse, 96 beats/min; and respirations, 24 breaths/min. After the initial dose, which of the following is the priority assessment? 1. Monitoring weight loss 2. Monitoring temperature 3. Monitoring blood pressure 4. Monitoring potassium level

3. Monitoring blood pressure Rationale: Bumetanide is a loop diuretic. Hypotension is a common side effect associated with the use of this medication. The other options also require assessment but are not the priority. **priority ABCs—airway, breathing, and circulation**

201.) A nurse is preparing to administer eardrops to an infant. The nurse plans to: 1. Pull up and back on the ear and direct the solution onto the eardrum. 2. Pull down and back on the ear and direct the solution onto the eardrum. 3. Pull down and back on the ear and direct the solution toward the wall of the canal. 4. Pull up and back on the ear lobe and direct the solution toward the wall of the canal.

3. Pull down and back on the ear and direct the solution toward the wall of the canal. Rationale: When administering eardrops to an infant, the nurse pulls the ear down and straight back. In the adult or a child older than 3 years, the ear is pulled up and back to straighten the auditory canal. The medication is administered by aiming it at the wall of the canal rather than directly onto the eardrum.

37.) The client has begun medication therapy with pancrelipase (Pancrease MT). The nurse evaluates that the medication is having the optimal intended benefit if which effect is observed? 1. Weight loss 2. Relief of heartburn 3. Reduction of steatorrhea 4. Absence of abdominal pain

3. Reduction of steatorrhea Rationale: Pancrelipase (Pancrease MT) is a pancreatic enzyme used in clients with pancreatitis as a digestive aid. The medication should reduce the amount of fatty stools (steatorrhea). Another intended effect could be improved nutritional status. It is not used to treat abdominal pain or heartburn. Its use could result in weight gain but should not result in weight loss if it is aiding in digestion.

111.) A client arrives at the health care clinic and tells the nurse that he has been doubling his daily dosage of bupropion hydrochloride (Wellbutrin) to help him get better faster. The nurse understands that the client is now at risk for which of the following? 1. Insomnia 2. Weight gain 3. Seizure activity 4. Orthostatic hypotension

3. Seizure activity Rationale: Bupropion does not cause significant orthostatic blood pressure changes. Seizure activity is common in dosages greater than 450 mg daily. Bupropion frequently causes a drop in body weight. Insomnia is a side effect, but seizure activity causes a greater client risk.

98.) The nurse is assigned to care for a client with cytomegalovirus retinitis and acquired immunodeficiency syndrome who is receiving foscarnet. The nurse should check the latest results of which of the following laboratory studies while the client is taking this medication? 1. CD4 cell count 2. Serum albumin 3. Serum creatinine 4. Lymphocyte count

3. Serum creatinine Rationale: Foscarnet is toxic to the kidneys. Serum creatinine is monitored before therapy, two to three times per week during induction therapy, and at least weekly during maintenance therapy. Foscarnet may also cause decreased levels of calcium, magnesium, phosphorus, and potassium. Thus these levels are also measured with the same frequency.

211.) A client with epilepsy is taking the prescribed dose of phenytoin (Dilantin) to control seizures. A phenytoin blood level is drawn, and the results reveal a level of 35 mcg/ml. Which of the following symptoms would be expected as a result of this laboratory result? 1. Nystagmus 2. Tachycardia 3. Slurred speech 4. No symptoms, because this is a normal therapeutic level

3. Slurred speech Rationale: The therapeutic phenytoin level is 10 to 20 mcg/mL. At a level higher than 20 mcg/mL, involuntary movements of the eyeballs (nystagmus) appear. At a level higher than 30 mcg/mL, ataxia and slurred speech occur.

187.) A clinic nurse prepares to administer an MMR (measles, mumps, rubella) vaccine to a child. How is this vaccine best administered? 1. Intramuscularly in the deltoid muscle 2. Subcutaneously in the gluteal muscle 3. Subcutaneously in the outer aspect of the upper arm 4. Intramuscularly in the anterolateral aspect of the thigh

3. Subcutaneously in the outer aspect of the upper arm Rationale: The MMR vaccine is administered subcutaneously in the outer aspect of the upper arm. The gluteal muscle is most often used for intramuscular injections. The MMR vaccine is not administered by the intramuscular route.

46.) A postoperative client has received a dose of naloxone hydrochloride for respiratory depression shortly after transfer to the nursing unit from the postanesthesia care unit. After administration of the medication, the nurse checks the client for: 1. Pupillary changes 2. Scattered lung wheezes 3. Sudden increase in pain 4. Sudden episodes of diarrhea

3. Sudden increase in pain Rationale: Naloxone hydrochloride is an antidote to opioids and may also be given to the postoperative client to treat respiratory depression. When given to the postoperative client for respiratory depression, it may also reverse the effects of analgesics. Therefore, the nurse must check the client for a sudden increase in the level of pain experienced. Options 1, 2, and 4 are not associated with this medication.

245.) A client taking carbamazepine (Tegretol) asks the nurse what to do if he misses one dose. The nurse responds that the carbamazepine should be: 1. Withheld until the next scheduled dose 2. Withheld and the health care provider is notified immediately 3. Taken as long as it is not immediately before the next dose 4. Withheld until the next scheduled dose, which should then be doubled

3. Taken as long as it is not immediately before the next dose Rationale: Carbamazepine is an anticonvulsant that should be taken around the clock, precisely as directed. If a dose is omitted, the client should take the dose as soon as it is remembered, as long as it is not immediately before the next dose. The medication should not be double dosed. If more than one dose is omitted, the client should call the health care provider.

103.) A nurse is caring for a hospitalized client who has been taking clozapine (Clozaril) for the treatment of a schizophrenic disorder. Which laboratory study prescribed for the client will the nurse specifically review to monitor for an adverse effect associated with the use of this medication? 1. Platelet count 2. Cholesterol level 3. White blood cell count 4. Blood urea nitrogen level

3. White blood cell count Rationale: Hematological reactions can occur in the client taking clozapine and include agranulocytosis and mild leukopenia. The white blood cell count should be checked before initiating treatment and should be monitored closely during the use of this medication. The client should also be monitored for signs indicating agranulocytosis, which may include sore throat, malaise, and fever. Options 1, 2, and 4 are unrelated to this medication.

81.) A client with trigeminal neuralgia is being treated with carbamazepine (Tegretol). Which laboratory result would indicate that the client is experiencing an adverse reaction to the medication? 1. Sodium level, 140 mEq/L 2. Uric acid level, 5.0 mg/dL 3. White blood cell count, 3000 cells/mm3 4. Blood urea nitrogen (BUN) level, 15 mg/dL

3. White blood cell count, 3000 cells/mm3 Rationale: Adverse effects of carbamazepine (Tegretol) appear as blood dyscrasias, including aplastic anemia, agranulocytosis, thrombocytopenia, leukopenia, cardiovascular disturbances, thrombophlebitis, dysrhythmias, and dermatological effects. Options 1, 2, and 4 identify normal laboratory values.

30-A client is receiving a secondary infusion of erythromycin 1 gram in 100 mL dextrose 5% in water to be infused in 45 minutes. How many mL/hour should the nurse program the infusion pump? Round to the nearest whole number

33 ml/hr

61.) A home health care nurse is visiting a client with elevated triglyceride levels and a serum cholesterol level of 398 mg/dL. The client is taking cholestyramine (Questran). Which of the following statements, if made by the client, indicates the need for further education? 1. "Constipation and bloating might be a problem." 2. "I'll continue to watch my diet and reduce my fats." 3. "Walking a mile each day will help the whole process." 4. "I'll continue my nicotinic acid from the health food store."

4. "I'll continue my nicotinic acid from the health food store." Rationale: Nicotinic acid, even an over-the-counter form, should be avoided because it may lead to liver abnormalities. All lipid-lowering medications also can cause liver abnormalities, so a combination of nicotinic acid and cholestyramine resin is to be avoided. Constipation and bloating are the two most common side effects. Walking and the reduction of fats in the diet are therapeutic measures to reduce cholesterol and triglyceride levels.

62.) A client is on nicotinic acid (niacin) for hyperlipidemia and the nurse provides instructions to the client about the medication. Which statement by the client would indicate an understanding of the instructions? 1. "It is not necessary to avoid the use of alcohol." 2. "The medication should be taken with meals to decrease flushing." 3. "Clay-colored stools are a common side effect and should not be of concern." 4. "Ibuprofen (Motrin) taken 30 minutes before the nicotinic acid should decrease the flushing."

4. "Ibuprofen (Motrin) taken 30 minutes before the nicotinic acid should decrease the flushing." Rationale: Flushing is a side effect of this medication. Aspirin or a nonsteroidal anti-inflammatory drug can be taken 30 minutes before taking the medication to decrease flushing. Alcohol consumption needs to be avoided because it will enhance this side effect. The medication should be taken with meals, this will decrease gastrointestinal upset. Taking the medication with meals has no effect on the flushing. Clay-colored stools are a sign of hepatic dysfunction and should be immediately reported to the health care provider (HCP).

66.) Trimethoprim-sulfamethoxazole (TMP-SMZ) is prescribed for a client. A nurse should instruct the client to report which symptom if it developed during the course of this medication therapy? 1. Nausea 2. Diarrhea 3. Headache 4. Sore throat

4. Sore throat Rationale: Clients taking trimethoprim-sulfamethoxazole (TMP-SMZ) should be informed about early signs of blood disorders that can occur from this medication. These include sore throat, fever, and pallor, and the client should be instructed to notify the health care provider if these symptoms occur. The other options do not require health care provider notification.

218.) A film-coated form of diflunisal has been prescribed for a client for the treatment of chronic rheumatoid arthritis. The client calls the clinic nurse because of difficulty swallowing the tablets. Which initial instruction should the nurse provide to the client? 1. "Crush the tablets and mix them with food." 2. "Notify the health care provider for a medication change." 3. "Open the tablet and mix the contents with food." 4. "Swallow the tablets with large amounts of water or milk."

4. "Swallow the tablets with large amounts of water or milk." Rationale: Diflunisal may be given with water, milk, or meals. The tablets should not be crushed or broken open. Taking the medication with a large amount of water or milk should be tried before contacting the health care provider.

87.) A client with acute muscle spasms has been taking baclofen (Lioresal). The client calls the clinic nurse because of continuous feelings of weakness and fatigue and asks the nurse about discontinuing the medication. The nurse should make which appropriate response to the client? 1. "You should never stop the medication." 2. "It is best that you taper the dose if you intend to stop the medication." 3. "It is okay to stop the medication if you think that you can tolerate the muscle spasms." 4. "Weakness and fatigue commonly occur and will diminish with continued medication use."

4. "Weakness and fatigue commonly occur and will diminish with continued medication use." Rationale: The client should be instructed that symptoms such as drowsiness, weakness, and fatigue are more intense in the early phase of therapy and diminish with continued medication use. The client should be instructed never to withdraw or stop the medication abruptly, because abrupt withdrawal can cause visual hallucinations, paranoid ideation, and seizures. It is best for the nurse to inform the client that these symptoms will subside and encourage the client to continue the use of the medication.

163.) A client with trigeminal neuralgia tells the nurse that acetaminophen (Tylenol) is taken on a frequent daily basis for relief of generalized discomfort. The nurse reviews the client's laboratory results and determines that which of the following indicates toxicity associated with the medication? 1. Sodium of 140 mEq/L 2. Prothrombin time of 12 seconds 3. Platelet count of 400,000 cells/mm3 4. A direct bilirubin level of 2 mg/dL

4. A direct bilirubin level of 2 mg/dL Rationale: In adults, overdose of acetaminophen (Tylenol) causes liver damage. Option 4 is an indicator of liver function and is the only option that indicates an abnormal laboratory value. The normal direct bilirubin is 0 to 0.4 mg/dL. The normal platelet count is 150,000 to 400,000 cells/mm3. The normal prothrombin time is 10 to 13 seconds. The normal sodium level is 135 to 145 mEq/L.

75.) A client with myasthenia gravis becomes increasingly weak. The health care provider prepares to identify whether the client is reacting to an overdose of the medication (cholinergic crisis) or increasing severity of the disease (myasthenic crisis). An injection of edrophonium (Enlon) is administered. Which of the following indicates that the client is in cholinergic crisis? 1. No change in the condition 2. Complaints of muscle spasms 3. An improvement of the weakness 4. A temporary worsening of the condition

4. A temporary worsening of the condition Rationale: An edrophonium (Enlon) injection, a cholinergic drug, makes the client in cholinergic crisis temporarily worse. This is known as a negative test. An improvement of weakness would occur if the client were experiencing myasthenia gravis. Options 1 and 2 would not occur in either crisis.

56.) Heparin sodium is prescribed for the client. The nurse expects that the health care provider will prescribe which of the following to monitor for a therapeutic effect of the medication? 1. Hematocrit level 2. Hemoglobin level 3. Prothrombin time (PT) 4. Activated partial thromboplastin time (aPTT)

4. Activated partial thromboplastin time (aPTT) Rationale: The PT will assess for the therapeutic effect of warfarin sodium (Coumadin) and the aPTT will assess the therapeutic effect of heparin sodium. Heparin sodium doses are determined based on these laboratory results. The hemoglobin and hematocrit values assess red blood cell concentrations.

242.) A client who was started on anticonvulsant therapy with clonazepam (Klonopin) tells the nurse of increasing clumsiness and unsteadiness since starting the medication. The client is visibly upset by these manifestations and asks the nurse what to do. The nurse's response is based on the understanding that these symptoms: 1. Usually occur if the client takes the medication with food 2. Are probably the result of an interaction with another medication 3. Indicate that the client is experiencing a severe untoward reaction to the medication 4. Are worse during initial therapy and decrease or disappear with long-term use

4. Are worse during initial therapy and decrease or disappear with long-term use Rationale: Drowsiness, unsteadiness, and clumsiness are expected effects of the medication during early therapy. They are dose related and usually diminish or disappear altogether with continued use of the medication. It does not indicate that a severe side effect is occurring. It is also unrelated to interaction with another medication. The client is encouraged to take this medication with food to minimize gastrointestinal upset. **Eliminate options 2 and 3 first because they are comparable or alike and because of the word "severe" in option 3**

4.) The camp nurse asks the children preparing to swim in the lake if they have applied sunscreen. The nurse reminds the children that chemical sunscreens are most effective when applied: 1. Immediately before swimming 2. 15 minutes before exposure to the sun 3. Immediately before exposure to the sun 4. At least 30 minutes before exposure to the sun

4. At least 30 minutes before exposure to the sun Rationale: Sunscreens are most effective when applied at least 30 minutes before exposure to the sun so that they can penetrate the skin. All sunscreens should be reapplied after swimming or sweating.

178.) Methylergonovine (Methergine) is prescribed for a client with postpartum hemorrhage caused by uterine atony. Before administering the medication, the nurse checks which of the following as the important client parameter? 1. Temperature 2. Lochial flow 3. Urine output 4. Blood pressure

4. Blood pressure Rationale: Methylergonovine is an ergot alkaloid used for postpartum hemorrhage. It stimulates contraction of the uterus and causes arterial vasoconstriction. Ergot alkaloids are avoided in clients with significant cardiovascular disease, peripheral disease, hypertension, eclampsia, or preeclampsia. These conditions are worsened by the vasoconstrictive effects of the ergot alkaloids. The nurse would check the client's blood pressure before administering the medication and would follow agency protocols regarding withholding of the medication. Options 1, 2, and 3 are items that are checked in the postpartum period, but they are unrelated to the use of this medication.

237.) A client who is on lithium carbonate (Lithobid) will be discharged at the end of the week. In formulating a discharge teaching plan, the nurse will instruct the client that it is most important to: 1. Avoid soy sauce, wine, and aged cheese. 2. Have the lithium level checked every week. 3. Take medication only as prescribed because it can become addicting. 4. Check with the psychiatrist before using any over-the-counter (OTC) medications or prescription medications.

4. Check with the psychiatrist before using any over-the-counter (OTC) medications or prescription medications. Rationale: Lithium is the medication of choice to treat manic-depressive illness. Many OTC medications interact with lithium, and the client is instructed to avoid OTC medications while taking lithium. Lithium is not addicting, and, although serum lithium levels need to be monitored, it is not necessary to check these levels every week. A tyramine-free diet is associated with monoamine oxidase inhibitors.

19.) Tamoxifen is prescribed for the client with metastatic breast carcinoma. The nurse understands that the primary action of this medication is to: 1. Increase DNA and RNA synthesis. 2. Promote the biosynthesis of nucleic acids. 3. Increase estrogen concentration and estrogen response. 4. Compete with estradiol for binding to estrogen in tissues containing high concentrations of receptors.

4. Compete with estradiol for binding to estrogen in tissues containing high concentrations of receptors. Rationale: Tamoxifen is an antineoplastic medication that competes with estradiol for binding to estrogen in tissues containing high concentrations of receptors. Tamoxifen is used to treat metastatic breast carcinoma in women and men. Tamoxifen is also effective in delaying the recurrence of cancer following mastectomy. Tamoxifen reduces DNA synthesis and estrogen response.

217.) A health care provider prescribes auranofin (Ridaura) for a client with rheumatoid arthritis. Which of the following would indicate to the nurse that the client is experiencing toxicity related to the medication? 1. Joint pain 2. Constipation 3. Ringing in the ears 4. Complaints of a metallic taste in the mouth

4. Complaints of a metallic taste in the mouth Rationale: Ridaura is the one gold preparation that is given orally rather than by injection. Gastrointestinal reactions including diarrhea, abdominal pain, nausea, and loss of appetite are common early in therapy, but these usually subside in the first 3 months of therapy. Early symptoms of toxicity include a rash, purple blotches, pruritus, mouth lesions, and a metallic taste in the mouth.

124.) A client with chronic renal failure is receiving ferrous sulfate (Feosol). The nurse monitors the client for which common side effect associated with this medication? 1. Diarrhea 2. Weakness 3. Headache 4. Constipation

4. Constipation Rationale: Feosol is an iron supplement used to treat anemia. Constipation is a frequent and uncomfortable side effect associated with the administration of oral iron supplements. Stool softeners are often prescribed to prevent constipation. **Focus on the name of the medication. Recalling that oral iron can cause constipation will easily direct you to the correct option.**

155.) Mycophenolate mofetil (CellCept) is prescribed for a client as prophylaxis for organ rejection following an allogeneic renal transplant. Which of the following instructions does the nurse reinforce regarding administration of this medication? 1. Administer following meals. 2. Take the medication with a magnesium-type antacid. 3. Open the capsule and mix with food for administration. 4. Contact the health care provider (HCP) if a sore throat occurs.

4. Contact the health care provider (HCP) if a sore throat occurs. Rationale: Mycophenolate mofetil should be administered on an empty stomach. The capsules should not be opened or crushed. The client should contact the HCP if unusual bleeding or bruising, sore throat, mouth sores, abdominal pain, or fever occurs because these are adverse effects of the medication. Antacids containing magnesium and aluminum may decrease the absorption of the medication and therefore should not be taken with the medication. The medication may be given in combination with corticosteroids and cyclosporine. **neutropenia can occur with this medication**

157.) A client receiving nitrofurantoin (Macrodantin) calls the health care provider's office complaining of side effects related to the medication. Which side effect indicates the need to stop treatment with this medication? 1. Nausea 2. Diarrhea 3. Anorexia 4. Cough and chest pain

4. Cough and chest pain Rationale: Gastrointestinal (GI) effects are the most frequent adverse reactions to this medication and can be minimized by administering the medication with milk or meals. Pulmonary reactions, manifested as dyspnea, chest pain, chills, fever, cough, and the presence of alveolar infiltrates on the x-ray, would indicate the need to stop the treatment. These symptoms resolve in 2 to 4 days following discontinuation of this medication. **Eliminate options 1, 2, and 3 because they are similar GI-related side effects. Also, use the ABCs— airway, breathing, and circulation**

132.) The client with non-Hodgkin's lymphoma is receiving daunorubicin (DaunoXome). Which of the following would indicate to the nurse that the client is experiencing a toxic effect related to the medication? 1. Fever 2. Diarrhea 3. Complaints of nausea and vomiting 4. Crackles on auscultation of the lungs

4. Crackles on auscultation of the lungs Rationale: Cardiotoxicity noted by abnormal electrocardiographic findings or cardiomyopathy manifested as congestive heart failure is a toxic effect of daunorubicin. Bone marrow depression is also a toxic effect. Nausea and vomiting are frequent side effects associated with the medication that begins a few hours after administration and lasts 24 to 48 hours. Fever is a frequent side effect, and diarrhea can occur occasionally. The other options, however, are not toxic effects. **keep in mind that the question is asking about a toxic effect and think: ABCs—airway, breathing, and circulation**

117.) A nurse has given the client taking ethambutol (Myambutol) information about the medication. The nurse determines that the client understands the instructions if the client immediately reports: 1. Impaired sense of hearing 2. Distressing gastrointestinal side effects 3. Orange-red discoloration of body secretions 4. Difficulty discriminating the color red from green

4. Difficulty discriminating the color red from green Rationale: Ethambutol causes optic neuritis, which decreases visual acuity and the ability to discriminate between the colors red and green. This poses a potential safety hazard when driving a motor vehicle. The client is taught to report this symptom immediately. The client is also taught to take the medication with food if gastrointestinal upset occurs. Impaired hearing results from antitubercular therapy with streptomycin. Orange-red discoloration of secretions occurs with rifampin (Rifadin).

190.) A child is hospitalized with a diagnosis of lead poisoning. The nurse assisting in caring for the child would prepare to assist in administering which of the following medications? 1. Activated charcoal 2. Sodium bicarbonate 3. Syrup of ipecac syrup 4. Dimercaprol (BAL in Oil)

4. Dimercaprol (BAL in Oil) Rationale: Dimercaprol is a chelating agent that is administered to remove lead from the circulating blood and from some tissues and organs for excretion in the urine. Sodium bicarbonate may be used in salicylate poisoning. Syrup of ipecac is used in the hospital setting in poisonings to induce vomiting. Activated charcoal is used to decrease absorption in certain poisoning situations. Note that dimercaprol is prepared with peanut oil, and hence should be avoided by clients with known or suspected peanut allergy.

203.) A nurse is preparing to give the postcraniotomy client medication for incisional pain. The family asks the nurse why the client is receiving codeine sulfate and not "something stronger." In formulating a response, the nurse incorporates the understanding that codeine: 1. Is one of the strongest opioid analgesics available 2. Cannot lead to physical or psychological dependence 3. Does not cause gastrointestinal upset or constipation as do other opioids 4. Does not alter respirations or mask neurological signs as do other opioids

4. Does not alter respirations or mask neurological signs as do other opioids Rationale: Codeine sulfate is the opioid analgesic often used for clients after craniotomy. It is frequently combined with a nonopioid analgesic such as acetaminophen for added effect. It does not alter the respiratory rate or mask neurological signs as do other opioids. Side effects of codeine include gastrointestinal upset and constipation. The medication can lead to physical and psychological dependence with chronic use. It is not the strongest opioid analgesic available.

159.) A nurse is caring for a client receiving morphine sulfate subcutaneously for pain. Because morphine sulfate has been prescribed for this client, which nursing action would be included in the plan of care? 1. Encourage fluid intake. 2. Monitor the client's temperature. 3. Maintain the client in a supine position. 4. Encourage the client to cough and deep breathe.

4. Encourage the client to cough and deep breathe. Rationale: Morphine sulfate suppresses the cough reflex. Clients need to be encouraged to cough and deep breathe to prevent pneumonia. **ABCs—airway, breathing, and circulation**

223.) A client with a psychotic disorder is being treated with haloperidol (Haldol). Which of the following would indicate the presence of a toxic effect of this medication? 1. Nausea 2. Hypotension 3. Blurred vision 4. Excessive salivation

4. Excessive salivation Rationale: Toxic effects include extrapyramidal symptoms (EPS) noted as marked drowsiness and lethargy, excessive salivation, and a fixed stare. Akathisia, acute dystonias, and tardive dyskinesia are also signs of toxicity. Hypotension, nausea, and blurred vision are occasional side effects.

236.) A client is being treated for depression with amitriptyline hydrochloride. During the initial phases of treatment, the most important nursing intervention is: 1. Prescribing the client a tyramine-free diet 2. Checking the client for anticholinergic effects 3. Monitoring blood levels frequently because there is a narrow range between therapeutic and toxic blood levels of this medication 4. Getting baseline postural blood pressures before administering the medication and each time the medication is administered

4. Getting baseline postural blood pressures before administering the medication and each time the medication is administered Rationale: Amitriptyline hydrochloride is a tricyclic antidepressant often used to treat depression. It causes orthostatic changes and can produce hypotension and tachycardia. This can be frightening to the client and dangerous because it can result in dizziness and client falls. The client must be instructed to move slowly from a lying to a sitting to a standing position to avoid injury if these effects are experienced. The client may also experience sedation, dry mouth, constipation, blurred vision, and other anticholinergic effects, but these are transient and will diminish with time.

196.) A client has been prescribed amikacin (Amikin). Which of the following priority baseline functions should be monitored? 1. Apical pulse 2. Liver function 3. Blood pressure 4. Hearing acuity

4. Hearing acuity Rationale: Amikacin (Amikin) is an antibiotic. This medication can cause ototoxicity and nephrotoxicity; therefore, hearing acuity tests and kidney function studies should be performed before the initiation of therapy. Apical pulse, liver function studies, and blood pressure are not specifically related to the use of this medication.

175.) A nurse notes that a client is receiving lamivudine (Epivir). The nurse determines that this medication has been prescribed to treat which of the following? 1. Pancreatitis 2. Pharyngitis 3. Tonic-clonic seizures 4. Human immunodeficiency virus (HIV) infection

4. Human immunodeficiency virus (HIV) infection Rationale: Lamivudine is a nucleoside reverse transcriptase inhibitor and antiviral medication. It slows HIV replication and reduces the progression of HIV infection. It also is used to treat chronic hepatitis B and is used for prophylaxis in health care workers at risk of acquiring HIV after occupational exposure to the virus. **Note the letters "-vir" in the trade name for this medication**

192.) A nurse is collecting medication information from a client, and the client states that she is taking garlic as an herbal supplement. The nurse understands that the client is most likely treating which of the following conditions? 1. Eczema 2. Insomnia 3. Migraines 4. Hyperlipidemia

4. Hyperlipidemia Rationale: Garlic is an herbal supplement that is used to treat hyperlipidemia and hypertension. An herbal supplement that may be used to treat eczema is evening primrose. Insomnia has been treated with both valerian root and chamomile. Migraines have been treated with feverfew.

73.) A client with myasthenia gravis is suspected of having cholinergic crisis. Which of the following indicate that this crisis exists? 1. Ataxia 2. Mouth sores 3. Hypotension 4. Hypertension

4. Hypertension Rationale: Cholinergic crisis occurs as a result of an overdose of medication. Indications of cholinergic crisis include gastrointestinal disturbances, nausea, vomiting, diarrhea, abdominal cramps, increased salivation and tearing, miosis, hypertension, sweating, and increased bronchial secretions.

76.) Carbidopa-levodopa (Sinemet) is prescribed for a client with Parkinson's disease, and the nurse monitors the client for adverse reactions to the medication. Which of the following indicates that the client is experiencing an adverse reaction? 1. Pruritus 2. Tachycardia 3. Hypertension 4. Impaired voluntary movements

4. Impaired voluntary movements Rationale: Dyskinesia and impaired voluntary movement may occur with high levodopa dosages. Nausea, anorexia, dizziness, orthostatic hypotension, bradycardia, and akinesia (the temporary muscle weakness that lasts 1 minute to 1 hour, also known as the "on-off phenomenon") are frequent side effects of the medication.

212.) Mannitol (Osmitrol) is being administered to a client with increased intracranial pressure following a head injury. The nurse assisting in caring for the client knows that which of the following indicates the therapeutic action of this medication? 1. Prevents the filtration of sodium and water through the kidneys 2. Prevents the filtration of sodium and potassium through the kidneys 3. Decreases water loss by promoting the reabsorption of sodium and water in the loop of Henle 4. Induces diuresis by raising the osmotic pressure of glomerular filtrate, thereby inhibiting tubular reabsorption of water and solutes

4. Induces diuresis by raising the osmotic pressure of glomerular filtrate, thereby inhibiting tubular reabsorption of water and solutes Rationale: Mannitol is an osmotic diuretic that induces diuresis by raising the osmotic pressure of glomerular filtrate, thereby inhibiting tubular reabsorption of water and solutes. It is used to reduce intracranial pressure in the client with head trauma.

200.) A client is seen in the clinic for complaints of skin itchiness that has been persistent over the past several weeks. Following data collection, it has been determined that the client has scabies. Lindane is prescribed, and the nurse is asked to provide instructions to the client regarding the use of the medication. The nurse tells the client to: 1. Apply a thick layer of cream to the entire body. 2. Apply the cream as prescribed for 2 days in a row. 3. Apply to the entire body and scalp, excluding the face. 4. Leave the cream on for 8 to 12 hours and then remove by washing.

4. Leave the cream on for 8 to 12 hours and then remove by washing. Rationale: Lindane is applied in a thin layer to the entire body below the head. No more than 30 g (1 oz) should be used. The medication is removed by washing 8 to 12 hours later. Usually, only one application is required.

119.) A client with diabetes mellitus who has been controlled with daily insulin has been placed on atenolol (Tenormin) for the control of angina pectoris. Because of the effects of atenolol, the nurse determines that which of the following is the most reliable indicator of hypoglycemia? 1. Sweating 2. Tachycardia 3. Nervousness 4. Low blood glucose level

4. Low blood glucose level Rationale: β-Adrenergic blocking agents, such as atenolol, inhibit the appearance of signs and symptoms of acute hypoglycemia, which would include nervousness, increased heart rate, and sweating. Therefore, the client receiving this medication should adhere to the therapeutic regimen and monitor blood glucose levels carefully. Option 4 is the most reliable indicator of hypoglycemia.

205.) A nurse is assisting in preparing to administer acetylcysteine (Mucomyst) to a client with an overdose of acetaminophen (Tylenol). The nurse prepares to administer the medication by: 1. Administering the medication subcutaneously in the deltoid muscle 2. Administering the medication by the intramuscular route in the gluteal muscle 3. Administering the medication by the intramuscular route, mixed in 10 mL of normal saline 4. Mixing the medication in a flavored ice drink and allowing the client to drink the medication through a straw

4. Mixing the medication in a flavored ice drink and allowing the client to drink the medication through a straw Rationale: Because acetylcysteine has a pervasive odor of rotten eggs, it must be disguised in a flavored ice drink. It is consumed preferably through a straw to minimize contact with the mouth. It is not administered by the intramuscular or subcutaneous route. **Knowing that the medication is a solution that is also used for nebulization treatments will assist you to select the option that indicates an oral route**

36.) The client has a PRN prescription for ondansetron (Zofran). For which condition should this medication be administered to the postoperative client? 1. Paralytic ileus 2. Incisional pain 3. Urinary retention 4. Nausea and vomiting

4. Nausea and vomiting Rationale: Ondansetron is an antiemetic used to treat postoperative nausea and vomiting, as well as nausea and vomiting associated with chemotherapy. The other options are incorrect.

143.) A client has just taken a dose of trimethobenzamide (Tigan). The nurse plans to monitor this client for relief of: 1. Heartburn 2. Constipation 3. Abdominal pain 4. Nausea and vomiting

4. Nausea and vomiting Rationale: Trimethobenzamide is an antiemetic agent used in the treatment of nausea and vomiting. The other options are incorrect.

17.) The client with ovarian cancer is being treated with vincristine (Oncovin). The nurse monitors the client, knowing that which of the following indicates a side effect specific to this medication? 1. Diarrhea 2. Hair loss 3. Chest pain 4. Numbness and tingling in the fingers and toes

4. Numbness and tingling in the fingers and toes Rationale: A side effect specific to vincristine is peripheral neuropathy, which occurs in almost every client. Peripheral neuropathy can be manifested as numbness and tingling in the fingers and toes. Depression of the Achilles tendon reflex may be the first clinical sign indicating peripheral neuropathy. Constipation rather than diarrhea is most likely to occur with this medication, although diarrhea may occur occasionally. Hair loss occurs with nearly all the antineoplastic medications. Chest pain is unrelated to this medication.

2.) Oral iron supplements are prescribed for a 6-year-old child with iron deficiency anemia. The nurse instructs the mother to administer the iron with which best food item? 1. Milk 2. Water 3. Apple juice 4. Orange juice

4. Orange juice Rationale: Vitamin C increases the absorption of iron by the body. The mother should be instructed to administer the medication with a citrus fruit or a juice that is high in vitamin C. Milk may affect absorption of the iron. Water will not assist in absorption. Orange juice contains a greater amount of vitamin C than apple juice.

15.) The client with small cell lung cancer is being treated with etoposide (VePesid). The nurse who is assisting in caring for the client during its administration understands that which side effect is specifically associated with this medication? 1. Alopecia 2. Chest pain 3. Pulmonary fibrosis 4. Orthostatic hypotension

4. Orthostatic hypotension Rationale: A side effect specific to etoposide is orthostatic hypotension. The client's blood pressure is monitored during the infusion. Hair loss occurs with nearly all the antineoplastic medications. Chest pain and pulmonary fibrosis are unrelated to this medication.

222.) A nurse has administered a dose of diazepam (Valium) to a client. The nurse would take which important action before leaving the client's room? 1. Giving the client a bedpan 2. Drawing the shades or blinds closed 3. Turning down the volume on the television 4. Per agency policy, putting up the side rails on the bed

4. Per agency policy, putting up the side rails on the bed Rationale: Diazepam is a sedative-hypnotic with anticonvulsant and skeletal muscle relaxant properties. The nurse should institute safety measures before leaving the client's room to ensure that the client does not injure herself or himself. The most frequent side effects of this medication are dizziness, drowsiness, and lethargy. For this reason, the nurse puts the side rails up on the bed before leaving the room to prevent falls. Options 1, 2, and 3 may be helpful measures that provide a comfortable, restful environment, but option 4 is the one that provides for the client's safety needs.

184.) A nurse preparing a client for surgery reviews the client's medication record. The client is to be nothing per mouth (NPO) after midnight. Which of the following medications, if noted on the client's record, should the nurse question? 1. Cyclobenzaprine (Flexeril) 2. Alendronate (Fosamax) 3. Allopurinol (Zyloprim) 4. Prednisone

4. Prednisone Rationale: Prednisone is a corticosteroid that can cause adrenal atrophy, which reduces the body's ability to withstand stress. Before and during surgery, dosages may be temporarily increased. Cyclobenzaprine is a skeletal muscle relaxant. Alendronate is a bone-resorption inhibitor. Allopurinol is an antigout medication.

189.) Prostaglandin E1 is prescribed for a child with transposition of the great arteries. The mother of the child asks the nurse why the child needs the medication. The nurse tells the mother that the medication: 1. Prevents hypercyanotic (blue or tet) spells 2. Maintains an adequate hormone level 3. Maintains the position of the great arteries 4. Provides adequate oxygen saturation and maintains cardiac output

4. Provides adequate oxygen saturation and maintains cardiac output Rationale: A child with transposition of the great arteries may receive prostaglandin E1 temporarily to increase blood mixing if systemic and pulmonary mixing are inadequate to maintain adequate cardiac output. Options 1, 2, and 3 are incorrect. In addition, hypercyanotic spells occur in tetralogy of Fallot. **Use the ABCs—airway, breathing, and circulation—to answer the question. The correct option addresses circulation**

13.) The client with squamous cell carcinoma of the larynx is receiving bleomycin intravenously. The nurse caring for the client anticipates that which diagnostic study will be prescribed? 1. Echocardiography 2. Electrocardiography 3. Cervical radiography 4. Pulmonary function studies

4. Pulmonary function studies Rationale: Bleomycin is an antineoplastic medication (Chemotheraputic Agents) that can cause interstitial pneumonitis, which can progress to pulmonary fibrosis. Pulmonary function studies along with hematological, hepatic, and renal function tests need to be monitored. The nurse needs to monitor lung sounds for dyspnea and crackles, which indicate pulmonary toxicity. The medication needs to be discontinued immediately if pulmonary toxicity occurs. Options 1, 2, and 3 are unrelated to the specific use of this medication.

144.) A client is taking docusate sodium (Colace). The nurse monitors which of the following to determine whether the client is having a therapeutic effect from this medication? 1. Abdominal pain 2. Reduction in steatorrhea 3. Hematest-negative stools 4. Regular bowel movements

4. Regular bowel movements Rationale: Docusate sodium is a stool softener that promotes the absorption of water into the stool, producing a softer consistency of stool. The intended effect is relief or prevention of constipation. The medication does not relieve abdominal pain, stop gastrointestinal (GI) bleeding, or decrease the amount of fat in the stools.

70.) Oxybutynin chloride (Ditropan XL) is prescribed for a client with neurogenic bladder. Which sign would indicate a possible toxic effect related to this medication? 1. Pallor 2. Drowsiness 3. Bradycardia 4. Restlessness

4. Restlessness Rationale: Toxicity (overdosage) of this medication produces central nervous system excitation, such as nervousness, restlessness, hallucinations, and irritability. Other signs of toxicity include hypotension or hypertension, confusion, tachycardia, flushed or red face, and signs of respiratory depression. Drowsiness is a frequent side effect of the medication but does not indicate overdosage.

44.) A client is receiving acetylcysteine (Mucomyst), 20% solution diluted in 0.9% normal saline by nebulizer. The nurse should have which item available for possible use after giving this medication? 1. Ambu bag 2. Intubation tray 3. Nasogastric tube 4. Suction equipment

4. Suction equipment Rationale: Acetylcysteine can be given orally or by nasogastric tube to treat acetaminophen overdose, or it may be given by inhalation for use as a mucolytic. The nurse administering this medication as a mucolytic should have suction equipment available in case the client cannot manage to clear the increased volume of liquefied secretions.

166.) Alendronate (Fosamax) is prescribed for a client with osteoporosis. The client taking this medication is instructed to: 1. Take the medication at bedtime. 2. Take the medication in the morning with breakfast. 3. Lie down for 30 minutes after taking the medication. 4. Take the medication with a full glass of water after rising in the morning.

4. Take the medication with a full glass of water after rising in the morning. Rationale: Precautions need to be taken with administration of alendronate to prevent gastrointestinal side effects (especially esophageal irritation) and to increase absorption of the medication. The medication needs to be taken with a full glass of water after rising in the morning. The client should not eat or drink anything for 30 minutes following administration and should not lie down after taking the medication.

67.) Phenazopyridine hydrochloride (Pyridium) is prescribed for a client for symptomatic relief of pain resulting from a lower urinary tract infection. The nurse reinforces to the client: 1. To take the medication at bedtime 2. To take the medication before meals 3. To discontinue the medication if a headache occurs 4. That a reddish orange discoloration of the urine may occur

4. That a reddish orange discoloration of the urine may occur Rationale: The nurse should instruct the client that a reddish-orange discoloration of urine may occur. The nurse also should instruct the client that this discoloration can stain fabric. The medication should be taken after meals to reduce the possibility of gastrointestinal upset. A headache is an occasional side effect of the medication and does not warrant discontinuation of the medication.

177.) A nurse is assisting in caring for a pregnant client who is receiving intravenous magnesium sulfate for the management of preeclampsia and notes that the client's deep tendon reflexes are absent. On the basis of this data, the nurse reports the finding and makes which determination? 1. The magnesium sulfate is effective. 2. The infusion rate needs to be increased. 3. The client is experiencing cerebral edema. 4. The client is experiencing magnesium toxicity.

4. The client is experiencing magnesium toxicity. Rationale: Magnesium toxicity can occur as a result of magnesium sulfate therapy. Signs of magnesium sulfate toxicity relate to the central nervous system depressant effects of the medication and include respiratory depression; loss of deep tendon reflexes; sudden decrease in fetal heart rate or maternal heart rate, or both; and sudden drop in blood pressure. Hyperreflexia indicates increased cerebral edema. An absence of reflexes indicates magnesium toxicity. The therapeutic serum level of magnesium for a client receiving magnesium sulfate ranges from 4 to 7.5 mEq/L (5 to 8 mg/dL).

232.) A client in the mental health unit is administered haloperidol (Haldol). The nurse would check which of the following to determine medication effectiveness? 1. The client's vital signs 2. The client's nutritional intake 3. The physical safety of other unit clients 4. The client's orientation and delusional status

4. The client's orientation and delusional status Rationale: Haloperidol is used to treat clients exhibiting psychotic features. Therefore, to determine medication effectiveness, the nurse would check the client's orientation and delusional status. Vital signs are routine and not specific to this situation. The physical safety of other clients is not a direct assessment of this client. Monitoring nutritional intake is not related to this situation.

99.) The client with acquired immunodeficiency syndrome and Pneumocystis jiroveci infection has been receiving pentamidine isethionate (Pentam 300). The client develops a temperature of 101° F. The nurse does further monitoring of the client, knowing that this sign would most likely indicate: 1. The dose of the medication is too low. 2. The client is experiencing toxic effects of the medication. 3. The client has developed inadequacy of thermoregulation. 4. The result of another infection caused by leukopenic effects of the medication.

4. The result of another infection caused by leukopenic effects of the medication. Rationale: Frequent side effects of this medication include leukopenia, thrombocytopenia, and anemia. The client should be monitored routinely for signs and symptoms of infection. Options 1, 2, and 3 are inaccurate interpretations.

121.) A client who is taking hydrochlorothiazide (HydroDIURIL, HCTZ) has been started on triamterene (Dyrenium) as well. The client asks the nurse why both medications are required. The nurse formulates a response, based on the understanding that: 1. Both are weak potassium-losing diuretics. 2. The combination of these medications prevents renal toxicity. 3. Hydrochlorothiazide is an expensive medication, so using a combination of diuretics is cost-effective. 4. Triamterene is a potassium-sparing diuretic, whereas hydrochlorothiazide is a potassium-losing diuretic.

4. Triamterene is a potassium-sparing diuretic, whereas hydrochlorothiazide is a potassium-losing diuretic. Rationale: Potassium-sparing diuretics include amiloride (Midamor), spironolactone (Aldactone), and triamterene (Dyrenium). They are weak diuretics that are used in combination with potassium-losing diuretics. This combination is useful when medication and dietary supplement of potassium is not appropriate. The use of two different diuretics does not prevent renal toxicity. Hydrochlorothiazide is an effective and inexpensive generic form of the thiazide classification of diuretics. **It is especially helpful to remember that hydrochlorothiazide is a potassium-losing diuretic and triamterene is a potassium-sparing diuretic**

140.) The client has a new prescription for metoclopramide (Reglan). On review of the chart, the nurse identifies that this medication can be safely administered with which condition? 1. Intestinal obstruction 2. Peptic ulcer with melena 3. Diverticulitis with perforation 4. Vomiting following cancer chemotherapy

4. Vomiting following cancer chemotherapy Rationale: Metoclopramide is a gastrointestinal (GI) stimulant and antiemetic. Because it is a GI stimulant, it is contraindicated with GI obstruction, hemorrhage, or perforation. It is used in the treatment of emesis after surgery, chemotherapy, and radiation.

104.) Disulfiram (Antabuse) is prescribed for a client who is seen in the psychiatric health care clinic. The nurse is collecting data on the client and is providing instructions regarding the use of this medication. Which is most important for the nurse to determine before administration of this medication? 1. A history of hyperthyroidism 2. A history of diabetes insipidus 3. When the last full meal was consumed 4. When the last alcoholic drink was consumed

4. When the last alcoholic drink was consumed Rationale: Disulfiram is used as an adjunct treatment for selected clients with chronic alcoholism who want to remain in a state of enforced sobriety. Clients must abstain from alcohol intake for at least 12 hours before the initial dose of the medication is administered. The most important data are to determine when the last alcoholic drink was consumed. The medication is used with caution in clients with diabetes mellitus, hypothyroidism, epilepsy, cerebral damage, nephritis, and hepatic disease. It is also contraindicated in severe heart disease, psychosis, or hypersensitivity related to the medication.

180.) A health care provider (HCP) writes a prescription for digoxin (Lanoxin), 0.25 mg daily. The nurse teaches the client about the medication and tells the client that it is important to: 1. Count the radial and carotid pulses every morning. 2. Check the blood pressure every morning and evening. 3. Stop taking the medication if the pulse is higher than 100 beats per minute. 4. Withhold the medication and call the HCP if the pulse is less than 60 beats per minute.

4. Withhold the medication and call the HCP if the pulse is less than 60 beats per minute. Rationale: An important component of taking this medication is monitoring the pulse rate; however, it is not necessary for the client to take both the radial and carotid pulses. It is not necessary for the client to check the blood pressure every morning and evening because the medication does not directly affect blood pressure. It is most important for the client to know the guidelines related to withholding the medication and calling the HCP. The client should not stop taking a medication.

*** The practical nurse (PN) is taking vital signs on a client who has been treated for melanoma in the past. Which findings would cause the PN to consult the charge nurse?

A mole that is purple in color (An asymmetrical mole, answer I chose) The practical nurse needs to consult the charge nurse about the asymmetrical mole. Melanoma is a skin cancer that is first identified by obvious change in the appearance of skin moles, which is one of the American Cancer Society's caution signs. The American Cancer Society uses the A, B, C, D method. A—asymmetry (a mole that is irregular in shape or two different looking halves); B—border; irregular, blurred, rough, or notched edges; C—changes in color or irregularity in the color of the appearance of the mole; D—diameter; moles larger than ¼ inch or 6 mm larger than a pencil.

A client who is receiving chemotherapy asks the nurse, "Why is so much of my hair falling out each day?" Which response by the nurse best explains the reason for alopecia? A) Chemotherapy affects the cells of the body that grow rapidly, both normal and malignant. B) Alopecia is a common side effect you will experience during long-term steroid therapy. C) Your hair will grow back completely after your course of chemotherapy is completed. D) The chemotherapy causes permanent alterations in your hair follicles that lead to hair loss.

A) Chemotherapy affects the cells of the body that grow rapidly, both normal and malignant. The common adverse effects of chemotherapy (nausea, vomiting, alopecia, bone marrow depression) are due to chemotherapy's effect on the rapidly reproducing cells, both normal and malignant (A). (B and D) do not provide correct information about chemotherapy-induced alopecia. Although (D) is a true statement, it does not effectively answer the client's question.

The nurse is assessing a client with bacterial meningitis. Which assessment finding indicates the client may have developed septic emboli? A) Cyanosis of the fingertips. B) Bradycardia and bradypnea. C) Presence of S3 and S4 heart sounds. D) 3+ pitting edema of the lower extremities.

A) Cyanosis of the fingertips. Septic emboli secondary to meningitis commonly lodge in the small arterioles of the extremities, causing a decrease in circulation to the hands (A) which may lead to gangrene. (B, C, and D) are abnormal findings, but do not indicate the development of septic emboli.

A postmenopausal client asks the nurse why she is experiencing discomfort during intercourse. What response is best for the nurse to provide? A) Estrogen deficiency causes the vaginal tissues to become dry and thinner. B) Infrequent intercourse results in the vaginal tissues losing their elasticity. C) Dehydration from inadequate fluid intake causes vulva tissue dryness. D) Lack of adequate stimulation is the most common reason for dyspareunia.

A) Estrogen deficiency causes the vaginal tissues to become dry and thinner. Estrogen deprivation decreases the moisture-secreting capacity of vaginal cells, so vaginal tissues tend to become thinner, drier (A), and the rugae become smoother which reduces vaginal stretching that contributes to dyspareunia. Dyspareunia is not related to (B or C). While (D) can contribute to discomfort during intercourse, the primary cause is hormone-related.

A middle-aged male client with diabetes continues to eat an abundance of foods that are high in sugar and fat. According to the Health Belief Model, which event is most likely to increase the client's willingness to become compliant with the prescribed diet? A) He visits his diabetic brother who just had surgery to amputate an infected foot. B) He is provided with the most current information about the dangers of untreated diabetes. C) He comments on the community service announcements about preventing complications associated with diabetes. D) His wife expresses a sincere willingness to prepare meals that are within his prescribed diet.

A) He visits his diabetic brother who just had surgery to amputate an infected foot. The loss of a limb by a family member (A) will be the strongest event or "cue to action" and is most likely to increase the perceived seriousness of the disease. (B, C, and D) may influence his behavior but do not have the personal impact of (A).

An 81-year-old male client has emphysema. He lives at home with his cat and manages self-care with no difficulty. When making a home visit, the nurse notices that his tongue is somewhat cracked and his eyeballs are sunken into his head. What nursing intervention is indicated? A) Help the client to determine ways to increase his fluid intake. B) Obtain an appointment for the client to see an ear, nose, and throat specialist. C) Schedule an appointment with an allergist to determine if the client is allergic to the cat. D) Encourage the client to slightly increase his use of oxygen at night and to always use humidified oxygen.

A) Help the client to determine ways to increase his fluid intake. The nurse should suggest creative methods to increase the intake of fluids (A), such as having disposable fruit juices readily available. Clients with COPD should have at least three liters of fluids a day. These clients often reduce fluid intake because of shortness of breath. (B) is not indicated. These symptoms are not indicative of an allergy (C). Many elderly depend on their pets for socialization and self-esteem. Humidified oxygen will not relieve these symptoms and increased oxygen levels will stifle the COPD client's trigger to breathe (D).

A client is placed on a respirator following a cerebral hemorrhage, and vecuronium bromide (Norcuron) 0.04 mg/kg q12h IV is prescribed. Which nursing diagnosis is the priority for this client? A) Impaired communication related to paralysis of skeletal muscles. B) High risk for infection related to increased intracranial pressure. C) Potential for injury related to impaired lung expansion. D) Social isolation related to inability to communicate.

A) Impaired communication related to paralysis of skeletal muscles. To increase the client's tolerance of endotracheal intubation and/or mechanical ventilation, a skeletal-muscle relaxant such as vecuronium is usually prescribed. Impaired communication (A) is a serious outcome because the client cannot communicate his/her needs. Although this client might also experience (D), it is not a priority when compared to (A). Infection is not related to increased intracranial pressure (B). The respirator will ensure that the lungs are expanded (C).

A 51-year-old truck driver who smokes two packs of cigarettes a day and is 30 pounds overweight is diagnosed with having a gastric ulcer. What content is most important for the nurse to include in the discharge teaching for this client? A) Information about smoking cessation. B) Diet instructions for a low-residue diet. C) Instructions on a weight-loss program. D) The importance of increasing milk in the diet.

A) Information about smoking cessation. Smoking has been associated with ulcer formation, and stopping or decreasing the number of cigarettes smoked per day is an important aspect of ulcer management (A). Diet management includes a reduction in high-fiber/high-roughage foods as well as spicy foods. (B) would be indicated for inflammatory bowel disease. Sodium and caloric intake are not the key elements in an ulcer diet. Although this client does need (C), the management of his ulcer is the key factor at this point. (D) would actually increase gastric acid production.

The nurse is caring for a client with syndrome of inappropriate antidiuretic hormone (SIADH), which is manifested by which symptoms? A) Loss of thirst, weight gain. B) Dependent edema, fever. C) Polydipsia, polyuria. D) Hypernatremia, tachypnea.

A) Loss of thirst, weight gain. SIADH occurs when the posterior pituitary gland releases too much ADH, causing water retention, a urine output of less than 20 ml/hour, and dilutional hyponatremia. Other indications of SIADH are loss of thirst, weight gain (A), irritability, muscle weakness, and decreased level of consciousness. (B) is not associated with SIADH. (C) is a finding associated with diabetes insipidus (a water metabolism problem caused by an ADH deficiency), not SIADH. The increase in plasma volume causes an increase in the glomerular filtration rate that inhibits the release of rennin and aldosterone, which results in an increased sodium loss in urine, leading to greater hyponatremia, not (D).

A client is admitted for further testing to confirm sarcoidosis. Which diagnostic test provides definitive information that the nurse should report to the healthcare provider? A) Lung tissue biopsy. B) Positive blood cultures. C) Magnetic resonance imaging (MRI). D) Computerized tomography (CT) of the thorax.

A) Lung tissue biopsy. Sarcoidosis is an inflammatory condition that is characterized by the formation of widespread granulomatous lesions involving a pulmonary primary site. Although chest radiography identifies sarcoidosis, lung tissue biopsy (A) obtained by bronchoscopy or bronchoalveolar lavage provides definitive confirmation. (B) does not provide results for sarcoidosis. Although MRI and CT identify pulmonary lesions, the (C and D) are not necessary and do not provide definitive confirmation.

32. Which of the following test results identify that a patient with an asthma attack is responding to treatment? A. A decreased exhaled nitric oxide B. An increase in CO2 levels C. A decrease in white blood cell count D. An increase in serum bicarbonate levels

A. A decreased exhaled nitric oxide. Nitric oxide levels are increased in the breath of people with asthma. A decrease in the exhaled nitric oxide concentration suggests that the treatment may be decreasing the lung inflammation associated with asthma.

Which assessment finding by the nurse during a client's clinical breast examination requires follow-up? A) Newly retracted nipple. B) A thickened area where the skin folds under the breast. C) Whitish nipple discharge. D) Tender lumpiness noted bilaterally throughout the breasts.

A) Newly retracted nipple. A newly retracted nipple (A), compared to a life-long finding, may be an indication of breast cancer and requires additional follow-up. The inframammary ridge (B) is a normal anatomic finding. Up to 80% of women may experience an intermittent nipple discharge (C), especially related to recent stimulation, and in most cases, nipple discharge is not related to malignancy. (D) is a classic finding for fibrocystic breast disease, a benign condition.

When teaching diaphragmatic breathing to a client with chronic obstructive pulmonary disease (COPD), which information should the nurse provide? A) Place a small book or magazine on the abdomen and make it rise while inhaling deeply. B) Purse the lips while inhaling as deeply as possible and then exhale through the nose. C) Wrap a towel around the abdomen and push against the towel while forcefully exhaling. D) Place one hand on the chest, one hand the abdomen and make both hands move outward.

A) Place a small book or magazine on the abdomen and make it rise while inhaling deeply. Diaphragmatic or abdominal breathing uses the diaphragm instead of accessory muscles to achieve maximum inhalation and to slow the respiratory rate. The client should protrude the abdomen on inhalation and contract it with exhalation, so (A) helps the client visualize the rise and fall of the abdomen. The client should purse the lips while exhaling, not (B). (C and D) are ineffective.

The nurse is assessing a client with chronic renal failure (CRF). Which finding is most important for the nurse to respond to first? A) Potassium 6.0 mEq. B) Daily urine output of 400 ml. C) Peripheral neuropathy. D) Uremic fetor.

A) Potassium 6.0 mEq. Hyperkalemia (normal serum level, 3.5 to 5.5 mEq) is a serious electrolyte disorder that can cause fatal arrhythmias, so (A) is the nursing priority. (B) is an expected finding associated with renal tubular destruction. In CRF, an increase in serum nitrogenous waste products, electrolyte imbalances, and demyelination of the nerve fibers contribute to the development of (C). (D) is a urinous odor of the breath related to the accumulation of blood urea nitrogen and is a common complication of CRF, but not as significant as hyperkalemia.

A client with heart disease is on a continuous telemetry monitor and has developed sinus bradycardia. In determining the possible cause of the bradycardia, the nurse assesses the client's medication record. Which medication is most likely the cause of the bradycardia? A) Propanolol (Inderal). B) Captopril (Capoten). C) Furosemide (Lasix). D) Dobutamine (Dobutrex).

A) Propanolol (Inderal). Inderal (A) is a beta adrenergic blocking agent, which causes decreased heart rate and decreased contractility. Neither (B), an ACE inhibitor, nor (C), a loop diuretic, causes bradycardia. (D) is a sympathomimetic, direct acting cardiac stimulant, which would increase the heart rate.

An adult client is admitted to the hospital burn unit with partial-thickness and full-thickness burns over 40% of the body surface area. In assessing the potential for skin regeneration, what should the nurse remember about full-thickness burns? A) Regenerative function of the skin is absent because the dermal layer has been destroyed. B) Tissue regeneration will begin several days following return of normal circulation. C) Debridement of eschar will delay the body's ability to regenerate normal tissue. D) Normal tissue formation will be preceded by scar formation for the first year.

A) Regenerative function of the skin is absent because the dermal layer has been destroyed. Full-thickness burns destroy the entire dermal layer. Included in this destruction is the regenerative tissue. For this reason, tissue regeneration does not occur, and skin grafting is necessary (A). (B, C, and D) are simply false.

After checking the urinary drainage system for kinks in the tubing, the nurse determines that a client who has returned from the post-anesthesia care has a dark, concentrated urinary output of 54 ml for the last 2 hours. What priority nursing action should be implemented? A) Report the findings to the surgeon. B) Irrigate the indwelling urinary catheter. C) Apply manual pressure to the bladder. D) Increase the IV flow rate for 15 minutes.

A) Report the findings to the surgeon. An adult who weighs 132 pounds (60 kg) should produce about 60 ml of urine hourly (1 ml/kg/hour). Dark, concentrated, and low volume of urine output should be reported to the surgeon. Although other actions (B, C, and D) may be indicated, the assessment findings should be reported to the healthcare provider.

A client is brought to the Emergency Center after a snow-skiing accident. Which intervention is most important for the nurse to implement? A) Review the electrocardiogram tracing. B) Obtain blood for coagulation studies. C) Apply a warming blanket. D) Provide heated PO fluids.

A) Review the electrocardiogram tracing. Airway, breathing, and circulation are priorities in client assessment and treatment. Continuous cardiac monitoring is indicated (A) because hypothermic clients have an increased risk for dysrhythmias. Coagulations studies (A) and re-warming procedures (C and D) can be initiated after a review of the ECG tracing (A).

A 46-year-old female client is admitted for acute renal failure secondary to diabetes and hypertension. Which test is the best indicator of adequate glomerular filtration? A) Serum creatinine. B) Blood Urea Nitrogen (BUN). C) Sedimentation rate. D) Urine specific gravity.

A) Serum creatinine. Creatinine (A) is a product of muscle metabolism that is filtered by the glomerulus, and blood levels of this substance are not affected by dietary or fluid intake. An elevated creatinine strongly indicates nephron loss, reducing filtration. (B) is also an indicator of renal activity, but it can be affected by non-renal factors such as hypovolemia and increased protein intake. (C) is a nonspecific test for acute or chronic inflammatory processes. (D) is useful in assessing hydration status, but not as useful in assessing glomerular function.

The nurse working in a postoperative surgical clinic is assessing a woman who had a left radical mastectomy for breast cancer. Which factor puts this client at greatest risk for developing lymphedema? A) She sustained an insect bite to her left arm yesterday. B) She has lost twenty pounds since the surgery. C) Her healthcare provider now prescribes a calcium channel blocker for hypertension. D) Her hobby is playing classical music on the piano.

A) She sustained an insect bite to her left arm yesterday. A radical mastectomy interrupts lymph flow, and the increased lymph flow that occurs in response to the insect bite increases the risk for the occurrence of lymphedema (A). (B) is not a factor. Lymphedema is not significantly related to vascular circulation (C). Only overuse of the arm, such as weight-lifting, would cause lymphedema--(D) would not.

The nurse formulates the nursing diagnosis of, Urinary retention related to sensorimotor deficit for a client with multiple sclerosis. Which nursing intervention should the nurse implement? A) Teach the client techniques of intermittent self-catheterization. B) Decrease fluid intake to prevent over distention of the bladder. C) Use incontinence briefs to maintain hygiene with urinary dribbling. D) Explain that anticholinergic drugs will decrease muscle spasticity.

A) Teach the client techniques of intermittent self-catheterization. Bladder control is a common problem for clients with multiple sclerosis. A client with urinary retention should receive instructions about self-catheterization (A) to prevent bladder distention. Adequate hydration, not (B), is important to reduce the risk of urinary tract infections by promoting elimination which reduces the time microorganisms spend in the bladder and by diluting the number of microorganisms in the bladder. Self-catheterization helps prevent dribbling, so (C) is unnecessary. Cholinergic drugs improve bladder muscle tone and help with bladder emptying, not (D).

The nurse is teaching a female client about the best time to plan sexual intercourse in order to conceive. Which information should the nurse provide? A) Two weeks before menstruation. B) Vaginal mucous discharge is thick. C) Low basal temperature. D) First thing in the morning.

A) Two weeks before menstruation. Ovulation typically occurs 14 days before menstruation begins (A), and sexual intercourse should occur within 24 hours of ovulation for conception to occur. High estrogen levels occur during ovulation and increase the vaginal mucous membrane characteristics, which become more "slippery" and stretchy, not (B). A rise in basal temperature, not (C), signals ovulation. The timing during the day is not as significant in determining conception as the day before and after ovulation (D).

A client who is sexually active with several partners requests an intrauterine device (IUD) as a contraceptive method. Which information should the nurse provide? A) Using an IUD offers no protection against sexually transmitted diseases (STD), which increase the risk for pelvic inflammatory disease (PID). B) Getting pregnant while using an IUD is common and is not the best contraceptive choice. C) Relying on an IUD may be a safer choice for monogamous partners, but a barrier method provides a better option in preventing STD transmission. D) Selecting a contraceptive device should consider choosing a successful method used in the past.

A) Using an IUD offers no protection against sexually transmitted diseases (STD), which increase the risk for pelvic inflammatory disease (PID). The use of an IUD provides the client with no protection from STDs (A). While pregnancy rates with the use of an IUD are somewhat higher, (B) is not therapeutic, but judgmental. (C) is judgmental and does not provide the client any information about use of an IUD. While talking about contraceptives may include (D), it is does not provide the best information to maintain the client's health.

To promote airway clearance in a patient with pneumonia, the nurse instructs the patient to do which of the following? (Select all that apply.) A. Splint the chest when coughing B. Maintain a semi-Fowler's position C. Maintain adequate fluid intake D. Instruct patient to cough at end of exhalation

A,C,D The nurse should instruct the patient to splint the chest while coughing. This will reduce discomfort and allow for a more effective cough. Maintaining adequate fluid intake liquefies secretions, allowing easier expectoration. Coughing at the end of exhalation promotes a more effective cough. The patient should be positioned in an upright sitting position (high-Fowler's) with head slightly flexed.

After diagnosis and initial treatment of a 3-year-old child with cystic fibrosis, the nurse provides home care instruction to the mother. Which statement by the child's mother indicates that she understands home care treatment to promote pulmonary functions? A. "Chest physiotherapy should be performed twice a day before a meal." B. "Administer a cough suppressant every 8 hours.' C. "Maintain supplemental oxygen at 4 to 6 L/minute." D. "Energy should be conserved by scheduling minimally strenuous activities."

A. "Chest physiotherapy should be performed twice a day before a meal."

29. The physician has prescribed salmeterol (Serevent) for a patient with asthma. In reviewing the use of dry powder inhalers (DPIs) with the patient, the nurse should provide which of the following instructions? A. "Close lips tightly around the mouthpiece and breathe in deeply and quickly." B. "To administer a DPI, you must use a spacer that holds the medicine so that you can inhale it." C. "Hold the inhaler several inches in front of your mouth and breathe in slowly, holding the medicine as long as possible." D. "You will know you have correctly used the DPI when you taste or sense the medicine going into your lungs."

A. "Close lips tightly around the mouthpiece and breathe in deeply and quickly." Dry powder inhalers do not require spacer devices. The patient should be instructed to breathe in deeply and quickly to ensure medicine moves down deeply into lungs. The patient may not taste or sense the medicine going into the lungs.

The physician has prescribed salmeterol (Serevent) for a patient with asthma. In reviewing the use of dry powder inhalers (DPIs) with the patient, the nurse should provide which of the following instructions? A. "Close lips tightly around the mouthpiece and breathe in deeply and quickly." B. "To administer a DPI, you must use a spacer that holds the medicine so that you can inhale it." C. "Hold the inhaler several inches in front of your mouth and breathe in slowly, holding the medicine as long as possible." D. "You will know you have correctly used the DPI when you taste or sense the medicine going into your lungs."

A. "Close lips tightly around the mouthpiece and breathe in deeply and quickly." Dry powder inhalers do not require spacer devices. The patient should be instructed to breathe in deeply and quickly to ensure medicine moves down deeply into lungs. The patient may not taste or sense the medicine going into the lungs.

A female client is seen in the Emergency Department with a broken arm. She is visibly anxious and tells the nurse "I am afraid my husband is going to kill me. The beatings are getting worse, and everyone says I should leave him, but I am afraid of what he will do to me and the children if I do leave." What question is most important for the nurse to ask the client? A. "Have you thought about what to do when you are in an unsafe situation?" B. "Have you considered attending family violence programs sponsored by the police department?" C. "Do you know a clergy person you can talk to?" D. "Did you experience violence in your family of origin?"

A. "Have you thought about what to do when you are in an unsafe situation?"

The nurse is administering a nystatin suspension (Mycostatin) for stomatitis. Which instruction will the nurse provide to the client when administering this medication? A. "Hold the medication in your mouth for a few minutes before swallowing it." B. "Do not drink or eat milk products for 1 hour prior to taking this medication." C. "Dilute the medication with juice to reduce the unpleasant taste and odor." D. "Take the medication before meals to promote increased absorption."

A. "Hold the medication in your mouth for a few minutes before swallowing it." Rationale: Mycostatin is prescribed for fungal infections of the mouth. The client should swish the medication in the mouth for 2 minutes and then swallow (A). (B) does not affect administration of this medication. The medication should not be diluted because this will reduce its effectiveness (C). (D) is not necessary.

During a health fair, a male client with emphysema tells the nurse that he fatigues easily. Assessment reveals marked clubbing of the fingernails and an increased anteroposterior chest diameter. Which instruction is best to provide the client? A. "Pace your activities and schedule rest periods." B. "Increase the amount of oxygen you use at night." C. "Obtain medical evaluation for antibiotic therapy." D. "Reduce your intake of fluids containing caffeine."

A. "Pace your activities and schedule rest periods." Rationale: Manifestations of emphysema include an increase in AP diameter (referred to as a barrel chest), nail bed clubbing, and fatigue. The nurse can provide instructions to promote energy management, such as pacing activities and scheduling rest periods (A). (B) may result in a decreased drive to breathe. The client is not exhibiting any symptoms of infection, so (C) is not necessary. (D) is less beneficial than (A).

29. Which of the following instructions are most appropriate in the home management of a patient who has undergone surgery for oral cancer? A. "You should drink plenty of fluids and eat foods you enjoy." B. "It is normal to have some leakage of saliva from the suture line." C. "Lying in a prone position helps decrease swelling at the suture line." D. "You should avoid foods high in protein while your suture line is healing."

A. "You should drink plenty of fluids and eat foods you enjoy." For patients who have undergone treatment for head and neck cancers, maintaining adequate nutrition is a challenge. The nurse encourages the patient to increase fluids to prevent dehydration and liquefy secretions. These patients are more likely to eat foods that they enjoy and can tolerate.

Which of the following instructions are most appropriate in the home management of a patient who has undergone surgery for oral cancer? A. "You should drink plenty of fluids and eat foods you enjoy." B. "It is normal to have some leakage of saliva from the suture line." C. "Lying in a prone position helps decrease swelling at the suture line." D. "You should avoid foods high in protein while your suture line is healing."

A. "You should drink plenty of fluids and eat foods you enjoy." For patients who have undergone treatment for head and neck cancers, maintaining adequate nutrition is a challenge. The nurse encourages the patient to increase fluids to prevent dehydration and liquefy secretions. These patients are more likely to eat foods that they enjoy and can tolerate.

A male client with arterial peripheral vascular disease (PVD) complains of pain in his feet. Which instruction should the nurse give to the UPA to quickly relieve the client's pain? A. Help the client to dangle his legs. B. Apply compression stockings. C. Assist with passive leg exercises. D. Ambulate three times daily.

A. A client who has arterial PVD may benefit from a dependent position which can be achieved by dangling by improving blood flow and relieving pain. (B) is indicated for venous insufficiency and (C) is indicated for bed rest. (D) is indicated to facilitate collateral circulation and may improve long term complaints of pain.

The nurse is observing an unlicensed assistive personnel (UPA) who is performing morning care for a bedfast client with Huntington disease. Which care measure is most important for the nurse to supervise? A. Oral care B. Bathing C. Foot care D. Catheter care

A. A client with Huntington disease experiences problems with motor skills such as swallowing and is at high risk for aspiration. (B, C, D) do not pose life-threatening consequences.

Which of the following test results identify that a patient with an asthma attack is responding to treatment? A. A decreased exhaled nitric oxide B. An increase in CO2 levels C. A decrease in white blood cell count D. An increase in serum bicarbonate levels

A. A decreased exhaled nitric oxide. Nitric oxide levels are increased in the breath of people with asthma. A decrease in the exhaled nitric oxide concentration suggests that the treatment may be decreasing the lung inflammation associated with asthma.

The nurse on a busy surgical unit is assigning client care to a registered nurse (RN) and a practical nurse (PN). Which client is best to assign to the PN? A. A preoperative client who has developed urinary retention and need a urinary catheter inserted B. A postoperative client who is receiving a unit of packed red blood cells C. A preoperative client who is fearful and anxious about the impending surgery D. A postoperative client who is febrile, has a productive cough, and is complaining of pain

A. A preoperative client who has developed urinary retention and need a urinary catheter inserted

The nurse is preparing a teaching plan for a group of healthy adults. Which individual is most likely to maintain optimum health? A. A teacher whose blood glucose levels average 126 mg/dL daily with oral antidiabetic drugs B. An accountant whose blood pressure averages 140/96 mm Hg and who says he does not have time to exercise C. A stock broker whose total serum cholesterol level dropped to 290 mg/dL with diet modifications D. A recovering IV heroin user who contracted hepatitis more than 10 years ago

A. A teacher whose blood glucose levels average 126 mg/dL daily with oral antidiabetic drugs Rationale: The diabetic teacher (A) has assumed responsibility for self-care, so among those listed, is the most likely to maintain optimum health. (B) has expressed a lack of interest in health promotion. (C) continues to demonstrate a high-risk cholesterol level despite a reported attempt at dietary modifications. Previous IV drug use and a history of hepatitis (D) make this individual a health risk despite the fact that the individual is in recovery.

The nurse is preparing a teaching plan for a group of healthy adults. Which individual is most likely to maintain optimum health? A. A teacher whose blood glucose levels average 126 mg/dL daily with oral antidiabetic drugs B.An accountant whose blood pressure averages 140/96 mm Hg and who says he does not have time to exercise C. A stock broker whose total serum cholesterol level dropped to 290 mg/dL with diet modifications D. A recovering IV heroin user who contracted hepatitis more than 10 years ago

A. A teacher whose blood glucose levels average 126 mg/dL daily with oral antidiabetic drugs Rationale: The diabetic teacher has assumed responsibility for self-care, so among those listed, he or she is the most likely to maintain optimum health. Option B has expressed a lack of interest in health promotion. Option C continues to demonstrate a high-risk cholesterol level despite a reported attempt at dietary modifications. Previous IV drug use and a history of hepatitis make this individual a health risk despite the fact that the individual is in recovery.

The nurse is performing hourly neurologic checks for a client with a head injury. Which new assessment finding warrants immediate intervention by the nurse? A. A unilateral pupil that is dilated and nonreactive to light B. Client cries out when awakened by a verbal stimulus C. Client demonstrates a loss of memory of the events leading up to the injury D. Onset of nausea, headache, and vertigo

A. A unilateral pupil that is dilated and nonreactive to light Rationale: Any change in pupil size and reactivity is an indication of increasing intracranial pressure and should be reported to the health care provider immediately (A). (B) is a normal response to being awakened. (C and D) are common manifestations of head injury and are of less immediacy than (A).

The nurse is performing hourly neurologic checks for a client with a head injury. Which new assessment finding warrants immediate intervention by the nurse? A. A unilateral pupil that is dilated and nonreactive to light B. Client cries out when awakened by a verbal stimulus C. Client demonstrates a loss of memory of the events leading up to the injury D.Onset of nausea, headache, and vertigo

A. A unilateral pupil that is dilated and nonreactive to light Rationale: Any change in pupil size and reactivity is an indication of increasing intracranial pressure and should be reported to the health care provider immediately. Option B is a normal response to being awakened. Options C and D are common manifestations of head injury and are of less immediacy than option A.

The nurse is assessing several clients at a homeless shelter. Which client(s) should the nurse refer for involuntary admission to a psychiatric facility? (select all that apply) A. A young adult who has lost his appetite because the communists are asking him to kill all conservative Catholics B. An adult male who has not eaten for three days and says he is Jesus Christ and that God loves all people C. An older woman who used to be a prostitute and resides in the shelter about three nights weekly D. A middle-aged woman who sits for long periods of time and mumbles to herself about wanting to die E. An adolescent male who works on the streets as a beggar to purchase marijuana, food, and shelter

A. A young adult who has lost his appetite because the communists are asking him to kill all conservative Catholics B. An adult male who has not eaten for three days and says he is Jesus Christ and that God loves all people D. A middle-aged woman who sits for long periods of time and mumbles to herself about wanting to die

23. A patient is having inspiratory stridor (crowing respiration) and the nurse suspects he is experiencing a laryngospasm. Which of the following would be most appropriate to implement for a patient experiencing a laryngospasm? A. Administer 100% oxygen. B. Position the patient in high Fowler's position. C. Insert a 16-gauge (large-bore) IV needle. D. Activate the emergency response team (code blue team) to the patient's room.

A. Administer 100% oxygen.A nurse should immediately administer 100% oxygen to the patient until the airway is fully reestablished, the larynx relaxes, and the spasms stop. Activating the emergency response team is not an immediate nursing action at this time because the nurse can administer the oxygen without the assistance of others. Positioning the patient in high Fowler's will not address the patient's need for immediate reoxygenation because of the patient's compromised respiratory state. Insertion of an IV device is not the first priority response but should be implemented after the nurse has assessed that the airway is stable.

The DASH (Dietary Approaches to Stop Hypertension) diet is prescribed for a client with uncontrolled hypertension. Which dietary choices should the nurse instruct the client to eat? A. Shredded wheat B. Avocado salad with olives C. Cheddar cheese D. Canned roasted almonds

A. Shredded wheat

A client with hypertension has been receiving ramipril (Altace), 5 mg PO, daily for 2 weeks and is scheduled to receive a dose at 0900. At 0830, the client's blood pressure is 120/70 mm Hg. Which action should the nurse take? A.Administer the prescribed dose at the scheduled time. B.Hold the dose and contact the health care provider. C.Hold the dose and recheck the blood pressure in 1 hour. D.Check the health care provider's prescription to clarify the dose.

A. Administer the prescribed dose at the scheduled time Rationale: The client's blood pressure is within normal limits, indicating that the ramipril, an antihypertensive, is having the desired effect and should be administered. Options B and C would be appropriate if the client's blood pressure was excessively low (<100 mm Hg systolic) or if the client were exhibiting signs of hypotension such as dizziness. This prescribed dose is within the normal dosage range, as defined by the manufacturer; therefore, option D is not necessary.

A client with hypertension has been receiving ramipril (Altace), 5 mg PO, daily for 2 weeks and is scheduled to receive a dose at 0900. At 0830, the client's blood pressure is 120/70 mm Hg. Which action should the nurse take? A. Administer the prescribed dose at the scheduled time. B. Hold the dose and contact the health care provider. C. Hold the dose and recheck the blood pressure in 1 hour. D. Check the health care provider's prescription to clarify dose.

A. Administer the prescribed dose at the scheduled time. Rationale: The client's blood pressure is within normal limits, indicating that the ramipril, an antihypertensive, is having the desired effect and should be administered (A). (B and C) would be appropriate if the client's blood pressure was excessively low (<100 mm Hg systolic) or if the client were exhibiting signs of hypotension such as dizziness. This prescribed dose is within the normal dosage range, as defined by the manufacturer; therefore, (D) is not necessary.

The nurse makes a supervisory home visit to observe an unlicensed assistive personnel (UAP) who is providing personal care for a client with Alzheimer's Disease. The nurse observes that whatever the client gets upset, the UAP changes the subject. What action should the nurse take in response to this observation? A. Affirm that the UAP is using an effective strategy to reduce the client's anxiety B. Meet with the UAP later to role model more assertive communication techniques C. Assume care of the client to ensure that effective communication is maintained D. Tell the UAP to offer more choices during the personal care to prevent anxiety

A. Affirm that the UAP is using an effective strategy to reduce the client's anxiety

34. The patient has an order for each of the following inhalers. Which of the following should the nurse offer to the patient at the onset of an asthma attack? A. Albuterol (Proventil) B. Beclomethasone (Beclovent) C. Ipratropium bromide (Atrovent) D. Salmeterol (Serevent)

A. Albuterol (Proventil) Albuterol is a short-acting bronchodilator that should initially be given when the patient experiences an asthma attack.

A charge nurse is establishing priorities of client needs at the beginning of the morning shift. Which client situation has the highest priority for immediate care? A. An elderly client who is trying to climb over the side rails B. A diabetic client who requires insulin before breakfast C. A postoperative client with unremitting nausea D. A 5-year-old child who is crying because the parents left

A. An elderly client who is trying to climb over the side rails

A 45-year-old man with asthma is brought to the emergency department by automobile. He is short of breath and appears frightened. During the initial nursing assessment, which of the following clinical manifestations might be present as an early symptom during an exacerbation of asthma? A. Anxiety B. Cyanosis C. Hypercapnia D. Bradycardia

A. Anxiety An early symptom during an asthma attack is anxiety because he is acutely aware of the inability to get sufficient air to breathe. He will be hypoxic early on with decreased PaCO2 and increased pH as he is hyperventilating.

What nursing intervention is particularly indicated for the second stage of labor? A. Assisting the client to push effectively so that expulsion of the fetus can be achieved B. Providing pain medication to increase the client's tolerance of labor pains C. Assessing the fetal heart rate and pattern for signs of fetal distress D. Monitoring effects of oxytocin administration to help achieve cervical dilation

A. Assisting the client to push effectively so that expulsion of the fetus can be achieved

When initially teaching a patient the supraglottic swallow following a radical neck dissection, with which of the following foods should the nurse begin? A. Cola B. Applesauce C. French fries D. White grape juice

A. ColaWhen learning the supraglottic swallow, it may be helpful to start with carbonated beverages because the effervescence provides clues about the liquid's position. Thin, watery fluids should be avoided because they are difficult to swallow and increase the risk of aspiration. Nonpourable pureed foods, such as applesauce, would decrease the risk of aspiration, but carbonated beverages are the better choice to start with.

During the shift report, the charge nurse informs a nurse that she has been assigned to another unit for the day. The nurse begins to sigh deeply and tosses about her belongings as she prepares to leave, making it known that she is very unhappy about being floated to the other unit. What is the best immediate action for the charge nurse to take? A. Continue with the shift report and talk to the nurse about the incident at a later time. B. Ask the nurse to call the house supervisor to see if she must be reassigned. C. Stop the shift report and remind the nurse that all staff are floated equally. D. Inform the nurse that her behavior is disruptive to the rest of the staff.

A. Continue with the shift report and talk to the nurse about the incident at a later time. Rationale: Continuing with the shift report (A) is the best immediate action because it allows the nurse who was floated some cooling off time. At a later time (after the nurse has cooled off) the charge nurse should discuss the conduct of the nurse in private. (B) encourages the nurse to shirk the float assignment. (C) is disruptive. Reprimanding the nurse in front of the staff would increase the nurse's hostility, so the nurse should be counseled in private (D).

During the shift report, the charge nurse informs a nurse that she has been assigned to another unit for the day. The nurse begins to sigh deeply and tosses about her belongings as she prepares to leave, making it known that she is very unhappy about being floated to the other unit. What is the best immediate action for the charge nurse to take? A. Continue with the shift report and talk to the nurse about the incident at a later time. B. Ask the nurse to call the house supervisor to see if she must be reassigned. C. Stop the shift report and remind the nurse that all staff are floated equally. D. Inform the nurse that her behavior is disruptive to the rest of the staff.

A. Continue with the shift report and talk to the nurse about the incident at a later time. Rationale: Continuing with the shift report is the best immediate action because it allows the nurse who was floated some cooling off time. At a later time (after the nurse has cooled off) the charge nurse should discuss the conduct of the nurse in private. Option B encourages the nurse to shirk the float assignment. Option C is disruptive. Reprimanding the nurse in front of the staff would increase the nurse's hostility, so the nurse should be counseled in private.

The nurse is conducting an osteoporosis screening clinic at a health fair. What information should the nurse provide to individuals who are at risk for osteoporosis? (Select all that apply.) A. Encourage alcohol and smoking cessation. B.Suggest supplementing diet with vitamin E. C. Promote regular weight-bearing exercises. D. Implement a home safety plan to prevent falls. E. Propose a regular sleep pattern of 8 hours nightly.

A. Encourage alcohol and smoking cessation. C. Promote regular weight-bearing exercises. D. Implement a home safety plan to prevent falls. Rationale: (A, C, and D) are factors that decrease the risk for developing osteoporosis. Vitamin D and calcium are important supplements to aid in the decrease of bone loss (B). Regular sleep patterns are important to overall health but are not identified with a decreasing risk for osteoporosis (E).

3. When caring for a patient with metastatic cancer, the nurse notes a hemoglobin level of 8.7 g/dl and hematocrit of 26%. The nurse would place highest priority on initiating interventions that will reduce which of the following? A. Fatigue B. Thirst C. Headache D. Abdominal pain

A. Fatigue The patient with a low hemoglobin and hematocrit (normal values approximately 13.5% to 17% and 40% to 54%, respectively) is anemic and would be most likely to experience fatigue. This symptom develops because of the lowered oxygen-carrying capacity that leads to reduced tissue oxygenation to carry out cellular functions.

When caring for a patient with metastatic cancer, the nurse notes a hemoglobin level of 8.7 g/dl and hematocrit of 26%. The nurse would place highest priority on initiating interventions that will reduce which of the following? A. Fatigue B. Thirst C. Headache D. Abdominal pain

A. Fatigue The patient with a low hemoglobin and hematocrit (normal values approximately 13.5% to 17% and 40% to 54%, respectively) is anemic and would be most likely to experience fatigue. This symptom develops because of the lowered oxygen-carrying capacity that leads to reduced tissue oxygenation to carry out cellular functions.

A client is diagnosed with an acute small bowel obstruction. Which assessment finding requires the most immediate intervention by the nurse? A. Fever of 102° F B. Blood pressure of 150/90 mm Hg C. Abdominal cramping D. Dry mucous membranes

A. Fever of 102° F Rationale: A sudden increase in temperature is an indicator of peritonitis. The nurse should notify the health care provider immediately (A). (B, C, and D) are also findings that require intervention by the nurse, but are of less priority than (A). (B) may indicate a hypertensive condition but is not as acute a condition as peritonitis. (C) is an expected finding in clients with small bowel obstruction and may require medication. (D) indicates probable fluid volume deficit, which requires fluid volume replacement.

A client is being discharged following radioactive seed implantation for prostate cancer. What is the most important information that the nurse should provide to this client's family? A. Follow exposure precautions. B. Encourage regular meals. C. Collect all urine. D. Avoid touching the client.

A. Follow exposure precautions. Rationale: Clients being treated for prostate cancer with radioactive seed implants should be instructed regarding the amount of time and distance needed to prevent excessive exposure (A) that would pose a hazard to others. (B) is a good suggestion to promote adequate nutrition but is not as important as (A). (C) is unnecessary. Contact with the client (D) is permitted but should be brief to limit radiation exposure.

A middle-aged woman, diagnosed with Graves' disease, asks the nurse about this condition. Which etiological pathology should the nurse include in the teaching plan about hyperthyroidism? (select all that apply) A. Graves' disease, an autoimmune condition, affects thyroid stimulating hormone receptors B. Large protruding eyeballs are a sign of hyperthyroid function C. Early treatment includes levothyroxine D. T3 and T4 hormone levels are increased E. Weight gain is a common complaint in hyperthyroidism

A. Graves' disease, an autoimmune condition, affects thyroid stimulating hormone receptors B. Large protruding eyeballs are a sign of hyperthyroid function D. T3 and T4 hormone levels are increased

An 81-year-old male client has emphysema. He lives at home with his cat and manages self-care with no difficulty. When making a home visit, the nurse notices that this client's tongue is somewhat cracked and his eyeballs appear sunken into his head. Which nursing intervention is indicated? A. Help the client determine ways to increase his fluid intake. B. Obtain an appointment for the client to have an eye examination. C. Instruct the client to use oxygen at night and increase the humidification. D. Schedule the client for tests to determine his sensitivity to cat hair.

A. Help the client determine ways to increase his fluid intake. Rationale: Clients with COPD should ingest 3 L of fluids daily but may experience a fluid deficit because of shortness of breath. The nurse should suggest creative methods to increase the intake of fluids (A), such as having fruit juices in disposable containers readily available. (B) is not indicated. Humidified oxygen will not effectively treat the client's fluid deficit, and there is no indication that the client needs supplemental oxygen at night (C). These symptoms are not indicative of (D) and may unnecessarily upset the client, who depends on his pet for socialization.

6. The nurse is caring for a patient admitted to the hospital with pneumonia. Upon assessment, the nurse notes a temperature of 101.4° F, a productive cough with yellow sputum and a respiratory rate of 20. Which of the following nursing diagnosis is most appropriate based upon this assessment? A. Hyperthermia related to infectious illness B. Ineffective thermoregulation related to chilling C. Ineffective breathing pattern related to pneumonia D. Ineffective airway clearance related to thick secretions

A. Hyperthermia related to infectious illness Because the patient has spiked a temperature and has a diagnosis of pneumonia, the logical nursing diagnosis is hyperthermia related to infectious illness. There is no evidence of a chill, and her breathing pattern is within normal limits at 20 breaths per minute. There is no evidence of ineffective airway clearance from the information given because the patient is expectorating sputum.

The nurse is caring for a patient admitted to the hospital with pneumonia. Upon assessment, the nurse notes a temperature of 101.4° F, a productive cough with yellow sputum and a respiratory rate of 20. Which of the following nursing diagnosis is most appropriate based upon this assessment? A. Hyperthermia related to infectious illness B. Ineffective thermoregulation related to chilling C. Ineffective breathing pattern related to pneumonia D. Ineffective airway clearance related to thick secretions

A. Hyperthermia related to infectious illness Because the patient has spiked a temperature and has a diagnosis of pneumonia, the logical nursing diagnosis is hyperthermia related to infectious illness. There is no evidence of a chill, and her breathing pattern is within normal limits at 20 breaths per minute. There is no evidence of ineffective airway clearance from the information given because the patient is expectorating sputum.

A client is placed on a mechanical ventilator following a cerebral hemorrhage, and vecuronium bromide (Norcuron), 0.04 mg/kg every 12 hours IV, is prescribed. What is the priority nursing diagnosis for this client? A. Impaired communication related to paralysis of skeletal muscles B. High risk for infection related to increased intracranial pressure C. Potential for injury related to impaired lung expansion D. Social isolation related to inability to communicate

A. Impaired communication related to paralysis of skeletal muscles Rationale: To increase the client's tolerance of endotracheal intubation and/or mechanical ventilation, a skeletal muscle relaxant such as vecuronium is usually prescribed. (A) is a serious outcome because the client cannot communicate his or her needs. Although this client might also experience (D), it is not a priority when compared with (A). Infection is not related to increased intracranial pressure (B). The respirator will ensure that the lungs are expanded, so (C) is incorrect.

A client is placed on a mechanical ventilator following a cerebral hemorrhage, and vecuronium bromide, 0.04 mg/kg every 12 hours IV, is prescribed. What is the priority nursing diagnosis for this client? A.Impaired communication related to paralysis of skeletal muscles B.High risk for infection related to increased intracranial pressure C.Potential for injury related to impaired lung expansion D.Social isolation related to inability to communicate

A. Impaired communication related to paralysis of skeletal muscles Rationale: To increase the client's tolerance of endotracheal intubation and/or mechanical ventilation, a skeletal muscle relaxant such as vecuronium is usually prescribed. Option A is a serious outcome because the client cannot communicate his or her needs. Although this client might also experience option D, it is not a priority when compared with option A. Infection is not related to increased intracranial pressure. The respirator will ensure that the lungs are expanded, so option C is incorrect.

49. The nurse evaluates that a patient is experiencing the expected beneficial effects of ipratropium (Atrovent) after noting which of the following assessment findings? A. Increased peak flow readings B. Increased level of consciousness C. Decreased sputum production D. Increased respiratory rate

A. Increased peak flow readings. Ipratropium is a bronchodilator that should lead to increased PEFRs.

The nurse evaluates that a patient is experiencing the expected beneficial effects of ipratropium (Atrovent) after noting which of the following assessment findings? A. Increased peak flow readings B. Increased level of consciousness C. Decreased sputum production D. Increased respiratory rate

A. Increased peak flow readings. Ipratropium is a bronchodilator that should lead to increased PEFRs.

A client is admitted voluntarily to the in-patient psychiatric unit for multi-substance dependency. While the client is in a group therapy, a peace officer approaches the nurse's station demanding to arrest the client. How should the nurse respond? A. Inform the officer that client information cannot be released B. Ensure safety for other client's by handcuffing the client during removal C. Refer the peace officer to the client's healthcare provider D. Call the unit security code and have the peace officer escorted off the unit.

A. Inform the officer that client information cannot be released

An emaciated homeless client presents to the emergency department complaining of a productive cough, with blood-tinged sputum and night sweats. Which action is most important for the emergency department triage nurse to implement for this client? A. Initiate airborne infection precautions. B. Place a surgical mask on the client. C. Don an isolation gown and latex gloves. D. Start protective (reverse) isolation precautions.

A. Initiate airborne infection precautions. Rationale: This client is exhibiting classic symptoms of tuberculosis (TB), and the client is from a high-risk population for TB. Therefore, airborne infection precautions (A), which are indicated for TB, should be used with this client. (B) is used with droplet precautions. There is no evidence that (C or D) would be warranted at this time.

1. When assessing a patient's nutritional-metabolic pattern related to hematologic health, the nurse would: A. Inspect the skin for petechiae. B. Ask the patient about joint pain. C. Assess for vitamin C deficiency. D. Determine if the patient can perform ADLs.

A. Inspect the skin for petechiae. Any changes in the skin's texture or color should be explored when assessing the patient's nutritional-metabolic pattern related to hematologic health. The presences of petechiae or ecchymotic areas could be indicative of hematologic deficiencies related to poor nutritional intake or related causes.

When assessing a patient's nutritional-metabolic pattern related to hematologic health, the nurse would: A. Inspect the skin for petechiae. B. Ask the patient about joint pain. C. Assess for vitamin C deficiency. D. Determine if the patient can perform ADLs.

A. Inspect the skin for petechiae. Any changes in the skin's texture or color should be explored when assessing the patient's nutritional-metabolic pattern related to hematologic health. The presences of petechiae or ecchymotic areas could be indicative of hematologic deficiencies related to poor nutritional intake or related causes.

A male client with major depression who is taking fluoxetine calls the psychiatric clinic complaining of be more agitated, irritable, and anxious than usual. Which intervention should the nurse implement? A. Instruct the client to seek medical attention immediately. B. Explain that these are common side effects of Prozac. C. Encourage him to take the fluoxetine at night with a snack. D. Tell the client to have a complete blood count (CBC) drawn.

A. Instruct the client to seek medical attention immediately. Agitation, irritability, anxiety, and mania are early signs of serotonin syndrome, a rare but fatal reaction to SSRIs, so the client should seek medical attention immediately.

The nurse who has administered a first dose of oral prednisone (Deltasone) to the patient with asthma writes on the care plan to begin monitoring which of the following patient parameters? A. Intake and output B. Bowel sounds C. Apical pulse D. Deep tendon reflexes

A. Intake and output Corticosteroids such as prednisone can lead to fluid retention. For this reason, it is important to monitor the patient's intake and output.

Based on the clinical manifestations of Cushing syndrome, which nursing intervention would be appropriate for a client who is newly diagnosed with Cushing syndrome? A. Monitor blood glucose levels daily. B. Increase intake of fluids high in potassium. C. Encourage adequate rest between activities. D. Offer the client a sodium-enriched menu.

A. Monitor Blood Glucose Levels Cushing syndrome results from a hypersecretion of glucocorticoids in the adrenal cortex. Clients with Cushing syndrome often develop diabetes mellitus. Monitoring of serum glucose levels assesses for increased blood glucose levels so that treatment can begin early. A common finding in Cushing syndrome is generalized edema. Although potassium is needed, it is generally obtained from food intake, not by offering potassium-enhanced fluids. Fatigue is usually not an overwhelming factor in Cushing syndrome, so an emphasis on the need for rest is not indicated A low-calorie, low-carbohydrate, low-sodium diet is not recommended.

Based on the clinical manifestations of Cushing's syndrome, which nursing intervention would be appropriate for a client who is newly diagnosed with Cushing's syndrome? A. Monitor blood glucose levels daily. B. Increase intake of fluids high in potassium. C. Encourage adequate rest between activities. D. Offer the client a sodium-enriched menu.

A. Monitor blood glucose levels daily. Rationale: Cushing's syndrome results from a hypersecretion of glucocorticoids in the adrenal cortex. Clients with Cushing's syndrome often develop diabetes mellitus. Monitoring of serum glucose levels (A) assesses for increased blood glucose levels so that treatment can begin early. A common finding in Cushing's syndrome is generalized edema. Although potassium is needed, it is generally obtained from food intake, not by offering potassium-enhanced fluids (B). Fatigue is usually not an overwhelming factor in Cushing's syndrome, so an emphasis on the need for rest (C) is not indicated A low-calorie, low-carbohydrate, low-sodium diet is not recommended (D).

An older female who ambulates with a quad-cane prefers to use a wheelchair because she has a halting and unsteady gait at times. Which interventions should the nurse implement? (select all that apply) A. Move personal items within client's reach B. Lower bed to the lowest possible position C. Raise all bed rails when client is resting D. Give directions to call for assistance E. Assist client to the bathroom q 2 hours F. Encourage the use of the wheelchair

A. Move personal items within client's reach B. Lower bed to the lowest possible position D. Give directions to call for assistance E. Assist client to the bathroom q 2 hours

The nurse is interviewing a client with schizophrenia. Which client behavior requires immediate intervention? A. Muscle spasms of the back and neck B. Rocks back and forth in the chair C. Shuffling gait and stooped posture D. Lip smacking and frequent eye blinking

A. Muscle spasms of the back and neck

The nurse is observing an unlicensed assistive personnel (UAP) performing morning care for a bedridden client with Huntington's disease. Which care measure is most important for the nurse to supervise? A. Oral care B. Bathing C. Foot care D. Catheter care

A. Oral care Rationale: The client with Huntington's disease experiences problems with motor skills such as swallowing and is at high risk for aspiration, so the highest priority for the nurse to observe is the UAP's ability to perform oral care safely (A). (B, C, and D) do not necessarily require registered nurse (RN) supervision because they do not ordinarily pose life-threatening consequences.

The nurse is observing an unlicensed assistive personnel (UAP) performing morning care for a bedridden client with Huntington disease. Which care measure is most important for the nurse to supervise? A. Oral care B.Bathing C. Foot care D. Catheter care

A. Oral care Rationale: The client with Huntington disease experiences problems with motor skills such as swallowing and is at high risk for aspiration, so the highest priority for the nurse to observe is the UAP's ability to perform oral care safely. Options B, C, and D do not necessarily require registered nurse (RN) supervision because they do not ordinarily pose life-threatening consequences.

A 15-year-old client with a spinal cord injury develops spastic leg tremors, sweating, and a headache. Which action should the nurse implement? A. Palpate the bladder for distention B. Obtain an oxygen saturation level C. Administer a prescribed analgesic D. Encourage dorsiflexion of the feet

A. Palpate the bladder for distention

8. While obtaining the admission assessment data, which of the following characteristics would a nurse expect a patient with anemia to report? A. Palpitations B. Blurred vision C. Increased appetite D. Feeling of warm flushing sensation

A. Palpitations Patients experiencing moderate anemia (hemoglobin [Hb] 6 to 10 g/dL) may experience dyspnea (shortness of breath), palpitations, diaphoresis (profound perspiration) with exertion, and chronic fatigue. Blurred vision is associated in patients experiencing profound anemia states. Anorexia is common in patients with severe anemia, as well. Patients with anemia often appear pale and complain of feeling cold because of compensatory vasoconstriction of the subcutaneous capillaries.

While obtaining the admission assessment data, which of the following characteristics would a nurse expect a patient with anemia to report? A. Palpitations B. Blurred vision C. Increased appetite D. Feeling of warm flushing sensation

A. Palpitations Patients experiencing moderate anemia (hemoglobin [Hb] 6 to 10 g/dL) may experience dyspnea (shortness of breath), palpitations, diaphoresis (profound perspiration) with exertion, and chronic fatigue. Blurred vision is associated in patients experiencing profound anemia states. Anorexia is common in patients with severe anemia, as well. Patients with anemia often appear pale and complain of feeling cold because of compensatory vasoconstriction of the subcutaneous capillaries.

A 63-year-old client with type 2 diabetes mellitus is admitted for treatment of an ulcer on the heel of the left foot that has not healed with wound care. The nurse observes that the entire left foot is darker in color than the right foot. Which additional symptom should the nurse expect to find? A. Pedal pulses will be weak or absent in the left foot. B. The client will state that the left foot is usually warm. C. Flexion and extension of the left foot will be limited. D. Capillary refill of the client's left toes will be brisk.

A. Pedal pulses will be weak or absent in the left foot. Rationale: Symptoms associated with decreased blood supply are weak or absent pedal and tibial pulses (A). The client with diabetes experiences vascular scarring as a result of atherosclerotic changes in the peripheral vessels. This results in compromised perfusion to the dependent extremities, which further delays wound healing in the affected foot. Although flexion and extension may be limited (C), depending on the degree of damage, this is not always the case. (B and D) are signs of adequate perfusion of the foot, which would not be expected in this client.

A 63-year-old client with type 2 diabetes mellitus is admitted for treatment of an ulcer on the heel of the left foot that has not healed with wound care. The nurse observes that the entire left foot is darker in color than the right foot. Which additional symptom should the nurse expect to find? A.Pedal pulses will be weak or absent in the left foot. B.The client will state that the left foot is usually warm C.Flexion and extension of the left foot will be limited. D.Capillary refill of the client's left toes will be brisk.

A. Pedal pulses will be weak or absent in the left foot. Rationale: Symptoms associated with decreased blood supply are weak or absent pedal and tibial pulses. The client with diabetes experiences vascular scarring as a result of atherosclerotic changes in the peripheral vessels. This results in compromised perfusion to the dependent extremities, which further delays wound healing in the affected foot. Although flexion and extension may be limited, depending on the degree of damage, this is not always the case. Options B and D are signs of adequate perfusion of the foot, which would not be expected in this client.

An older male adult resident of an extended care facility receives a prescription for diphenhydramine (Benadryl) 25 mg PO to treat generalized pruritus. Two hours after administration of the drug, he continues to experience itching, is confused, and has an unsteady gait. What action should the nurse implement first? A. Place the client on fall precautions B. Apply soft limb restraints to extremities C. Give a second dose of Benadryl D. Lubricate the skin with an emollient

A. Place the client on fall precautions

The nurse is caring for a one-week infant who has a ventriculoperitoneal (VP) shunt that was placed 2 days after birth. Which findings are an indication of a postoperative complication? (select all that apply) A. Poor feeding and vomiting B. Leakage of CSF form the incisional site C. Hyperactive bowel sounds D. Abdominal distension E. White blood cell count of 10, 000/mm^3

A. Poor feeding and vomiting B. Leakage of CSF form the incisional site D. Abdominal distension

45. The nurse reviews pursed lip breathing with a patient newly diagnosed with emphysema. The nurse reinforces that this technique will assist respiration by which of the following mechanisms? A. Preventing bronchial collapse and air trapping in the lungs during exhalation B. Increasing the respiratory rate and giving the patient control of respiratory patterns C. Loosening secretions so that they may be coughed up more easily D. Promoting maximal inhalation for better oxygenation of the lungs

A. Preventing bronchial collapse and air trapping in the lungs during exhalation The focus of pursed lip breathing is to slow down the exhalation phase of respiration, which decreases bronchial collapse and subsequent air trapping in the lungs during exhalation.

While assessing a client's chest tube (CT), the nurse discovers bubbling in the water seal chamber of the chest tube collections device. The client's vital sign are: blood pressure 80/40 mmHg, heart rate 120 beats/minute, respiratory rate 32 breaths/minute, oxygen saturation 88%. Which interventions should the nurse implement? (select all that apply) A. Provide supplemental oxygen B. Auscultate bilateral lung fields C. Administer a nebulizer treatment D. Reinforce occlusive CT dressing E. Give PRN dose of pain medication

A. Provide supplemental oxygen B. Auscultate bilateral lung fields D. Reinforce occlusive CT dressing

The legs of a client who is receiving hospice care have become mottled in appearance. When the nurse observes the unlicensed assistive personnel (UAP) place a heating pad on the mottled areas, what action should the nurse take? A. Remove the heating pads and pace a soft blanket over the client's legs and feet B. Advise the UAP to observe the client's skin while the heating pads are in place C. Evaluate the client's feet on a pillow and monitor the client's pedal pulses frequently D. Instruct the UAP to reposition the heating pads to the sides of the legs and feet

A. Remove the heating pads and pace a soft blanket over the client's legs and feet

What modification is most important for the nurse to recommend to a client with high cholesterol? A. Replace beed with fish or poultry B. Include six servings of fruits and vegetables daily C. Limit portion sizes using the "Plate Method" D. Use vegetables oils in food preparation

A. Replace beed with fish or poultry

A client who had a below-the-knee amputation is experiencing severe phantom limb pain (PLP) and asks the nurse if mirror therapy will make the pain stop. Which response by the nurse is likely to be most helpful? A. Research indicated that mirror therapy is effective in reducing phantom limb pain B. You can try mirror therapy, but do not expect complete elimination of the pain C. Transcutaneous electrical nerve stimulation (TENS) has been found to be more effective D. Where did you learn about the use of mirror therapy in treating phantom limb pain?

A. Research indicated that mirror therapy is effective in reducing phantom limb pain

The scrub nurse places the fenestrated drapes to expose the operative area for a client who is having a hepatic tumor removed. The scrub nurse should assist with applying the sterile impermeable adhesive drape or surgical skin barrier to which area? A. Right subcostal B. Right lumbar C. Low transverse D. Midline abdominal

A. Right subcostal

If a nurse is caring for an 80-year-old patient with a temperature of 100.4° F, crackles at the right lung base, pain with deep inspiration, and dyspnea, which of the following orders is the nurse's priority? A. Sputum specimen for culture and sensitivity B. Codeine 15 mg orally every 6 hours as needed C. Incentive spirometer every 2 hours while awake D. Amoxicillin (Amoxil) 500 mg orally 4 times a day

A. Sputum specimen for culture and sensitivity The patient presents with signs of a respiratory infection. To initiate the most effective therapy, the health care prescriber must know the pathogen causing the infection. Therefore, the sputum specimen is the nurse's priority. If the antibiotic is administered before the specimen is obtained, the results of the culture might not be as accurate and could impair the effectiveness of therapy. After the specimen is obtained, the nurse can administer codeine for coughing and begin the incentive spirometry to mobilize secretions and improve the patient's ability to expectorate the secretions.

A nurse stops at the site of a motorcycle accident and finds a young adult male lying face down in the road in a puddle of water. It is raining, no one is available to send for help, and the cell phone is in the car about 50 feet away. What action should the nurse take first? A. Stabilize the victim's neck and roll over to evaluate his status B. Examine the victim's body surfaces for arterial bleeding C. Open the airway and initiate resuscitative measures D. Return to the car to call emergency response 911 for help

A. Stabilize the victim's neck and roll over to evaluate his status

In assessing an older client with dementia for sundowning syndrome, what assessment technique is best for the nurse to use? A. Observe for tiredness at the end of the day. B. Perform a neurologic exam and mental status exam. C. Monitor for medication side effects. D. Assess for decreased gross motor movement.

A. Sundowning syndrome is a pattern of agitated behavior in the evening, believed to be associated with tiredness at the end of the day combined with fewer orienting stimuli, such as activities and interactions. (B, C, & D) with not provide information about this syndrome.

A client with Type 1 diabetes mellitus (DM) is admitted for an emergency cholecystectomy. To prevent diabetic ketoacidosis (DKA), which intervention is most important for the nurse to implement? A. Supplement insulin needs using a sliding scale B. Monitor the client for polyuria and dehydration C. Measure urine ketones when blood glucose exceeds 300 mg/dl D. Teach the client to monitor blood glucose levels frequently when ill

A. Supplement insulin needs using a sliding scale

A client diagnosed with angina pectoris complains of chest pain while ambulating in the hallway. Which action should the nurse implement first? A. Support the client to a sitting position. B. Ask the client to walk slowly back to the room. C. Administer a sublingual nitroglycerin tablet. D. Provide oxygen via nasal cannula.

A. Support the client to a sitting position. Rationale: The nurse should safely assist the client to a resting position (A) and then perform (C and D). The client must cease all activity immediately, which will decrease the oxygen requirement of the myocardial muscle. After these interventions are implemented, the client can be escorted back to the room via wheelchair or stretcher (B).

After attending a class on reducing cancer risk factors, a client selects bran flakes with 2% milk and orange slices from a breakfast menu. In evaluating the client's learning, the nurse affirms that the client has made good choices and makes what additional recommendation? A. Switch to skim milk. B. Switch to orange juice. C. Add a source of protein. D. Add herbal tea.

A. Switch to skim milk. Rationale: Dietary recommendations to reduce cancer risk include reduced consumption of fats, with increased consumption of fruits, vegetables, and fiber. (A) promotes reduced fat consumption. Orange slices provide more fiber than orange juice (B, C, and D) are not standard recommendations for reducing cancer risk.

A client with hypertension has been receiving ramipril (Altace) 5 mg PO daily for 2 weeks and is scheduled to receive a dose at 0900. At 0830 the client's blood pressure is 120/70. Which action should the nurse take? A. Administer the dose as prescribed. B. Hold the dose and contact the healthcare provider. C. Hold the dose and recheck the blood pressure in 1 hour. D. Check the healthcare provider's prescription to clarify the dose.

A. The BP is WNL and indicates that the medication is working. (B & C) would be indicated if the BP was low (systole below 100). (D) is not required because the dose is within manufacture's recommendations.

In caring for a client with acute diverticulitis, which assessment data warrants immediate nursing intervention? A. The client has a rigid hard abdomen and elevated WBC. B. The client has left lower quadrant pain and an elevated temperature. C. The client is refusing to eat any of the meal and is complaining of nausea. D. The client has not had a bowel movement in 2 days and has a soft abdomen.

A. The client has a rigid hard abdomen and elevated WBC. Rationale: A hard rigid abdomen and elevated WBC is indicative of peritonitis (A), which is a medical emergency and should be reported to the health care provider immediately. (B and C) are expected clinical manifestations of diverticulitis. (D) does not warrant immediate intervention.

A client with cirrhosis states that his disease was cause by a blood transfusion. What information should the nurse obtain first to provide effective client teaching? A. The year the blood transfusion was received B. The amount of alcohol the client drinks C. How long the client has had cirrhosis D. The client's normal coping mechanisms

A. The nurse should first verify the clients explanation (A) since it may be accurate due to prior to 1990 blood was not screened for Hep C and hep C can cause cirrhosis. Not all cirrhosis is caused is caused by alcoholism (B) (C & D) provide useful but less relevant information.

The nurse is investigating a client injury that occurred when a mechanical lift malfunctioned while moving a client from the bed to a orthopedic-chair. Which question should the nurse ask first? A. Was the equipment used according to policy? B. Which day of the week did the injury occur? C. When was the lift last checked by maintenance? D. What mitigating circumstances led to the injury?

A. Was the equipment used according to policy?

9. Before beginning a transfusion of RBCs, which of the following actions by the nurse would be of highest priority to avoid an error during this procedure? A. Check the identifying information on the unit of blood against the patient's ID bracelet. B. Select new primary IV tubing primed with lactated Ringer's solution to use for the transfusion. C. Add the blood transfusion as a secondary line to the existing IV and used the IV controller to maintain correct flow. D. Remain with the patient for 60 minutes after beginning the transfusion to watch for signs of a transfusion reaction. The patient's identifying information (name, date of birth, medical record number) on the identification bracelet should exactly match the information on the blood bank tag that has been placed on the unit of blood. If any information does not match, the transfusions should not be hung because of possible error and risk to the patient.

A. The patient's identifying information (name, date of birth, medical record number) on the identification bracelet should exactly match the information on the blood bank tag that has been placed on the unit of blood. If any information does not match, the transfusions should not be hung because of possible error and risk to the patient.

Before beginning a transfusion of RBCs, which of the following actions by the nurse would be of highest priority to avoid an error during this procedure? A. Check the identifying information on the unit of blood against the patient's ID bracelet. B. Select new primary IV tubing primed with lactated Ringer's solution to use for the transfusion. C. Add the blood transfusion as a secondary line to the existing IV and used the IV controller to maintain correct flow. D. Remain with the patient for 60 minutes after beginning the transfusion to watch for signs of a transfusion reaction. The patient's identifying information (name, date of birth, medical record number) on the identification bracelet should exactly match the information on the blood bank tag that has been placed on the unit of blood. If any information does not match, the transfusions should not be hung because of possible error and risk to the patient.

A. The patient's identifying information (name, date of birth, medical record number) on the identification bracelet should exactly match the information on the blood bank tag that has been placed on the unit of blood. If any information does not match, the transfusions should not be hung because of possible error and risk to the patient.

The nurse is caring for a postoperative patient with sudden onset of respiratory distress. The physician orders a STAT ventilation-perfusion scan. Which of the following explanations should the nurse provide to the patient about the procedure? A. This test involves injection of a radioisotope to outline the blood vessels in the lungs, followed by inhalation of a radioisotope gas. B. This test will use special technology to examine cross sections of the chest with use of a contrast dye. C. This test will use magnetic fields to produce images of the lungs and chest. D. This test involves injecting contrast dye into a blood vessel to outline the blood vessels of the lungs.

A. This test involves injection of a radioisotope to outline the blood vessels in the lungs, followed by inhalation of a radioisotope gas.A ventilation-perfusion scan has two parts. In the perfusion portion, a radioisotope is injected into the blood and the pulmonary vasculature is outlined. In the ventilation part, the patient inhales a radioactive gas that outlines the alveoli.

The nurse is caring for a client with a chest tube to water seal drainage that was inserted 10 days ago because of a ruptured bullae and pneumothorax. Which finding should the nurse report to the health care provider before the chest tube is removed? A. Tidaling of water in water seal chamber B. Bilateral muffled breath sounds at bases C. Temperature of 101° F D.Absence of chest tube drainage for 2 days

A. Tidaling of water in water seal chamber Rationale: Tidaling (rising and falling of water with respirations) in the water seal chamber should be reported to the health care provider before the chest tube is removed to rule out an unresolved pneumothorax or persistent air leak, which is characteristic of a ruptured bullae caused by abnormally wide changes in negative intrathoracic pressure. Option B may indicate hypoventilation from chest tube discomfort and usually improves when the chest tube is removed. Option C usually indicates an infection, which may not be related to the chest tube. Option D is an expected finding.

A client is placed on a mechanical ventilator following a cerebral hemorrhage, and vecuronium bromide (Norcuron) 0.04 mg/kg every 12 hours IV is prescribed. What is the priority nursing diagnosis for this client? A. Impaired communication related to paralysis of skeletal muscles. B. Hight risk or infection related to increased ICP. C. Potential for injury related to impaired lung expansion. D. Social isolation related to inability to communicate.

A. To increase the client's tolerance of the endotracheal intubation and/or mechanical ventilation, a skeletal-muscle relaxant such as vecuronium is usually prescribed. (A) is a serious outcome because the client cannot communicate his/her needs. (D) is not as much of a priority. (B) infection is not related to ICP. (C) is incorrect because the ventilator will ensure that the lungs are expanded.

A client in septic shock jas a double lumen central venous catheter with one liter of 0.9% Normal Saline solution infusing at 1 ml/hour through one lumen and Total Parenteral Nutrition (TPN) Infusing at 50 ml/hour through one port. The nurse prepares newly prescribed IV antibiotics that should take 45 minutes to infuse. What intervention should the nurse implement? A. Use a secondary port of the Normal Saline solution to administer the antibiotic B. Add the antibiotic to the TPN solution, and continue the normal saline solution C. Stop the TPN infusion for the time needed to administer the prescribed antibiotic D. Add the antibiotic to the Normal Saline solution and continue both infusions

A. Use a secondary port of the Normal Saline solution to administer the antibiotic

6. A nurse is working on a respiratory care unit where many of the patients are affected by asthma. Which of the following actions by the nurse would most likely increase respiratory difficulty for the patients? A. Wearing perfume to work B. Encouraging patients to ambulate daily C. Allowing the patients to eat green leafy vegetables D. Withholding antibiotic therapy until cultures are obtained

A. Wearing perfume to work People with asthma should avoid extrinsic allergens and irritants (e.g., dust, pollen, smoke, certain foods, colognes and perfumes, certain types of medications) because their airways become inflamed, producing shortness of breath, chest tightness, and wheezing. Many green leafy vegetables are rich in vitamins, minerals, and proteins, which incorporate healthy lifestyle patterns into the patients' daily living routines. Routine exercise is a part of a prudent lifestyle, and for patients with asthma the physical and psychosocial effects of ambulation can incorporate feelings of well-being, strength, and enhancement of physical endurance. Antibiotic therapy is always initiated after cultures are obtained so that the sensitivity to the organism can be readily identified.

A nurse is working on a respiratory care unit where many of the patients are affected by asthma. Which of the following actions by the nurse would most likely increase respiratory difficulty for the patients? A. Wearing perfume to work B. Encouraging patients to ambulate daily C. Allowing the patients to eat green leafy vegetables D. Withholding antibiotic therapy until cultures are obtained

A. Wearing perfume to work People with asthma should avoid extrinsic allergens and irritants (e.g., dust, pollen, smoke, certain foods, colognes and perfumes, certain types of medications) because their airways become inflamed, producing shortness of breath, chest tightness, and wheezing. Many green leafy vegetables are rich in vitamins, minerals, and proteins, which incorporate healthy lifestyle patterns into the patients' daily living routines. Routine exercise is a part of a prudent lifestyle, and for patients with asthma the physical and psychosocial effects of ambulation can incorporate feelings of well-being, strength, and enhancement of physical endurance. Antibiotic therapy is always initiated after cultures are obtained so that the sensitivity to the organism can be readily identified.

The nurse is caring for a patient with an acute exacerbation of asthma. Following initial treatment, which of the following findings indicates to the nurse that the patient's respiratory status is improving? A. Wheezing becomes louder B. Vesicular breath sounds decrease C. Aerosol bronchodilators stimulate coughing D. The cough remains nonproductive

A. Wheezing becomes louder The primary problem during an exacerbation of asthma is narrowing of the airway and subsequent diminished air exchange. As the airways begin to dilate, wheezing gets louder because of better air exchange.

When planning care for a client with acute pancreatitis, which nursing intervention has the highest priority? A. Withhold food and fluid intake B. Initiate IV fluid replacement C. Administer antiemetics as needed D. Evaluate intake and output ratio

A. Withhold food and fluid intake

In making client care assignments, which client is best to assign to the practical nurse (PN) working on the unit with a nurse? A. an immobile client receiving low molecular weight heparin Q12H B. a client who is receiving a continuous infusion of heparin and gets out of bed BID C. A client how is being titrated off a heparin infusion and started on PO warfarin D. An ambulatory client receiving warfarin with an INR of 5 seconds.

A. an immobile client receiving low molecular weight heparin Q12H

28. To find the infection site associated with acute lymphangitis, the nurse should look _____ to the inflammation. A. distal B. anterior C. proximal D. contralateral

A. distal The nurse should assess distal to swelling to locate the initial site of infection. Examining proximal, contralateral, or anterior to the inflammation does not describe swelling associated with infection.

35. A patient is admitted to the hospital with fever, chills, a productive cough with rusty sputum, and pleuritic chest pain. Pneumococcal pneumonia is suspected. An appropriate nursing diagnosis for the patient based on the patient's manifestations is A. hyperthermia related to acute infectious process. B. chronic pain related to ineffective pain management. C. risk for injury related to disorientation and confusion. D. ineffective airway clearance related to retained secretions.

A. hyperthermia related to acute infectious process. The patient with pneumococcal pneumonia is acutely ill with fever and the systemic manifestations of fever, such as chills, thirst, headache, and malaise. Interventions that monitor temperature and aid in lowering body temperature are appropriate. Ineffective airway clearance would be manifested by adventitious breath sounds and difficulty producing secretions. Disorientation and confusion are not noted in this patient and are not typical unless the patient is very hypoxemic. Pleuritic pain is an acute pain that is due to inflammation of the pleura.

A patient is admitted to the hospital with fever, chills, a productive cough with rusty sputum, and pleuritic chest pain. Pneumococcal pneumonia is suspected. An appropriate nursing diagnosis for the patient based on the patient's manifestations is A. hyperthermia related to acute infectious process. B. chronic pain related to ineffective pain management. C. risk for injury related to disorientation and confusion. D. ineffective airway clearance related to retained secretions.

A. hyperthermia related to acute infectious process. The patient with pneumococcal pneumonia is acutely ill with fever and the systemic manifestations of fever, such as chills, thirst, headache, and malaise. Interventions that monitor temperature and aid in lowering body temperature are appropriate. Ineffective airway clearance would be manifested by adventitious breath sounds and difficulty producing secretions. Disorientation and confusion are not noted in this patient and are not typical unless the patient is very hypoxemic. Pleuritic pain is an acute pain that is due to inflammation of the pleura.

A male client who has never smoked but has had COPD for the past 5 years is now being assessed for cancer of the lung. The nurse knows that he is most likely to develop which type of lung cancer? A. Adenocarcinoma B. Oat-cell carcinoma C. Malignant melanoma D. Squamous-cell carcinoma

A. is the only lung cancer not related to cigarette smoking related to lung scarring and fibrosis from preexisting pulmonary diseases such as TB and COPD. (B& D) are related to smoking. (C) is a skin cancer

16. To ensure the correct amount of oxygen delivery for a patient receiving 35% oxygen via a Venturi mask, it is most important that the nurse A. keep the air-entrainment ports clean and unobstructed. B. apply an adaptor to increase humidification of the oxygen. C. drain moisture condensation from the oxygen tubing every hour. D. keep the flow rate high enough to keep the bag from collapsing during inspiration.

A. keep the air-entrainment ports clean and unobstructed. Oxygen is delivered to a small jet in the center of a wide-based cone. Air is entrained (pulled through) openings in the cone as oxygen flows through the small jet. The degree of restriction or narrowness of the jet determines the amount of entrainment and the dilution of pure oxygen with room air and thus the concentration of oxygen. Although applying an adaptor can increase the humidification with the Venturi mask, it is not the best answer, because an open port is essential to proper functioning. Draining moisture condensation from the oxygen tubing is performed as often as needed, not on an hourly schedule. A plastic face mask with a reservoir bag needs to have sufficient flow rate to keep the bag inflated.

To ensure the correct amount of oxygen delivery for a patient receiving 35% oxygen via a Venturi mask, it is most important that the nurse A. keep the air-entrainment ports clean and unobstructed. B. apply an adaptor to increase humidification of the oxygen. C. drain moisture condensation from the oxygen tubing every hour. D. keep the flow rate high enough to keep the bag from collapsing during inspiration.

A. keep the air-entrainment ports clean and unobstructed. Oxygen is delivered to a small jet in the center of a wide-based cone. Air is entrained (pulled through) openings in the cone as oxygen flows through the small jet. The degree of restriction or narrowness of the jet determines the amount of entrainment and the dilution of pure oxygen with room air and thus the concentration of oxygen. Although applying an adaptor can increase the humidification with the Venturi mask, it is not the best answer, because an open port is essential to proper functioning. Draining moisture condensation from the oxygen tubing is performed as often as needed, not on an hourly schedule. A plastic face mask with a reservoir bag needs to have sufficient flow rate to keep the bag inflated.

When caring for a 2-day-old infant, the nurse observes that the babies legs are flexed with limited abduction. Based on the finding, what action should the nurse take next? A. notify the healthcare provider B. continue care since this is a normal finding C. document the finding in the record D. perform range of motion to the joint

A. notify the healthcare provider limited abduction could indicate developmental hip dysplasia and the HCP should be notified

In reviewing the preoperative laboratory findings of an adult male client who is scheduled for colon resection in the morning, the nurse notes that the client has a hemoglobin of 9 grams/dl. After verifying the accuracy of this finding with the laboratory, what action should the nurse take? A. notify the surgeon of the laboratory findings B. collect another specimen for analysis C. confirm the availability of compatible units of blood D. administer a bolus of normal saline preoperatively

A. notify the surgeon of the laboratory findings

While caring for a patient with respiratory disease, a nurse observes that the oxygen saturation drops from 94% to 85% when the patient ambulates. The nurse should determine that A. supplemental oxygen should be used when the patient exercises. B. ABG determinations should be done to verify the oxygen saturation reading. C. this finding is a normal response to activity and that the patient should continue to be monitored. D. the oximetry probe should be moved from the finger to the earlobe for an accurate oxygen saturation measurement during activity.

A. supplemental oxygen should be used when the patient exercises.An oxygen saturation lower than 90% indicates inadequate oxygenation. If the drop is related to activity of some type, supplemental oxygen is indicated.

3. The nurse notices clear nasal drainage in a patient newly admitted with facial trauma, including a nasal fracture. The nurse should: A. test the drainage for the presence of glucose. B. suction the nose to maintain airway clearance. C. document the findings and continue monitoring. D. apply a drip pad and reassure the patient this is normal.

A. test the drainage for the presence of glucose. Clear nasal drainage suggests leakage of cerebrospinal fluid (CSF). The drainage should be tested for the presence of glucose, which would indicate the presence of CSF.

Zolpidem tartrate, 1.75 mg PRN at bedtime, is prescribed for rest. The scored tablets are labeled 3.5 mg per tablet. How many tablets should the nurse plan to administer? A.½ tablet B.1 tablet C.1½ tablets D.2 tablets

A. ½ tablet Rationale: 1.75 is ordered. 3.5 is available. 1.75/3.5 time one tab equlas 05. Or one half tablet. Options B, C, and D are incorrect.

The nurse is administering a nystatin suspension for stomatitis. Which instruction will the nurse provide to the client when administering this medication? A."Hold the medication in your mouth for a few minutes before swallowing it." B. "Do not drink or eat milk products for 1 hour prior to taking this medication." C. "Dilute the medication with juice to reduce the unpleasant taste and odor." D. "Take the medication before meals to promote increased absorption."

A."Hold the medication in your mouth for a few minutes before swallowing it." Rationale: Nystatin suspension is prescribed for fungal infections of the mouth. The client should swish the medication in the mouth for 2 minutes and then swallow. Option B does not affect administration of this medication. The medication should not be diluted because this will reduce its effectiveness. Option D is not necessary.

During a health fair, a male client with emphysema tells the nurse that he fatigues easily. Assessment reveals marked clubbing of the fingernails and an increased anteroposterior chest diameter. Which instruction is best to provide the client? A."Pace your activities and schedule rest periods." B."Increase the amount of oxygen you use at night." C."Obtain medical evaluation for antibiotic therapy." D."Reduce your intake of fluids containing caffeine."

A."Pace your activities and schedule rest periods." Rationale: Manifestations of emphysema include an increase in AP diameter (referred to as a barrel chest), nail bed clubbing, and fatigue. The nurse can provide instructions to promote energy management, such as pacing activities and scheduling rest periods. Option B may result in a decreased drive to breathe. The client is not exhibiting any symptoms of infection, so option C is not necessary. Option D is less beneficial than option A.

The nurse is conducting an osteoporosis screening clinic at a health fair. What information should the nurse provide to individuals who are at risk for osteoporosis? (Select all that apply.) A.Encourage alcohol and smoking cessation. B.Suggest supplementing diet with vitamin E. C. Promote regular weight-bearing exercises. D.Implement a home safety plan to prevent falls. E.Propose a regular sleep pattern of 8 hours nightly.

A.Encourage alcohol and smoking cessation. C.Promote regular weight-bearing exercises. D.Implement a home safety plan to prevent falls. Rationale: Options A, C, and D are factors that decrease the risk for developing osteoporosis. Vitamin D and calcium are important supplements to aid in the decrease of bone loss. Regular sleep patterns are important to overall health but are not identified with a decreasing risk for osteoporosis.

Which content about self-care should the nurse include in the teaching plan of a female client who has genital herpes? (Select all that apply.) A. Encourage annual physical and Pap smear. B. Take antiviral medication as prescribed. C. Use condoms to avoid transmission to others. D. Warm sitz baths may relieve itching. E. Use Nystatin suppositories to control itching. F. Use a douche with weak vinegar solution to decrease itching.

A.Encourage annual physical and Pap smear. B.Take antiviral medication as prescribed. C.Use condoms to avoid transmission to others. D.Warm sitz baths may relieve itching. Rationale: The nurse should include (A, B, C, and D) in the teaching plan of a female client with genital herpes. (E) is specific for Candida infections, and option (F) is used to treat Trichomonas.

A client is diagnosed with an acute small bowel obstruction. Which assessment finding requires the most immediate intervention by the nurse? A.Fever of 102° F B.Blood pressure of 150/90 mm Hg C.abdominal cramping D. Dry mucous membranes

A.Fever of 102° F Rationale: A sudden increase in temperature is an indicator of peritonitis. The nurse should notify the health care provider immediately. Options B, C, and D are also findings that require intervention by the nurse but are of less priority than option A. Option B may indicate a hypertensive condition but is not as acute a condition as peritonitis. Option C is an expected finding in clients with small bowel obstruction and may require medication. Option D indicates probable fluid volume deficit, which requires fluid volume replacement.

The nurse is performing an assessment with a client using the Glasgow Coma Scale. Which result would indicate the best neurological status for the client? A. 2 B. 5 C. 13 D. 20

Answer: C The higher the Glasgow Coma Scale number, the better neurological status is present. The scale sum ups the score in three areas; Eyes Open; Best verbal response and Best motor response. The overall total scores range from 3-15.

A client is being discharged following radioactive seed implantation for prostate cancer. What is the most important information that the nurse should provide to this client's family? A. Follow exposure precautions. B. Encourage regular meals. C. Collect all urine. D. Avoid touching the client

A.Follow exposure precautions. Rationale: Clients being treated for prostate cancer with radioactive seed implants should be instructed regarding the amount of time and distance needed to prevent excessive exposure that would pose a hazard to others. Option B is a good suggestion to promote adequate nutrition but is not as important as option A. Option C is unnecessary. Contact with the client is permitted but should be brief to limit radiation exposure

An 81-year-old male client has emphysema. He lives at home with his cat and manages self-care with no difficulty. When making a home visit, the nurse notices that this client's tongue is somewhat cracked and his eyeballs appear sunken into his head. Which nursing intervention is indicated? A.Help the client determine ways to increase his fluid intake. B.Obtain an appointment for the client to have an eye examination. C.Instruct the client to use oxygen at night and increase the humidification. D.Schedule the client for tests to determine his sensitivity to cat hair.

A.Help the client determine ways to increase his fluid intake. Rationale: Clients with COPD should ingest 3 L of fluids daily but may experience a fluid deficit because of shortness of breath. The nurse should suggest creative methods to increase the intake of fluids, such as having fruit juices in disposable containers readily available. Option B is not indicated. Humidified oxygen will not effectively treat the client's fluid deficit, and there is no indication that the client needs supplemental oxygen at night. These symptoms are not indicative of option D and may unnecessarily upset the client, who depends on his pet for socialization.

An emaciated homeless client presents to the emergency department complaining of a productive cough, with blood-tinged sputum and night sweats. Which action is most important for the emergency department triage nurse to implement for this client? A. Initiate airborne infection precautions. B. Place a surgical mask on the client. C. Don an isolation gown and latex gloves. D. Start protective (reverse) isolation precautions.

A.Initiate airborne infection precautions. Rationale: This client is exhibiting classic symptoms of tuberculosis (TB), and the client is from a high-risk population for TB. Therefore, airborne infection precautions, which are indicated for TB, should be used with this client. Option B is used with droplet precautions. There is no evidence that option C or D would be warranted at this time.

A client is admitted to the hospital with a diagnosis of severe acute diverticulitis. Which nursing intervention has the highest priority? A.Place the client on NPO status. B. Assess the client's temperature. C. Obtain a stool specimen. D.Administer IV fluids.

A.Place the client on NPO status. Rationale: A client with acute severe diverticulitis is at risk for peritonitis and intestinal obstruction and should be made NPO to reduce risk of intestinal rupture. Options B, C, and D are important but are less of a priority than option A, which is implemented to prevent a severe complication.

During assessment of a client in the intensive care unit, the nurse notes that the client's breath sounds are clear on auscultation, but jugular vein distention and muffled heart sounds are present. Which intervention should the nurse implement? A.Prepare the client for a pericardial tap. B.Administer intravenous furosemide (Lasix). C.Assist the client to cough and breathe deeply. D.Instruct the client to restrict oral fluid intake.

A.Prepare the client for a pericardial tap. Rationale: The client is exhibiting symptoms of cardiac tamponade, a collection of fluid in the pericardial sac that results in a reduction in cardiac output, which is a potentially fatal complication of pericarditis. Treatment for tamponade is a pericardial tap. Lasix IV is not indicated for treatment of pericarditis. Because the client's breath sounds are clear, option C is not a priority. Fluids are frequently increased in the initial treatment of tamponade to compensate for the decrease in cardiac output, but this is not the same priority as option A.

A client diagnosed with angina pectoris complains of chest pain while ambulating in the hallway. Which action should the nurse implement first? A. Support the client to a sitting position. B. Ask the client to walk slowly back to the room. C. Administer a sublingual nitroglycerin tablet. D. Provide oxygen via nasal cannula.

A.Support the client to a sitting position. Rationale: The nurse should safely assist the client to a resting position and then perform options C and D. The client must cease all activity immediately, which will decrease the oxygen requirement of the myocardial muscle. After these interventions are implemented, the client can be escorted back to the room via wheelchair or stretcher.

After attending a class on reducing cancer risk factors, a client selects bran flakes with 2% milk and orange slices from a breakfast menu. In evaluating the client's learning, the nurse affirms that the client has made good choices and makes what additional recommendation? A.Switch to skim milk. B. Switch to orange juice. C. Add a source of protein. D. Add herbal tea.

A.Switch to skim milk. Rationale: Dietary recommendations to reduce cancer risk include reduced consumption of fats, with increased consumption of fruits, vegetables, and fiber. Option A promotes reduced fat consumption. Orange slices provide more fiber than orange juice. Options B, C, and D are not standard recommendations for reducing cancer risk.

In caring for a client with acute diverticulitis, which assessment data warrants immediate nursing intervention? A.The client has a rigid hard abdomen and elevated WBC. B.The client has left lower quadrant pain and an elevated temperature. C.The client is refusing to eat any of the meal and is complaining of nausea. D.The client has not had a bowel movement in 2 days and has a soft abdomen.

A.The client has a rigid hard abdomen and elevated WBC. Rationale: A hard rigid abdomen and elevated WBC is indicative of peritonitis, which is a medical emergency and should be reported to the health care provider immediately. Options B and C are expected clinical manifestations of diverticulitis. Option D does not warrant immediate intervention.

43. Select all that apply. Which of the following is included in a comprehensive respiratory assessment? A. Pulse oximetry B. Chest auscultation C. Apical radial pulse D. Nail-bed assessment E. Evaluation of respiratory effort F. Rate and character of respirations

ABDEF The total assessment of the respiratory system includes pulse oximetry; auscultation; skin and nail-bed assessment for the detection of cyanosis; and rate, character, and degree of effort of respirations. The apical radial pulse is a cardiac assessment.

42. Select all that apply. Which of the following nursing actions can help clear tracheobronchial secretions in a patient with cystic fibrosis? A. Postural drainage B. Suppressing the cough C. Ensuring adequate hydration D. Administering mucolytic aerosols E. Encouraging the patient to lie flat F. Administering water-soluble vitamins

ACD Postural drainage, adequate hydration, and administration of mucolytic aerosols all encourage coughing and the clearing of secretions. A patient with cystic fibrosis will be more comfortable sitting upright.

41. Select all that apply. During initial assessment, a nurse should record which of the following manifestations of respiratory distress? A. Tachypnea B. Nasal flaring C. Thready pulse D. Panting or grunting E. Use of intercostal muscles F. An inspiratory-to-expiratory ratio of 1:2

AD Manifestations of respiratory distress include tachypnea, grunting and panting on respiration, central cyanosis, use of accessory muscles, and flaring nares.

The practical nurse is reinforcing osteoporosis prevention education to a group of senior citizens. What life style choices will help decrease the risk of developing osteoporosis? (Select all that apply.)

Alcohol in moderation and smoking cessation. Regular weight-bearing exercises. Consumption of a diet rich in calcium and vitamin D. Alcohol in moderation and smoking cessation, regular weight-bearing exercises at least 30 minutes a day, and consumption of a diet rich in calcium and vitamin D are life style choices that decrease the risk for developing osteoporosis.

Which diet modification is most important for the nurse to include when counseling a client with heart failure? A. Reduce sodium intake. B. Increase protein intake. C. Increase vegetables intake. D. Reduce carbohydrate intake.

Answer: A Chronic diseases such as congestive heart failure require that a client make dietary changes to manage the disease. It is important that a client with congestive heart failure be aware of sources of dietary sodium, as sodium may exacerbate the effects of congestive heart failure.

A client comes to the emergency department with a compliant of vomiting and abdominal pain. The triage nurse notes the client is jaundiced and icterus and has scattered bruises on their arms and vascular lesions "spider angiomas" on their nose and cheeks. Upon further assessment, the client's abdomen appears to be obtunded with dilated abdominal veins and is dull upon percussion. The assessment findings are most likely reflective of which condition? Cirrhosis. Pancreatitis. Gastric ulcer. Diverticulitis.

Answer: A Clients diagnosed with cirrhosis clinical signs and symptoms vary based on the damage to the liver. Clinical signs and symptoms can consist of: jaundice and icterus; dry skin; rashes; petechiae and/or scattered ecchymoses; vascular lesions known as "spider angiomas" located on the nose, cheeks, upper thorax, and shoulders. Ascites if present will appear as an obtunded abdomen with dilated veins which would sound dull upon percussion due the fluid in the peritoneal cavity

The clinic nurse is assessing the blood pressure of a client diagnosed hypertension. How should the nurse assess this client's blood pressure? A. Obtain blood pressure readings from both the client's arms. B. Take a blood pressure with the client sitting down and then standing up. C. Compare the blood pressure readings from the upper extremities and lower extremities. D. A blood pressure should be taken in the same arm at the beginning and at the end of the check-up.

Answer: A Clients diagnosed with hypertension have a high incidence of atherosclerosis. It is best to obtain a blood pressure from both arms of the client and record in the client's electronic medical record documenting the blood pressure reading for each arm.

An emergency department nurse is triaging an unaccompanied, unconscious client. Upon inspection the nurse notices some paradoxical movement of the anterior lower chest area. The client's blood pressure is 88/54mmHg. The heart rate is 112 beats per minute and the client's oxygen saturation via pulse oximetry is 91% on room air. Based on these findings which condition should the nurse suspect? A. Lung tumor. B. Broken ribs. C. Pneumothorax. D. Pulmonary infiltrates.

Answer: A The client's presenting symptom of paradoxical movement of the thorax cavity is indicative of broken ribs or one rib that is fractured in more than one place. This condition is referred to as a "flail chest". The paradoxical chest movement is often accompanied by client complaints of pain, especially when coughing or trying to breath deeply. Other symptoms include dyspnea, cyanosis, tachycardia and hypotension depending upon how severe the injury.

An unconscious client with a mean arterial pressure (MAP) below 60mmHg; temperature of 96° F (35.5° C) has skin which is cool and moist. During the primary assessment process, the MAP dropped another 10mmHg and appears to be not responding to the bolus IV infusion of NS. The nurse should anticipate the healthcare provider to prescribe what due to the client's response to the resuscitation efforts? A. Dobutamine to be initiated. B. Administration of dopamine. C. Infusion of sodium nitroprusside. D. Increased rate of the infusion of IV fluids.

Answer: A The nurse should anticipate the health care provider to prescribe the dopamine first to promote vasoconstriction to improve the mean arterial pressure before the initiation of the other medications. Dopamine is the drug of choice until it is determine whether this client is experiencing shock in relationship to hypovolemic or sepsis which is could be causing the massive vasodilation.

A client presents with persistent cough, fatigue, loss of appetite, and bloody sputum. The nurse should recognize that the client is exhibiting symptoms of which condition? A. Tuberculosis. B. Chronic obstructive pulmonary disease. C. Asthma. D. Cystic fibrosis.

Answer: A Tuberculosis is an infectious disease caused by airborne bacteria. Signs and symptoms of tuberculosis include cough, fatigue, loss of appetite and bloody sputum

A client with a chief complaint of severe, spasmodic pain in their (R) flank area that comes and goes accompanied with nausea and diaphoresis has additional assessment findings of hematuria and cloudiness appearance of their urine and a history of gout. The client denies any fevers or chills and is presently ad febrile.Based on these findings which condition would the nurse suspect? A. Urolithiasis. B. Urothelial cancer. C. Urinary incontinence. D. Urinary tract infection.

Answer: A Urolithiasis (kidney stones) is suspected based on the client's presenting signs and symptoms of severe, spasmodic pain in the (R) flank area that is sporadic, accompanied with hematuria and turbidity and being adfebrile. Clients with history of diabetes or gout (hyperuricemia) have an increased risk for stone formation.

The nurse is assessing the thyroid of a client with suspected hyperthyroidism. Which action should the nurse avoid while performing this assessment? A. Vigorous palpation of the gland. B. Displacement of the trachea. C. Auscultation of the carotid artery. D. Movement of the vocal cords.

Answer: A Vigorous palpation of the thyroid gland should be avoided in clients suspected of hyperthyroidism. Vigorous palpation could stimulate a thyroid storm. In order to be able to assess the thyroid gland, the nurse needs to ask the client to their head to the side to be palpated. The nurse then gently displaces the trachea. If nodules are palpated, the nurse should auscultate over the site. The nurse should ask the client to swallow while palpating, so the gland may be felt as it moves upward.

The nurse is caring for a client who has increased intracranial pressure resulting from a head injury. Which interventions should the nurse not implement or implement with caution? (Select all that apply) A. Tracheal suctioning. B. Enema administration. C. Encouraging deep coughing. D. Medicating with stool softeners. E. Placing client in Fowler's position.

Answer: A B C Administration of an enema could cause a Valsalva reaction which increases intracranial pressure (ICP). Tracheal suction and hyperinflation, along with coughing causes increase in ICP. Stool softeners and Fowler's position are not risks.

The nurse is preparing client teaching for a post-menopausal female who is experiencing stress incontinence. Which instructions should be included to help reduce the occurrence of stress incontinence? A. Review of Kegel exercises. B. If overweight, weight reduction diet. C. Instructing the client to wipe from back to front. D. Avoidance of alcohol and caffeine food products. E. Encouraging the consumption of cranberry juice or pills.

Answer: A B D Stress incontinence is a condition which there is a small amount of urine loss during coughing, sneezing, jogging, or lifting. This is the result of weaken pelvic floor muscles due to aging and/or vaginal delivery(s) of an infant(s). Kegel exercises are exercises to help strengthen those muscles. Alcohol and caffeine should be avoided because it they are bladder irritants. If the client is overweight, they should be counsel to reduce their weight and given a review of weight-loss diet plans.

The client's lower extremity has an ulcer present. Which findings would indicate to the nurse that the ulcer is a venous ulcer? (Select all that apply) A. The ulcer is located in the ankle area. B. The lower extremities pulses are decreased. C. The area with the ulcer is located is edematous. D. The ulcer bed is pink with granulation tissue present. E. Dependent rubor is present in the extremity with the ulcer.

Answer: A C D Venous ulcers are a result of a faulty venous system. They are often chronic non-healing ulcers located in the ankle area with ankle or leg swelling often present. Pulses are present. The ulcer bed is pink with granulation tissue often present. There may be scarring present from previous ulcers and some ankle discoloration may be present.

A nursing is conducting a health history on a slightly obese 58 year old post-menopausal female who came to the clinic with complaint of vague abdominal and gastrointestinal symptoms indicative of ovarian cancer. Which questions should the nurse include in her interview? A. What type of birth control have you used in the past? B. Have you ever been diagnosed with a sexual transmitted disease? C. At what age did you start your period and experience menopause? D. When you had your children, did you bottle-feed or breastfed your children? E. Have you ever been pregnant and what was your age at your first pregnancy?

Answer: A C D E It is important to teach women over 50 years old to think "ovarian" if they have vague abdominal and gastrointestinal symptoms of non-known source. Ovarian cancer causes more deaths from the reproductive system in women over 50 years because it is not detected until its late stages. Risk factors for ovarian cancer include: older than 40 years old; family history of ovarian or breast cancer, colorectal cancer; diabetes mellitus; nulliparity; older than 30 years old at first pregnancy; history of breast and/or colorectal cancer; infertility; BRCA 1 or 2 gene; early menarche or late menopause; endometriosis; obesity and high-fat diet. Women whose choice of birth control was tubal ligation or oral birth control and/or who had breastfed their children are at less of risk of developing ovarian cancer.

What are the best interventions that can be done in the hospital setting to reduce the risk of an immunocompromised client becoming infected and possibly septic? A. When working with non-intact skin use aseptic technique. B. Changing out IV catheters and access lines every 24 hours. C. The removal of indwelling urinary catheters as soon as possible. D. Placing in negative pressure rooms with reverse airflow. E. Clients who are mechanically ventilate, when possible, weaned off the ventilators.

Answer: A C E The practice of aseptic technique when dealing with immunocompromised clients with non-intact skin and/or mucous membranes is crucial for the safety of the client. The removal of indwelling urinary catheters and IV catheters and access lines should be done as soon as possible. When possible, weaned off ventilators as soon as possible. All are interventions that can help minimize the risk of infection and possibly infection. A client should be placed in a positive pressure room if their absolute neutrophil count (ANC) is below 500.

Which nursing actions should be implemented to reduce the risk of the development of acute respiratory distress syndrome (ARDS)? A. Practicing scrupulous infection control guidelines. B. Implementing a restricted intake and documenting all output. C. Placing a cllient with active tuberculosis client in a negative pressure room. D. Adhering to aspiration precautions for clients with impaired swallowing and gag reflex. E. Raising the head of the bed to 30-45° for clients receiving enteral feedings.

Answer: A D E Both aspiration and systemic infections increase a client's chances of developing acute respiratory distress syndrome (ARDS). To reduce the risk of a client aspirating while receiving enteral feedings, the nurse should raise the head of the bed 30 to 45 degrees. If the client has impaired swallow or reduced gag reflex, aspirations precautions should be put in place and followed. To help decrease the risk of a systemic infection, the nurse should adhere to scrupulous infection control guidelines.

The triage nurse is assessing a client with a six week history of a persistent cough producing rusty colored mucus, 10 pounds (4.54kg) weight loss, decreased appetite and night sweats. Based on the client's history which nursing action should the nurse do next? A. Request a prescription for chest x-ray and lab work. B. Place a mask on the client and place the client in a private room. C. Assess the client's vital signs to include a pulse oximetry reading. D. Auscultate the lung fields as the client takes deep and slow breaths.

Answer: B Based on the client's presenting history and complaints, the nurse needs to place a mask on the client and place the client in a private room, until tuberculosis (TB) can be ruled out. Clinical manifestations of TB are a persistent cough, weight loss, anorexia, night sweats, fevers or chills, dyspnea or hemoptysis.

The triage nurse is assessing a client with a six week history of a persistent cough producing rusty colored mucus, 10 pounds (4.54kg) weight loss, decreased appetite and night sweats. Based on the client's history which nursing action should the nurse do next? A. Request a prescription for chest x-ray and lab work. B. Place a mask on the client and place the client in a private room. C. Assess the client's vital signs to include a pulse oximetry reading. D. Auscultate the lung fields as the client takes deep and slow breaths.

Answer: B Based on the client's presenting history and complaints, the nurse needs to place a mask on the client and place the client in a private room, until tuberculosis (TB) can be ruled out. Clinical manifestations of TB are a persistent cough, weight loss, anorexia, night sweats, fevers or chills, dyspnea or hemoptysis.

The nurse is explaining the need to reduce salt intake to a client with primary hypertension. What explanation should the nurse provide? A. High salt can damage the lining of the blood vessels B. Too much salt can cause the kidneys to retain fluid C. Excessive salt can cause blood vessels to contract D. Salt can cause inflammation inside the blood vessels

B. Too much salt can cause the kidneys to retain fluid

The nurse is assigned a client who was admitted for a basilar fracture. Which finding is indicative of a complication that should be reported to the healthcare provider immediately? A. The client's report of some neck stiffness. B. The presence of new onset of post nasal drip. C. Client's request for acetaminophen for a headache. D. Slight increase of temperature to 100.1° F (37.8° C).

Answer: B Basilar fracture is a fracture that occurs at the base of the skull, usually along the paranasal sinus. If the client demonstrates any type of leakage of fluid from the ears or the nose it needs to be further evaluated for cerebral spinal fluid. The presence of post nasal drip needs to be investigated and reported immediately because it could be signs of of cerebral spinal fluid leaking from the nasal cavity and dripping down the back of the client's throat.

A nurse riding public transportation notices a child experiencing an asthma attack. The mother of the child pulls out of her purse a medicated discus dispenser of salmeterol xinafoate, a long-acting beta2 agonist, and immediately administers to the child. What is the anticipated outcome of the administration of this medication to this child? A. Asthmatic symptoms should resolve within 5 to 10 minutes after administration of medication. B. The child's asthma attack will be unresponsive to the administration of the medication. C. The medication administration will cause an increased in the child's heart rate. D. The bronchioles and alveoli in response to the medication will dilate and lung expansion will increase.

Answer: B During an asthma attack long-acting beta2 agonist (LABAs) delivered by inhaler directly to the bronchioles will have little effect. LABAs are used as a preventive medication and need time to build up an effect LABAs should be co-administered with inhaled steroids. Clients need to be instructed not to use LABAs to relieve an asthma attack or if wheezing becomes worse.

In clients with renal failure, which medication should the nurse anticipate the client may need to take with meals? A. Pepto-Bismol. B. Calcium carbonate. C. Lactose supplement. D. Simvastatin.

Answer: B Nutrition planning for clients with renal failure includes a reduction in foods containing potassium, phosphorous, and sodium. Calcium carbonate is often given with meals in clients with renal disease to act as a phosphorous binder.

A nurse is assessing a client who has had a 12 pound (5.45kg) weight gain in the last 72 hours and is exhibiting poor tissue perfusion and pulmonary congestion. Which condition is most consistent with these findings? A. Tetralogy of Fallot. B. Left-sided heart failure. C. Right-sided heart failure. D. Jugular venous distention.

Answer: B The client with rapid weight gain and poor peripheral perfusion is experiencing heart failure. Ineffective contractions of the left ventricular results in decreased cardiac output which results in poor tissue perfusion and increased congestion in the lungs due to the ineffective pumping and emptying of the left ventricle. This process results in pulmonary congestion and fluid retention.

A client with a blunt chest trauma has developed a tension pneumothorax. Which piece of equipment should a nurse anticipate and obtain immediately for the healthcare provider to use to help assist the client with this condition? A. A chest tube. B. A large bore needle. C. An elastic bounding wrap. D. A non-permeable occlusive dressing.

Answer: B The initial management of a tension pneumothorax is the insertion of a large bore needle in the second intercostal space in the midclavicular line of the affected side to relieve some of the trapped air on that side of the pleural cavity causing the lung to collapse. The procedure is followed by the placement of a chest tube in the 4th intercostal space and is attached to a water seal until the lung re-inflates.

The nurse is assigned to provide care to a client with traumatic brain injury following a motor vehicle accident. The treatment has included iv fluid of hypertonic saline, a mannitol IV bolus, and a hypothermia blanket. The client has been placed in a barbiturate coma and has been placed on mechanical ventilator. What is the goal of these medical interventions ? A. Decreased the risk of seizure activity B. Prevent increased intracranial pressure C. Increased the client's Glasgow Coma scale Score D. Promote perfusion and oxygenated to the brain perfusion

Answer: B The medical interventions of administering hypertonic saline and osmotic diuretic mannitol are to pull fluid out of the intracranial space and into the vascular system for excretion. Placing the client into a hypothermic state with a hypothermic blanket is to slow down the brain's metabolism. Placing the client into a barbiturate coma and using mechanical ventilation causes vasoconstriction of the cerebral vessels and decreases the oxygen demands of the brain. All these interventions are implemented to help decrease the risk of increased intracranial pressure.

The nurse is assigned a client who was admitted for a basilar fracture. Which finding is indicative of a complication that should be reported to the healthcare provider immediately? A. The client's report of some neck stiffness. B. The presence of new onset of post nasal drip. C. Client's request for acetaminophen for a headache. D. Slight increase of temperature to 100.1° F (37.8° C).

Answer: B Basilar fracture is a fracture that occurs at the base of the skull, usually along the paranasal sinus. If the client demonstrates any type of leakage of fluid from the ears or the nose it needs to be further evaluated for cerebral spinal fluid. The presence of post nasal drip needs to be investigated and reported immediately because it could be signs of of cerebral spinal fluid leaking from the nasal cavity and dripping down the back of the client's throat.

A client, who had a laparoscopy cholecystectomy two days ago, calls the clinic's triage nurse in the morning, complaining that they have been awake all night feel feeling restless and anxious, like something isn't right with them and are afraid of dying. Which is the most appropriate nurse's response? A. Questioned the client, if they are experiencing epigastric pain when eating. B. Advise the client to return to the clinic now to be seen by their gastric surgeon. C. Explained to the client, it may be discomfort from the gas used during the procedure. D. Instruct the client to monitor their temperature every four hours and report if temp >100.5°F (37.8°C).

Answer: B Clients status post invasive procedures are at risk of hemorrhaging. As a result of the hemorrhaging, the client could go into shock. A feeling of impending doom, accompanied with restlessness and anxiety could be a signs that the client may be hemorrhaging internally. The nurse needs to instruct the client to return to the clinic now to be seen by their gastric surgeon.

The nurse is evaluating lab results in the client's electronic medical record. Which lab value would indicate poor blood glucose control with a state of hyperglycemia during the past few weeks? A. Triglycerides 135mg/dL. B. Hemoglobin A1c 9.2%. C. C-peptide 1.2 nanograms/ml.. D. Total cholesterol 200mg/dl.

Answer: B Hemoglobin A1c measures plasma blood sugar levels over a period of 2-3 months. The normal value of A1c in clients without diabetes is 4-5.9%. For most clients with diabetes an A1c below 7% is the target. A level higher than 9% indicates the client had poor glucose control with hyperglycemia during the past few weeks.

The nurse is conducting a physical assessment with a client who has a history of unexplained weight gain, thinning of hair, thickened skin, low blood pressure and pulse. The client also reports an inability to concentrate, memory loss, cold intolerance and constipation. What is the most likely cause of these physical findings? A. Crohn's disease. B. Hypothyroidism. C. Graves' disease. D. Diabetes mellitus.

Answer: B Hypothyroidism is one of the most common medical disorders in the U.S. It occurs when the thyroid fails to secrete sufficient hormones, slowing down important metabolic processes.

A client complained their eye was feeling itchy after their cataract surgery. The post operative eye appears to be reddened and slightly swollen. Based on these findings, what should the nurse do? A. Contact the ophthalmologist immediately and report the findings. B. Reassure the client and explained to them itchiness is an expectant finding. C. Lowered the client's head of bed and roll client onto their side of the affected eye. D. Instruct the client to keep their eye closed and apply a warm compress onto the affected eye.

Answer: B Mild eye itching and a bloodshot appearance of the eye, accompanied with slight eyelid swelling status post cataract surgery is considered an expectant finding during the healing process. If a finding of significant swelling or bruising is present this is considered abnormal and a cool compress should be applied to the affected eye.

A nurse is assessing a client who is 24 hours post-op hypophysectomy (removal of the pituitary gland). The client complains to the nurse of postnasal drip that is bothering them and causing them excessive swallowing. The client's complaint may be indicative of what? A. An allergic reaction. B. Possible CSF leakage. C. Sign of nerve damage. D. An expected post-op finding.

Answer: B S/P hypophysectomy, any sign of postnasal drip and increase in swallowing may be a sign of CSF leakage. Any drainage from the post-surgical nasal packaging that has an appearance of a light yellow color surrounding the clear drainage, also may be indicative of a CSF leak. This sign is commonly referred to as a "halo sign".

The nurse is reviewing client education with a parent and their teenager. Which interventions should help minimize the occurrence of sickle cell crisis? (Select all that apply) A. Receive a yearly pneumonia vaccination. B. Always wear socks and gloves when outside on cold days. C. Engage in vigorous aerobic exercise at least 3-4 times a week. D. You should drink at least 3-4 liters of non-caffeinated liquid a day.

Answer: B D E Avoid cigarettes and/or tobacco products and second hand smoke. Rationale In order to help minimize the occurrence of a sickle cell crisis, the client should be instructed to: drink 3-4 liters of non-caffeinated liquid/day; avoid alcoholic beverages; avoid cigarettes and/or tobacco to include second-hand smoke; receive an annual flu shot; a pneumonia shot every 5 years or according or the CDC guidelines; avoid being too hot or cold; wear socks and gloves outside on cold days; avoid high altitudes; avoid strenuous exercise; and engage in mild, low imp

A client is admitted with a diagnosis of right-sided heart failure. Which conditions are known to contribute to the development of right-sided heart failure? A. Atrial septal defect. B. Left ventricular failure. C. Coronary artery disease D. Pulmonary hypertension. E. Right ventricular myocardial infarction.

Answer: B D E Heart failure of the right side is usually caused by poorly managed left-sided heart failure due to the left ventricular failure, a myocardial infarction involving the right ventricular or pulmonary hypertension. Each of these conditions puts added workload on the right ventricle resulting in heart failure.

A client status-post 48 hours external fixation placement of their (L) femur suddenly becomes anxious and restless. Assessment findings include increased RR to 28 breathes/minute and HR to 110 beats/minute; (L) foot and toes are pink and warm to touch; capillary refill is +2, with slight swelling present and a slight macular rash present on the client's upper arms, chest and neck. Based on their presenting clinical signs and symptoms which condition is the client most likely experiencing? A. Acute osteomyelitis. B. Post-traumatic stress. C. Fat embolism syndrome. D. Compartment syndrome.

Answer: C Due to the fact the client is status-post surgical repair to the (L) femur they are at an increased risk for a release of fat globules from the yellow marrow. These fat globules once released into the blood stream form into fat emboli which commonly clog the small blood vessels, the lungs are most often affected. This causes fat embolism syndrome. If one of these fat globules becomes lodged in the lungs; classical clinical signs and symptoms are often decreasing level of consciousness, anxiety, respiratory distress, increase in the respiratory rate and heart rate, accompanied with slight fever, hemoptysis and petechiae which is generally seen on the client's neck, upper arms , chest and abdomen.

The nurse is caring for a client who takes carbidopa/levodopa for treatment of Parkinson's symptoms. What side effect of this medication should the nurse be aware of when helping the client ambulate? A. Shortness of breath. B. Incontinence. C. Syncope. D. Uncontrolled bleeding

Answer: C Carbidopa/levodopa (Sinemet) is a baseline medication used to treat clients with Parkinson's disease. Clients who take Sinemet may experience syncope, so the nurse should be aware of an increased risk of falling.

A female client has had a scleral buckling with the use of silicone oil for repair of partial retinal detachment of her right eye. The client is transferred to the surgical floor. The client is lying on her left side with her head in the midline position during the nurse's assessment. A protective shield is covering the client's right eye. Which is the most important nursing intervention the nurse should implement post-operatively? A. Offer pain medication continuously around the clock. B. Keep both of the eyes covered with protective eye shields. C. Ensure the client remains on her left side with her head midline. D. Administer topical corticosteroids and immunosuppressants.

Answer: C The most important nursing intervention for a client following retinal detachment repair in which gas or silicone oil placed in the eye to promote retinal attachment is to maintain the client's head in the same position the surgeon has placed it. The gas or oil bubble is used to float up against the retina to hold the retina in place until it heals in place.

A client status-post 48 hours external fixation placement of their (L) femur suddenly becomes anxious and restless. Assessment findings include increased RR to 28 breathes/minute and HR to 110 beats/minute; (L) foot and toes are pink and warm to touch; capillary refill is +2, with slight swelling present and a slight macular rash present on the client's upper arms, chest and neck. Based on their presenting clinical signs and symptoms which condition is the client most likely experiencing? A. Acute osteomyelitis. B. Post-traumatic stress. C. Fat embolism syndrome. D. Compartment syndrome.

Answer: C Due to the fact the client is status-post surgical repair to the (L) femur they are at an increased risk for a release of fat globules from the yellow marrow. These fat globules once released into the blood stream form into fat emboli which commonly clog the small blood vessels, the lungs are most often affected. This causes fat embolism syndrome. If one of these fat globules becomes lodged in the lungs; classical clinical signs and symptoms are often decreasing level of consciousness, anxiety, respiratory distress, increase in the respiratory rate and heart rate, accompanied with slight fever, hemoptysis and petechiae which is generally seen on the client's neck, upper arms , chest and abdomen.

A nurse is assigned a client who was being treated for early sepsis with IV antibiotics and IV fluids. During the client's assessment the nurse notes that the client's blood pressure which was 86/52mmHg is now 118/78mmHg; their skin which was cool is now appears pink and warm to touch and their heart rate has increased from 75-89 beats per minute. How should the nurse interpret these assessment findings? Select the most appropriate statement. A. The client appears to be responding well to the antibiotic therapy. B. The status of the client is improving as evident by the skin changes. C. The condition of the client may be getting worse rather than improving. D. The circulation is improving as evidenced of the increased blood pressure and skin.

Answer: C In early sepsis the clients will exhibit mild hypotension; slight decrease in urine output and increase respirations which result in decrease cardiac output. Their skin on their extremities may appear slightly pale and cool to touch. If the sepsis does not respond to interventions, the client will shift into severe sepsis and the body will attempt to compensate by increasing the heart rate which will increase the stroke volume in turn increased the cardiac output, return the blood pressure and central venous pressure back to normal. The increased cardiac output and vasodilation will make the appearance of the skin to become pink and warm to touch. Unfortunately, this compensation is actually a sign of the client's condition worsening and is only temporary, in which the client's cardiac output will drastically reduce and the client then progresses into septic shock.

The nurse is working with a client who was born with an inherited disorder in which the nephrons of their kidneys are affected and usually do not become symptomatic until they reach their 30's when their nephron and kidney function become less effective. These clients usually experience hypertension as a result of this condition. This client is most likely diagnosed with which condition? A. Cystitis. B. Urolithiasis. C. Polycystic disease. D. Urothelial cancer.

Answer: C Polycystic kidney disease is an inherited disorder in which the nephrons of the kidneys are affected with fluid-filled cysts. At birth, there are only a few cysts located in the nephrons. Once a person reaches their 30's the small cysts usually become larger as they fill up with fluid and then create a compromise on the nephron and kidney function.

To reduce the risk of postoperative infection, the nurse should teach the client how to use which medical device? A. Pulse oximeter. B. CPAP machine. C. Incentive spirometer. D. Apnea monitor.

Answer: C Post-operative clients have limited mobility related to pain and activity restrictions. Incentive spirometry therapy promotes full inflation of the lungs and decreases the risk of pulmonary infections

Which abnormal finding should the nurse anticipate when performing heart auscultation on an older adult client with congestive heart failure? A. Pericardial rub. B. Split S2 heart sound. C. S3 heart sound. D. Systolic murmur.

Answer: C S3 is a rare extra heart sound heard during diastole over the aortic valve area, the second intercostal space. A third heart sound is benign in younger clients, but it may be one of the first indications of congestive heart in older adult clients.

What is the most reliable method a nurse should use to document the status of fluid retention in a client diagnosed with heart failure? A. Assess their skin turgor. B. Auscultate their lung sounds. C. Weigh them daily in the morning. D. Evaluate them for the presence of edema.

Answer: C The best way to determine the fluid status of a client diagnosed with heart failure is measuring the client's weight early in the morning every day. Weight gain or loss is the most reliable way to assess the fluid status of these clients.

A nurse is assigned the care of a client who is presently experiencing hypovolemic shock. The client's MAP has decreased by 20 mmHg from its baseline, tissue ischemia and anoxia of non-vital organs is occurring, pulses are weak, urine output is absent and the client's skin is cool and moist. The client appears to be confused and extremely anxious. Which stage of hypovolemic shock do these clinical signs and symptoms indicate? A. Initial stage. B. Refractory Stage. C. Progressive stage. D. Non-progressive stage.

Answer: C The client is exhibiting the clinical signs and symptoms of the progressive stage of hypovolemic shock which if allowed to progress will lead into the refractory stage and eventually death. The clinical signs and symptoms of hypovolemic progresses in four stages, if the condition causing the shock and/or interventions is not done and/or successful. If the interventions and the cause of the shock are corrected within 1-2 hours after onset, then the effects are temporary and irreversible.

The community health nurse is assessing an older client and notices that the client walks with short, hesitant steps. The client walks with a slow, shuffling motion and with very little arm movement. At rest, the client has tremors. The nurse also notes that the client speaks in a very soft, low-pitched voice and has difficulty finding the right words. Which condition most likely explains the client's behaviors? A. Fibromyalgia. B. Polyneuropathy. C. Parkinson's disease. D. Wernicke-Korsakoff Syndrome.

Answer: C The client is presenting clinical signs of Parkinson's disease. Parkinson's disease is a debilitating neurologic disorder involving the basal ganglia and substantia nigra. Clients with this condition experience depletion of dopamine which causes difficulty with initiation and coordination of voluntary movement. The disease is characterized by muscle rigidity, akinesia, postural instability, tremors, dysarthria, and a mask like facial appearance.

An unlicensed assistant personnel (UAP) is providing a bed bath to a client who is 48 hours post radical neck dissection due to oral cancer. Upon turning the client to the side, the UAP notices the client's neck dressing start to ooze bright red blood. The UAP immediately applies pressure to the neck dressing and calls the nurse. What should the nurse do first? A. Raise the head of the client's bed to 45° and flex the client's knees. B. Switch out their gloved hand for the UAP's non -gloved hand. Correct C. Instruct the UAP to gently take their hand off the client's dressing. D. Maintain direct pressure on dressing and transport client to the operating room.

Answer: C The oozing blood indicates a leak of the carotid artery rather than a rupture. A client who is status post radical neck dissection should be monitor for the development of carotid artery leakage or rupture. A ruptured artery will appear as large amounts of bright red blood spurting quickly. A carotid artery with a leak will appear as oozing of bright red blood. If a carotid leakage is suspected, do not touch the area because additional pressure could cause immediate rupture; instead call the Rapid Response Team. The UAP should stop applying pressure. Direct pressure is only applied to a ruptured carotid artery.

A client receives a bone marrow transplant (engraftment) from a matched donor from the bone marrow donor registry. What data determines that the engraftment was successful? A. The client's bone marrow starts to produce their own red blood cells. B. The client's antibodies take over and macrophage the donor's donated cells. C. The analysis of the client's bone marrow reveals only the donor's cells are present. D. The presence of the client's white blood cells, red blood cells and platelet counts beginning to rise.

Answer: C When the donor's stem cells are the only ones present in the client's bone marrow, this indicates a successful engraftment. It usually takes a bone marrow transplant an average 21 days for engraftment.

The nurse is assisting in the evaluation of a client for a possible diagnosis of heart failure. Which assessment tool would not be to diagnose the presence or extent of heart failure? A. Chest x-ray. B. Echocardiogram. C. Electrocardiogram. D. Radionuclide studies.

Answer: C Chest x-rays that reveal enlarged hearts are indicative of left ventricular failure; echocardiogram considered the best tool to diagnosed heart failure; and radionuclide studies can confirm the presence and the cause of heart failure. Electrocardiograms can be indicative of ventricular hypertrophy, dysrhythmias, myocardial ischemia, injury or infarction; however they cannot detect the presence or the extent of heart failure.

A client's assessment findings of hoarseness and audible wheezing noted at 2-3 feet (0.61-0.91 meters); trachea deviation towards the left; breathing pattern with prolonged exhalation followed by periods of shallow breathing; tenderness and increased fremitus with light palpation on the left side of the thorax cavity's base; and an occupation history as an oil refinery plant manager for the past ten years would be indicative of which condition? A. Flail chest. B. Emphysema. C. Lung cancer. D. Pneumothorax.

Answer: C Cigarette smoking is a major risk factor and responsible for 85% of all lung cancer deaths. The other 15% causes of lung cancer are due to environmental exposure to asbestos, petroleum distillates, radiation, tar, and uranium. Pulmonary signs and symptoms of lung cancer can be the presence of hoarseness; wheezing; decreased or absent breath sounds; prolonged exhalations alternating with periods of shallow breathing; rapid shallow breathing; areas of tenderness or masses palpated on chest wall; increase fremitus in areas of tumors; decreased or absent fremitus with bronchial obstruction; tracheal deviation; pleural friction rub; asymmetry of diaphragm movement and use of respiratory accessory muscles.

A client who is status post thyroidectomy 12 hours ago appears to be becoming increasingly anxious and is complaining about being uncomfortable and extremely thirsty.The nurse notes the client is lying in bed supine with the head of the bed elevated 30°; their neck dressing appears to be dry and intact; and NS @60ml/hr is infusing without any signs and/or symptoms of infiltration. Which intervention should the nurse implement first? A. Offer the client their prescribed pain medication. B. Obtain a prescription to obtain a serum calcium level. C. Reassure the client and state to them it appears you are anxious. D. Gently roll the client to the side and inspect the back of their neck area.

Answer: D The nurse needs to gently roll the client to the side and inspect back of their neck area to ensure they are not bleeding. A client is at risk of hemorrhaging within the first 24 hours status post a thyroidectomy. One of the first signs of hemorrhaging is "thirst". Signs of restlessness or agitation and anxiety can also be a sign of hemorrhaging after an invasive procedure.

A client with a systemic infection has hypoxia despite 100% FiO2 supplementation. The client has pulmonary edema and dyspnea. The nurse notes an increased use of accessory respiratory muscles. The client's chest x-ray indicates dense pulmonary infiltrates with a "ground-glass" appearance. Based on the client's clinical signs and symptoms, which condition is most likely present? A. Pulmonary emboli. B. Legionnaire's disease. C. Pulmonary hypertension. D. Acute respiratory distress syndrome

Answer: D The client's respiratory distress indicates an increased risk for developing acute respiratory distress syndrome (ARDS). ARDS is an inflammatory response that injures the alveolar-capillary membrane, causing the alveoli to fill up with protein-containing fluid which interferes with the O2/CO2 gas exchange. Aspiration and systemic infections are considered risk factors for the development of ARDS. The overall effect of this condition on the lungs is a decreased lung volume and compliance as the result of the damaged alveolar. This disease has a very high mortality rate and those who do survive have a very poor quality of life due to neuropsychologic deficits.

A client is diagnosed with the hepatitis B virus (HBV) infection. The practical nurse (PN) who has been vaccinated with the hepatitis B series is assigned the client. What precautions should the PN implement when providing caring for this client?

Appropriate PPE should be worn if the possibility of increased risk of bodily fluid exposure. Standard precautions should be implemented at all times regardless of a client's or health care personnel health status. Hepatitis B is spread from contact of bodily fluids, mask and gloves are not necessary if there is no increased risk of exposure to bodily fluids. Health care personnel who have not been vaccinated for hepatitis B can still provide care to the client; they need to ensure to follow standard precaution protocol.

The nurse is taking a history of a newly diagnosed Type 2 diabetic who is beginning treatment. Which subjective information is most important for the nurse to note? A) A history of obesity. B) An allergy to sulfa drugs. C) Cessation of smoking three years ago. D) Numbness in the soles of the feet.

B) An allergy to sulfa drugs. An allergy to sulfa drugs may make the client unable to use some of the most common antihyperglycemic agents (sulfonylureas). The nurse needs to highlight this allergy for the healthcare provider. (A) is common and warrants counseling, but does not have the importance of (B). (C) does increase the risk for vascular disease, but it is not as important to the treatment regimen as (B). Diabetic neuropathy, as indicated by (D), is common with diabetics, but when the serum glucose is decreased, new onset numbness can possibly improve.

During suctioning, a client with an uncuffed tracheostomy tube begins to cough violently and dislodges the tracheostomy tube. Which action should the nurse implement first? A) Notify the healthcare provider for reinsertion. B) Attempt to reinsert the tracheostomy tube. C) Position the client in a lateral position with the neck extended. D) Ventilate client's tracheostomy stoma with a manual bag-mask.

B) Attempt to reinsert the tracheostomy tube. The nurse should attempt to reinsert the tracheostomy tube (B) by using a hemostat to open the tracheostomy or by grasping the retention sutures (if present) to spread the opening in insert a replacement tube (with its obturator) into the stoma. Once in place, the obturator should immediately be removed. (A, C, and D) place the client at risk of airway obstruction.

A 58-year-old client who has been post-menopausal for five years is concerned about the risk for osteoporosis because her mother has the condition. Which information should the nurse offer? A) Osteoporosis is a progressive genetic disease with no effective treatment. B) Calcium loss from bones can be slowed by increasing calcium intake and exercise. C) Estrogen replacement therapy should be started to prevent the progression osteoporosis. D) Low-dose corticosteroid treatment effectively halts the course of osteoporosis.

B) Calcium loss from bones can be slowed by increasing calcium intake and exercise. Post-menopausal females are at risk for osteoporosis due to the cessation of estrogen secretion, but a regimen including calcium, vitamin D, and weight-bearing exercise can prevent further bone loss (B). Osteoporosis can be managed with conservative therapy, such as bone metabolism regulators and estrogen replacement therapy (ERT) to improve bone density, but it is not a genetic disease (A). Although ERT is effective in managing osteoporosis, an increased risk for cancer and heart disease should be considered for individual clients. Corticosteroid therapy promotes bone resorption and is counterproductive in maintaining or increasing bone density (D).

The nurse notes that the only ECG for a 55-year-old male client scheduled for surgery in two hours is dated two years ago. The client reports that he has a history of "heart trouble," but has no problems at present. Hospital protocol requires that those over 50 years of age have a recent ECG prior to surgery. What nursing action is best for the nurse to implement? A) Ask the client what he means by "heart trouble." B) Call for an ECG to be performed immediately. C) Notify surgery that the ECG is over two years old. D) Notify the client's surgeon immediately.

B) Call for an ECG to be performed immediately. Clients over the age of 40 and/or with a history of cardiovascular disease, should receive ECG evaluation prior to surgery, generally 24 hours to two weeks before. (B) should be implemented to ensure that the client's current cardiovascular status is stable. Additional data might be valuable (A), but since time is limited, the priority is to obtain the needed ECG. Documentation of vital signs is important, but does not replace the need for the ECG (C). The surgeon only needs to be notified if the ECG cannot be completed, or if there is a significant problem (D).

The nurse is caring for a client with a continuous feeding through a percutaneous endoscopic gastrostomy (PEG) tube. Which intervention should the nurse include in the plan of care? A) Flush the tube with 50 ml of water q 8 hours. B) Check for tube placement and residual volume q4 hours. C) Obtain a daily x- ray to verify tube placement. D) Position on left side with head of bed elevated 45 degrees.

B) Check for tube placement and residual volume q4 hours. Tube placement and residual volume should be checked before each feeding (B). Tube placement is checked by aspiration of stomach contents and measurement of pH. It is important to check for residual volume because gastric emptying is often delayed during illness. There is an increased risk for aspiration of the feeding with increased residual volume. (A, C, and D) are not correct procedures to follow.

Which milestone indicates to the nurse successful achievement of young adulthood? A) Demonstrates a conceptualization of death and dying. B) Completes education and becomes self-supporting. C) Creates a new definition of self and roles with others. D) Develops a strong need for parental support and approval.

B) Completes education and becomes self-supporting. Transitioning through young adulthood is characterized by establishing independence as an adult, and includes developmental tasks such as completing education, beginning a career, and becoming self-supporting (B). (A and C) are characteristic of adolescence. Although strong bonds with parents are an expected finding for this age group, the need for support and approval (D) indicates dependency, which is a developmental delay.

The nurse is caring for a client with a stroke resulting in right-sided paresis and aphasia. The client attempts to use the left hand for feeding and other self-care activities. The spouse becomes frustrated and insists on doing everything for the client. Based on this data, which nursing diagnosis should the nurse document for this client? A) Situational low self-esteem related to functional impairment and change in role function. B) Disabled family coping related to dissonant coping style of significant person. C) Interrupted family processes related to shift in health status of family member. D) Risk for ineffective therapeutic regimen management related to complexity of care.

B) Disabled family coping related to dissonant coping style of significant person. A stroke affects the whole family and in this case the spouse probably thinks that she is helping and needs to feel that she is contributing to the client's care. Her help is noted as being incongruent with attempts of self-care by the client thereby disabling family coping (B). The scenario does not discuss the client's self-esteem (A), interrupted family processes (C) or the risk for ineffective therapeutic regimen (D).

A 49-year-old female client arrives at the clinic for an annual exam and asks the nurse why she becomes excessively diaphoretic and feels warm during nighttime. What is the nurse's best response? A) Explain the effect of the follicle-stimulating and luteinizing hormones. B) Discuss perimenopause and related comfort measures. C) Assess lung fields and for a cough productive of blood-tinged mucous. D) Ask if a fever above 101º F has occurred in the last 24 hours.

B) Discuss perimenopause and related comfort measures. The perimenopausal period begins about 10 years before menopause with the cessation of menstruation at the average ages of 52 to 54. Lower estrogen levels causes FSH and LH secretion in bursts (surges), which triggers vasomotor instability, night sweats, and hot flashes, so discussions about the perimenopausal body's changes, comfort measures (B), and treatment options should be provided. In-depth pathophysiology of the symptoms (A) may only confuse the client. There is no indication that the client has tuberculosis and an infection, so (C and D) are not indicated.

Which intervention should the nurse implement for a female client diagnosed with pelvic relaxation disorder? A) Describe proper administration of vaginal suppositories and cream. B) Encourage the client to perform Kegel exercises 10 times daily. C) Explain the importance of using condoms when having sexual intercourse. D) Discuss the importance of keeping a diary of daily temperature and menstrual cycle events.

B) Encourage the client to perform Kegel exercises 10 times daily. Pelvic relaxation disorders are structural disorders resulting from weakening support tissues of the pelvis. (B) helps strengthen the surrounding muscles. Medication will not help correct a cystocele, rectocele, or uterine prolapse (A). (C) will help prevent sexually transmitted diseases. (D) is used to identify fertile times during the woman's menstrual cycle.

A male client receives a local anesthetic during surgery. During the post-operative assessment, the nurse notices the client is slurring his speech. Which action should the nurse take? A) Determine the client is anxious and allow him to sleep. B) Evaluate his blood pressure, pulse, and respiratory status. C) Review the client's pre-operative history for alcohol abuse. D) Continue to monitor the client for reactivity to anesthesia.

B) Evaluate his blood pressure, pulse, and respiratory status. Slurred speech in the post-operative client who received a local anesthetic is an atypical finding and may indicate neurological deficits that require further assessment, so obtaining the client's vital signs (B) will provide information about possible cardiovascular complications, such as stroke. The client's anxiety (A), a history of alcohol abuse (D), or local anesthesia (D) are unrelated to the client's sudden onset of slurred speech.

A 32-year-old female client complains of severe abdominal pain each month before her menstrual period, painful intercourse, and painful defecation. Which additional history should the nurse obtain that is consistent with the client's complaints? A) Frequent urinary tract infections. B) Inability to get pregnant. C) Premenstrual syndrome. D) Chronic use of laxatives.

B) Inability to get pregnant. Dysmenorrhea, dyspareunia, and difficulty or painful defecation are common symptoms of endometriosis, which is the abnormal displacement of endometrial tissue in the dependent areas of the pelvic peritoneum. A history of infertility (B) is another common finding associated with endometriosis. Although (A, C, and D) are common, nonspecific gynecological complaints, the most common complaints of the client with endometriosis are pain and infertility.

Which reaction should the nurse identify in a client who is responding to stimulation of the sympathetic nervous system? A) Pupil constriction. B) Increased heart rate. C) Bronchial constriction. D) Decreased blood pressure.

B) Increased heart rate. Any stressor that is perceived as threatening to homeostasis acts to stimulate the sympathetic nervous system and manifests as a flight-or-fight response, which includes an increase in heart rate (B). (A, C, and D) are responses of the parasympathetic nervous system.

A client who has heart failure is admitted with a serum potassium level of 2.9 mEq/L. Which action is most important for the nurse to implement? A) Give 20 mEq of potassium chloride. B) Initiate continuous cardiac monitoring. C) Arrange a consultation with the dietician. D) Teach about the side effects of diuretics.

B) Initiate continuous cardiac monitoring. Hypokalemia (normal 3.5 to 5 mEq/L) causes changes in myocardial irritability and ECG waveform, so it is most important for the nurse to initiate continuous cardiac monitoring (B) to identify ventricular ectopy or other life-threatening dysrhythmias. Potassium chloride (A) should be given after cardiac monitoring is initiated so that the effects of potassium replacement on the cardiac rhythm can be monitored. (C and D) should be implemented when the client is stable.

A 20-year-old female client calls the nurse to report a lump she found in her breast. Which response is the best for the nurse to provide? A) Check it again in one month, and if it is still there schedule an appointment. B) Most lumps are benign, but it is always best to come in for an examination. C) Try not to worry too much about it, because usually, most lumps are benign. D) If you are in your menstrual period it is not a good time to check for lumps.

B) Most lumps are benign, but it is always best to come in for an examination. (B) provides the best response because it addresses the client's anxiety most effectively and encourages prompt and immediate action for a potential problem. (A) postpones treatment if the lump is malignant, and does not relieve the client's anxiety. (C and D) provide false reassurance and do not help relieve anxiety.

A client with a 16-year history of diabetes mellitus is having renal function tests because of recent fatigue, weakness, elevated blood urea nitrogen, and serum creatinine levels. Which finding should the nurse conclude as an early symptom of renal insufficiency? A) Dyspnea. B) Nocturia. C) Confusion. D) Stomatitis.

B) Nocturia As the glomerular filtration rate decreases in early renal insufficiency, metabolic waste products, including urea, creatinine, and other substances, such phenols, hormones, electrolytes, accumulate in the blood. In the early stage of renal insufficiency, polyuria results from the inability of the kidneys to concentrate urine and contribute to nocturia (B). (A, C, and D) are more common in the later stages of renal failure.

Which client should the nurse recognize as most likely to experience sleep apnea? A) Middle-aged female who takes a diuretic nightly. B) Obese older male client with a short, thick neck. C) Adolescent female with a history of tonsillectomy. D) School-aged male with a history of hyperactivity disorder.

B) Obese older male client with a short, thick neck. Sleep apnea is characterized by lack of respirations for 10 seconds or more during sleep and is due to the loss of pharyngeal tone which allows the pharynx to collapse during inspiration and obstructs air flow through the nose and mouth. With obstructive sleep apnea, the client is often obese or has a short, thick neck as in (B). (A, C, and D) are not typically prone to sleep apnea.

Which intervention should the nurse plan to implement when caring for a client who has just undergone a right above-the-knee amputation? A) Maintain the residual limb on three pillows at all times. B) Place a large tourniquet at the client's bedside. C) Apply constant, direct pressure to the residual limb. D) Do not allow the client to lie in the prone position.

B) Place a large tourniquet at the client's bedside. A large tourniquet should be placed in plain sight at the client's bedside (B). If severe bleeding occurs, the tourniquet should be readily available and applied to the residual limb to control hemorrhage. The residual limb should not be placed on a pillow (A) because a flexion contracture of the hip may result. (C) should be avoided because it may compromise wound healing. (D) should be encouraged to stretch the flexor muscles and to prevent flexion contracture of the hip.

During CPR, when attempting to ventilate a client's lungs, the nurse notes that the chest is not moving. What action should the nurse take first? A) Use a laryngoscope to check for a foreign body lodged in the esophagus. B) Reposition the head to validate that the head is in the proper position to open the airway. C) Turn the client to the side and administer three back blows. D) Perform a finger sweep of the mouth to remove any vomitus.

B) Reposition the head to validate that the head is in the proper position to open the airway. The most frequent cause of inadequate aeration of the client's lungs during CPR is improper positioning of the head resulting in occlusion of the airway (B). A foreign body can occlude the airway, but this is not common unless choking preceded the cardiac emergency, and (A, C and D) should not be the nurse's first action.

The nurse is planning care for a client who has a right hemispheric stroke. Which nursing diagnosis should the nurse include in the plan of care? A) Impaired physical mobility related to right-sided hemiplegia. B) Risk for injury related to denial of deficits and impulsiveness. C) Impaired verbal communication related to speech-language deficits. D) Ineffective coping related to depression and distress about disability.

B) Risk for injury related to denial of deficits and impulsiveness. With right-brain damage, a client experience difficulty in judgment and spatial perception and is more likely to be impulsive and move quickly, which placing the client at risk for falls (B). Although clients with right and left hemisphere damage may experience impaired physical mobility, the client with right brain damage will manifest physical impairments on the contralateral side of the body, not the same side (A). The client with a left-brain injury may manifest right-sided hemiplegia with speech or language deficits (C). A client with left-brain damage is more likely to be aware of the deficits and experience grief related to physical impairment and depression (D).

Physical examination of a comatose client reveals decorticate posturing. Which statement is accurate regarding this client's status based upon this finding? A) A cerebral infectious process is causing the posturing. B) Severe dysfunction of the cerebral cortex has occurred. C) There is a probable dysfunction of the midbrain. D) The client is exhibiting signs of a brain tumor.

B) Severe dysfunction of the cerebral cortex has occurred. Decorticate posturing (adduction of arms at shoulders, flexion of arms on chest with wrists flexed and hands fisted and extension and adduction of extremities) is seen with severe dysfunction of the cerebral cortex (B). (A) is characteristic of meningitis. (C) is characterized by decerebrate posturing (rigid extension and pronation of arms and legs). A client with (D) may exhibit decorticate posturing, depending on the position of the tumor and the condition of the client.

The nurse working on a telemetry unit finds a client unconscious and in pulseless ventricular tachycardia (VT). The client has an implanted automatic defibrillator. What action should the nurse implement? A) Prepare the client for transcutaneous pacemaker. B) Shock the client with 200 joules per hospital policy. C) Use a magnet to deactivate the implanted pacemaker. D) Observe the monitor until the onset of ventricular fibrillation.

B) Shock the client with 200 joules per hospital policy. The client must be externally shocked (B) to restore an effective cardiac rhythm. The automatic defibrillator is obviously malfunctioning. (A) will not be effective during ventricular tachycardia, since it is used for asystole. Since the defibrillator is not functioning, (C) is not warranted. The client should be treated immediately to restore cardiac output (D).

Which description of symptoms is characteristic of a client diagnosed with trigeminal neuralgia (tic douloureux)? A) Tinnitus, vertigo, and hearing difficulties. B) Sudden, stabbing, severe pain over the lip and chin. C) Facial weakness and paralysis. D) Difficulty in chewing, talking, and swallowing.

B) Sudden, stabbing, severe pain over the lip and chin. Trigeminal neuralgia is characterized by paroxysms of pain, similar to an electric shock, in the area innervated by one or more branches of the trigeminal nerve (5th cranial) (B). (A) would be characteristic of Méniére's disease (8th cranial nerve). (C) would be characteristic of Bell's palsy (7th cranial nerve). (D) would be characteristic of disorders of the hypoglossal cranial nerve (12th).

Which instruction should the nurse teach a female client about the prevention of toxic shock syndrome? A. "Get immunization against human papillomavirus (HPV)." B. "Change your tampon frequently." C. "Empty your bladder after intercourse." D. "Obtain a yearly flu vaccination."

B. "Change your tampon frequently." Rationale: Certain strains of Staphylococcus aureus produce a toxin that can enter the bloodstream through the vaginal mucosa. Changing the tampon frequently (B) reduces the exposure to these toxins, which are the primary cause of toxic shock syndrome. (A) helps prevent cervical cancer, not toxic shock syndrome. (C) can lessen the incidence of urinary tract infection. (D) can help prevent some individuals from contracting the flu and pneumonia, but no relationship to toxic shock syndrome has been proven.

A 58-year-old client, who has no health problems, asks the nurse about the Pneumovax vaccine. The nurse's response to the client should be based on which information? A) The vaccine is given annually before the flu season to those over 50 years of age. B) The immunization is administered once to older adults or persons with a history of chronic illness. C) The vaccine is for all ages and is given primarily to those persons traveling overseas to areas of infection. D) The vaccine will prevent the occurrence of pneumococcal pneumonia for up to five years.

B) The immunization is administered once to older adults or persons with a history of chronic illness. It is usually recommended that persons over 65 years of age and those with a history of chronic illness receive the vaccine once in a lifetime (B). (Some resources recommend obtaining the vaccine at 50 years of age.) The influenza vaccine is given once a year, not the Pneumovax (A). Although the vaccine might be given to a person traveling overseas, that is not the main rationale for administering the vaccine (C). It is usually given once in a lifetime (D), but with immunosuppressed clients or clients with a history of pneumonia re-vaccination is sometimes required.

The nurse is planning the care for a client who is admitted with the syndrome of inappropriate antidiuretic hormone secretion (SIADH). Which interventions should the nurse include in this client's plan of care? (Select all that apply.) A. Salt-free diet B. Quiet environment C. Deep tendon reflex assessments D. Neurologic checks E. Daily weights F. Unrestricted intake of free water

B, C, D, E. SIADH results in water retention and dilutional hyponatremia, which causes neurologic change when serum sodium levels are less than 115 mEq/L. The nurse should maintain a quiet environment (B) to prevent overstimulation that can lead to periods of disorientation, assess deep tendon reflexes (C) and neurologic checks (D) to monitor for neurologic deterioration. Daily weights (E) should be monitored to assess for fluid overload: 1 kg weight gain equals 1 L of fluid retention, which further dilutes serum sodium levels. (A and F) contribute to dilutional hyponatremia.

53. A patient is being discharged from the emergency department after being treated for epistaxis. In teaching the family first aid measures in the event the epistaxis would recur, which of the following measures would the nurse suggest? (Select all that apply.) A. Tilt patients head backwards B. Apply ice compresses to the nose C. Pinch the entire soft lower portion of the nose D. Partially insert a small gauze pad into the bleeding nostril

B,C,D First aid measures to control epistaxis includes placing the patient in a sitting position, leaning forward. Tilting the head back does not stop the bleeding, but rather allows the blood to enter the nasopharynx, which could result in aspiration or nausea/vomiting from swallowing blood. All of the other options are appropriate first aid treatment of epistaxis.

A patient is being discharged from the emergency department after being treated for epistaxis. In teaching the family first aid measures in the event the epistaxis would recur, which of the following measures would the nurse suggest? (Select all that apply.) A. Tilt patients head backwards B. Apply ice compresses to the nose C. Pinch the entire soft lower portion of the nose D. Partially insert a small gauze pad into the bleeding nostril

B,C,D First aid measures to control epistaxis includes placing the patient in a sitting position, leaning forward. Tilting the head back does not stop the bleeding, but rather allows the blood to enter the nasopharynx, which could result in aspiration or nausea/vomiting from swallowing blood. All of the other options are appropriate first aid treatment of epistaxis.

47. A patient has been receiving oxygen per nasal cannula while hospitalized for COPD. The patient asks the nurse whether oxygen use will be needed at home. Which of the following would be the most appropriate response by the nurse? A. "Long-term home oxygen therapy should be used to prevent respiratory failure." B. "Oxygen will be needed when your oxygen saturation drops to 88% and you have symptoms of hypoxia. C. "Long-term home oxygen therapy should be used to prevent heart problems related to emphysema." D. "Oxygen will not be needed until or unless you are in the terminal stages of this disease."

B. "Oxygen will be needed when your oxygen saturation drops to 88% and you have symptoms of hypoxia.Long-term oxygen therapy in the home should be considered when the oxygen saturation is 88% or less and the patient has signs of tissue hypoxia, such as cor pulmonale, erythrocytosis, or impaired mental status.

The nurse notes a physician's order written at 10:00 AM for 2 units of packed red blood cells to be administered to a patient who is anemic secondary to chronic blood loss. If the transfusion is picked up at 11:30, the nurse should plan to hang the unit no later than which of the following times? A. 11:45 AM B. 12:00 noon C. 12:30 PM D. 3:30 PM

B. 12:00 noon The nurse must hang the unit of packed red blood cells within 30 minutes of signing them out from the blood bank

After reviewing the Braden Scale findings of residents at a long-term facility, the charge nurse should to tell the unlicensed assistive personnel (UAP) to prioritize skin care for which client? A. An older adult who is unable to communicate elimination needs B. A older man who sheets are damp each time he turned C. A woman with osteoporosis who is unable to bear weight D. A poorly nourished client who requires liquid supplements

B. A older man who sheets are damp each time he turned

The nurse is interviewing a client who is taking interferon-alfa-2a (Roferon-A) and ribavirin (Virazole) combination therapy for hepatitis C. The client reports experiencing overwhelming feelings of depression. What action should the nurse implement first? A. Recommend mental health counseling. B. Review the medications actions and interactions. C. Assess for the client's daily activity level. D. Provide information regarding a support group.

B. Alpha-interferon and ribavirin combination therapy can cause severe depression. (A, B, C) may be implemented after physiological aspect of the situation are assessed.

When assigning clients on a medical-surgical floor to an RN and a PN, it is best for the charge nurse to assign which client to the PN? A. A young adult with bacterial meningitis with recent seizures B. An older adult client with pneumonia and viral meningitis C. A female client in isolation with meningococcal meningitis D. A male client 1 day postoperative after drainage of a brain abscess

B. An older adult client with pneumonia and viral meningitis Rationale: The most stable client is (B). (A, C, and D) are all at high risk for increased intracranial pressure and require the expertise of the RN for assessment and management of care.

The nurse notes that a client who is scheduled for surgery the next morning has an elevated blood urea nitrogen (BUN) level. Which condition is most likely to have contributed to this finding? A. Myocardial infarction 2 months ago B. Anorexia and vomiting for the past 2 days C. Recently diagnosed type 2 diabetes mellitus D. Skeletal traction for a right hip fracture

B. Anorexia and vomiting for the past 2 days Rationale: The blood urea nitrogen (BUN) level indicates the effectiveness of the kidneys in filtering waste from the blood. Dehydration, which could be caused by vomiting, would cause an increased the BUN level (B). (A) would affect serum enzyme levels, not the BUN level. (C) would primarily affect the blood glucose level; renal failure that could increase the BUN level would be unlikely in a client newly diagnosed with type 2 diabetes. Effects of (D) might affect the complete blood count (CBC) but would not directly increase the BUN level.

An elderly client is experiencing disturbed sleep patterns. Which interventions should the nurse implement to help the client attain maximal sleep function? (select all that apply) A. Discuss need for adult day services B. Assess daytime somnolence C. Recommend evaluation of home safety D. Review medication side effect profiles E. Refer to social worker for elder care

B. Assess daytime somnolence D. Review medication side effect profiles

An older male client comes to the outpatient clinic complaining of pain in his left calf. The nurse notices a reddened area on the calf of his right leg that is warm to the touch, and the nurse suspects that the client may have thrombophlebitis. Which additional assessment is most important for the nurse to perform? A. Measure the client's calf circumference. B. Auscultate the client's breath sounds. C. Observe for ecchymosis and petechiae. D. Obtain the client's blood pressure.

B. Auscultate the client's breath sounds. Rationale: All these techniques provide useful assessment data. The most important is to auscultate the client's breath sounds (B) because the client may have a pulmonary embolus secondary to the thrombophlebitis. (A) may provide data that support the nurse's suspicion of thrombophlebitis. (C) is the least helpful assessment because bruising is not a typical finding associated with thrombophlebitis. (D) is always useful in evaluating the client's response to a problem but is of less immediate priority than breath sound auscultation.

During the transfer of a client who had major abdominal surgery this morning, the postanesthesia care unit (PACU) nurse reports that the client, who is awake and responsive, continues to report pain and nausea after receiving morphine 2 mg IV and ondansetron 4 mg IV 45 minutes ago. Which elements of SBAR communication are missing from the report given by the PACU nurse? A. Situation B. Background C. Assessment D. Recommendation E. Rationales

B. Background C. Assessment D. Recommendation

An older male client diagnosed with end-stage chronic obstructive pulmonary disease (COPD) is on strict bedrest, and asks the nurse, "Why can't I get out of bed? What response is best for the nurse to provide? A. A high-Fowler's position promotes lung expansion B. Bed rest decreases your body's need for oxygen C. You are so weak that you are at risk for falling D. hospital policy requires that you are assisted to get up

B. Bed rest decreases your body's need for oxygen

A client with chronic asthma is admitted to postanesthesia complaining of pain at level 8 of 10, with a BP of 124/78, pulse of 88 beats/min, and respirations of 20 breaths/min. The postanesthesia recovery prescription is, "Morphine 2 to 4 mg IV push while in recovery for pain level over 5." What intervention should the nurse implement? A. Give the medication as prescribed to decrease the client's pain. B. Call the anesthesia provider for a different medication for pain. C. Use nonpharmacologic techniques before giving the medication. D. Reassess pain level in 30 Minutes and medicate if it remains elevated.

B. Call for a different medication because morphine and meperidine (Demerol) have histamine-releasing narcotics and should be avoided when a client has asthma. (A) puts the client at risk for asthma attack. (C & D) disregard the clients prescription and pain relief.

A client with chronic asthma is admitted to the PACU complaining of pain at a level of 8 on a 1 to 10 scale, with a blood pressure of 124/78 mm Hg, pulse of 88 beats/min, and respirations of 20 breaths/min. The PACU recovery prescription is, "Morphine, 2 to 4 mg IV push, while in recovery for pain level over 5." Which intervention should the nurse implement? A. Give the medication as prescribed to decrease the client's pain. B. Call the anesthesia provider for a different medication for pain. C. Use nonpharmacologic techniques before giving the medication. D. Reassess the pain level in 30 minutes and medicate if it remains elevated.

B. Call the anesthesia provider for a different medication for pain. Rationale: The nurse should call the provider for a different medication (B) because morphine is a histamine-releasing opioid and should be avoided when the client has asthma. (A) is unsafe because it puts the client at risk for an asthma exacerbation. Even if the drug were safe for the client, (C and D) both disregard the prescription and the client's need for pain relief in the immediate postoperative period.

A client with chronic asthma is admitted to the PACU complaining of pain at a level of 8 on a 1 to 10 scale, with a blood pressure of 124/78 mm Hg, pulse of 88 beats/min, and respirations of 20 breaths/min. The PACU recovery prescription is "Morphine, 2 to 4 mg IV push, while in recovery for pain level over 5." Which intervention should the nurse implement? AGive the medication as prescribed to decrease the client's pain. B.Call the anesthesia provider for a different medication for pain. C.Use nonpharmacologic techniques before giving the medication. D.Reassess the pain level in 30 minutes and medicate if it remains elevated.

B. Call the anesthesia provider for a different medication for pain. Rationale: The nurse should call the provider for a different medication because morphine is a histamine-releasing opioid and should be avoided when the client has asthma. Option A is unsafe because it puts the client at risk for an asthma exacerbation. Even if the drug were safe for the client, options C and D both disregard the prescription and the client's need for pain relief in the immediate postoperative period.

24. A nurse is preparing to establish oxygen therapy for a patient with COPD, and the physician's prescription reads "oxygen per nasal cannula at 5 L per minute." Which of the following actions should the nurse take? A. Administer the oxygen as prescribed. B. Call the physician and question the correct flow rate of the oxygen. C. Establish the oxygen as prescribed and obtain an ABG. D. Change the delivery device from a nasal cannula to a simple oxygen mask.

B. Call the physician and question the correct flow rate of the oxygen. The nurse should call the physician immediately and question the flow rate for delivery of the oxygen before implementation. Oxygen is used cautiously in patients with COPD because of longstanding hypoxemia serving as the respiratory drive mechanism. If high levels of oxygen are administered, the respiratory drive can be obliterated. Changing the device to a simple oxygen mask may alter the oxygen concentration being delivered to the patient and will further enhance the obliteration of the patient's respiratory drive. Obtaining an ABG sample is not a priority at this time, and the action does not address the validity of the prescribed oxygen dosing for the patient.

A nurse is preparing to establish oxygen therapy for a patient with COPD, and the physician's prescription reads "oxygen per nasal cannula at 5 L per minute." Which of the following actions should the nurse take? A. Administer the oxygen as prescribed. B. Call the physician and question the correct flow rate of the oxygen. C. Establish the oxygen as prescribed and obtain an ABG. D. Change the delivery device from a nasal cannula to a simple oxygen mask.

B. Call the physician and question the correct flow rate of the oxygen. The nurse should call the physician immediately and question the flow rate for delivery of the oxygen before implementation. Oxygen is used cautiously in patients with COPD because of longstanding hypoxemia serving as the respiratory drive mechanism. If high levels of oxygen are administered, the respiratory drive can be obliterated. Changing the device to a simple oxygen mask may alter the oxygen concentration being delivered to the patient and will further enhance the obliteration of the patient's respiratory drive. Obtaining an ABG sample is not a priority at this time, and the action does not address the validity of the prescribed oxygen dosing for the patient.

Which change in laboratory values indicates to the nurse that a client with rheumatoid arthritis may be experiencing an adverse effect of methotrexate (Mexate) therapy? A. Increase in rheumatoid factor B. Decrease in hemoglobin level C. Increase in blood glucose level D. Decrease in erythrocyte sedimentation rate (ESR; sed rate)

B. Decrease in hemoglobin level Rationale: Methotrexate is an immunosuppressant. A common side effect is bone marrow depression, which would be reflected by a decrease in the hemoglobin level (B). (A) indicates disease progression but is not a side effect of the medication. (C) is not related to methotrexate. (D) indicates that inflammation associated with the disease has diminished.

The nurse administers an oral antiviral to a client with shingles. Which finding is most important for the nurse to report to the healthcare provider? A. elevated liver function tests B. Decreased white blood cell count C. Vomiting and diarrhea D. Pruritus and muscle aches

B. Decreased white blood cell count

The nurse is caring for a client with a fractured right elbow. Which assessment finding has the highest priority and requires immediate intervention? A. Ecchymosis over the right elbow area B. Deep unrelenting pain in the right arm C. An edematous right elbow D. The presence of crepitus in the right elbow

B. Deep unrelenting pain in the right arm Rationale: Compartment syndrome is a condition involving increased pressure and constriction of the nerves and vessels within an anatomic compartment, causing pain uncontrolled by opioids, and neurovascular compromise (B). (A) is an expected finding. (C) related to compartment syndrome cannot be seen, and any visible edema is an expected finding related to the injury. (D) is an expected finding.

20. If a patient states, "It's hard for me to breathe and I feel short-winded all the time," what is the most appropriate terminology to be applied in documenting this assessment by a nurse? A. Apnea B. Dyspnea C. Tachypnea D. Respiratory fatigue

B. Dyspnea Dyspnea is a subjective description reflective of the patient's statement indicating difficulty in breathing. Apnea refers to absence of breath or breathing. Tachypnea refers to an increased rate of breathing, usually greater than 20 breaths per minute. Respiratory fatigue is subjective and usually refers to the patient exhibiting signs and symptoms associated with a comprehensive respiratory assessment including laborious breathing, use of accessory muscles, and slowing of respirations.

A client in the third trimester of pregnancy reports that she feels some "lumpy places" in her breasts and that her nipples sometimes leak yellowish fluid. She has an appointment with her healthcare provider in two weeks. What action should the nurse take? A. Reschedule the client's appointment for the following day B. Explain that this normal secretion can be assessed at the next visit C. Tell the client to begin nipple stimulation to prepare for breast feeding D. Recommend that the client start wearing a supportive brassiere

B. Explain that this normal secretion can be assessed at the next visit

When completing a mental health assessment, the nurse wishes to obtain information about a client's insight and judgement. What interview technique is most useful in assessing these mental abilities? A. Ask the client to describe what is meant by the proverb, "A penny saved is a penny earned." B. Have the client describe what should be done if a child was observed alone in a locked car on a hot day C. Show the client a list of 3 words a the beginning of the interview, and ask for word recall later. D. Request that the client complete a mathematical operation, such as counting backward by 9s form 100

B. Have the client describe what should be done if a child was observed alone in a locked car on a hot day

In assessing a client at 34-weeks gestation, the nurse notes that she has a slightly elevated total T4 with a slightly enlarged thyroid, a hematocrit of 28%, a heart rate of 92 beats per minute, and a systolic murmur. Which finding requires follow-up? A.Elevated parathyroid hormone level B. Hematocrit of 28% C. Heart rate of 92 beats per minute D. Systolic murmur

B. Hematocrit of 28%

A client diagnosed with chronic kidney disease (CKD) 2 years ago is regularly treated at a community hemodialysis facility. Before his scheduled dialysis treatment, which electrolyte imbalance should the nurse anticipate? A. Hypophosphatemia B. Hypocalcemia C. Hyponatremia D. Hypokalemia

B. Hypocalcemia Rationale: Hypocalcemia (B) develops in CKD because of chronic hyperphosphatemia, not (A). Increased phosphate levels cause the peripheral deposition of calcium and resistance to vitamin D absorption needed for calcium absorption. Prior to dialysis, the nurse would expect to find the client hypernatremic and hyperkalemic, not with (C or D).

Which of the following nursing interventions is of the highest priority in helping a patient expectorate thick secretions related to pneumonia? A. Humidify the oxygen as able B. Increase fluid intake to 3L/day if tolerated. C. Administer cough suppressant q4hr. D. Teach patient to splint the affected area.

B. Increase fluid intake to 3L/day if tolerated. Although several interventions may help the patient expectorate mucus, the highest priority should be on increasing fluid intake, which will liquefy the secretions so that the patient can expectorate them more easily. Humidifying the oxygen is also helpful, but is not the primary intervention. Teaching the patient to splint the affected area may also be helpful, but does not liquefy the secretions so that they can be removed.

A client with congestive heart failure and atrial fibrillation develops ventricular ectopy with a pattern of 8 ectopic beats/min. Which action should the nurse take based on this observation? A. Assess for bilateral jugular vein distention. B. Increase oxygen flow via nasal cannula. C. Administer PRN furosemide (Lasix). D.Auscultate for a pleural friction rub.

B. Increase oxygen flow via nasal cannula. Rationale: This client should have the oxygen flow immediately increased to promote oxygenation of the myocardium. Ventricular ectopy, characterized by multiple PVCs, is often caused by myocardial ischemia exacerbated by hypokalemia. The nurse would expect the client in congestive heart failure to have some degree of option A, which does not exacerbate the ectopy. Option C could create a more severe hypokalemia, which could increase the ectopy. The client is not exhibiting signs of option D.

The nurse is caring for a client who is 1 day post-acute myocardial infarction. The client is receiving oxygen at 2 L/min via nasal cannula and has a peripheral saline lock. The nurse notes that the client is having eight premature ventricular contractions (PVCs) per minute. Which intervention should the nurse implement first? A. Obtain an IV pump for antiarrhythmic infusion. B. Increase the client's oxygen flow rate. C. Prepare for immediate countershock. D. Gather equipment for endotracheal intubation.

B. Increase the client's oxygen flow rate. Rationale: Increasing the oxygen flow rate (B) provides more oxygen to the client's myocardium and may decrease myocardial irritability as manifested by the frequent PVCs. (A) can be delegated and is a lower priority action than (B). Defibrillation may eventually be necessary, but (C) is not the immediate treatment for frequent PVCs. (D) may become necessary if the client stops breathing, but is not indicated at this time.

During a home visit, the nurse determines that a male client is experiencing symptoms that should be controlled by his prescribed medication. The client states that he forgot when he was supposed to take his medication. What is the priority nursing problem when the nurse develops the plan of care for the client? A. Self neglect related to loss of cognitive function B. Ineffective health maintenance related to lack of knowledge C. Situational low self-esteem related to symptoms of illness D. Noncompliance related to lifestyle change

B. Ineffective health maintenance related to lack of knowledge

Which condition should the nurse anticipate as a potential problem in a female client with a neurogenic bladder? A. Stress incontinence B. Infection C. Painless gross hematuria D. Peritonitis

B. Infection Rationale: Infection (B) is the major complication resulting from stasis of urine and subsequent catheterization. (A) is the involuntary loss of urine through an intact urethra as a result of a sudden increase in intraabdominal pressure. (C) is the most common symptom of bladder cancer. (D) is the most common and serious complication of peritoneal dialysis.

Which condition should the nurse anticipate as a potential problem in a female client with a neurogenic bladder? A. Stress incontinence. B. Infection. C. Painless, gross hematuria. D. Peritonitis.

B. Infection is the major complication resulting from stasis of urine and subsequent catheterization. (A) is the involuntary loss of urine through an intact urethra as a result of suddenly increased pressure. (C) is the most common symptom of bladder cancer. (D) is the most common and serious complication of peritoneal dialysis.

8. The nurse receives a physician's order to transfuse fresh frozen plasma to a patient suffering from an acute blood loss. Which of the following procedures is most appropriate for infusing this blood product? A. Hand the fresh frozen plasma as a piggyback to a new bag of primary IV solution without KCl. B. Infuse the fresh frozen plasma as rapidly as the patient will tolerate. C. Hang the fresh frozen plasma as a piggyback to the primary IV solution. D. Infuse the fresh frozen plasma as a piggyback to a primary solution of normal saline.

B. Infuse the fresh frozen plasma as rapidly as the patient will tolerate. The fresh frozen plasma should be administered as rapidly as possible and should be used within 2 hours of thawing. Fresh frozen plasma is infused using any straight-line infusion set. Any existing IV should be interrupted while the fresh frozen plasma is infused, unless a second IV line has been started for the transfusion.

The nurse receives a physician's order to transfuse fresh frozen plasma to a patient suffering from an acute blood loss. Which of the following procedures is most appropriate for infusing this blood product? A. Hand the fresh frozen plasma as a piggyback to a new bag of primary IV solution without KCl. B. Infuse the fresh frozen plasma as rapidly as the patient will tolerate. C. Hang the fresh frozen plasma as a piggyback to the primary IV solution. D. Infuse the fresh frozen plasma as a piggyback to a primary solution of normal saline.

B. Infuse the fresh frozen plasma as rapidly as the patient will tolerate. The fresh frozen plasma should be administered as rapidly as possible and should be used within 2 hours of thawing. Fresh frozen plasma is infused using any straight-line infusion set. Any existing IV should be interrupted while the fresh frozen plasma is infused, unless a second IV line has been started for the transfusion.

A female college student is admitted to the Emergency Department following indigestion of alcohol and pain medication. A nasogastric tube and subclavian line are placed. The nurse auscultates audible breath sounds on the right side, faint sounds on the left side, and chest involvement that occurs only on the right side of the thorax. Which procedure should the nurse prepare for first? A. removal of the subclavian line and preparation for jugular insertion B. Insertion of 16g needle at the 4th intercostal space midclavicular line C. Placement of an endotracheal tube and mechanical ventilation D. Retraction of the nasogastric tube by a length of 2 cm

B. Insertion of 16g needle at the 4th intercostal space midclavicular line Based on the location of the audible breath sounds and the client's chest movements, the nurse should suspect that the subclavian line has pierced the client's left lung, causing a pneumothorax. Therefore the treatment of choice is B.

A male client with Addison's disease tells the nurse that he is taking hydrocortisone in a divided daily dose. He reports increasing fatigue and weakness. What action should the nurse take? A. Advise the client to skip the next scheduled dose of hydrocortisone B. Interview the client about any sources of increased stress in his life C. Instruct the client to limit his intake of oral fluids, especially at night D. Encourage the client to increase daily exercise and physical activity

B. Interview the client about any sources of increased stress in his life

When assigning clients on a medical-surgical floor to a RN and a LPN, it is best for the charge nurse to assign which client to the LPN? A. A child with bacterial meningitis with recent seizures. B. An older adult client with pneumonia and viral meningitis. C. A female client in isolation wiht meningococcal meningitis. D. A male client 1 day post-op after drainage of a brain abscess.

B. Is the most stable. A, C, D have an increased risk for elevated ICP.

2. Which of the following is a factor significant in the development of anemia in men? A. Condom use B. Large hemorrhoids C. A diet high in cholesterol D. Smoking one pack of cigarettes daily

B. Large hemorrhoids Gastrointestinal (GI) tract bleeding is a common etiologic factor in men and may result from peptic ulcers, hiatal hernia, gastritis, cancer, hemorrhoids, diverticula, ulcerative colitis, or salicylate poisoning.

What intervention should the nurse implement to prevent edema and promote healing of a client's incision resulting from an above-the-knee amputation? A. Keep the residual limb in a dependent position B. Maintain the residual limb in a compression dressing C. Inspect the incision hourly for the first 24 hours postoperatively D. Elevate the affected limb on two pillows at all times

B. Maintain the residual limb in a compression dressing

The nurse is planning discharge care for a male client with metastatic cancer. The client tells the nurse that he plans to return to work despite pain, fatigue, and impending death. Which goal is most important to include in this client's plan of care? A. Implement decisions about future hospice services within the next 3 months B. Maintains pain level below 4 when implementing outpatient pain clinic strategies C. Requests home health care if independence becomes compromised for 5 days D. Arranges for short term counseling if stressors impact work schedule for 2 weeks

B. Maintains pain level below 4 when implementing outpatient pain clinic strategies

A male client with hepatitis A is admitted with elevated hepatic enzymes and jaundice. Which intervention should the nurse implement? A. Provide cloth gowns for the client to wear B. Meticulous hand washing after each client contact C. Place the client in strict airborne precautions D. Use plastic utensils with each meal tray

B. Meticulous hand washing after each client contact

Which abnormal lab finding indicates that a client with diabetes needs further evaluation for diabetic nephropathy? A. Hypokalemia B. Microalbuminauria C. Elevated serum lipids D. Ketonuria

B. Microalbuminuria is the earliest sign of nephropathy and indicates the need for follow-up evaluation. Hyperkalemia (A) is associated with end stage renal disease caused by diabetic nephropathy. (C) may be elevated in end stage renal disease. (D) may signal the onset of DKA.

A tornado warning alarm has been activated at the local hospital. Which action should the charge nurse working on a surgical unit implement first? A. Instruct the nursing staff to close all window blinds and curtains in clients' rooms. B. Move clients and visitors into the hallways and close all doors to clients' rooms. C. Visually confirm the location of the tornado by checking the windows on the unit. D. Assist all visitors with evacuation down the stairs in a calm and orderly manner.

B. Move clients and visitors into the hallways and close all doors to clients' rooms. Rationale: In the event of a tornado, all persons should be moved into the hallways, away from windows, to prevent flying debris from causing injury (B). Although (A) may help decrease the amount of flying debris, it is not safe to leave clients in rooms with closed blinds; (B) is a higher priority at this time. Hospital staff should stay away from windows to avoid injury and should focus on client evacuation into hallways rather than (C). (D) is not the first action that should be taken.

A client with superficial burn to the face, neck. and hands resulting from a house fire is admitted to the burn unit. Which assessment finding indicated to the nurse that the client should be monitored for carbon monoxide poisoning? A. Expiratory stridor and nasal flaring B. Mucus membrane cherry red color C. Carbonaceous particles in sputum D. Pulse oximetry reading of 80 percent

B. Mucus membrane cherry red color

The nurse assesses a postoperative client whose skin is cool, pale, and moist. The client is very restless and has scant urine output. Oxygen is being administered at 2 L/min, and a saline lock is in place. Which intervention should the nurse implement first? A. Measure the urine specific gravity. B. Obtain IV fluids for infusion per protocol. C. Prepare for insertion of a central venous catheter. D. Auscultate the client's breath sounds.

B. Obtain IV fluids for infusion per protocol. Rationale: The client is at risk for hypovolemic shock because of the postoperative status and is exhibiting early signs of shock. A priority intervention is the initiation of IV fluids (B) to restore tissue perfusion. (A, C, and D) are all important interventions, but are of less priority than (B).

A 25-year-old client was admitted yesterday after a motor vehicle collision. Neurodiagnostic studies have shown a basal skull fracture in the middle fossa. Assessment on admission revealed both halo and Battle signs. Which new symptom indicates that the client is likely to be experiencing a common life-threatening complication associated with a basal skull fracture? A. Bilateral jugular venous distention B. Oral temperature of 102° F C. Intermittent focal motor seizures D. Intractable pain in the cervical region

B. Oral temperature of 102° F Rationale: Clients with basilar skull fractures are at high risk for infection of the brain, as indicated by an increased oral temperature (B), because the fracture leaves the meninges open to bacterial invasion. Clients may experience (C and D), but these findings do not pose as great a life-threatening risk as infection. Jugular distention (A) is not a typical complication of basal skull fractures.

A 25-year-old client was admitted yesterday after a motor vehicle collision. Neurodiagnostic studies have shown a basal skull fracture in the middle fossa. Assessment on admission revealed both halo and Battle signs. Which new symptom indicates that the client is likely to be experiencing a common life-threatening complication associated with a basal skull fracture? A. Bilateral jugular venous distention B.Oral temperature of 102° F C.Intermittent focal motor seizures D.Intractable pain in the cervical region

B. Oral temperature of 102° F Rationale: Clients with basilar skull fractures are at high risk for infection of the brain, as indicated by an increased oral temperature, because the fracture leaves the meninges open to bacterial invasion. Clients may experience options C and D, but these findings do not pose as great a life-threatening risk as infection. Jugular distention is not a typical complication of basal skull fractures.

A client with metastatic cancer who was taking hydromorphone (Dilaudid) PO at home is now receiving the medication IV while in the hospital. To evaluate if the client is receiving an equianalgesic dose of the Dilaudid, what assessment should the nurse complete? A. Respiratory rate B. Pain scale C. Level of consciousness D. Blood pressure

B. Pain scale

The nurse has received funding to design a health promotion project for African-American women who are at risk for developing breast cancer. Which resource is most important in designing this program? A. A lasting of African-American women who live in the community B. Participation of community leaders in planning the program C. Morbidity data for breast cancer in women of all races D. Techical assistacne to produce a video on breast self-examination

B. Participation of community leaders in planning the program

A client on telemetry has a pattern of uncontrolled atrial fibrillation with a rapid ventricular response. Based on this finding, the nurse anticipates assisting the physician with which treatment? A. Administer lidocaine,75 mg intravenous push. B. Perform synchronized cardioversion. C. Defibrillate the client as soon as possible. D. Administer atropine, 0.4 mg intravenous push.

B. Perform synchronized cardioversion. Rationale: With uncontrolled atrial fibrillation, the treatment of choice is synchronized cardioversion (B) to convert the cardiac rhythm back to normal sinus rhythm. (A) is a medication used for ventricular dysrhythmias. (C) is not for a client with atrial fibrillation; it is reserved for clients with life-threatening dysrhythmias, such as ventricular fibrillation and unstable ventricular tachycardia. (D) is the drug of choice in symptomatic sinus bradycardia, not atrial fibrillation.

A clients morning laboratory test results include leukocytes 3,500/mm^3 or 3.5x10^9/L (SI). Based on the laboratory result, which complaint is this client most likely to report to the nurse? A. Inability to walk without shortness of breath B. Persistent cough with yellow-colored sputum C. Superficial cuts do not readily stop bleeding D. A red streak and pain in right calf muscle

B. Persistent cough with yellow-colored sputum The leukocyte level is below normal (WBC 5,000-10,000). WBC are integral to the body's response to infection, so the client's report of yellow-colored sputum is characteristic of purulent sputum, a sign of infection

The nurse is planning care for a client with diabetes mellitus who has gangrene of the toes to the midfoot. Which goal should be included in this client's plan of care? A. Restore skin integrity. B. Prevent infection. C. Promote healing. D. Improve nutrition.

B. Prevent infection. Rationale: The nurse is planning care for a client with diabetes mellitus who has gangrene of the toes to the midfoot. Which goal should be included in this client's plan of care?Rationale: The prevention of infection is a priority goal for this client. Gangrene is the result of necrosis (tissue death). If infection develops, there is insufficient circulation to fight the infection and the infection can result in osteomyelitis or sepsis. Because tissue death has already occurred, options A and C are unattainable goals. Option D is important but of less priority than option B.

While performing a skin inspection for a female adult client, the nurse observes a rash that is well circumscribed, has silvery scales and plaques, and is located on the elbows and knees. These assessment findings are likely to indicate which condition? A. Tinea corporis B. Psoriasis C. Herpes Zoster D. Drug reaction

B. Psoriasis

17. The nurse is caring for a 73-year-old patient who underwent a left total knee arthroplasty. On the third postoperative day, the patient complains of shortness of breath, slight chest pain, and that "something is wrong." Temperature is 98.4o F, blood pressure 130/88, respirations 36, and oxygen saturation 91% on room air. Which of the following should the nurse first suspect as the etiology of this episode? A. Septic embolus from the knee joint B. Pulmonary embolus from deep vein thrombosis C. New onset of angina pectoris D. Pleural effusion related to positioning in the operating room

B. Pulmonary embolus from deep vein thrombosis The patient presents the classic symptoms of pulmonary embolus: acute onset of symptoms, tachypnea, shortness of breath, and chest pain.

The nurse is caring for a 73-year-old patient who underwent a left total knee arthroplasty. On the third postoperative day, the patient complains of shortness of breath, slight chest pain, and that "something is wrong." Temperature is 98.4o F, blood pressure 130/88, respirations 36, and oxygen saturation 91% on room air. Which of the following should the nurse first suspect as the etiology of this episode? A. Septic embolus from the knee joint B. Pulmonary embolus from deep vein thrombosis C. New onset of angina pectoris D. Pleural effusion related to positioning in the operating room

B. Pulmonary embolus from deep vein thrombosis The patient presents the classic symptoms of pulmonary embolus: acute onset of symptoms, tachypnea, shortness of breath, and chest pain.

The nurse is caring for six clients on a medical-surgical unit. Which interventions should the nurse delegate to the unlicensed assistive personnel (UAP)? (select all that apply) A. Assess daily weights for trends B. Record vital signs every four hours C. Assist with ambulation as prescribed D. Provide oral care after meals E. Monitor for signs of dehydration

B. Record vital signs every four hours C. Assist with ambulation as prescribed D. Provide oral care after meals

An older female client with dementia is transferred from a long term care unit to an acute care unit. The client's children express concern that their mother's confusion is worsening. How should the nurse respond? A. "It is to be expected that older people will experience progressive confusion." B. "Confusion in an older person often follows relocation to new surroundings." C. "The dementia is progressing rapidly, but we will do everything we can to keep your mother safe." D. "The acute care staff is not as experienced as the long-term care staff at dealing with dementia."

B. Relocation often results in confusion among older clients and is stressful to clients of all ages. (A) is an inaccurate stereotype. (C) is most likely false there are many factors that cause increased temporary confusion. (D) may be true but does not offer the family a sense of security about the care.

A female client with a nasogastric tube attached to low suction states that she is nauseated. The nurse assesses that there has been no drainage through the nasogastric tube in the last 2 hours. Which action should the nurse take first? A. Irrigate the nasogastric tube with sterile normal saline. B. Reposition the client on her side. C. Advance the nasogastric tube 5 cm. D. Administer an intravenous antiemetic as prescribed.

B. Reposition the client on her side. Rationale: The immediate priority is to determine if the tube is functioning correctly, which would then relieve the client's nausea. The least invasive intervention, repositioning the client (B), should be attempted first, followed by (A and C), unless either of these interventions is contraindicated. If these measures are unsuccessful, the client may require (D).

A female client with a nasogastric tube attached to low suction states that she is nauseated. The nurse assesses that there has been no drainage through the nasogastric tube in the last 2 hours. Which action should the nurse take first? A.Irrigate the nasogastric tube with sterile normal saline. B.Reposition the client on her side. C.Advance the nasogastric tube 5 cm. D.Administer an intravenous antiemetic as prescribed.

B. Reposition the client on her side. Rationale: The immediate priority is to determine if the tube is functioning correctly, which would then relieve the client's nausea. The least invasive intervention, repositioning the client, should be attempted first, followed by options A and C, unless either of these interventions is contraindicated. If these measures are unsuccessful, the client may require option D.

The nurse is managing clients who are mechanically ventilated. The client with which assessment finding requires the most immediate intervention by the nurse? A. Audible voice when client is trying to communicate B. Restrained and restless with a low volume alarm C. High pressure alarm when client is coughing D. Diminished breath sounds in the right posterior base

B. Restrained and restless with a low volume alarm

The nurse is interviewing a client who is taking interferon-alfa-2a (Roferon-A) and ribavirin (Virazole) combination therapy for hepatitis C. The client reports experiencing overwhelming feelings of depression. Which action should the nurse implement first? A. Recommend mental health counseling. B. Review the medication actions and interactions. C. Assess for the client's daily activity level. D. Provide information regarding a support group.

B. Review the medication actions and interactions. Rationale: Interferon-alfa-2a and ribavirin combination therapy can cause severe depression (B); therefore, it is most important for the nurse to review the medication effects and report these to the health care provider. (A, C, and D) might be implemented after the physiologic aspects of the situation have been assessed.

The nurse observes ventricular fibrillation on telemetry and, on entering the client's bathroom, finds the client unconscious on the floor. Which intervention should the nurse implement first? A. Administer an antidysrhythmic medication. B. Start cardiopulmonary resuscitation. C. Prepare for mechanical ventilation. D. Assess the client's pulse oximetry.

B. Start cardiopulmonary resuscitation. Rationale: Ventricular fibrillation is a life-threatening dysrhythmia, and CPR should be started immediately until the crash cart arrives. Options A and C are appropriate, but CPR is the priority action until a defibrillator is available, which is the most effective treatment for ventricular fibrillation. The client is dying, and option D does not address the seriousness of this situation.

A newborn is apnea for 20 seconds. What action should the nurse implement? A. Place oxygen cannula near the nares B. Stimulate by gently rubbing infant's trunk C. Begin cardiopulmonary resuscitation D. Suction the infant's nano-oropharynx

B. Stimulate by gently rubbing infant's trunk

What is the most important nursing priority for a client who has been admitted for a possible kidney stone? A. Reducing dairy products in the diet B. Straining all urine C. Measuring intake and output D. Increasing fluid intake

B. Straining all urine Rationale: Straining all urine (B) is the most important nursing action to take in this case. Encouraging fluid intake (D) is important for any client who may have a kidney stone, but is even more important to strain all urine. Straining urine will enable the nurse to determine when the kidney stone has been passed and may prevent the need for surgery. (C) is not the highest priority action. (A) is usually not recommended until the stone is obtained and the content of the stone is determined. Even then, dietary restrictions are controversial.

Which description of symptoms is characteristic of a client diagnosed with trigeminal neuralgia (tic douloureux)? A. Tinnitus, vertigo, and hearing difficulties B. Sudden, stabbing, severe pain over the lip and chin C. Unilateral facial weakness and paralysis D. Difficulty in chewing, talking, and swallowing

B. Sudden, stabbing, severe pain over the lip and chin Rationale: Trigeminal neuralgia is characterized by paroxysms of pain, similar to an electric shock, in the area innervated by one or more branches of the trigeminal nerve (cranial V) (B). (A) would be characteristic of Ménière's syndrome (cranial nerve VIII). (C) would be characteristic of Bell's palsy (cranial nerve VII). (D) would be characteristic of disorders of the hypoglossal (cranial nerve XII).

An unlicensed assistive personnel (UAP) is teamed with a nurse who is caring for four clients. Client A is admitted to the medical unit for heart failure (HF) Client B has just returned from surgery Client C, with chronic obstructive pulmonary disease (COPD), needs new oxygen tubing Client D is waiting to be discharged Which activity should the nurse delegate to the UAP as having the highest priority? A. Obtain the daily weight for the client with HF B. Take vital signs of the client who just returned form surgery C. Gather equipment for oxygen tubing change for the client with COPD D. Assist client into the wheelchair for discharge

B. Take vital signs of the client who just returned form surgery

A male client returns to the mental health clinic for assistance with his anxiety reaction that is manifested by a rapid heartbeat, sweating, shaking, and nausea while driving over the bay bridge. What action in the treatment plan should the nurse implement? A.Tell client to drive over the bridge until fear is manageable B. Teach client to listen to music or audio books while driving C. Encourage client to have spouse drive in stressful places D. Recommend that the client avoid driving over the bridge

B. Teach client to listen to music or audio books while driving

A client with a prescription for "do not resuscitate" (DNR) begins to manifest signs of impending death. After notifying the family of the client's status, what priority action should the nurse implement? A. The healthcare provider should be notified of the client's status B. The client's need for pain medication should be determined C. The chaplain should be requested to come to the client's bedside D. The client's signs of impending death should be documented

B. The client's need for pain medication should be determined

A 58-year-old client who has no health problems asks the nurse about receiving the pneumococcal vaccine (Pneumovax). Which statement given by the nurse would offer the client accurate information about this vaccine? A. The vaccine is given annually before the flu season to those older than 50 years. B. The immunization is administered once to older adults or those at risk for illness. C. The vaccine is for all ages and is given primarily to those persons traveling overseas to areas of infection. D. The vaccine will prevent the occurrence of pneumococcal pneumonia for up to 5 years.

B. The immunization is administered once to older adults or those at risk for illness. Rationale: It is usually recommended that persons older than 65 years and those with a history of chronic illness should receive the vaccine once in their lifetime (B). Some recommend receiving the vaccine at 50 years of age. The influenza vaccine is given once a year, not Pneumovax (A). Although the vaccine might be given to a person traveling overseas, that is not the main rationale for administering the vaccine (C). The vaccine is usually given once in a lifetime (D), but with immunosuppressed clients or clients with a history of pneumonia, revaccination is sometimes required.

A 55-year-old male client is admitted to the coronary care unit having suffered an acute myocardial infarction (MI). Within 24 hours of the occurrence, the nurse can expect to find which systemic sign? A. Elevated serum amylase level B. Elevated CM-MB level C. Prolonged prothrombin time (PT) D. Elevated serum BUN and creatinine

B. Tissue damage in the myocardium causes the release of cardiac enzymes into the blood system. An elevated CM-MB is a recognized indicator of an MI. It peaks 12 - 24 hours and returns to normal within 48 - 78 hours. (A) would indicate pancreatitis or a gastric disorder. (D) Although an elevated BUN might be related to an acute MI it is usually associated with dehydration, high protein intake or gastrointestinal bleeding and creatine levels indicate renal damage. (C) Indicates effective anticoagulation therapy.

A client slips and falls while getting out of bed and the charge nurse instructs the nurse who is caring for the client to complete an incident report. What is the main purpose in having the nurse complete the incident report? A. To ensure that the nurse caring for the client takes responsibility for the incident. B. To provide computer documentation of the incident as a basis for further investigation C. To protect the nurse and the hospital against charges of malpractice D. To reprimand the nurse for not providing safe care for the client

B. To provide computer documentation of the incident as a basis for further investigation

Which description of symptoms is characteristic of a client with diagnosed with trigeminal neuralgia (tic douloureux)? A. Tinnitus, vertigo, and hearing difficulties. B. Sudden, stabbing, severe pain over the lip and chin. C. Unilateral facial weakness and paralysis. D. Difficulty in talking, chewing, and swallowing.

B. Trigeminal neuralgia is characterized by paroxysms of pain, similar to an electric shock, in the area innervated by one or more branches of the trigeminal nerve. A. Characteristic of Meniere's C. Characteristic of Bell palsey D. Characteristic of disorders of the hypoglossal (12th cranial nerve)

A client with dementia who is cared for at home by her husband becomes increasingly confused in the evening. Her husband reports to the home health care nurse that his wife often believes that she is waiting for the oil to be changed in her car and insists on leaving. Which recommendation should the nurse provide to this husband? A. Remind his wife that she is just a little confused B. Try to interest his wife in a different activity C. Give his wife a PRN dose of haloperidol (Haldol) D. Show his wife her car that is still in the garage

B. Try to interest his wife in a different activity

A nurse is assisting an 82-year-old client with ambulation and is concerned that the client may fall. Which area contains the older person's center of gravity? A. Head and neck B. Upper torso C. Bilateral arms D. Feet and legs

B. Upper torso Rationale: Stooped posture results in the upper torso (B) becoming the center of gravity for older persons. The center of gravity for adults is the hips. However, as a person grows older, a stooped posture is common because of changes caused by osteoporosis and normal bone degeneration. Furthermore, the knees, hips, and elbows flex. The head and neck (A) and feet and legs (D) are not the center of gravity in the older adult. Although the arms (C) comprise a part of the upper torso, they do not reflect the best and most complete answer.

The nurse plans to collect a 24-hour urine specimen for a creatine clearance test. Which instruction should the nurse provide to the adult male client? A. Urinate immediately into a urinal, and the lab will collect the specimen every 6 hours, for the next 24 hours B. Urinate at a specified time, discard this urine, and collect all subsequent urine during the next 24 hours C. For the next 24 hours, notify nurse when the bladder is full, and the nurse will collect catheterized specimens D. Cleanse around the meatus, discard first portion of voiding, and collect the rest in a sterile bottle

B. Urinate at a specified time, discard this urine, and collect all subsequent urine during the next 24 hours

48. Before discharge, the nurse discusses activity levels with a 61-year-old patient with COPD and pneumonia. Which of the following exercise goals is most appropriate once the patient is fully recovered from this episode of illness? A. Slightly increase activity over the current level. B. Walk for 20 minutes a day, keeping the pulse rate less than 130 beats per minute. C. Limit exercise to activities of daily living to conserve energy. D. Swim for 10 min/day, gradually increasing to 30 min/day.

B. Walk for 20 minutes a day, keeping the pulse rate less than 130 beats per minute. The patient will benefit from mild aerobic exercise that does not stress the cardiorespiratory system. The patient should be encouraged to walk for 20 min/day, keeping the pulse rate less than 75% to 80% of maximum heart rate (220 minus patient's age).

Before discharge, the nurse discusses activity levels with a 61-year-old patient with COPD and pneumonia. Which of the following exercise goals is most appropriate once the patient is fully recovered from this episode of illness? A. Slightly increase activity over the current level. B. Walk for 20 minutes a day, keeping the pulse rate less than 130 beats per minute. C. Limit exercise to activities of daily living to conserve energy. D. Swim for 10 min/day, gradually increasing to 30 min/day.

B. Walk for 20 minutes a day, keeping the pulse rate less than 130 beats per minute. The patient will benefit from mild aerobic exercise that does not stress the cardiorespiratory system. The patient should be encouraged to walk for 20 min/day, keeping the pulse rate less than 75% to 80% of maximum heart rate (220 minus patient's age).

The nurse is assessing a male client with acute pancreatitis. Which finding requires the most immediate intervention by the nurse? A. The client's amylase level is three times higher than the normal level. B. While the nurse is taking the client's blood pressure, he has a carpal spasm. C. On a 1 to 10 scale, the client tells the nurse that his epigastric pain is at 7. D. The client states that he will continue to drink alcohol after going home.

B. While the nurse is taking the client's blood pressure, he has a carpal spasm. Rationale: A positive Trousseau sign (B) indicates hypocalcemia and always requires further assessment and intervention, regardless of the cause (40% to 75% of those with acute pancreatitis experience hypocalcemia, which can have serious, systemic effects). A key diagnostic finding of pancreatitis is serum amylase and lipase levels that are two to five times higher than the normal value (A). Severe boring pain is an expected symptom for this diagnosis (C), but dealing with the hypocalcemia is a priority over administering an analgesic. Long-term planning and teaching (D) do not have the same immediate importance as a positive Trousseau sign.

Debilitating anginal pain can be decreased in some clients by the administration of beta-blocking agents such as nadolol (Corgard). Which client requires the nurse to use extreme caution when administering Corgard? A. A 56-year-old air traffic controller who had bypass surgery 2 years ago. B. A 47-year-old kindergarten teacher diagnosed with asthma 40 years ago C. A 52-year-old unemployed stock broker who refuses treatment for alcoholism D. A 60-year-old retired librarian who takes a diuretic daily for hypertension.

B. asthma must be carefully monitored because beta blockers because it can induce cardiogenic shock and reduce bronchodilation efforts. (A & D) this medication is indicated and (C) it is not contraindicated.

1. A nurse is reviewing the hematologic test results for a patient in whom the hematocrit (Hct) is reported at a reading of 30%. Based on this result, the nurse should interpret that the patient A. is susceptible to bleeding disorders. B. has fewer red blood cells than normal. C. is experiencing an inflammatory response. D. is experiencing an acute hemolytic crisis.

B. has fewer red blood cells than normal.The Hct is the measure of the volume of red blood cells in whole blood expressed as a percentage. This test is useful in the diagnosis of anemia, polycythemia, and abnormal hydration states. Patients who are susceptible to bleeding disorders likely will have a low platelet count. The inflammatory response may best be evaluated by examination of results that include the white blood cell count with differential analysis. Acute hemolytic crisis develops in patients receiving blood components in which incompatibility occurs or in patients with bleeding disorders or conditions that promote cellular damage, such as damage associated with shock.

A nurse is reviewing the hematologic test results for a patient in whom the hematocrit (Hct) is reported at a reading of 30%. Based on this result, the nurse should interpret that the patient A. is susceptible to bleeding disorders. B. has fewer red blood cells than normal. C. is experiencing an inflammatory response. D. is experiencing an acute hemolytic crisis.

B. has fewer red blood cells than normal.The Hct is the measure of the volume of red blood cells in whole blood expressed as a percentage. This test is useful in the diagnosis of anemia, polycythemia, and abnormal hydration states. Patients who are susceptible to bleeding disorders likely will have a low platelet count. The inflammatory response may best be evaluated by examination of results that include the white blood cell count with differential analysis. Acute hemolytic crisis develops in patients receiving blood components in which incompatibility occurs or in patients with bleeding disorders or conditions that promote cellular damage, such as damage associated with shock.

18. Nursing interventions for the patient with aplastic anemia are directed toward the prevention of the complications of A. fatigue and dyspnea. B. hemorrhage and infection. C. thromboemboli and gangrene. D. cardiac arrhythmias and heart failure.

B. hemorrhage and infection. Hemorrhage from thrombocytopenia and infection from neutropenia are the greatest risks for the patient with aplastic anemia. The patient will experience fatigue from anemia, but bleeding and infection are the major causes of death in aplastic anemia.

3. In preparing the preoperative teaching plan for a patient who is to undergo a total laryngectomy, a nurse should give highest priority to the A. tracheostomy being in place for 2 to 3 days. B. patient's not being able to speak normally again. C. insertion of a gastrostomy feeding tube during surgery. D. patient's not being able to perform deep-breathing exercises.

B. patient's not being able to speak normally again. Patients who have a total laryngectomy have a permanent tracheostomy and will need to learn how to speak using alternative methods, such as an artificial larynx. The tracheostomy will be permanent to allow normal breathing patterns and air exchange. After surgery, the patient's nutrition is supplemented with enteral feedings, and when the patient can swallow secretions, oral feedings can begin. Deep-breathing exercises should be performed with the patient at least every 2 hours to prevent further pulmonary complications.

In preparing the preoperative teaching plan for a patient who is to undergo a total laryngectomy, a nurse should give highest priority to the A. tracheostomy being in place for 2 to 3 days. B. patient's not being able to speak normally again. C. insertion of a gastrostomy feeding tube during surgery. D. patient's not being able to perform deep-breathing exercises.

B. patient's not being able to speak normally again. Patients who have a total laryngectomy have a permanent tracheostomy and will need to learn how to speak using alternative methods, such as an artificial larynx. The tracheostomy will be permanent to allow normal breathing patterns and air exchange. After surgery, the patient's nutrition is supplemented with enteral feedings, and when the patient can swallow secretions, oral feedings can begin. Deep-breathing exercises should be performed with the patient at least every 2 hours to prevent further pulmonary complications.

36. The resurgence in TB resulting from the emergence of multidrug-resistant strains of Mycobacterium tuberculosis is primarily the result of A. a lack of effective means to diagnose TB. B. poor compliance with drug therapy in patients with TB. C. the increased population of immunosuppressed individuals with AIDS. D. indiscriminate use of antitubercular drugs in treatment of other infections.

B. poor compliance with drug therapy in patients with TB. Drug-resistant strains of TB have developed because TB patients' compliance to drug therapy has been poor and there has been general decreased vigilance in monitoring and follow-up of TB treatment. Antitubercular drugs are almost exclusively used for TB infections. TB can be effectively diagnosed with sputum cultures. The incidence of TB is at epidemic proportions in patients with HIV, but this does not account for drug-resistant strains of TB.

The resurgence in TB resulting from the emergence of multidrug-resistant strains of Mycobacterium tuberculosis is primarily the result of A. a lack of effective means to diagnose TB. B. poor compliance with drug therapy in patients with TB. C. the increased population of immunosuppressed individuals with AIDS. D. indiscriminate use of antitubercular drugs in treatment of other infections.

B. poor compliance with drug therapy in patients with TB. Drug-resistant strains of TB have developed because TB patients' compliance to drug therapy has been poor and there has been general decreased vigilance in monitoring and follow-up of TB treatment. Antitubercular drugs are almost exclusively used for TB infections. TB can be effectively diagnosed with sputum cultures. The incidence of TB is at epidemic proportions in patients with HIV, but this does not account for drug-resistant strains of TB.

2. A patient admitted to the emergency department with tension pneumothorax and mediastinal shift following an automobile crash is most likely to exhibit A. bradycardia. B. severe hypotension. C. mediastinal flutter. D. a sucking chest wound.

B. severe hypotension. Mediastinal shift may cause compression of the lung in the direction of the shift and compression, traction, torsion, or kinking of the great vessels. Blood return to the heart is dangerously impaired and causes a subsequent decrease in cardiac output and blood pressure. Tachycardia is a clinical manifestation of tension pneumothorax. An uncovered opened pneumothorax is associated with a sucking chest wound and mediastinal flutter.

A patient admitted to the emergency department with tension pneumothorax and mediastinal shift following an automobile crash is most likely to exhibit A. bradycardia. B. severe hypotension. C. mediastinal flutter. D. a sucking chest wound.

B. severe hypotension. Mediastinal shift may cause compression of the lung in the direction of the shift and compression, traction, torsion, or kinking of the great vessels. Blood return to the heart is dangerously impaired and causes a subsequent decrease in cardiac output and blood pressure. Tachycardia is a clinical manifestation of tension pneumothorax. An uncovered opened pneumothorax is associated with a sucking chest wound and mediastinal flutter.

25. A 75-year-old obese patient who is snoring loudly and having periods of apnea several times each night is most likely experiencing A. narcolepsy. B. sleep apnea. C. sleep deprivation. D. paroxysmal nocturnal dyspnea.

B. sleep apnea. Sleep apnea is most common in obese patients. Typical symptoms include snoring and periods of apnea. Narcolepsy is when a patient falls asleep unexpectedly. Sleep deprivation could result from sleep apnea. Paroxysmal nocturnal dyspnea occurs when a patient has shortness of breath during the night.

A 75-year-old obese patient who is snoring loudly and having periods of apnea several times each night is most likely experiencing A. narcolepsy. B. sleep apnea. C. sleep deprivation. D. paroxysmal nocturnal dyspnea.

B. sleep apnea. Sleep apnea is most common in obese patients. Typical symptoms include snoring and periods of apnea. Narcolepsy is when a patient falls asleep unexpectedly. Sleep deprivation could result from sleep apnea. Paroxysmal nocturnal dyspnea occurs when a patient has shortness of breath during the night.

What is the most important nursing priority for a client who has been admitted for a possible kidney stone? A. Reducing dairy products in the diet B. Straining all urine C. Measuring intake and output D. Increasing fluid intake

B.Straining all urine Rationale: Straining all urine is the most important nursing action to take in this case. Encouraging fluid intake is important for any client who may have a kidney stone, but it is even more important to strain all urine. Straining urine will enable the nurse to determine when the kidney stone has been passed and may prevent the need for surgery. Option C is not the highest priority action. Option A is usually not recommended until the stone is obtained and the content of the stone is determined. Even then, dietary restrictions are controversial.

An older male client comes to the outpatient clinic complaining of pain in his left calf. The nurse notices a reddened area on the calf of his right leg that is warm to the touch, and the nurse suspects that the client may have thrombophlebitis. Which additional assessment is most important for the nurse to perform? A.Measure the client's calf circumference. B. Auscultate the client's breath sounds. C.Observe for ecchymosis and petechiae. D. Obtain the client's blood pressure.

B.Auscultate the client's breath sounds. Rationale: All these techniques provide useful assessment data. The most important is to auscultate the client's breath sounds because the client may have a pulmonary embolus secondary to the thrombophlebitis. Option A may provide data that support the nurse's suspicion of thrombophlebitis. Option C is the least helpful assessment because bruising is not a typical finding associated with thrombophlebitis. Option D is always useful in evaluating the client's response to a problem but is of less immediate priority than breath sound auscultation.

A client diagnosed with chronic kidney disease (CKD) 2 years ago is regularly treated at a community hemodialysis facility. Before his scheduled dialysis treatment, which electrolyte imbalance should the nurse anticipate? A. Hypophosphatemia B. Hypocalcemia C. Hyponatremia D. Hypokalemia

B.Hypocalcemia Rationale: Hypocalcemia develops in CKD because of chronic hyperphosphatemia, not option A. Increased phosphate levels cause the peripheral deposition of calcium and resistance to vitamin D absorption needed for calcium absorption. Prior to dialysis, the nurse would expect to find the client hypernatremic and hyperkalemic, not with option C or D.

The nurse is caring for a client who is one day post-acute myocardial infarction. The client is receiving oxygen at 2 L/min via nasal cannula and has a peripheral saline lock. The nurse notes that the client is having eight premature ventricular contractions (PVCs) per minute. Which intervention should the nurse implement first? A.Obtain an IV pump for antiarrhythmic infusion. B.Increase the client's oxygen flow rate. C.Prepare for immediate countershock. D.Gather equipment for endotracheal intubation.

B.Increase the client's oxygen flow rate. Rationale: Increasing the oxygen flow rate provides more oxygen to the client's myocardium and may decrease myocardial irritability as manifested by the frequent PVCs. Option A can be delegated and is a lower priority action than option B. Defibrillation may eventually be necessary, but option C is not the immediate treatment for frequent PVCs. Option D may become necessary if the client stops breathing but is not indicated at this time.

A 43-year-old homeless, malnourished female client with a history of alcoholism is transferred to the ICU. She is placed on telemetry, and the rhythm strip shown is obtained. The nurse palpates a heart rate of 160 beats/min, and the client's blood pressure is 90/54 mm Hg. Based on these findings, which IV medication should the nurse administer? A. Amiodarone (Cordarone) B. Magnesium sulfate C. Lidocaine (Xylocaine) D. Procainamide (Pronestyl)

B.Magnesium sulfate Rationale: Because the client has chronic alcoholism, she is likely to have hypomagnesemia. Option B is the recommended drug for torsades de pointes, which is a form of polymorphic ventricular tachycardia (VT) usually associated with a prolonged QT interval that occurs with hypomagnesemia. Options A and D increase the QT interval, which can cause the torsades to worsen. Option C is the antiarrhythmic of choice in most cases of drug-induced monomorphic VT, not torsades.

A tornado warning alarm has been activated at the local hospital. Which action should the charge nurse working on a surgical unit implement first? A. Instruct the nursing staff to close all window blinds and curtains in clients' rooms. B. Move clients and visitors into the hallways and close all doors to clients' rooms. C. Visually confirm the location of the tornado by checking the windows on the unit. D. Assist all visitors with evacuation down the stairs in a calm and orderly manner.

B.Move clients and visitors into the hallways and close all doors to clients' rooms. Rationale: In the event of a tornado, all persons should be moved into the hallways, away from windows, to prevent flying debris from causing injury. Although option A may help decrease the amount of flying debris, it is not safe to leave clients in rooms with closed blinds; option B is a higher priority at this time. Hospital staff should stay away from windows to avoid injury and should focus on client evacuation into hallways rather than option C. Option D is not the first action that should be taken.

A client on telemetry has a pattern of uncontrolled atrial fibrillation with a rapid ventricular response. Based on this finding, the nurse anticipates assisting the physician with which treatment? A.Administer lidocaine, 75 mg intravenous push. B.Perform synchronized cardioversion. C.Defibrillate the client as soon as possible. D.Administer atropine, 0.4 mg intravenous push.

B.Perform synchronized cardioversion. Rationale: With uncontrolled atrial fibrillation, the treatment of choice is synchronized cardioversion to convert the cardiac rhythm back to normal sinus rhythm. Option A is a medication used for ventricular dysrhythmias. Option C is not for a client with atrial fibrillation; it is reserved for clients with life-threatening dysrhythmias, such as ventricular fibrillation and unstable ventricular tachycardia. Option D is the drug of choice in symptomatic sinus bradycardia, not atrial fibrillation.

The nurse is interviewing a client who is taking interferon-alfa-2a (Roferon-A) and ribavirin (Virazole) combination therapy for hepatitis C. The client reports experiencing overwhelming feelings of depression. Which action should the nurse implement first? A. Recommend mental health counseling. B. Review the medication actions and interactions. C. Assess for the client's daily activity level. D. Provide information regarding a support group

B.Review the medication actions and interactions. Rationale: Interferon-alfa-2a and ribavirin combination therapy can cause severe depression; therefore, it is most important for the nurse to review the medication effects and report these to the health care provider. Options A, C, and D might be implemented after the physiologic aspects of the situation have been assessed.

The nurse teaches a client with type 2 diabetes nutritional strategies to decrease obesity. Which food items chosen by the client indicate understanding of the teaching? (Select all that apply.) A. White bread B. Salmon C.Broccoli D. Whole milk E. Banana

B.Salmon C.Broccoli E.Banana Rationale: Options B, C, and E provide fresh fruits, lean meats and fish, vegetables, whole grains, and low-fat dairy products. All are recommended by the American Diabetes Association (ADA) and are a part of the My Plate guidelines recommended by the U.S. Department of Agriculture (USDA). Whole milk is high in fat and is not recommended by the ADA. White bread is milled, a process that removes the essential nutrients. It should be avoided for weight loss and is a poor choice for the client with diabetes.

Which description of symptoms is characteristic of a client diagnosed with trigeminal neuralgia (tic douloureux)? A.Tinnitus, vertigo, and hearing difficulties B.Sudden, stabbing, severe pain over the lip and chin C.Unilateral facial weakness and paralysis D.Difficulty in chewing, talking, and swallowing

B.Sudden, stabbing, severe pain over the lip and chin Rationale: Trigeminal neuralgia is characterized by paroxysms of pain, similar to an electric shock, in the area innervated by one or more branches of the trigeminal nerve (cranial V). Option A would be characteristic of Ménière syndrome (cranial nerve VIII). Option C would be characteristic of Bell palsy (cranial nerve VII). Option D would be characteristic of disorders of the hypoglossal (cranial nerve XII).

A 58-year-old client who has no health problems asks the nurse about receiving the pneumococcal vaccine. Which statement given by the nurse would offer the client accurate information about this vaccine? A. The vaccine is given annually before the flu season to those older than 50 years. B. The immunization is administered once to older adults or those at risk for illness. C.The vaccine is for all ages and is given primarily to those persons traveling overseas to areas of infection. D. The vaccine will prevent the occurrence of pneumococcal pneumonia for up to 5 years.

B.The immunization is administered once to older adults or those at risk for illness. Rationale: It is usually recommended that persons older than 65 years and those with a history of chronic illness should receive the vaccine once in their lifetime. Some recommend receiving the vaccine at 50 years of age. The influenza vaccine is given once a year. Although the vaccine might be given to a person traveling overseas, that is not the main rationale for administering the vaccine. The vaccine is usually given once in a lifetime, but with immunosuppressed clients or clients with a history of pneumonia, revaccination is sometimes required.

A hospitalized client is receiving nasogastric tube feedings via a small-bore tube and a continuous pump infusion. He begins to cough and produces a moderate amount of white sputum. Which action should the nurse take first? A.Auscultate the client's breath sounds. B.Turn off the continuous feeding pump. C.Check placement of the nasogastric tube. D.Measure the amount of residual feeding.

B.Turn off the continuous feeding pump. Rationale: A productive cough may indicate that the feeding has been aspirated. The nurse should first stop the feeding to prevent further aspiration. Options A, C, and D should all be performed before restarting the tube feeding if no evidence of aspiration is present and the tube is in place.

A nurse is assisting an 82-year-old client with ambulation and is concerned that the client may fall. Which area contains the older person's center of gravity? A. Head and neck B. Upper torso C. Bilateral arms D. Feet and legs

B.Upper torso Rationale: Stooped posture results in the upper torso becoming the center of gravity for older persons. The center of gravity for adults is the hips. However, as a person grows older, a stooped posture is common because of changes caused by osteoporosis and normal bone degeneration. Furthermore, the knees, hips, and elbows flex. The head and neck and feet and legs are not the center of gravity in the older adult. Although the arms comprise a part of the upper torso, they do not reflect the best and most complete answer.

The nurse is assessing a male client with acute pancreatitis. Which finding requires the most immediate intervention by the nurse? A. The client's amylase level is three times higher than the normal level. B.While the nurse is taking the client's blood pressure, he has a carpal spasm. C.On a 1 to 10 scale, the client tells the nurse that his epigastric pain is at 7. D.The client states that he will continue to drink alcohol after going home.

B.While the nurse is taking the client's blood pressure, he has a carpal spasm. Rationale: A positive Trousseau sign indicates hypocalcemia and always requires further assessment and intervention, regardless of the cause (40% to 75% of those with acute pancreatitis experience hypocalcemia, which can have serious, systemic effects). A key diagnostic finding of pancreatitis is serum amylase and lipase levels that are two to five times higher than the normal value. Severe boring pain is an expected symptom for this diagnosis, but dealing with the hypocalcemia is a priority over administering an analgesic. Long-term planning and teaching do not have the same immediate importance as a positive Trousseau sign.

40. Select all that apply. Atelectasis can be caused by A. long-term smoking. B. inadequate surfactant. C. localized airway obstruction. D. an increase in lung expansion. E. an increase in elastic recoil.

BCE The collapse of lung tissue has several causes, including reduced lung expansion, localized airway obstruction, inadequate surfactant, and an increase in elastic recoil. Smoking, although harmful, does not in itself cause atelectasis.

Select all that apply. Atelectasis can be caused by A. long-term smoking. B. inadequate surfactant. C. localized airway obstruction. D. an increase in lung expansion. E. an increase in elastic recoil.

BCE The collapse of lung tissue has several causes, including reduced lung expansion, localized airway obstruction, inadequate surfactant, and an increase in elastic recoil. Smoking, although harmful, does not in itself cause atelectasis.

21. Select all that apply. Which of the following are significant risk factors for leukemia? A. Being a longtime smoker B. Employment in an oil refinery C. History of hemophilia in parent D. Having Down syndrome E. Having a twin brother with leukemia F. Treatment with an alkylating agent = 3 years ago

BDEF Exposure to chemical agents, treatment with alkylating cancer drugs, leukemia in a sibling, and the patient's having Down syndrome are all risk factors for leukemia.

Select all that apply. Which of the following are significant risk factors for leukemia? A. Being a longtime smoker B. Employment in an oil refinery C. History of hemophilia in parent D. Having Down syndrome E. Having a twin brother with leukemia F. Treatment with an alkylating agent = 3 years ago

BDEF Exposure to chemical agents, treatment with alkylating cancer drugs, leukemia in a sibling, and the patient's having Down syndrome are all risk factors for leukemia.

A nurse is reinforcing teaching about urinary tract infections (UTIs) with a client. Which of the following manifestations should the nurse include?

Back pain

14. Which of the following foods is high in iron? A. Citrus fruits B. Milk products C. Yellow vegetables D. Green leafy vegetables

D. Green leafy vegetables Green leafy vegetables are high in iron. Foods cooked in iron pots and foods such as liver (the richest source), oysters, lean meats, kidney beans, whole wheat bread, kale, spinach, egg yolk, turnip tops, beet greens, carrots, apricots, and raisins are also high in iron.

Based on the analysis of the client's atrial fibrillation, the nurse should prepare the client for which treatment protocol? A) Diuretic therapy. B) Pacemaker implantation. C) Anticoagulation therapy. D) Cardiac catheterization.

C) Anticoagulation therapy. The client is experiencing atrial fibrillation, and the nurse should prepare the client for anticoagulation therapy (C) which should be prescribed before rhythm control therapies to prevent cardioembolic events which result from blood pooling in the fibrillating atria. (A, B, and D) are not indicated.

The nurse is teaching a client with maple syrup urine disease (MSUD), an autosomal recessive disorder, about the inheritance pattern. Which information should the nurse provide? A) This recessive disorder is carried only on the X chromosome. B) Occurrences mainly affect males and heterozygous females. C) Both genes of a pair must be abnormal for the disorder to occur. D) One copy of the abnormal gene is required for this disorder.

C) Both genes of a pair must be abnormal for the disorder to occur. Maple syrup urine disease (MSUD) is a type of autosomal recessive inheritance disorder in which both genes of a pair must be abnormal for the disorder to be expressed (C). MSUD is not an x-linked (A and B) dominant or recessive disorder or an autosomal dominant inheritance disorder. Both genes of a pair, not (D), must be present.

Which healthcare practice is most important for the nurse to teach a postmenopausal client? A) Wear layers of clothes if experiencing hot flashes. B) Use a water-soluble lubricant for vaginal dryness. C) Consume adequate foods rich in calcium. D) Participate in stimulating mental exercises.

C) Consume adequate foods rich in calcium. Bone density loss associated with osteoporosis increases at a more rapid rate when estrogen levels begin to fall, so the most important healthcare practice during menopause is ensuring an adequate calcium (C) intake to help maintain bone density and prevent osteoporosis. Although practices such as (A and B) may reduce some of the discomforts for a postmenopausal female, calcium intake is more important than comfort measures. Although social and mental exercises stimulate thought, there is no scientific evidence that mental exercises (D) prevent dementia or common forgetfulness associated with reduced hormonal levels.

A client is admitted to the medical intensive care unit with a diagnosis of myocardial infarction. The client's history indicates the infarction occurred ten hours ago. Which laboratory test result should the nurse expect this client to exhibit? A) Elevated LDH. B) Elevated serum amylase. C) Elevated CK-MB. D) Elevated hematocrit.

C) Elevated CK-MB. The cardiac isoenzyme CK-MB (C) is the most sensitive and most reliable indicator of myocardial damage of all the cardiac enzymes. It peaks within 12 to 20 hours after myocardial infarction (MI). (A) is a cardiac enzyme that peaks around 48 hours after an MI. (B) is expected with acute pancreatitis. (D) would be expected in a client with a fluid volume deficit, which is not a typical finding in MI.

Which postmenopausal client's complaint should the nurse refer to the healthcare provider? A) Breasts feel lumpy when palpated. B) History of white nipple discharge. C) Episodes of vaginal bleeding. D) Excessive diaphoresis occurs at night.

C) Episodes of vaginal bleeding. Postmenopausal vaginal bleeding (C) may be an indication of endometrial cancer, which should be reported to the healthcare provider. Compared to a new-onset of a single lump, breasts that feel lumpy (A) overall may be a normal variant or a finding consistent with nonmalignant fibrocystic disease. Up to 80% of women experience (B), depending on sexual stimulation or hormonal levels, and is no longer recommended as a reportable symptom when discovered during breast self-exam (BSE). The client may need further teaching concerning (D), a disturbing symptom, but it is not as important as (C).

The nurse is completing an admission interview and assessment on a client with a history of Parkinson's disease. Which question should provide information relevant to the client's plan of care? A) Have you ever experienced any paralysis of your arms or legs? B) Have you ever sustained a severe head injury? C) Have you ever been 'frozen' in one spot, unable to move? D) Do you have headaches, especially ones with throbbing pain?

C) Have you ever been 'frozen' in one spot, unable to move? Clients with Parkinson's disease frequently experience difficulty in initiating, maintaining, and performing motor activities. They may even experience being rooted to the spot and unable to move (C). Parkinson's disease does not cause (A). Parkinson's disease is not usually associated with (B), nor does it typically cause (D).

The nurse knows that lab values sometimes vary for the older client. Which data should the nurse expect to find when reviewing laboratory values of an 80-year-old male? A) Increased WBC, decreased RBC. B) Increased serum bilirubin, slightly increased liver enzymes. C) Increased protein in the urine, slightly increased serum glucose levels. D) Decreased serum sodium, an increased urine specific gravity.

C) Increased protein in the urine, slightly increased serum glucose levels. In older adults, the protein found in urine slightly rises probably as a result of kidney changes or subclinical urinary tract infections. The serum glucose increases slightly due to changes in the kidney. The specific gravity declines by age 80 from 1.032 to 1.024.

The nurse is working with a 71-year-old obese client with bilateral osteoarthritis (OA) of the hips. What recommendation should the nurse make that is most beneficial in protecting the client's joints? A) Increase the amount of calcium intake in the diet. B) Apply alternating heat and cold therapies. C) Initiate a weight-reduction diet to achieve a healthy body weight. D) Use a walker for ambulation to lessen weight-bearing on the hips.

C) Initiate a weight-reduction diet to achieve a healthy body weight. Achieving a healthy weight (C) is critical to protect the joints of clients with OA. Increasing the amount of calcium in the client's diet (A) will not protect hip joints from the effects of OA. Thermal therapies may lessen pain and stiffness from OA but are not protective of the joints (B). Assistive devices such as a walker may be beneficial to help avoid falls and assist in ambulation but are not protective against OA's effects (D).

During an interview with a client planning elective surgery, the client asks the nurse, "What is the advantage of having a preferred provider organization insurance plan?" Which response is best for the nurse to provide? A) Long-term relationships with healthcare providers are more likely. B) There are fewer healthcare providers to choose from than in an HMO plan. C) Insurance coverage of employees is less expensive to employers. D) An individual can become a member of a PPO without belonging to a group.

C) Insurance coverage of employees is less expensive to employers. The financial advantage of (C) is the feature of a PPO that is most relevant to the average consumer. The nurse must have knowledge about PPOs, which provide discounted rates to large employers who provide insurance coverage for their employees. In return, the insurance company receives a large pool of clients for their facilities. (A, B, and D) are not accurate representations of the PPO.

The nurse is planning care to prevent complication for a client with multiple myeloma. Which intervention is most important for the nurse to include? A) Safety precautions during activity. B) Assess for changes in size of lymph nodes. C) Maintain a fluid intake of 3 to 4 L per day. D) Administer narcotic analgesic around the clock.

C) Maintain a fluid intake of 3 to 4 L per day. Multiple myeloma is a malignancy of plasma cells that infiltrate bone causing demineralization and hypercalcemia, so maintaining a urinary output of 1.5 to 2 L per day requires an intake of 3 to 4 L (C) to promote excretion of serum calcium. Although the client is at risk for pathologic fractures due to diffuse osteoporosis, mobilization and weight bearing (A) should be encouraged to promote bone reabsorption of circulating calcium, which can cause renal complications. (B) is a component of ongoing assessment. Chronic pain management (D) should be included in the plan of care, but prevention of complications related to hypercalcemia is most important.

During lung assessment, the nurse places a stethoscope on a client's chest and instructs him/her to say "99" each time the chest is touched with the stethoscope. What should be the correct interpretation if the nurse hears the spoken words "99" very clearly through the stethoscope? A) This is a normal auscultatory finding. B) May indicate pneumothorax. C) May indicate pneumonia. D) May indicate severe emphysema.

C) May indicate pneumonia. This test (whispered pectoriloquy) demonstrates hyperresonance and helps determine the clarity with which spoken words are heard upon auscultation. Normally, the spoken word is not well transmitted through lung tissue, and is heard as a muffled or unclear transmission of the spoken word. Increased clarity of a spoken word is indicative of some sort of consolidation process (e.g., tumor, pneumonia) (C), and is not a normal finding (A). When lung tissue is filled with more air than normal, the voice sounds are absent or very diminished (e.g., pneumothorax, severe emphysema) (B and D).

The nurse is assessing a client who smokes cigarettes and has been diagnosed with emphysema. Which finding should the nurse expect this client to exhibit? A) A decreased total lung capacity. B) Normal arterial blood gases. C) Normal skin coloring. D) An absence of sputum.

C) Normal skin coloring. The differentiation between the "pink puffer" and the "blue bloater" is a well-known method of differentiating clients exhibiting symptoms of emphysema (normal color but puffing respirations) from those exhibiting symptoms of chronic bronchitis (edematous, cyanotic, shallow respirations) (C). Total lung capacity is increased in emphysema since these clients have hyperinflated lungs (A). Arterial blood gases are typically abnormal (B). (D) is indicative of bronchitis, while clients with emphysema usually have copious amounts of thick, white sputum.

A 67-year-old woman who lives alone is admitted after tripping on a rug in her home and fractures her hip. Which predisposing factor probably led to the fracture in the proximal end of her femur? A) Failing eyesight resulting in an unsafe environment. B) Renal osteodystrophy resulting from chronic renal failure. C) Osteoporosis resulting from hormonal changes. D) Cardiovascular changes resulting in small strokes which impair mental acuity.

C) Osteoporosis resulting from hormonal changes. The most common cause of a fractured hip in elderly women is osteoporosis, resulting from reduced calcium in the bones as a result of hormonal changes in later life (C). (A) may or may not have contributed to the accident, but it had nothing to do with the hip being involved. (B) is not a common condition of the elderly; it is common in chronic renal failure. (D) may occur in some people, but does not affect the fragility of the bones as osteoporosis does.

An elderly male client comes to the geriatric screening clinic complaining of pain in his left calf. The nurse notices a reddened area on the calf of his right leg which is warm to the touch and suspects it might be thrombophlebitis. Which type of pain should further confirm this suspicion? A) Pain in the calf awakening him from a sound sleep. B) Calf pain on exertion which stops when standing in one place. C) Pain in the calf upon exertion which is relieved by rest and elevating the extremity. D) Pain upon arising in the morning which is relieved after some stretching and exercise.

C) Pain in the calf upon exertion which is relieved by rest and elevating the extremity. Thrombophlebitis pain is relieved by rest and elevation of the extremity (C). It typically occurs with exercise at the site of the thrombus, and is aggravated by placing the extremity in a dependent position, such as standing in one place (B). (A and D) describe pain that is not common with thrombophlebitis.

In assessing a client diagnosed with primary hyperaldosteronism, the nurse expects the laboratory test results to indicate a decreased serum level of which substance? A) Sodium. B) Antidiuretic hormone. C) Potassium. D) Glucose.

C) Potassium. Clients with primary aldosteronism exhibit a profound decline in the serum levels of potassium (C) (hypokalemia)--hypertension is the most prominent and universal sign. (A) is normal or elevated, depending on the amount of water reabsorbed with the sodium. (B) is decreased with diabetes insipidus. (D) is not affected by primary aldosteronism.

A client who was in a motor vehicle collision was admitted to the hospital and the right knee was placed in skeletal traction. The nurse has documented this nursing diagnosis in the client's medical record: "Potential for impairment of skin integrity related to immobility from traction." Which nursing intervention is indicated based on this diagnosis statement? A) Release the traction q4h to provide skin care. B) Turn the client for back care while suspending traction. C) Provide back and skin care while maintaining the traction. D) Give back care after the client is released from traction.

C) Provide back and skin care while maintaining the traction. (C) indicates that back care is performed while traction is left intact, which is the correct intervention for maintaining skin integrity. Maintaining skin integrity and providing back care is difficult when a client is in traction, but it cannot be delayed until the client is removed from traction (D). The nurse should never release the traction (A and B).

A client with multiple sclerosis has experienced an exacerbation of symptoms, including paresthesias, diplopia, and nystagmus. Which instruction should the nurse provide? A) Stay out of direct sunlight. B) Restrict intake of high protein foods. C) Schedule extra rest periods. D) Go to the emergency room immediately.

C) Schedule extra rest periods. Exacerbations of the symptoms of MS occur most commonly as the result of fatigue and stress. Extra rest periods should be scheduled (C) to reduce the symptoms. (A, B, and D) are not necessary.

When preparing a client who has had a total laryngectomy for discharge, which instruction is most important for the nurse to include in the discharge teaching? A) Recommend that the client carry suction equipment at all times. B) Instruct the client to have writing materials with him at all times. C) Tell the client to carry a medic alert card stating that he is a total neck breather. D) Tell the client not to travel alone.

C) Tell the client to carry a medic alert card stating that he is a total neck breather. It is imperative that total neck breathers carry a medic alert notice (C) so that if they have a cardiac arrest, mouth-to-neck breathing can be done. Mouth-to-mouth resuscitation will not help them. They do not need to carry (A) nor refrain from (D). There are many alternative means of communication for clients who have had a laryngectomy; depending on (B) is probably the least effective. How do you know he can read and write?

Which of the following foods is high in iron? A. Citrus fruits B. Milk products C. Yellow vegetables D. Green leafy vegetables

D. Green leafy vegetables Green leafy vegetables are high in iron. Foods cooked in iron pots and foods such as liver (the richest source), oysters, lean meats, kidney beans, whole wheat bread, kale, spinach, egg yolk, turnip tops, beet greens, carrots, apricots, and raisins are also high in iron.

A client with early breast cancer receives the results of a breast biopsy and asks the nurse to explain the meaning of staging and the type of receptors found on the cancer cells. Which explanation should the nurse provide? A) Lymph node involvement is not significant. B) Small tumors are aggressive and indicate poor prognosis. C) The tumor's estrogen receptor guides treatment options. D) Stage I indicates metastasis.

C) The tumor's estrogen receptor guides treatment options. Treatment decisions (C) and prediction of prognosis are related to the tumor's receptor status, such as estrogen and progesterone receptor status which commonly are well-differentiated, have a lower chance of recurrence, and are receptive to hormonal therapy. Tumor staging designates tumor size and spread of breast cancer cells into axillary lymph nodes, which is one of the most important prognostic factors in early-stage breast cancer, not (A). Larger tumors are more likely to indicate poor prognosis, not (B). Stage I indicates the cancer is localized and has not spread systemically (D).

Which information about mammograms is most important to provide a post-menopausal female client? A) Breast self-examinations are not needed if annual mammograms are obtained. B) Radiation exposure is minimized by shielding the abdomen with a lead-lined apron. C) Yearly mammograms should be done regardless of previous normal x-rays. D) Women at high risk should have annual routine and ultrasound mammograms.

C) Yearly mammograms should be done regardless of previous normal x-rays. The current breast screening recommendation is a yearly mammogram after age 40 (C). Breast self-exam (A) continues to be a priority recommendation for all women because a small lump (or tumor) is often first felt by a woman before a mammogram is obtained. The radiation exposure from a mammogram is low, so (B) is not normally provided. The frequency of using routine and ultrasound mammograms (D) in women with high-risk variables, such as a history of breast cancer, the presence of BRC1 and BRC2 genes, or 2 first-degree relatives with breast cancer, should be recommended and followed closely by the healthcare provider.

A client has taken steroids for 12 years to help manage chronic obstructive pulmonary disease (COPD). When making a home visit, which nursing function is of greatest importance to this client? Assess the client's A) pulse rate, both apically and radially. B) blood pressure, both standing and sitting. C) temperature. D) skin color and turgor.

C) temperature. It is very important to check the client's temperature (C). Infection is the most common factor precipitating respiratory distress. Clients with COPD who are on maintenance doses of corticosteroids are particularly predisposed to infection. (A and B) are important data for baseline and ongoing assessment, but they are not as important as temperature measurement for this client who is taking steroids. Assessment of skin color and turgor is less important (D).

39. Select all that apply. Which of the following are clinical manifestations of tension pneumothorax? A. Midline trachea B. Severe hypertension C. Progressive cyanosis D. A loud bruit on affected side E. Asymmetrical chest wall movement F. Subcutaneous emphysema in the neck

C,E, F The indicators of tension pneumothorax are asymmetrical chest wall movement, severe hypotension, subcutaneous emphysema in the neck and upper chest, and progressive cyanosis.

Which of the following statements made by a patient with COPD indicates a need for further education regarding the use of an ipratropium inhaler? A. "I should rinse my mouth following the two puffs to get rid of the bad taste." B. "I should wait at least 1 to 2 minutes between each puff of the inhaler." C. "If my breathing gets worse, I should keep taking extra puffs of the inhaler until I can breathe more easily." D. "Because this medication is not fast-acting, I cannot use it in an emergency if my breathing gets worse.

C. "If my breathing gets worse, I should keep taking extra puffs of the inhaler until I can breathe more easily." The patient should not take extra puffs of the inhaler at will to make breathing easier. Excessive treatment could trigger paradoxical bronchospasm, which would worsen the patient's respiratory status.

19. Which of the following statements made by a nurse would indicate proper teaching principles regarding feeding and tracheostomies? A. "Follow each spoon of food consumed with a drink of fluid." B. "Thin your foods to a liquid consistency whenever possible." C. "Tilt your chin forward toward the chest when swallowing your food." D. "Make sure your cuff is overinflated before eating if you have swallowing problems."

C. "Tilt your chin forward toward the chest when swallowing your food." A nurse should instruct a patient to tilt the chin toward the chest, which will close the glottis and allow food to enter the normal passageway. Ideally, foods should be of a thick consistency to enable effective swallowing and reduce the risk of aspiration. Overinflation of the cuff causes swallowing difficulties. Fluids should be consumed in small amounts after swallowing to prevent the risk of aspiration.

A central venous catheter has been inserted via a jugular vein and a radiography has confirmed placement of the catheter. A prescription has been received for stat medication but IV fluids have not yet been started. What action should the nurse take prior to administering the prescribed medication? A. Assess for signs of jugular vein distention. B. Obtain the needed intravenous solution. C. Administer a bolus of normal saline solution. D. Flush the line with heparinized saline.

C. A medication can be administered central line without IV fluids, flush with normal saline to remove heparin that may counteract with the medication. (B) is used following the medication and a second saline bolus. (A) will not impact the the med administration and is not a priority. (B) Administration of the stat medication is more of a priority than (B).

A female nurse who took drugs from the unit for personal use was temporarily released from duty. After completion of mandatory counseling, the nurse has asked administration to allow her to return to work. When the nurse administrator approaches the charge nurse with the impaired nurse's request, what action is best for the charge nurse to take? A.Since treatment is completed, assign the nurse to routine RN responsibilities B. Ask to meet with the impaired nurse's therapist before allowing her back on the unit C. All the impaired nurse to return to work and monitor medication administration D. Meet with the staff to assess their feelings about the impaired nurse's return to the unit

C. All the impaired nurse to return to work and monitor medication administration

About 85 victims of a train derailment are brought to the Emergency Department of a small rural hospital. An older male with extensive crush injuries to his lower extremities and pelvis has a blood pressure of 42/28, a thready pulse of 120 beats/minute, and a respiratory rate of 10 breaths/minute with periods of apnea. Using the disaster triage system, which action should the nurse take? A. Obtain the crash cart and defibrillator B. Transport to radiology department C. Assign a black triage color D. Initiate a large bore IV infusion

C. Assign a black triage color

An adult, Muslim client with ulcerative colitis was admitted to the post-surgical unit earlier today following a bowel resection with temporary colostomy. Which intervention is most important for the nurse to implement? A. Teach the client how to perform stoma care B. Allow family members to visit whenever they wish C. Assign a care provider of the same gender D. Evaluate the client's current nutritional status

C. Assign a care provider of the same gender

Client census is often used to determine staffing needs. Which method of obtaining census determination for a particular unit provides the best formula for determining long-range staffing patterns? A. Midnight census B. Oncoming shift census C. Average daily census D. Hourly census

C. Average daily census Rationale: An average daily census (C) is determined by trend data and takes into account seasonal and daily fluctuations, so it is the best method for determining staffing needs. (A) and (B) provide data at a certain point in time and that data could change quickly. It is unrealistic to expect to obtain an hourly census (D), and such data would only provide information about a certain point in time.

A male client with multiple myeloma is admitted with pneumonia and pancytopenia. The nurse reviews the complete blood cell count findings and identifies a platelet count of 20,000 cells/mm^3. Which intervention should the nurse include in the client's plan of care? A. Pace activities between planned rest periods. B. Monitor intake and output C. Avoid intramuscular injections D. Limit exposure to visitors with respiratory infections

C. Avoid intramuscular injections

A client taking tamoxifen citrate following a lumpectomy reports several problems to the nurse. It is most important for the nurse to follow-up on which reported problem? A. Erratic menstrual periods B. Bone pain C. Calf tenderness D. Anorexia and nausea

C. Calf tenderness

The nurse is assessing the normal development of a 9-year-old male infant. Which information should the nurse obtain from the mother? A. Has the child started to walk? B. Is the baby able to lift his head when prone? C. Can the child sit alone? D. Does the baby roll from abdomen to back?

C. Can the child sit alone?

A 12-lead electrocardiogram (ECG) indicated a ST elevation in leads V1 to V4, for a client who reports having chest pain. The healthcare provider prescribes tissue plasminogen activator (t-PA). Prior to initiating the infusion, which intervention is most important for the nurse to implement? A. Place the ECG findings in the client's record B. Obtain a signed informed consent C. Complete pre-infusion checklist D. Insert two large bore IV sites

C. Complete pre-infusion checklist

During the change of shift report, the charge nurse reviews the infusions being received by clients on the oncology unit. The client receiving which infusion should be assessed first? A. Continuous IV infusion of magnesium B. One-time infusion of albumin C. Continuous epidural infusion of morphine D. Intermittent infusion of IV vancomycin

C. Continuous epidural infusion of morphine Rationale: All four of these clients have the potential to have significant complications. The client with the morphine epidural infusion is at highest risk for respiratory depression and should be assessed first. Option A can cause hypotension. The client receiving option B is at lowest risk for serious complications. Although option D can cause nephrotoxicity and phlebitis, these problems are not as immediately life threatening as option C.

A client exposed to tuberculosis is scheduled to begin prophylactic treatment with isoniazid. Which information is most important for the nurse to note before administering the initial dose? A. Conversion of the client's PPD test from negative to positive B. Length of time of the exposure to tuberculosis C. Current diagnosis of hepatitis B D. History of intravenous drug abuse

C. Current diagnosis of hepatitis B

The nurse is caring for a critically ill client with cirrhosis of the liver who has a nasogastric tube draining bright red blood. The nurse notes that the client's serum hemoglobin and hematocrit levels are decreased. Which additional change in laboratory data should the nurse expect? A. Increased serum albumin level B. Decreased serum creatinine C. Decreased serum ammonia level D. Increased liver function test results

C. Decreased serum ammonia level Rationale: The breakdown of glutamine in the intestine and the increased activity of colonic bacteria from the digestion of proteins increase ammonia levels in clients with advanced liver disease, so removal of blood, a protein source, from the intestine results in a reduced level of ammonia (C). (A, B, and D) will not be significantly affected by the removal of blood.

During assessment of a 2-year-old infant, the nurse notices a bluish-black discoloration over the lumbosacral area. Which action should the nurse take? A. Ask the mother about the discoloration B. Report possible child abuse to protective services C. Document the finding in the report D. Gently rub the area with skin cream to promote healing

C. Document the finding in the report

The home health nurse is assessing a male client being treated for Parkinson's disease with carbidopa-levodopa (Sinemet). The nurse observes that he does not demonstrate any apparent emotion when speaking and rarely blinks. Which intervention should the nurse implement? A. Perform a complete cranial nerve assessment. B. Instruct the client that he may be experiencing medication toxicity. C. Document the presence of these assessment findings. D. Advise the client to seek immediate medical evaluation.

C. Document the presence of these assessment findings. Rationale: A masklike expression and infrequent blinking are common clinical features of parkinsonism. The nurse should document these expected findings (C). (A and D) are not necessary. Signs of toxicity (B) of levodopa-carbidopa (Sinemet) include dyskinesia, hallucinations, and psychosis.

While completing an admission assessment for a client with unstable angina, which closed ended question should the nurse ask about the client's chest pain? A. Tell me about the activities that cause your pain? B. When did you first notice the pain in your chest? C. Does your pain occur when walking short decrease? D. How do you feel when the pain becomes noticeable?

C. Does your pain occur when walking short decrease?

A client's serum potassium test level is 6 mEq/L. The laboratory indicates, "Specimen is hemolyzed." What action should the nurse take? A. Encourage the client to eat a banana B. Notify the healthcare provider of the laboratory finding C. Draw a new blood specimen D. Obtain a prescription for sodium polystyrene sulfonate (Kayexalate)

C. Draw a new blood specimen

43. The nurse evaluates that nursing interventions to promote airway clearance in a patient admitted with COPD are successful based on which of the following findings? A. Absence of dyspnea B. Improved mental status C. Effective and productive coughing D. PaO2 within normal range for the patient

C. Effective and productive coughing The issue of the question is airway clearance, which is most directly evaluated as successful if the patient can engage in effective and productive coughing.

The nurse evaluates that nursing interventions to promote airway clearance in a patient admitted with COPD are successful based on which of the following findings? A. Absence of dyspnea B. Improved mental status C. Effective and productive coughing D. PaO2 within normal range for the patient

C. Effective and productive coughing The issue of the question is airway clearance, which is most directly evaluated as successful if the patient can engage in effective and productive coughing.

The nurse initiates a one-to-one relationship with a 35-year-old depressed female client who was recently admitted to the psychiatric facility. Which nursing action is most effective in promoting the development of a therapeutic relationship? A. At your first meeting clearly define the unit's rules and policies B. Obtain client data from her family to use at the first meeting with the client C. Ensure that scheduled appointments begin according to the schedule D. Re-direct all conversations to discussions about feelings of low self-esteem

C. Ensure that scheduled appointments begin according to the schedule

A newly hired male unlicensed assistive personnel (UAP) is assigned to a home healthcare team along with two experienced UAP's. Which intervention should the home health nurse implement to ensure adequate care for all clients? A. Assign the newly hired UAP to clients who require the least complex level of care B. Ask the most experienced UAP on the team to partner with the newly hired UAP C. Evaluate the newly hired UAP's level of competency by observing him deliver care D. Review the UAP's skills checklist and experience wit the person who hired him

C. Evaluate the newly hired UAP's level of competency by observing him deliver care

During the admission interview, the nurse learns that a newly admitted adult client has a six month history of reoccurring somatic pain. Which problem is most important for the nurse to further explore with the client? A. Nausea and vomiting B. Episodes of injury related to falls C. Feelings of depression D. Periods of anxiety and restlessness

C. Feelings of depression

46. Nursing assessment findings of jugular vein distention and pedal edema would be indicative of which of the following complications of emphysema? A. Acute respiratory failure B. Pulmonary edema caused by left-sided heart failure C. Fluid volume excess secondary to cor pulmonale D. Secondary respiratory infection

C. Fluid volume excess secondary to cor pulmonale Cor pulmonale is a right-sided heart failure caused by resistance to right ventricular outflow due to lung disease. With failure of the right ventricle, the blood emptying into the right atrium and ventricle would be slowed, leading to jugular venous distention and pedal edema.

Nursing assessment findings of jugular vein distention and pedal edema would be indicative of which of the following complications of emphysema? A. Acute respiratory failure B. Pulmonary edema caused by left-sided heart failure C. Fluid volume excess secondary to cor pulmonale D. Secondary respiratory infection

C. Fluid volume excess secondary to cor pulmonale Cor pulmonale is a right-sided heart failure caused by resistance to right ventricular outflow due to lung disease. With failure of the right ventricle, the blood emptying into the right atrium and ventricle would be slowed, leading to jugular venous distention and pedal edema.

Immediately after an elective cardioversion for rapid supraventricular tachycardia (SVT), a male client who was premedicated with hydromorphone (Dilaudid) and midazolam (Versed) is difficult to arouse. His vital signs are: oxygen saturation 94% while receiving oxygen at 2 L/minute per nasal cannula, heart rate 78 beats/minute, respirations 6 breaths/minute, and blood pressure 102/70. Which intervention should the nurse implement? A. Increase oxygen to 4 L/minute B. Prepare for another cardioversion C. Give IV naloxone (Narcan) D. Infuse normal saline IV bolus

C. Give IV naloxone (Narcan)

When a nurse assesses a client receiving total parenteral nutrition (TPN), which laboratory value is most important for the nurse to monitor regularly? A. Albumin B. Calcium C. Glucose D. Alkaline phosphatase

C. Glucose Rationale: TPN solutions contain high concentrations of glucose, so the blood glucose level is often monitored as often as q6h because of the risk for hyperglycemia (C). (A) is monitored periodically because an increase in the albumin level, a serum protein, is generally a desired effect of TPN. (B) may be added to TPN solutions, but calcium imbalances are not generally a risk during TPN administration. (D) may be decreased in the client with malnutrition who receives TPN, but abnormal values, reflecting liver or bone disorders, are not a common complication of TPN administration.

When a nurse assesses a client receiving total parenteral nutrition (TPN), which laboratory value is most important for the nurse to monitor regularly? A. Albumin B. Calcium C. Glucose D. Alkaline phosphatase

C. Glucose Rationale: TPN solutions contain high concentrations of glucose, so the blood glucose level is often monitored as often as q6h because of the risk for hyperglycemia. Option A is monitored periodically because an increase in the albumin level, a serum protein, is generally a desired effect of TPN. Option B may be added to TPN solutions, but calcium imbalances are not generally a risk during TPN administration. Option D may be decreased in the client with malnutrition who receives TPN, but abnormal values, reflecting liver or bone disorders, are not a common complication of TPN administration.

Which of the following positions is most appropriate for the nurse to place a patient experiencing an asthma exacerbation? A. Supine B. Lithotomy C. High-Fowler's D. Reverse Trendelenburg

C. High-Fowler'sThe patient experiencing an asthma attack should be placed in high-Fowler's position to allow for optimal chest expansion and enlist the aid of gravity during inspiration.

28. While teaching a patient with asthma about the appropriate use of a peak flow meter, the nurse instructs the patient to do which of the following? A. Use the flow meter each morning after taking medications to evaluate their effectiveness. B. Empty the lungs and then inhale quickly through the mouthpiece to measure how fast air can be inhaled. C. Keep a record of the peak flow meter numbers if symptoms of asthma are getting worse. D. Increase the doses of the long-term control medication if the peak flow numbers decrease.

C. Keep a record of the peak flow meter numbers if symptoms of asthma are getting worse. It is important to keep track of peak flow readings daily and when the patient's symptoms are getting worse. The patient should have specific directions as to when to call the physician based on personal peak flow numbers. Peak flow is measured by exhaling into the meters and should be assessed before and after medications to evaluate their effectiveness.

While teaching a patient with asthma about the appropriate use of a peak flow meter, the nurse instructs the patient to do which of the following? A. Use the flow meter each morning after taking medications to evaluate their effectiveness. B. Empty the lungs and then inhale quickly through the mouthpiece to measure how fast air can be inhaled. C. Keep a record of the peak flow meter numbers if symptoms of asthma are getting worse. D. Increase the doses of the long-term control medication if the peak flow numbers decrease.

C. Keep a record of the peak flow meter numbers if symptoms of asthma are getting worse. It is important to keep track of peak flow readings daily and when the patient's symptoms are getting worse. The patient should have specific directions as to when to call the physician based on personal peak flow numbers. Peak flow is measured by exhaling into the meters and should be assessed before and after medications to evaluate their effectiveness.

A client in the emergency department is bleeding profusely from a gunshot wound to the abdomen. In what position should the nurse immediately place the client to promote maintenance of the client's blood pressure above a systolic pressure of 90 mm Hg? A. Place the client in a 45-degree Trendelenburg position to promote cerebral blood flow. B. Turn the client prone to place pressure on the abdominal wound to help staunch the bleeding. C. Maintain the client in a supine position to reduce diaphragmatic pressure and visualize the wound. D. Put the client on the right side to apply pressure to the liver and spleen to stop hemorrhaging.

C. Maintain the client in a supine position to reduce diaphragmatic pressure and visualize the wound. Rationale: Placing the client in a supine position (C) reduces diaphragmatic pressure, thereby enhancing oxygenation, and allows for visualization of the abdominal wound. (A) compromises diaphragmatic expansion and inhibits pressoreceptor activity. (B) places the client at risk of evisceration of the abdominal wound and increased bleeding. (D) will not stop internal bleeding in the liver and spleen caused by the gunshot wound.

6. What type of anemia is associated with folate deficiency? A. Microcytic B. Pernicious C. Megaloblastic D. Iron deficiency

C. Megaloblastic Megaloblastic anemia is nutritional anemia; large immature red blood cells with a decreased oxygen-carrying capacity can occur as a result of impaired DNA synthesis. Folic acid is used in the synthesis of DNA and helps convert B12 to coenzyme form. Folic acid is needed for growth and development of red blood cells. Microcytic anemia is anemia with abnormally small erythrocytes (red blood cells) in Hb. This anemia is associated with vitamin B6 (pyridoxine) deficiency. Pernicious anemia is caused by a deficiency of vitamin B12. Iron-deficiency anemia results from loss of blood or deficient intake of iron foods or disease states in which the body does not absorb or utilize iron as it should.

What type of anemia is associated with folate deficiency? A. Microcytic B. Pernicious C. Megaloblastic D. Iron deficiency

C. Megaloblastic Megaloblastic anemia is nutritional anemia; large immature red blood cells with a decreased oxygen-carrying capacity can occur as a result of impaired DNA synthesis. Folic acid is used in the synthesis of DNA and helps convert B12 to coenzyme form. Folic acid is needed for growth and development of red blood cells. Microcytic anemia is anemia with abnormally small erythrocytes (red blood cells) in Hb. This anemia is associated with vitamin B6 (pyridoxine) deficiency. Pernicious anemia is caused by a deficiency of vitamin B12. Iron-deficiency anemia results from loss of blood or deficient intake of iron foods or disease states in which the body does not absorb or utilize iron as it should.

During report, the nurse learns that a client with tumor lysis syndrome is receiving an IV infusion containing insulin. Which assessment should the nurse complete first? A. Review the client's history for diabetes mellitus. B. Observe the extremity distal to the IV site. C. Monitor the client's serum potassium and blood glucose levels. D. Evaluate the client's oxygen saturation and breath sounds.

C. Monitor the client's serum potassium and blood glucose levels. Rationale: Clients with tumor lysis syndrome may experience hyperkalemia, requiring the addition of insulin to the IV solution to reduce the serum potassium level. It is most important for the nurse to monitor the client's serum potassium and blood glucose levels to ensure that they are not at dangerous levels (C). (A, B, and D) provide valuable assessment data but are of less priority than (C).

20. During assessment of a 45-year-old patient with asthma, the nurse notes wheezing and dyspnea. The nurse interprets that these symptoms are related to which of the following pathophysiologic changes? A. Laryngospasm B. Overdistention of the alveoli C. Narrowing of the airway D. Pulmonary edema

C. Narrowing of the airwayNarrowing of the airway leads to reduced airflow, making it difficult for the patient to breathe and producing the characteristic wheezing.

During assessment of a 45-year-old patient with asthma, the nurse notes wheezing and dyspnea. The nurse interprets that these symptoms are related to which of the following pathophysiologic changes? A. Laryngospasm B. Overdistention of the alveoli C. Narrowing of the airway D. Pulmonary edema

C. Narrowing of the airwayNarrowing of the airway leads to reduced airflow, making it difficult for the patient to breathe and producing the characteristic wheezing.

In assessing a client with an arteriovenous (AV) shunt who is scheduled for dialysis today, the nurse notes the absence of a thrill or bruit at the shunt site. What action should the nurse take? A. Advise the client that the shunt is intact and ready for dialysis as scheduled. B. Encourage the client to keep the shunt site elevated above the level of the heart. C. Notify the health care provider of the findings immediately. D. Flush the site at least once with a heparinized saline solution.

C. Notify the health care provider of the findings immediately. Rationale: Absence of a thrill or bruit indicates that the shunt may be obstructed. The nurse should notify the health care provider (C) so that intervention can be initiated to restore function of the shunt. (A) is incorrect. (B) will not resolve the obstruction. An AV shunt is internal and cannot be flushed (D) without access using special needles.

16. While ambulating a patient with metastatic lung cancer, the nurse observes a drop in oxygen saturation from 93% to 86%. Which of the following nursing interventions is most appropriate based upon these findings? A. Continue with ambulation as this is a normal response to activity. B. Move the oximetry probe from the finger to the earlobe for more accurate monitoring during activity. C. Obtain a physician's order for supplemental oxygen to be used during ambulation and other activity. D. Obtain a physician's order for arterial blood gas determinations to verify the oxygen saturation.

C. Obtain a physician's order for supplemental oxygen to be used during ambulation and other activity. An oxygen saturation level that drops below 90% with activity indicates that the patient is not tolerating the exercise and needs to have supplemental oxygen applied.

7. During care of a patient with multiple myeloma, an important nursing intervention is A. limiting activity to prevent pathologic fractures. B. assessing for changes in size and characteristics of lymph nodes. C. maintaining a fluid intake of 3 to 4 L/day to dilute calcium load. D. administering narcotic analgesics continuously to control bone pain.

C. maintaining a fluid intake of 3 to 4 L/day to dilute calcium load. Adequate hydration must be maintained to minimize problems from hypercalcemia. The goal of a urinary output of 1.5 to 2 L/day requires an intake of 3 to 4 L/day.

While ambulating a patient with metastatic lung cancer, the nurse observes a drop in oxygen saturation from 93% to 86%. Which of the following nursing interventions is most appropriate based upon these findings? A. Continue with ambulation as this is a normal response to activity. B. Move the oximetry probe from the finger to the earlobe for more accurate monitoring during activity. C. Obtain a physician's order for supplemental oxygen to be used during ambulation and other activity. D. Obtain a physician's order for arterial blood gas determinations to verify the oxygen saturation.

C. Obtain a physician's order for supplemental oxygen to be used during ambulation and other activity. An oxygen saturation level that drops below 90% with activity indicates that the patient is not tolerating the exercise and needs to have supplemental oxygen applied.

11. When assessing a patient's respiratory status, which of the following nonrespiratory data are most important for the nurse to obtain? A. Height and weight B. Neck circumference C. Occupation and hobbies D. Usual daily fluid intake

C. Occupation and hobbiesMany respiratory problems occur as a result of chronic exposure to inhalation irritants. Common occupational sources of inhalation irritants include mines, granaries, farms, lawn care companies, paint, plastics and rubber manufacture, and building remodeling. Hobbies associated with inhalation irritants include woodworking, metal finishing, furniture refinishing, painting, and ceramics. Daily fluids, height, and weight are more related to respiratory problems secondary to cardiac issues.

A client arrives in the emergency department (ED) with slurred speech and right-sided weakness. Which information is most important? A. Family history of stroke B. Changes in vision C. Onset of symptoms D. Severity of headache

C. Onset of symptoms

The nurse determines that a patient is experiencing common adverse effects from the inhaled corticosteroid beclomethasone (Beclovent) after noting which of the following? A. Adrenocortical dysfunction and hyperglycemia B. Elevation of blood glucose and calcium levels C. Oropharyngeal candidiasis and hoarseness D. Hypertension and pulmonary edema

C. Oropharyngeal candidiasis and hoarseness Oropharyngeal candidiasis and hoarseness are common adverse effects from the use of inhaled corticosteroids because the medication can lead to overgrowth of organisms and local irritation if the patient does not rinse the mouth following each dose.

A 62-year-old woman who lives alone tripped on a rug in her home and fractured her hip. Which predisposing factor most likely contributed to the fracture in the proximal end of her femur? A. Failing eyesight resulting in an unsafe environment B. Renal osteodystrophy resulting from chronic kidney disease (CKD) C. Osteoporosis resulting from declining hormone levels D. Cerebral vessel changes causing transient ischemic attacks

C. Osteoporosis resulting from declining hormone levels Rationale: The most common cause of a fractured hip in older women is osteoporosis, resulting from reduced calcium in the bones as a result of hormonal changes in the perimenopausal years (C). (A) may or may not have contributed to the accident, but eye changes were not involved in promoting the hip fracture. (B) is not a common condition of older people but is associated with CKD. Although (D) may result in transient ischemic attacks (TIAs) or stroke, it will not result in fragility of the bones, as does osteoporosis.

A 62-year-old woman who lives alone tripped on a rug in her home and fractured her hip. Which predisposing factor most likely contributed to the fracture in the proximal end of her femur? A.Failing eyesight resulting in an unsafe environment B.Renal osteodystrophy resulting from chronic kidney disease (CKD) C.Osteoporosis resulting from declining hormone levels D.Cerebral vessel changes causing transient ischemic attacks

C. Osteoporosis resulting from declining hormone levels Rationale: The most common cause of a fractured hip in older women is osteoporosis, resulting from reduced calcium in the bones as a result of hormonal changes in the perimenopausal years. Option A may or may not have contributed to the accident, but eye changes were not involved in promoting the hip fracture. Option B is not a common condition of older people but is associated with CKD. Although option D may result in transient ischemic attacks (TIAs) or stroke, it will not result in fragility of the bones, as does osteoporosis.

The nurse is completing an admission inter for a client with Parkinson disease. Which question will provide addition information about manifestations the client is likely to experience? A. "Have you ever experienced and paralysis of your arms or legs?" B. " Do you have frequent blackout spells?" C. "Have you ever been 'frozen' in one spot, unable to move?" D. "Do you have headaches, especially ones with throbbing pain?"

C. Parkinson clients frequently experience difficulty in initiating, maintaining, and performing motor activities. They may even experience being rooted, unable to move. (A, B, D) Does not typically occur in Parkinson.

15. When reviewing the results of a 83-year-old patient's blood tests, which of the following findings would be of most concern to the nurse? A. Platelets of 150,000/µl B. Serum iron of 50 mcg/dl C. Partial thromboplastin time (PTT) of 60 seconds D. Erythrocyte sedimentation rate (ESR) of 35 mm in 1 hour

C. Partial thromboplastin time (PTT) of 60 seconds In aging, the partial thromboplastin time (PTT) is normally decreased, so an abnormally high PTT of 60 seconds is an indication that bleeding could readily occur. Platelets are unaffected by aging, and 150,000 is a normal count. Serum iron levels are decreased and the erythrocyte sedimentation rate (ESR) is significantly increased with aging, as are reflected in these values.

30. Which of the following conditions or factors in a 64-year-old patient diagnosed with head and neck cancer most likely contributed to this health problem? A. Patient's hobby is oil painting. B. Patient's father also had head and neck cancer. C. Patient uses chewing tobacco and drinks beer daily. D. Patient quit school at age 16 and has worked in a butcher shop for more than 40 years.

C. Patient uses chewing tobacco and drinks beer daily. Many environmental risk factors contribute to the development of head and neck cancer, although the actual cause is unknown. There does not appear to be a genetic predisposition to this type of cancer. The two most important risk factors are tobacco and alcohol use, especially in combination. Other risk factors include chewing tobacco, pipe smoking, marijuana use, voice abuse, chronic laryngitis, exposure to industrial chemicals or hardwood dust, and poor oral hygiene.

Which of the following conditions or factors in a 64-year-old patient diagnosed with head and neck cancer most likely contributed to this health problem? A. Patient's hobby is oil painting. B. Patient's father also had head and neck cancer. C. Patient uses chewing tobacco and drinks beer daily. D. Patient quit school at age 16 and has worked in a butcher shop for more than 40 years.

C. Patient uses chewing tobacco and drinks beer daily. Many environmental risk factors contribute to the development of head and neck cancer, although the actual cause is unknown. There does not appear to be a genetic predisposition to this type of cancer. The two most important risk factors are tobacco and alcohol use, especially in combination. Other risk factors include chewing tobacco, pipe smoking, marijuana use, voice abuse, chronic laryngitis, exposure to industrial chemicals or hardwood dust, and poor oral hygiene.

A 71-year-old patient is admitted with acute respiratory distress related to cor pulmonale. Which of the following nursing interventions is most appropriate during admission of this patient? A. Delay any physical assessment of the patient and review with the family the patient's history of respiratory problems. B. Perform a comprehensive health history with the patient to review prior respiratory problems. C. Perform a physical assessment of the respiratory system and ask specific questions related to this episode of respiratory distress. D. Complete a full physical examination to determine the effect of the respiratory distress on other body functions.

C. Perform a physical assessment of the respiratory system and ask specific questions related to this episode of respiratory distress.Because the patient is having respiratory difficulty, the nurse should ask specific questions about this episode and perform a physical assessment of this system. Further history taking and physical examination of other body systems can proceed once the patient's acute respiratory distress is being managed.

Before placing a client's dentures in the sink for cleansing, what action should the nurse take? A. Fill the sink with half-strength peroxide solution B. Loosen dried secretions with a toothbrush C. Place a washcloth on the bottom of the sink D. Hold the dentures in a stream of running water

C. Place a washcloth on the bottom of the sink

Which of the following conditions is manifested by unexplained shortness of breath and a high mortality rate? A. Bleeding ulcer B. Transient ischemia C. Pulmonary embolism D. MI

C. Pulmonary embolism A high mortality rate is associated with a pulmonary embolism. A pulmonary embolism is an obstruction of the pulmonary artery caused by an embolus. It presents with hypoxia, anxiety, restlessness, and shortness of breath. Bleeding ulcers, MI, and transient ischemia are not associated with such a high mortality rate.

The nurse is preparing a 50 ml dose of 50% Dextrose IV for a client with insulin shock. How should the nurse administer the medication? A. Dilute Dextrose in one liter of 0.9% Normal Saline solution B. Mix Dextrose in a 50 ml piggyback for a total volume of 100 ml. C. Push the undiluted Dextrose slowly through the currently infusing IV D. Ask the pharmacist to add the Dextrose to a TPN solution

C. Push the undiluted Dextrose slowly through the currently infusing IV

When checking a third-grader's height and weight, the school notes that these measurements have not changed in the last year. The child is currently taking daily vitamins, albuterol, and methylphenidate for attention deficit hyperactivity disorder (ADHD). Which intervention should the nurse implement? A. Report the finding to the parents B. Document findings in the child's school life C. Refer child to the family healthcare provider D. Encourage child to get more sleep

C. Refer child to the family healthcare provider

A female client, newly diagnosed with breast cancer, is scheduled for a mastectomy next week. During the preoperative assessment, she complains that her husband has become withdrawn and complains about her irritability, and frequently crying. How should the nurse respond? A. Explain that a positive attitude helps reduce preoperative complications B. Encourage the spouse to be more supportive at this difficult time C. Refer the couple to a counselor to help them with coping strategies D. Inquire if the couple has met with a minister to discuss their feelings

C. Refer the couple to a counselor to help them with coping strategies

The nurse includes frequent oral care in the plan of care for a client scheduled for an esophagogastrostomy for esophageal cancer. This intervention is included in the client's plan of care to address which nursing diagnosis? A.Fluid volume deficit B.Self-care deficit C. Risk for infection D.Impaired nutrition

C. Risk for infection Rationale: The primary reason for performing frequent mouth care preoperatively is to reduce the risk of postoperative infection because these clients may be regurgitating retained food particles, blood, or pus from the tumor. Meticulous oral care should be provided several times a day before surgery. Although oral care will be of benefit to the client who may also be experiencing option A, B, or D, these problems are not the primary reason for the provision of frequent oral care.

A client receives a prescription for an intramuscular pain medication. The nurse uses the Z-track method to administer the injection. Which rationale supports the nurse's use of this method? A. Prevents injury to the underlying bones, nerves, and blood vessels B Minimizes client's discomfort at the injection site C. Seals needle track to avoid medication leakage through the tissue D. Ensures medication reaches the intramuscular site

C. Seals needle track to avoid medication leakage through the tissue

When educating a client after a total laryngectomy, which instruction would be most important for the nurse to include in the discharge teaching? A. Recommend that the client carry suction equipment at all times. B. Instruct the client to have writing materials with him at all times. C. Tell the client to carry a medical alert card that explains his condition. D. Caution the client not to travel outside the United States alone.

C. Tell the client to carry a medical alert card that explains his condition. Rationale: Neck breathers carry a medical alert card (C) that notifies health care personnel of the need to use mouth to stoma breathing in the event of a cardiac arrest in this client. Mouth to mouth resuscitation will not establish a patent airway. (A and D) are not necessary. There are many alternative means of communication for clients who have had a laryngectomy; dependence on writing messages (B) is probably the least effective.

When educating a client after a total laryngectomy, which instruction would be most important for the nurse to include in the discharge teaching? A.Recommend that the client carry suction equipment at all times. B.Instruct the client to have writing materials with him at all times. C.Tell the client to carry a medical alert card that explains his condition. D.Caution the client not to travel outside the United States alone.

C. Tell the client to carry a medical alert card that explains his condition. Rationale: Neck breathers carry a medical alert card that notifies health care personnel of the need to use mouth to stoma breathing in the event of a cardiac arrest in this client. Mouth to mouth resuscitation will not establish a patent airway. Options A and D are not necessary. There are many alternative means of communication for clients who have had a laryngectomy; dependence on writing messages is probably the least effective.

A 55-year-old male client has been admitted to the hospital with a medical diagnosis of chronic obstructive pulmonary disease (COPD). Which risk factor is the most significant in the development of this client's COPD? A. The client's father was diagnosed with COPD in his 50s. B. A close family member contracted tuberculosis last year. C. The client smokes one to two packs of cigarettes per day. D. The client has been 40 pounds overweight for 15 years.

C. The client smokes one to two packs of cigarettes per day. Rationale: Smoking, considered to be a modifiable risk factor, is the most significant risk factor for the development of COPD (C). The exact mechanism of genetic and hereditary implications (A) for the development of COPD is still under investigation, although exposure to similar predisposing factors (e.g., smoking or inhaling second-hand smoke) may increase the likelihood of COPD incidence among family members. (B and D) do not exceed the risks associated with cigarette smoking in the development of COPD.

A 55-year-old male client has been admitted to the hospital with a medical diagnosis of chronic obstructive pulmonary disease (COPD). Which risk factor is the most significant in the development of this client's COPD? A.The client's father was diagnosed with COPD in his 50s. B.A close family member contracted tuberculosis last year. C.The client smokes one to two packs of cigarettes per day. D.The client has been 40 pounds overweight for 15 years.

C. The client smokes one to two packs of cigarettes per day. Rationale: Smoking, considered to be a modifiable risk factor, is the most significant risk factor for the development of COPD. The exact mechanism of genetic and hereditary implications for the development of COPD is still under investigation, although exposure to similar predisposing factors (e.g., smoking or inhaling secondhand smoke) may increase the likelihood of COPD incidence among family members. Options B and D do not exceed the risks associated with cigarette smoking in the development of COPD.

During report, the nurse learns that a client with tumor lysis syndrome is receiving an IV infusion containing insulin. Which assessment should the nurse complete first? A. Review the client's history for diabetes mellitus. B. Observe the extremity distal to the IV site. C. Monitor the client's serum potassium and blood glucose. D. Evaluate the client's oxygen saturation and breath sounds.

C. The client with tumor lysis syndrome may experience hyperkalemia, therefor it is important to monitor serum potassium and blood glucose levels. (A, B, D) are not as priority.

The community mental health nurse is planning to visit four clients with schizophrenia. Which client should the nurse see first? A. The young woman who believes she is to blame for her divorce B. The client with a history of substance abuse who is living in a halfway house C. The father who took his children from school because aliens were after them D. The client who needs to be evaluated for medication compliance.

C. The father who took his children from school because aliens were after them

12. If a nurse is assessing a patient whose recent blood gas determination indicated a pH of 7.32 and respirations are measured at 32 breaths/min, which of the following is the most appropriate nursing assessment? A. The rapid breathing is causing the low pH. B. The nurse should sedate the patient to slow down respirations. C. The rapid breathing is an attempt to compensate for the low pH. D. The nurse should give the patient a paper bag to breathe into to correct the low pH.

C. The rapid breathing is an attempt to compensate for the low pH. The respiratory system influences pH (acidity) through control of carbon dioxide exhalation. Thus, rapid breathing increases the pH. Breathing into a paper bag aids a patient who is hyperventilating; in respiratory alkalosis, it aids in lowering the pH. The use of sedation can cause respiratory depression and hypoventilation, resulting in an even lower pH.

If a nurse is assessing a patient whose recent blood gas determination indicated a pH of 7.32 and respirations are measured at 32 breaths/min, which of the following is the most appropriate nursing assessment? A. The rapid breathing is causing the low pH. B. The nurse should sedate the patient to slow down respirations. C. The rapid breathing is an attempt to compensate for the low pH. D. The nurse should give the patient a paper bag to breathe into to correct the low pH.

C. The rapid breathing is an attempt to compensate for the low pH. The respiratory system influences pH (acidity) through control of carbon dioxide exhalation. Thus, rapid breathing increases the pH. Breathing into a paper bag aids a patient who is hyperventilating; in respiratory alkalosis, it aids in lowering the pH. The use of sedation can cause respiratory depression and hypoventilation, resulting in an even lower pH.

A client is admitted to a medical unit with a diagnosis of gastritis and chronic heavy alcohol abuse. What should the nurse administer to prevent the development of Wernicke's syndrome? A. Lorazepam (Ativan) B. Famotidine (Pepcid) C. Thiamine (Vitamin B1) D. Atenolol (Tenormin)

C. Thiamine (Vitamin B1)

A home health nurse knows that a 70-year-old male client who is convalescing at home following a hip replacement is at risk for developing pressure ulcers. Which physical characteristic of aging puts the client at risk? A. 16% increase in overall body fat B. Reduced melanin production C. Thinning of the skin, with loss of elasticity D. Calcium loss in the bones

C. Thinning of the skin, with loss of elasticity Rationale: Thin nonelastic skin (C) is an important factor in pressure formation. The proportion of body fat to lean mass increases with age (A) and might help decrease ulcer tendency. (B) causes gray hair. (D) can contribute to broken bones, but it is probably not a factor in pressure ulcer formation.

A client who is at 36 weeks gestation is admitted with severe preeclampsia. After a 6 gram loading dose of magnesium sulfate is administered, an intravenous infusion of magnesium sulfate at a rate of 2 grams/hour is initiated. Which assessment finding warrants immediate intervention by the nurse? A. Blood pressure 162/94 B. Complaint of headache C. Urine output 20 ml/hour D. Nausea and vomiting

C. Urine output 20 ml/hour urinary output of less than 30 ml/hour indicates that the kidneys are being affected by the high level of magnesium, which is excreted through kidneys.

33. A patient with an acute pharyngitis is seen at the clinic with fever and severe throat pain that affects swallowing. On inspection the throat is reddened and edematous with patchy yellow exudates. The nurse anticipates that collaborative management will include A. treatment with antibiotics. B. treatment with antifungal agents. C. a throat culture or rapid strep antigen test. D. treatment with medication only if the pharyngitis does not resolve in 3 to 4 days.

C. a throat culture or rapid strep antigen test. Although inadequately treated β-hemolytic streptococcal infections may lead to rheumatic heart disease or glomerulonephritis, antibiotic treatment is not recommended until strep infections are definitely diagnosed with culture or antigen tests. The manifestations of viral and bacterial infections are similar, and appearance is not diagnostic except when candidiasis is present.

A patient with an acute pharyngitis is seen at the clinic with fever and severe throat pain that affects swallowing. On inspection the throat is reddened and edematous with patchy yellow exudates. The nurse anticipates that collaborative management will include A. treatment with antibiotics. B. treatment with antifungal agents. C. a throat culture or rapid strep antigen test. D. treatment with medication only if the pharyngitis does not resolve in 3 to 4 days.

C. a throat culture or rapid strep antigen test. Although inadequately treated β-hemolytic streptococcal infections may lead to rheumatic heart disease or glomerulonephritis, antibiotic treatment is not recommended until strep infections are definitely diagnosed with culture or antigen tests. The manifestations of viral and bacterial infections are similar, and appearance is not diagnostic except when candidiasis is present.

A client in the intensive care unit is being mechanically ventilated , has an indwelling urinary catheter in place, and is exhibiting signs of restlessness. Which action should the nurse take first? A. administer PRN dose of lorazepam B. auscultate bilateral breath sounds C. check urinary catheter for obstruction D. review the heart rhythm on cardiac monitor

C. check urinary catheter for obstruction

32. An excess of carbon dioxide in the blood causes an increased respiratory rate and volume because CO2 A. displaces oxygen on hemoglobin, leading to a decreased PaO2. B. causes an increase in the amount of hydrogen ions available in the body. C. combines with water to form carbonic acid, lowering the pH of cerebrospinal fluid. D. directly stimulates chemoreceptors in the medulla to increase respiratory rate and volume.

C. combines with water to form carbonic acid, lowering the pH of cerebrospinal fluid. A combination of excess CO2 and H2O results in carbonic acid, which lowers the pH of the cerebrospinal fluid and stimulates an increase in the respiratory rate. Peripheral chemoreceptors in the carotid and aortic bodies also respond to increases in PaCO2 to stimulate the respiratory center. Excess CO2 does not increase the amount of hydrogen ions available in the body but does combine with the hydrogen of water to form an acid.

An excess of carbon dioxide in the blood causes an increased respiratory rate and volume because CO2 A. displaces oxygen on hemoglobin, leading to a decreased PaO2. B. causes an increase in the amount of hydrogen ions available in the body. C. combines with water to form carbonic acid, lowering the pH of cerebrospinal fluid. D. directly stimulates chemoreceptors in the medulla to increase respiratory rate and volume.

C. combines with water to form carbonic acid, lowering the pH of cerebrospinal fluid. A combination of excess CO2 and H2O results in carbonic acid, which lowers the pH of the cerebrospinal fluid and stimulates an increase in the respiratory rate. Peripheral chemoreceptors in the carotid and aortic bodies also respond to increases in PaCO2 to stimulate the respiratory center. Excess CO2 does not increase the amount of hydrogen ions available in the body but does combine with the hydrogen of water to form an acid.

34. Following a supraglottic laryngectomy, the patient is taught how to use the supraglottic swallow to minimize the risk of aspiration. In teaching the patient about this technique, the nurse instructs the patient to A. perform Valsalva maneuver immediately after swallowing. B. breathe between each Valsalva maneuver and cough sequence. C. cough after swallowing to remove food from the top of the vocal cords. D. practice swallowing thin, watery fluids before attempting to swallow solid foods.

C. cough after swallowing to remove food from the top of the vocal cords. A supraglottic laryngectomy involves removal of the epiglottis and false vocal cords, and the removal of the epiglottis allows food to enter the trachea. Supraglottic swallowing requires performance of the Valsalva maneuver before placing food in the mouth and swallowing. The patient then coughs to remove food from the top of the vocal cords, swallows again, and then breathes after the food has been removed from the vocal cords.

With the client's eyes closed, the nurse places a common object in the client's hand and asks the client to describe the object. The client accurately names the object. How should the nurse document this assessment finding? A. No paraesthesia present B. Short term memory intact C. Active range of motion D. Positive for stereognosis

D. Positive for stereognosis

Following a supraglottic laryngectomy, the patient is taught how to use the supraglottic swallow to minimize the risk of aspiration. In teaching the patient about this technique, the nurse instructs the patient to A. perform Valsalva maneuver immediately after swallowing. B. breathe between each Valsalva maneuver and cough sequence. C. cough after swallowing to remove food from the top of the vocal cords. D. practice swallowing thin, watery fluids before attempting to swallow solid foods.

C. cough after swallowing to remove food from the top of the vocal cords. A supraglottic laryngectomy involves removal of the epiglottis and false vocal cords, and the removal of the epiglottis allows food to enter the trachea. Supraglottic swallowing requires performance of the Valsalva maneuver before placing food in the mouth and swallowing. The patient then coughs to remove food from the top of the vocal cords, swallows again, and then breathes after the food has been removed from the vocal cords.

If a patient has pernicious anemia, the nurse should provide information regarding A. frequent bouts of dyspnea. B. risks relative to dehydration. C. deficiency of intrinsic factor. D. lack of any effective treatment for this condition.

C. deficiency of intrinsic factor. Pernicious anemia is a type of anemia caused by failure of absorption of vitamin B12 (cobalamin). The most common cause is lack of intrinsic factor, a glucoprotein produced by the parietal cells of the gastric lining.

The nurse presenting information about fetal development to a group of parents who are attending a Lamaze birthing class. When discussing cephalocaudal fetal development, which information should the nurse provide? A. A set order in fetal development is expected B. growth normally occurs within one organ at a time C. development progresses from head to rump D. organ formation is directed from brain development

C. development progresses from head to rump fetal systems are developed in pre-determined order, best described in the direction of head to rump

18. A nurse establishes the presence of a tension pneumothorax when assessment findings reveal a(n) A. absence of lung sounds on the affected side. B. inability to auscultate tracheal breath sounds. C. deviation of the trachea toward the side opposite the pneumothorax. D. shift of the point of maximal impulse (PMI) to the left, with bounding pulses.

C. deviation of the trachea toward the side opposite the pneumothorax. Tension pneumothorax is caused by rapid accumulation of air in the pleural space, causing severely high intrapleural pressure. This results in collapse of the lung, and the mediastinum shifts toward the unaffected side, which is subsequently compressed.

A nurse establishes the presence of a tension pneumothorax when assessment findings reveal a(n) A. absence of lung sounds on the affected side. B. inability to auscultate tracheal breath sounds. C. deviation of the trachea toward the side opposite the pneumothorax. D. shift of the point of maximal impulse (PMI) to the left, with bounding pulses.

C. deviation of the trachea toward the side opposite the pneumothorax. Tension pneumothorax is caused by rapid accumulation of air in the pleural space, causing severely high intrapleural pressure. This results in collapse of the lung, and the mediastinum shifts toward the unaffected side, which is subsequently compressed.

37. The chronic inflammation of the bronchi characteristic of chronic obstructive pulmonary disease (COPD) results in A. collapse of small bronchioles on expiration. B. permanent, abnormal dilation of the bronchi. C. hyperplasia of mucus-secreting cells and bronchial edema. D. destruction of the elastic and muscular structures of the bronchial wall.

C. hyperplasia of mucus-secreting cells and bronchial edema. Chronic bronchitis is characterized by chronic inflammation of the bronchial lining, with edema and increased mucus production. Collapse of small bronchioles on expiration is common in emphysema, and abnormal dilation of the bronchi because of destruction of the elastic and muscular structures is characteristic of bronchiectasis.

The chronic inflammation of the bronchi characteristic of chronic obstructive pulmonary disease (COPD) results in A. collapse of small bronchioles on expiration. B. permanent, abnormal dilation of the bronchi. C. hyperplasia of mucus-secreting cells and bronchial edema. D. destruction of the elastic and muscular structures of the bronchial wall.

C. hyperplasia of mucus-secreting cells and bronchial edema. Chronic bronchitis is characterized by chronic inflammation of the bronchial lining, with edema and increased mucus production. Collapse of small bronchioles on expiration is common in emphysema, and abnormal dilation of the bronchi because of destruction of the elastic and muscular structures is characteristic of bronchiectasis.

8. A person complains of fatigue and malaise and has a slight temperature elevation for 2 days before symptoms of influenza (fever, chest congestion, and productive cough) become noticeable. During the time immediately before the illness is diagnosed, the patient A. could avoid contracting the disease if treatment is begun with antibiotics. B. is unable to spread the disease because it is still in the incubation period. C. is in the prodromal stage and is highly contagious and able to spread the disease. D. has a nosocomial infection, which affects approximately two million individuals a year.

C. is in the prodromal stage and is highly contagious and able to spread the disease. The prodromal stage is a short period of time (hours to several days) immediately preceding the onset of an illness during which the patient is very contagious. Antibiotics are not effective against viral illnesses. The incubation period is the time from entry of the organism to the onset of symptoms and, in some viral illnesses, may be contagious. Nosocomial infections are those acquired in a hospital, and this scenario does not suggest the source of the infection.

A person complains of fatigue and malaise and has a slight temperature elevation for 2 days before symptoms of influenza (fever, chest congestion, and productive cough) become noticeable. During the time immediately before the illness is diagnosed, the patient A. could avoid contracting the disease if treatment is begun with antibiotics. B. is unable to spread the disease because it is still in the incubation period. C. is in the prodromal stage and is highly contagious and able to spread the disease. D. has a nosocomial infection, which affects approximately two million individuals a year.

C. is in the prodromal stage and is highly contagious and able to spread the disease. The prodromal stage is a short period of time (hours to several days) immediately preceding the onset of an illness during which the patient is very contagious. Antibiotics are not effective against viral illnesses. The incubation period is the time from entry of the organism to the onset of symptoms and, in some viral illnesses, may be contagious. Nosocomial infections are those acquired in a hospital, and this scenario does not suggest the source of the infection.

During care of a patient with multiple myeloma, an important nursing intervention is A. limiting activity to prevent pathologic fractures. B. assessing for changes in size and characteristics of lymph nodes. C. maintaining a fluid intake of 3 to 4 L/day to dilute calcium load. D. administering narcotic analgesics continuously to control bone pain.

C. maintaining a fluid intake of 3 to 4 L/day to dilute calcium load. Adequate hydration must be maintained to minimize problems from hypercalcemia. The goal of a urinary output of 1.5 to 2 L/day requires an intake of 3 to 4 L/day.

A client who was splashed with a chemical has both eyes covered with bandages. When assisting the client with eating, which intervention should the nurse instruct the unlicensed assistive personal (UAP) to implement? A. feed the client the entire meal B. provide with only finer food C. orient the client to location of food on the plate D. ask family member to visit during meal time to assist with feeding

C. orient the client to location of food on the plate

5. Absorption of vitamin B12 may be decreased in older adults because of decreased A. intestinal motility. B. production of bile by the liver. C. production of intrinsic factor by the stomach. D. synthesis of cobalamin (vitamin B12) by intestinal bacteria.

C. production of intrinsic factor by the stomach. Older persons are at risk for deficiency of cobalamin (pernicious anemia) because of a naturally occurring reduction of the intrinsic factor by the stomach mucosa. Absorption of cobalamin relies on intrinsic factor. Both must be present for absorption. Megaloblastic anemia is related to folate dysfunction. Intestinal motility (peristalsis) is the motion that moves food down the GI tract. The rhythmic contractions of muscles cause wave-like motions. Lack of peristalsis is called "paralytic ileus." Bile is produced in the liver, is stored and concentrated in the gallbladder, and is released into the duodenum when fat is eaten. Bile emulsifies fats and prepares them for enzyme digestion in order for the nutrient to be absorbed into lymph and eventually into blood vessels to the liver. Vitamin K (the blood-clotting vitamin) is synthesized by intestinal bacteria.

Absorption of vitamin B12 may be decreased in older adults because of decreased A. intestinal motility. B. production of bile by the liver. C. production of intrinsic factor by the stomach. D. synthesis of cobalamin (vitamin B12) by intestinal bacteria.

C. production of intrinsic factor by the stomach. Older persons are at risk for deficiency of cobalamin (pernicious anemia) because of a naturally occurring reduction of the intrinsic factor by the stomach mucosa. Absorption of cobalamin relies on intrinsic factor. Both must be present for absorption. Megaloblastic anemia is related to folate dysfunction. Intestinal motility (peristalsis) is the motion that moves food down the GI tract. The rhythmic contractions of muscles cause wave-like motions. Lack of peristalsis is called "paralytic ileus." Bile is produced in the liver, is stored and concentrated in the gallbladder, and is released into the duodenum when fat is eaten. Bile emulsifies fats and prepares them for enzyme digestion in order for the nutrient to be absorbed into lymph and eventually into blood vessels to the liver. Vitamin K (the blood-clotting vitamin) is synthesized by intestinal bacteria.

9. In older adults, infection after exposure to respiratory illness is most likely to A. result in similar rates of infection as in the younger adult. B. be easily prevented with the use of antibiotics after being exposed. C. result in serious lower respiratory infection related to weakened respiratory muscles and fewer cilia. D. be less serious because the older adult has less contact with younger children who are most likely to carry serious infections.

C. result in serious lower respiratory infection related to weakened respiratory muscles and fewer cilia. Changes in the older adult respiratory system make older adults more susceptible to infections that can be very serious and life threatening. Use of antibiotics to "prevent" lung infections is not recommended and is ineffective for viral infections.

In older adults, infection after exposure to respiratory illness is most likely to A. result in similar rates of infection as in the younger adult. B. be easily prevented with the use of antibiotics after being exposed. C. result in serious lower respiratory infection related to weakened respiratory muscles and fewer cilia. D. be less serious because the older adult has less contact with younger children who are most likely to carry serious infections.

C. result in serious lower respiratory infection related to weakened respiratory muscles and fewer cilia. Changes in the older adult respiratory system make older adults more susceptible to infections that can be very serious and life threatening. Use of antibiotics to "prevent" lung infections is not recommended and is ineffective for viral infections.

38. In teaching the patient with COPD about the need for physical exercise, the nurse informs the patient that A. all patients with COPD should be able to increase walking gradually up to 20 min/day. B. a bronchodilator inhaler should be used to relieve exercise-induced dyspnea immediately after exercise. C. shortness of breath is expected during exercise but should return to baseline within 5 minutes after the exercise. D. monitoring the heart rate before and after exercise is the best way to determine how much exercise can be tolerated.

C. shortness of breath is expected during exercise but should return to baseline within 5 minutes after the exercise.Shortness of breath usually increases during exercise, but the activity is not being overdone if breathing returns to baseline within 5 minutes after stopping. Bronchodilators can be administered 10 minutes before exercise but should not be administered for at least 5 minutes after activity to allow recovery. Patients are encouraged to walk 15 to 20 minutes a day with gradual increases, but actual patterns will depend on patient tolerance. Dyspnea most frequently limits exercise and is a better indication of exercise tolerance than is heart rate in the patient with COPD.

Client census is often used to determine staffing needs. Which method of obtaining census determination for a particular unit provides the best formula for determining long-range staffing patterns? A. Midnight census B.Oncoming shift census C.Average daily census D.Hourly census

C.Average daily census Rationale: An average daily census is determined by trend data and takes into account seasonal and daily fluctuations, so it is the best method for determining staffing needs. Options A and B provide data at a certain point in time, and that data could change quickly. It is unrealistic to expect to obtain an hourly census, and such data would only provide information about a certain point in time.

In teaching the patient with COPD about the need for physical exercise, the nurse informs the patient that A. all patients with COPD should be able to increase walking gradually up to 20 min/day. B. a bronchodilator inhaler should be used to relieve exercise-induced dyspnea immediately after exercise. C. shortness of breath is expected during exercise but should return to baseline within 5 minutes after the exercise. D. monitoring the heart rate before and after exercise is the best way to determine how much exercise can be tolerated.

C. shortness of breath is expected during exercise but should return to baseline within 5 minutes after the exercise.Shortness of breath usually increases during exercise, but the activity is not being overdone if breathing returns to baseline within 5 minutes after stopping. Bronchodilators can be administered 10 minutes before exercise but should not be administered for at least 5 minutes after activity to allow recovery. Patients are encouraged to walk 15 to 20 minutes a day with gradual increases, but actual patterns will depend on patient tolerance. Dyspnea most frequently limits exercise and is a better indication of exercise tolerance than is heart rate in the patient with COPD.

The nurse is preparing an older male client with Parkinson's disease for discharge to home. Which instruction should the nurse provide to promote independence and reduce risk for injury? (select all that apply) A. use a long stride when walking B. try to swing arms while walking C. take scheduled medication on time D. place small rugs over tile flooring E. consider use of recliner lift chair

C. take scheduled medication on time E. consider use of recliner lift chair

16. If a patient with blood type O Rh- is given AB Rh- blood, the nurse would expect A. the patient's Rh factor to react with the RBCs of the donor blood. B. no adverse reaction because the patient has no antibodies against the donor blood. C. the anti-A and anti-B antibodies in the patient's blood to hemolyze the donor blood. D. the anti-A and anti-B antibodies in the donor blood to hemolyze the patient's blood.

C. the anti-A and anti-B antibodies in the patient's blood to hemolyze the donor blood. A patient with O Rh+ blood has no A or B antigens on the red cell but does have anti-A and anti-B antibodies in the blood and has an Rh antigen. AB Rh- blood has both A and B antigens on the red cell but no Rh antigen and no anti-A or anti-B antibodies. If the AB Rh- blood is given to the patient with O Rh+ blood, the antibodies in the patient's blood will react with the antigens in the donor blood, causing hemolysis of the donor cells. There will be no Rh reaction because the donor blood has no Rh antigen.

If a patient with blood type O Rh- is given AB Rh- blood, the nurse would expect A. the patient's Rh factor to react with the RBCs of the donor blood. B. no adverse reaction because the patient has no antibodies against the donor blood. C. the anti-A and anti-B antibodies in the patient's blood to hemolyze the donor blood. D. the anti-A and anti-B antibodies in the donor blood to hemolyze the patient's blood.

C. the anti-A and anti-B antibodies in the patient's blood to hemolyze the donor blood. A patient with O Rh+ blood has no A or B antigens on the red cell but does have anti-A and anti-B antibodies in the blood and has an Rh antigen. AB Rh- blood has both A and B antigens on the red cell but no Rh antigen and no anti-A or anti-B antibodies. If the AB Rh- blood is given to the patient with O Rh+ blood, the antibodies in the patient's blood will react with the antigens in the donor blood, causing hemolysis of the donor cells. There will be no Rh reaction because the donor blood has no Rh antigen.

14. Upon entering the room of a patient who has just returned from surgery for total laryngectomy and radical neck dissection, a nurse should recognize a need for intervention when finding A. a gastrostomy tube that is clamped. B. the patient coughing blood-tinged secretions from the tracheostomy. C. the patient positioned in a lateral position with the head of the bed flat. D. 200 ml of serosanguineous drainage in the patient's portable drainage device.

C. the patient positioned in a lateral position with the head of the bed flat. After total laryngectomy and radical neck dissection, a patient should be placed in a semi-Fowler's position to decrease edema and limit tension on the suture line.

Upon entering the room of a patient who has just returned from surgery for total laryngectomy and radical neck dissection, a nurse should recognize a need for intervention when finding A. a gastrostomy tube that is clamped. B. the patient coughing blood-tinged secretions from the tracheostomy. C. the patient positioned in a lateral position with the head of the bed flat. D. 200 ml of serosanguineous drainage in the patient's portable drainage device.

C. the patient positioned in a lateral position with the head of the bed flat. After total laryngectomy and radical neck dissection, a patient should be placed in a semi-Fowler's position to decrease edema and limit tension on the suture line.

A nurse is caring for a client who is in the oliguric-anuric stage of acute kidney injury. The client reports diarrhea, a dull headache, palpitations, and muscle tingling and weakness. Which of the following actions should the nurse take first?

Check the client's electrolyte values

The nurse is caring for a critically ill client with cirrhosis of the liver who has a nasogastric tube draining bright red blood. The nurse notes that the client's serum hemoglobin and hematocrit levels are decreased. Which additional change in laboratory data should the nurse expect? A.Increased serum albumin level B.Decreased serum creatinine C.Decreased serum ammonia level D.Increased liver function test results

C.Decreased serum ammonia level Rationale: The breakdown of glutamine in the intestine and the increased activity of colonic bacteria from the digestion of proteins increase ammonia levels in clients with advanced liver disease, so removal of blood, a protein source, from the intestine results in a reduced level of ammonia. Options A, B, and D will not be significantly affected by the removal of blood.

The home health nurse is assessing a male client being treated for Parkinson disease with carbidopa-levodopa. The nurse observes that he does not demonstrate any apparent emotion when speaking and rarely blinks. Which intervention should the nurse implement? A.Perform a complete cranial nerve assessment. B.Instruct the client that he may be experiencing medication toxicity. C.Document the presence of these assessment findings. D.Advise the client to seek immediate medical evaluation.

C.Document the presence of these assessment findings Rationale: A masklike expression and infrequent blinking are common clinical features of parkinsonism. The nurse should document these expected findings. Options A and D are not necessary. Signs of toxicity of levodopa-carbidopa, include dyskinesia, hallucinations, and psychosis.

What is the correct location for placement of the hands for manual chest compressions during cardiopulmonary resuscitation (CPR) on the adult client? A.Just above the xiphoid process, on the upper third of the sternum B.Below the xiphoid process, midway between the sternum and the umbilicus C. Just above the xiphoid process, on the lower third of the sternum D. Below the xiphoid process, midway between the sternum and the first rib

C.Just above the xiphoid process, on the lower third of the sternum Rationale: The correct placement of the hands for chest compressions in CPR is just above the notch where the ribs meet the sternum on the lower part of the sternum. Option A is too high. Option B would not compress the heart. Option D would likely cause damage to both structures, possibly causing a puncture of the heart, and would not render effective compressions.

During report, the nurse learns that a client with tumor lysis syndrome is receiving an IV infusion containing insulin. Which assessment should the nurse complete first? A.Review the client's history for diabetes mellitus. B.Observe the extremity distal to the IV site C.Monitor the client's serum potassium and blood glucose levels. D.Evaluate the client's oxygen saturation and breath sounds.

C.Monitor the client's serum potassium and blood glucose levels. Rationale: Clients with tumor lysis syndrome may experience hyperkalemia, requiring the addition of insulin to the IV solution to reduce the serum potassium level. It is most important for the nurse to monitor the client's serum potassium and blood glucose levels to ensure that they are not at dangerous levels. Options A, B, and D provide valuable assessment data but are of less priority than option C.

In assessing a client with an arteriovenous (AV) shunt who is scheduled for dialysis today, the nurse notes the absence of a thrill or bruit at the shunt site. What action should the nurse take? A.Advise the client that the shunt is intact and ready for dialysis as scheduled. B.Encourage the client to keep the shunt site elevated above the level of the heart. C.Notify the health care provider of the findings immediately. D.Flush the site at least once with a heparinized saline solution.

C.Notify the health care provider of the findings immediately. Rationale: Absence of a thrill or bruit indicates that the shunt may be obstructed. The nurse should notify the health care provider so that intervention can be initiated to restore function of the shunt. Option A is incorrect. Option B will not resolve the obstruction. An AV shunt is internal and cannot be flushed without access using special needles.

Which age-related change in the older adult has the highest safety implication for the client? A.Change in height B.Hair loss C.Stooped posture D.Age spots

C.Stooped posture Rationale: Stooped posture results in the upper torso becoming the center of gravity for older persons. The center of gravity for adults is the hips. However, as a person grows older, a stooped posture is common because of changes caused by osteoporosis and normal bone degeneration. Furthermore, the knees, hips, and elbows flex. This age-related change can put the older adult at risk for falls. Options A, B, and D are age-related changes but are not high safety concerns.

A home health nurse knows that a 70-year-old male client who is convalescing at home following a hip replacement is at risk for developing pressure ulcers. Which physical characteristic of aging puts the client at risk? A.16% increase in overall body fat B.Reduced melanin production C.Thinning of the skin, with loss of elasticity D.Calcium loss in the bones

C.Thinning of the skin, with loss of elasticity Rationale: Thin nonelastic skin is an important factor in pressure formation. The proportion of body fat to lean mass increases with age and might help decrease ulcer tendency. Option B causes gray hair. Option D can contribute to broken bones, but it is probably not a factor in pressure ulcer formation.

A nurse caring for a client at risk for increased intracranial pressure is monitoring the client for manifestations that indicate that the pressure is increasing. To do this, the nurse should check the function of the third cranial nerve by performing which of the following data-collection activities? 1) Observing for facial asymmetry 2) Checking pupillary responses to light 3) Eliciting the gag reflex 4) Testing visual acuity

Checking pupillary responses to light

A nurse is caring for a client who comes to the clinic to be tested for tuberculosis (TB) after a close family contact tests positive. Which of the following measures should the nurse anticipate preparing for this client? 1) Tuberculin skin test 2) Sputum culture for acid fast bacillus (AFB) 3) BacilleCalmette-Guérin (bCG) vaccine 4) Chest x-ray

Chest x-ray

The nurse assesses a client who has been prescribed furosemide (Lasix) for cardiac disease. Which electrocardiographic change would be a concern for a client taking a diuretic? A. Tall, spiked T waves B. A prolonged QT interval C. A widening QRS complex D. Presence of a U wave

D. Presence of a U wave Rationale: A U wave (D) is a positive deflection following the T wave and is often present with hypokalemia (low potassium level). (A, B, and C) are all signs of hyperkalemia.

A client reports unprotected sexual intercourse one week ago and is worried about HIV exposure. An initial HIV antibody screen (ELISA) is obtained. The nurse teaches the client that seroconversion to HIV positive relies on antibody production by B lymphocytes after exposure to the virus. When should the nurse recommend the client return for repeat blood testing? A) 6 to 18 months. B) 1 to 12 months. C) 1 to 18 weeks. D) 6 to 12 weeks.

D) 6 to 12 weeks. Although the HIV antigen is detectable approximately 2 weeks after exposure, seroconversion to HIV positive may take up to 6 to 12 weeks (D) after exposure, so the client should return to repeat the serum screen for the presence of HIV antibodies during that time frame. (A) will delay treatment if the client tests positive. (B and C) may provide inaccurate results because the time frame maybe too early to reevaluate the client.

A client who is HIV positive asks the nurse, "How will I know when I have AIDS?" Which response is best for the nurse to provide? A) Diagnosis of AIDS is made when you have 2 positive ELISA test results. B) Diagnosis is made when both the ELISA and the Western Blot tests are positive. C) I can tell that you are afraid of being diagnosed with AIDS. Would you like for me to call your minister? D) AIDS is diagnosed when a specific opportunistic infection is found in an otherwise healthy individual.

D) AIDS is diagnosed when a specific opportunistic infection is found in an otherwise healthy individual. AIDS is diagnosed when one of several processes defined by the CDC is present in an individual who is not otherwise immunosuppressed (D) (PCP, candidacies, crytpococcus, cryptosporidiosis, Kaposi's sarcoma, CNS lymphomas). (A and B) identify the presence of HIV, indicating a high probability that in time the individual will develop AIDS, but do not necessarily denote the presence of AIDS. (C) is telling the client how he/she feels (afraid) and is dismissing the situation to the minister. This client is asking a question and specific medical information needs to be provided.

A client with diabetes mellitus is experiencing polyphagia. Which outcome statement is the priority for this client? A) Fluid and electrolyte balance. B) Prevention of water toxicity. C) Reduced glucose in the urine. D) Adequate cellular nourishment.

D) Adequate cellular nourishment. Diabetes mellitus Type 1 is characterized by hyperglycemia that precipitates glucosuria and polyuria (frequent urination), polydipsia (excessive thirst), and polyphagia (excessive hunger). Polyphagia is a consequence of cellular malnourishment when insulin deficiency prevents utilization of glucose for energy, so the outcome statement should include stabilization of adequate cellular nutrition (D). (A, B, and C) relate to subsequent osmolar fluid shifts related to glucosuria, polyuria, and polydipsia.

Two days postoperative, a male client reports aching pain in his left leg. The nurse assesses redness and warmth on the lower left calf. What intervention should be most helpful to this client? A) Apply sequential compression devices (SCDs) bilaterally. B) Assess for a positive Homan's sign in each leg. C) Pad all bony prominences on the affected leg. D) Advise the client to remain in bed with the leg elevated.

D) Advise the client to remain in bed with the leg elevated. The client is exhibiting symptoms of deep vein thrombosis (DVT), a complication of immobility. The initial care includes bedrest and elevation of the extremity (D). SCDs are used to prevent thrombophlebitis, not for treatment, when a clot might be dislodged (A). Once a client has thrombophlebitis, (B) is contraindicated because of the possibility of dislodging a clot. (C) is indicated to prevent pressure ulcers, but is not a therapeutic action for thrombophlebitis.

The nurse is assisting a client out of bed for the first time after surgery. What action should the nurse do first? A) Place a chair at a right angle to the bedside. B) Encourage deep breathing prior to standing. C) Help the client to sit and dangle legs on the side of the bed. D) Allow the client to sit with the bed in a high Fowler's position.

D) Allow the client to sit with the bed in a high Fowler's position. The first step is to raise the head of the bed to a high Fowler's position (D), which allow venous return to compensate from lying flat and vasodilating effects of perioperative drugs. (A, B, and C) are implemented after (D).

A client taking furosemide (Lasix), reports difficulty sleeping. What question is important for the nurse to ask the client? A) What dose of medication are you taking? B) Are you eating foods rich in potassium? C) Have you lost weight recently? D) At what time do you take your medication?

D) At what time do you take your medication? The nurse needs to first determine at what time of day the client takes the Lasix (D). Because of the diuretic effect of Lasix, clients should take the medication in the morning to prevent nocturia. The actual dose of medication (A) is of less importance than the time taken. (B) is not related to the insomnia. (C) is valuable information about the effect of the diuretic, but is not likely to be related to insomnia.

What types of medications should the nurse expect to administer to a client during an acute respiratory distress episode? A) Vasodilators and hormones. B) Analgesics and sedatives. C) Anticoagulants and expectorants. D) Bronchodilators and steroids.

D) Bronchodilators and steroids. Besides supplemental oxygen, the ARDS client needs medications to widen air passages, increase air space, and reduce alveolar membrane inflammation, i.e., bronchodilators and steroids (D). (A) would not help the condition. (B) would further depress the client and compromise the ability to breathe. Anticoagulants would be contraindicated since clotting of the blood is not yet a problem, and expectorants are not appropriate for this critically ill client (C).

A client who is fully awake after a gastroscopy asks the nurse for something to drink. After confirming that liquids are allowed, which assessment action should the nurse consider a priority? A) Listen to bilateral lung and bowel sounds. B) Obtain the client's pulse and blood pressure. C) Assist the client to the bathroom to void. D) Check the client's gag and swallow reflexes.

D) Check the client's gag and swallow reflexes. Following gastroscopy, a client should remain nothing by mouth until the effects of local anesthesia have dissipated and the airway's protective reflexes, gag and swallow reflexes, have returned (D). (A, B, and C) are not the priority before reintroducing oral fluids after a gastroscopy.

A female client is brought to the clinic by her daughter for a flu shot. She has lost significant weight since the last visit. She has poor personal hygiene and inadequate clothing for the weather. The client states that she lives alone and denies problems or concerns. What action should the nurse implement? A) Notify social services immediately of suspected elderly abuse. B) Discuss the need for mental health counseling with the daughter. C) Explain to the client that she needs to take better care of herself. D) Collect further data to determine whether self-neglect is occurring.

D) Collect further data to determine whether self-neglect is occurring. Changes in weight and hygiene may be indicators of self-neglect or neglect by family members. Further assessment is needed (D) before notifying social services (A) or discussing a need for counseling (B). Until further information is obtained, explanations about the client's needs are premature (C).

The nurse assesses a client with advanced cirrhosis of the liver for signs of hepatic encephalopathy. Which finding should the nurse consider an indication of progressive hepatic encephalopathy? A) An increase in abdominal girth. B) Hypertension and a bounding pulse. C) Decreased bowel sounds. D) Difficulty in handwriting.

D) Difficulty in handwriting. A daily record in handwriting may provide evidence of progression or reversal of hepatic encephalopathy leading to coma (D). (A) is a sign of ascites. (B) are not seen with hepatic encephalopathy. (C) does not indicate an increase in serum ammonia level which is the primary cause of hepatic encephalopathy.

A client experiencing uncontrolled atrial fibrillation is admitted to the telemetry unit. What initial medication should the nurse anticipate administering to the client? A) Xylocaine (Lidocaine). B) Procainamide (Pronestyl). C) Phenytoin (Dilantin). D) Digoxin (Lanoxin).

D) Digoxin (Lanoxin). Digoxin (Lanoxin) (D) is administered for uncontrolled, symptomatic atrial fibrillation resulting in a decreased cardiac output. Digoxin slows the rate of conduction by prolonging the refractory period of the AV node, thus slowing the ventricular response, decreasing the heart rate, and effecting cardiac output. (A, B, and C) are not indicated in the initial treatment of uncontrolled atrial fibrillation.

The nurse is teaching a female client who uses a contraceptive diaphragm about reducing the risk for toxic shock syndrome (TSS). Which information should the nurse include? (Select all that apply.) A) Remove the diaphragm immediately after intercourse. B) Wash the diaphragm with an alcohol solution. C) Use the diaphragm to prevent conception during the menstrual cycle. D) Do not leave the diaphragm in place longer than 8 hours after intercourse. E) Contact a healthcare provider a sudden onset of fever grater than 101º F appears. F) Replace the old diaphragm every 3 months.

D) Do not leave the diaphragm in place longer than 8 hours after intercourse. E) Contact a healthcare provider a sudden onset of fever grater than 101º F appears. Correct selections are (D and E). The diaphragm needs to remain against the cervix for 6 to 8 hours to prevent pregnancy but should not remain for longer than 8 hours (D) to avoid the risk of TSS. If a sudden fever occurs, the client should notify the healthcare provider (E). (A) increases the risk of pregnancy, and (B) can reduce the integrity of the barrier contraceptive but neither prevents the risk of TSS. The diaphragm should not be used during menses (C) because it obstructs the menstrual flow and is not indicated because conception does not occur during this time. (F) is not necessary.

When providing discharge teaching for a client with osteoporosis, the nurse should reinforce which home care activity? A) A diet low in phosphates. B) Skin inspection for bruising. C) Exercise regimen, including swimming. D) Elimination of hazards to home safety.

D) Elimination of hazards to home safety. Discussion about fall prevention strategies is imperative for the discharged client with osteoporosis so that advice about safety measures can be given (D). A low phosphorus diet is not recommended in the treatment of osteoporosis (A). Bruising (B) is not a related symptom to osteoporosis. Weight-bearing exercise is most beneficial for clients with osteoporosis. Swimming (C) is not a weight-bearing exercise.

A client has undergone insertion of a permanent pacemaker. When developing a discharge teaching plan, the nurse writes a goal of, "The client will verbalize symptoms of pacemaker failure." Which symptoms are most important to teach the client? A) Facial flushing. B) Fever. C) Pounding headache. D) Feelings of dizziness.

D) Feelings of dizziness. Feelings of dizziness may occur as the result of a decreased heart rate, leading to decreased cardiac output (D). (A and C) will not occur as the result of pacemaker failure. (B) may be an indication of infection postoperatively, but is not an indication of pacemaker failure.

The nurse is receiving report from surgery about a client with a penrose drain who is to be admitted to the postoperative unit. Before choosing a room for this client, which information is most important for the nurse to obtain? A) If suctioning will be needed for drainage of the wound. B) If the family would prefer a private or semi-private room. C) If the client also has a Hemovac® in place. D) If the client's wound is infected.

D) If the client's wound is infected. Penrose drains provide a sinus tract or opening and are often used to provide drainage of an abscess. The fact that the client has a penrose drain should alert the nurse to the possibility that the client is infected. To avoid contamination of another postoperative client, it is most important to place an infected client in a private room (D). A penrose drain does not require (A). Although (B) is information that should be considered, it does not have the priority of (D). (C) is used to drain fluid from a dead space and is not important in choosing a room.

A client has a staging procedure for cancer of the breast and ask the nurse which type of breast cancer has the poorest prognosis. Which information should the nurse offer the client? A) Stage II. B) Invasive infiltrating ductal carcinoma. C) T1N0M0. D) Inflammatory with peau d'orange.

D) Inflammatory with peau d'orange. Inflammatory breast cancer, which has a thickened appearance like an orange peel (peau d'orange), is the most aggressive form of breast malignancies (D). Staging classifies cancer by the extension or spread of the disease, and (A) indicates limited local spread. (B) indicates cancer cells have spread from the ducts into the surrounding breast tissue only. TNM classification is used to indicate the extent of the disease process according to tumor size, regional spread lymph nodes involvement, and metastasis, and (C) indicates early cancer with small in situ involvement, no lymph node involvement, and no distant metastases.

Despite several eye surgeries, a 78-year-old client who lives alone has persistent vision problems. The visiting nurse is discussing painting the house with the client. The nurse suggests that the edge of the steps should be painted which color? A) Black. B) White. C) Light green. D) Medium yellow.

D) Medium yellow. Yellow is the easiest for a person with failing vision to see (D). (A) will be almost impossible to see at night because the shadows of the steps will be too difficult to determine, and would pose a safety hazard. (B) is very hard to see with a glare from the sun and it could hurt the eyes in the daytime to look at them. (C) is a pastel color and is difficult for elderly clients to see.

How should the nurse position the electrodes for modified chest lead one (MCL I) telemetry monitoring? A) Positive polarity right shoulder, negative polarity left shoulder, ground left chest nipple line. B) Positive polarity left shoulder, negative polarity right chest nipple line, ground left chest nipple line. C) Positive polarity right chest nipple line, negative polarity left chest nipple line, ground left shoulder. D) Negative polarity left shoulder, positive polarity right chest nipple line, ground left chest nipple line.

D) Negative polarity left shoulder, positive polarity right chest nipple line, ground left chest nipple line. In MCL I monitoring, the positive electrode is placed on the client's mid-chest to the right of the sternum, and the negative electrode is placed on the upper left part of the chest (D). The ground may be placed anywhere, but is usually placed on the lower left portion of the chest. (A, B, and C) describe incorrect placement of electrodes for telemetry monitoring.

What instruction should the nurse give a client who is diagnosed with fibrocystic changes of the breast? A) Observe cyst size fluctuations as a sign of malignancy. B) Use estrogen supplements to reduce breast discomfort. C) Notify the healthcare provider if whitish nipple discharge occurs. D) Perform a breast self-exam (BSE) procedure monthly.

D) Perform a breast self-exam (BSE) procedure monthly. Fibrocystic changes in the breast are related to excess fibrous tissue, proliferation of mammary ducts and cyst formation that cause edema and nerve irritation. These changes obscure typical diagnostic tests, such as mammography, due to an increased breast density. Women with fibrocystic breasts should be instructed to carefully perform monthly BSE (D) and consider changes in any previous "lumpiness." Fibrocystic disease does not increase the risk of breast cancer (A). Cyst size fluctuates with the menstrual cycle, and typically lessens after menopause, and responds with a heightened sensitivity to circulating estrogen (B), which is not indicated. Nipple discharge associated with fibrocystic breasts is often milky or watery-milky and is an expected finding (C).

During a health fair, a 72-year-old male client tells the nurse that he is experiencing shortness of breath. Auscultation reveals crackles and wheezing in both lungs. Suspecting that the client might have chronic bronchitis, which classic symptom should the nurse expect this client to have? A) Racing pulse with exertion. B) Clubbing of the fingers. C) An increased chest diameter. D) Productive cough with grayish-white sputum.

D) Productive cough with grayish-white sputum. Chronic bronchitis, one of the diseases comprising the diagnosis of COPD, is characterized by a productive cough with grayish-white sputum (D), which usually occurs in the morning and is often ignored by smokers. (A) is not related to chronic bronchitis; however, it is indicative of other problems such as ventricular tachycardia and should be explored. (B and C) are symptoms of emphysema and are not consistent with the other symptoms. (C) is usually referred to as a "barrel chest."

The healthcare provider prescribes aluminum and magnesium hydroxide (Maalox), 1 tablet PO PRN, for a client with chronic renal failure who is complaining of indigestion. What intervention should the nurse implement? A) Administer 30 minutes before eating. B) Evaluate the effectiveness 1 hour after administration. C) Instruct the client to swallow the tablet whole. D) Question the healthcare provider's prescription.

D) Question the healthcare provider's prescription. Magnesium agents are not usually used for clients with renal failure due to the risk of hypermagnesemia, so this prescription should be questioned by the nurse (D). (A, B, and C) are not recommended nursing actions for the administration of aluminum and magnesium hydroxide (Maalox).

A client with cirrhosis develops increasing pedal edema and ascites. What dietary modification is most important for the nurse to teach this client? A) Avoid high carbohydrate foods. B) Decrease intake of fat soluble vitamins. C) Decrease caloric intake. D) Restrict salt and fluid intake.

D) Restrict salt and fluid intake. Salt and fluid restrictions are the first dietary modifications for a client who is retaining fluid as manifested by edema and ascites (D). (A, B, and C) will not impact fluid retention.

In preparing a discharge plan for a 22-year-old male client diagnosed with Buerger's disease (thromboangiitis obliterans), which referral is most important? A) Genetic counseling. B) Twelve-step recovery program. C) Clinical nutritionist. D) Smoking cessation program.

D) Smoking cessation program. Buerger's disease is strongly related to smoking. The most effective means of controlling symptoms and disease progression is through smoking cessation (D). The cause of Buerger's disease is unknown; a genetic predisposition is possible, but (A) will not be of value. The client with Buerger's disease does not need referral to a 12-step program any more than the general population (B). Diet is not a significant factor in the disease, and general healthy diet guidelines can be provided by the nurse (C).

A 77-year-old female client is admitted to the hospital. She is confused, has no appetite, is nauseated and vomiting, and is complaining of a headache. Her pulse rate is 43 beats per minute. Which question is a priority for the nurse to ask this client or her family on admission? "Does the client A) have her own teeth or dentures?" B) take aspirin and if so, how much?" C) take nitroglycerin?" D) take digitalis?"

D) take digitalis?" Elderly persons are particularly susceptible to digitalis intoxication (D) which manifests itself in such symptoms as anorexia, nausea, vomiting, diarrhea, headache, and fatigue. Although it is important to obtain a complete medication history (B and C), the symptoms described are classic for digitalis toxicity, and assessment of this problem should be made promptly. (A) is irrelevant.

11. The nurse evaluates that discharge teaching for a patient hospitalized with pneumonia has been most effective when the patient states which of the following measures to prevent a relapse? A. "I will increase my food intake to 2400 calories a day to keep my immune system well." B. "I must use home oxygen therapy for 3 months and then will have a chest x-ray to reevaluate." C. "I will seek immediate medical treatment for any upper respiratory infections." D. "I should continue to do deep-breathing and coughing exercises for at least 6 weeks."

D. "I should continue to do deep-breathing and coughing exercises for at least 6 weeks." It is important for the patient to continue with coughing and deep breathing exercises for 6 to 8 weeks until all of the infection has cleared from the lungs. A patient should seek medical treatment for upper respiratory infections that persist for more than 7 days. Increased fluid intake, not caloric intake, is required to liquefy secretions. Home O2 is not a requirement unless the patient's oxygenation saturation is below normal.

The nurse evaluates that discharge teaching for a patient hospitalized with pneumonia has been most effective when the patient states which of the following measures to prevent a relapse? A. "I will increase my food intake to 2400 calories a day to keep my immune system well." B. "I must use home oxygen therapy for 3 months and then will have a chest x-ray to reevaluate." C. "I will seek immediate medical treatment for any upper respiratory infections." D. "I should continue to do deep-breathing and coughing exercises for at least 6 weeks."

D. "I should continue to do deep-breathing and coughing exercises for at least 6 weeks." It is important for the patient to continue with coughing and deep breathing exercises for 6 to 8 weeks until all of the infection has cleared from the lungs. A patient should seek medical treatment for upper respiratory infections that persist for more than 7 days. Increased fluid intake, not caloric intake, is required to liquefy secretions. Home O2 is not a requirement unless the patient's oxygenation saturation is below normal.

The nurse evaluates that teaching for the patient with iron deficiency anemia has been effective when the patient states A. "I will need to take the iron supplements the rest of my life." B. "I will increase my dietary intake of milk and milk products." C. "I should increase my activity to increase my aerobic capacity." D. "I should take the iron for several months after my blood is normal."

D. "I should take the iron for several months after my blood is normal." To replace the body's iron stores, iron supplements should be continued for 2 to 3 months after the Hb level returns to normal, but if the cause of the iron deficiency is corrected, the supplements do not need to be taken for a lifetime. Milk and milk products are poor sources of dietary iron. Activity should be gradually increased as Hb levels return to normal because aerobic capacity can be increased when adequate Hb is available.

The nurse is giving preoperative instructions to a 14-year-old client scheduled for surgery to correct a spinal curvature. Which statement by the client best demonstrates that learning has taken place? A. "I will read all the teaching booklets you gave me before surgery." B. "I have had surgery before, so I know what to expect afterward." C. "All the things people have told me will help me take care of my back." D. "Let me show you the method of turning I will use after surgery."

D. "Let me show you the method of turning I will use after surgery." Rationale: The outcome of learning is best demonstrated when the client not only verbalizes an understanding but can also provide a return demonstration (D). A 14-year-old client may or may not follow through with (A), and there is no measurement of learning. (B) may help the client understand the surgical process, but the type of surgery may have been very different, with differing postoperative care. In (C), the client may be saying what the nurse wants to hear without expressing any real understanding of what to do after surgery.

31. The nurse determines that the patient understood medication instructions about the use of a spacer device when taking inhaled medications after hearing the patient state which of the following as the primary benefit? A. "Now I will not need to breathe in as deeply when taking the inhaler medications." B. "This device will make it so much easier and faster to take my inhaled medications." C. "I will pay less for medication because it will last longer." D. "More of the medication will get down into my lungs to help my breathing."

D. "More of the medication will get down into my lungs to help my breathing." A spacer assists more medication to reach the lungs, with less being deposited in the mouth and the back of the throat.

4. The nurse is teaching a patient who is to undergo bone marrow aspiration. Which of the following statements made by the nurse would indicate correct instruction regarding the site for the aspiration procedure? A. "The health care provider will perform the aspiration by needle to the femur." B. "The health care provider will perform the aspiration by needle to the scapula." C. "The health care provider will perform the aspiration by needle to the antecubital fossa." D. "The health care provider will perform the aspiration by needle to the posterior iliac crest."

D. "The health care provider will perform the aspiration by needle to the posterior iliac crest." Bone marrow samples are commonly taken from the posterior iliac crest or, as an alternative, the sternum may be aspirated. These sites provide relative ease in accessing the bone marrow via the biopsy needle. The antecubital fossa, femur, and scapula do not allow access to bone marrow while also providing reduced risk of harm to the patient.

The nurse plans to administer 5,00o units of heparin, an anticoagulant. Which procedure should the nurse implement when administering this drug? A. Massage injection site after administration to ensure that the solution is dissolved B. Prior to injecting the solution, check for bleeding by aspirating the plunger C. Administer IM injections into the fatty portion of the upper arm D. Assess all needle insertion sites daily for hematoma and signs of inflammation

D. Assess all needle insertion sites daily for hematoma and signs of inflammation

The nurse is teaching a patient who is to undergo bone marrow aspiration. Which of the following statements made by the nurse would indicate correct instruction regarding the site for the aspiration procedure? A. "The health care provider will perform the aspiration by needle to the femur." B. "The health care provider will perform the aspiration by needle to the scapula." C. "The health care provider will perform the aspiration by needle to the antecubital fossa." D. "The health care provider will perform the aspiration by needle to the posterior iliac crest."

D. "The health care provider will perform the aspiration by needle to the posterior iliac crest." Bone marrow samples are commonly taken from the posterior iliac crest or, as an alternative, the sternum may be aspirated. These sites provide relative ease in accessing the bone marrow via the biopsy needle. The antecubital fossa, femur, and scapula do not allow access to bone marrow while also providing reduced risk of harm to the patient.

While the school nurse is teaching a group of 14-year-olds, one of the participants remarks, "You are too young to be our teacher! You're not much older than we are!" How should the nurse respond? A. "I think I am qualified to teach this group." B. "How old do you think I am?" C. "Do you think you can teach it any better?" D. "We need to stay focused on the topic."

D. "We need to stay focused on the topic."

A 60-year-old male client with cancer of the liver has been in a hepatic coma for the past 24 hours. On admission, the client signed a release of information to his family. His oldest son arrives from out of town and asks the nurse how his father is doing. Which response is best for the nurse to provide? A. "I know you are concerned about your father. Would you like to talk about your feelings?" B. The healthcare provider will be here this afternoon and can explain your father's condition to you and the rest of the family." C. "Your father's condition is extremely critical. Would you like me to call the hospital chaplain to talk with you?" D. "Your father has given no response for 24 hours. His condition is extremely critical at this time."

D. "Your father has given no response for 24 hours. His condition is extremely critical at this time."

5. Before starting a transfusion of packed red blood cells for an anemic patient, the nurse would arrange for a peer to monitor his or her other assigned patients for how many minutes when the nurse begins the transfusion? A. 60 B. 5 C. 30 D. 15

D. 15 As part of standard procedure, the nurse remains with the patient for the first 15 minutes after hanging a blood transfusion. Patients who are likely to have a transfusion reaction will more often exhibit signs within the first 15 minutes that the blood is infusing.

Before starting a transfusion of packed red blood cells for an anemic patient, the nurse would arrange for a peer to monitor his or her other assigned patients for how many minutes when the nurse begins the transfusion? A. 60 B. 5 C. 30 D. 15

D. 15 As part of standard procedure, the nurse remains with the patient for the first 15 minutes after hanging a blood transfusion. Patients who are likely to have a transfusion reaction will more often exhibit signs within the first 15 minutes that the blood is infusing.

42. The patient has an order for albuterol 5 mg via nebulizer. Available is a solution containing 1 mg/ml. How many milliliters should the nurse use to prepare the patient's dose? A. 0.2 B. 2.5 C. 3.75 D. 5.0

D. 5.0

The patient has an order for albuterol 5 mg via nebulizer. Available is a solution containing 1 mg/ml. How many milliliters should the nurse use to prepare the patient's dose? A. 0.2 B. 2.5 C. 3.75 D. 5.0

D. 5.0

1. The arterial blood gas (ABG) readings that indicate compensated respiratory acidosis are a PaCO2 of A. 30 mm Hg and bicarbonate level of 24 mEq/L. B. 30 mm Hg and bicarbonate level of 30 mEq/L. C. 50 mm Hg and bicarbonate level of 20 mEq/L. D. 50 mm Hg and bicarbonate level of 30 mEq/L.

D. 50 mm Hg and bicarbonate level of 30 mEq/L. If compensation is present, carbon dioxide and bicarbonate are abnormal (or nearly so) in opposite directions (e.g., one is acidotic and the other alkalotic).

The arterial blood gas (ABG) readings that indicate compensated respiratory acidosis are a PaCO2 of A. 30 mm Hg and bicarbonate level of 24 mEq/L. B. 30 mm Hg and bicarbonate level of 30 mEq/L. C. 50 mm Hg and bicarbonate level of 20 mEq/L. D. 50 mm Hg and bicarbonate level of 30 mEq/L.

D. 50 mm Hg and bicarbonate level of 30 mEq/L. If compensation is present, carbon dioxide and bicarbonate are abnormal (or nearly so) in opposite directions (e.g., one is acidotic and the other alkalotic).

The nurse know that a client taking diuretics must be assessed for the development of hypokalemia, and that hypokalemia will create changes in the client's normal ECG tracing. Which ECG change would be an expected finding in the client with hypokalemia? A. Tall, spiked T waves B. A prolonged QT interval C. A widening QRS complex D. Presence of a U wave

D. A U wave is a positive deflection following the T wave and is often present with hypokalemia. A, B, C indicate hyperkalemia.

What is the goal when planning nursing care for a client with edema and leg discoloration secondary to chronic venous insufficiency? A. Adequate oxygenation will be restored B. Client will manifest normal urine output C. Client will demonstrate improved fluid-balance D. Client's skin integrity will remain intact

D. Client's skin integrity will remain intact

The nurse witnesses a baseball player receive a blunt trauma to the back of the head with a softball. What assessment data should the nurse collect immediately? A. Reactivity of deep tendon reflexes, comparing upper with lower extremities B. Vital sign readings, excluding blood pressure if needed equipment is unavailable C. Memory of events that occurred before and after the blow to the head D. Ability to open the eyes spontaneously before any tactile stimuli are given

D. Ability to open the eyes spontaneously before any tactile stimuli are given Rationale: The level of consciousness (LOC) should be established immediately when a head injury has occurred. Spontaneous eye opening is a simple measure of alertness that indicates that arousal mechanisms are intact. Option A is not the best indicator of LOC. Although option B is important, vital signs are not the best indicators of LOC and can be evaluated after the client's LOC has been determined. Option C can be assessed after LOC has been established by assessing eye opening

A 74-year-old male client is admitted to the intensive care unit (ICU) with a diagnosis of respiratory failure secondary to pneumonia. Currently, he is ventilator-dependent, with settings of tidal volume (VT) of 750 mL and an intermittent mandatory ventilation (IMV) rate of 10 breaths/min. Arterial blood gas (ABG) results are as follows: pH, 7.48; Paco2, 30 mm Hg; Pao2, 64 mm Hg; HCO3, 25 mEq/L; and Fio2, 0.80. Which intervention should the nurse implement first? A. Increase the ventilator VT to 850 mL. B. Decrease the ventilator IMV to a rate of 8 breaths/min. C. Reduce the Fio2 to 0.70 and redraw ABGs. D. Add 5 cm positive end-expiratory pressure (PEEP).

D. Add 5 cm positive end-expiratory pressure (PEEP). Rationale: Adding PEEP (D) helps improve oxygenation while reducing Fio2 to a less toxic level. (A, B, and C) will not result in improved oxygenation and could cause further complications for this client, who is experiencing respiratory failure.

A 74-year-old male client is admitted to the intensive care unit (ICU) with a diagnosis of respiratory failure secondary to pneumonia. Currently, he is ventilator-dependent, with settings of tidal volume (VT) of 750 mL and an intermittent mandatory ventilation (IMV) rate of 10 breaths/min. Arterial blood gas (ABG) results are as follows: pH, 7.48; PaCO2, 30 mm Hg; PaO2, 64 mm Hg; HCO3, 25 mEq/L; and FiO2, 0.80. Which intervention should the nurse implement first? A.Increase the ventilator VT to 850 mL. B.Decrease the ventilator IMV to a rate of 8 breaths/min. C. Reduce the FiO2 to 0.70 and redraw ABGs. D.Add 5 cm positive end-expiratory pressure (PEEP).

D. Add 5 cm positive end-expiratory pressure (PEEP). Rationale: Adding PEEP helps improve oxygenation while reducing FiO2 to a less toxic level. Options A, B, and C will not result in improved oxygenation and could cause further complications for this client, who is experiencing respiratory failure.

Which client should the charge nurse on the oncology unit assign to an RN, rather than a practical nurse (PN) A. A young adult client who is experiencing fatigue while undergoing a series of external beam radiation treatments for stage 1 cancer. B. A middle-aged male client who has just undergone an excision biopsy and has been told that his tumor appears to be benign C. An adult client in remission after a series of chemotherapy treatments who is receiving intramuscular iron injections for anemia D. An elderly female client with cancer and her children who are trying to decide whether to change to palliative care measures or continue disease control

D. An elderly female client with cancer and her children who are trying to decide whether to change to palliative care measures or continue disease control

One day after a Billroth II surgery, a male client suddenly grabs his right chest and becomes pale and diaphoretic. Vital signs are assessed as blood pressure 100/80 mm Hg, pulse 110 beats/min, and respirations 36 breaths/min. Which action is most important for the nurse to take? A. Provide a paper bag for his hyperventilation. B. Administer a prescribed PRN analgesic. C. Have the client drink a glass of sweetened fruit juice. D. Apply oxygen at 2 L via nasal cannula.

D. Apply oxygen at 2 L via nasal cannula. Rationale: Pulmonary embolism and pneumothorax are risks associated with major abdominal surgery. The nurse should immediately provide oxygen while performing further assessment (D). A rapid respiratory rate should not be treated as hyperventilation (A). (B) should not be administered until more ominous causes are ruled out or treated. There is no evidence that the client is hypoglycemic (C).

One day after a Billroth II surgery, a male client suddenly grabs his right chest and becomes pale and diaphoretic. Vital signs are assessed as blood pressure 100/80 mm Hg, pulse 110 beats/min, and respirations 36 breaths/min. Which action is most important for the nurse to take? A. Provide a paper bag for his hyperventilation. B. Administer a prescribed PRN analgesic. C. Have the client drink a glass of sweetened fruit juice. D. Apply oxygen at 2 L via nasal cannula.

D. Apply oxygen at 2 L via nasal cannula. Rationale: Pulmonary embolism and pneumothorax are risks associated with major abdominal surgery. The nurse should immediately provide oxygen while performing further assessment. A rapid respiratory rate should not be treated as hyperventilation. Option B should not be administered until more ominous causes are ruled out or treated. There is no evidence that the client is hypoglycemic.

After administering a proton pump inhibitor (PPI), which action should the nurse take to evaluate the effectiveness of the medication? A. Auscultate for bowel sounds in all quadrants B. Monitor the client's serum electrolyte levels C. Measure the client's fluid intake and output D. Ask the client about gastrointestinal pain

D. Ask the client about gastrointestinal pain

A relative comes into the emergency department asking for information about a female adult client who was admitted in stable condition following a motor vehicle collision. What action should the triage nurse take? A. Determine what is happening with the client, then provide the relative with her current status B. Inform the relative that legally no information can be provided to him or her C. Ask the relative to wait in the waiting area until the healthcare provider can see him or her D. Ask the client if she would like to talk with the relative, then bring the relative to the bedside

D. Ask the client if she would like to talk with the relative, then bring the relative to the bedside

The nurse plans to administer 5,000 units of heparin, an anticoagulant. Which procedure should the nurse implement when administering this drug? A. Massage injection site after administration to ensure that the solution is dissolved. B. Prior to injecting the solution, check for bleeding by aspirating the plunger C. Administer IM injections into the fatty portion of the upper arm. D. Assess all needle insertion sites daily for hematoma and signs of inflammation

D. Assess all needle insertion sites daily for hematoma and signs of inflammation

The nurse notes that the client's drainage has decreased from 50 to 5 mL/hr 12 hours after chest tube insertion for hemothorax. What is the best initial action for the nurse to take? A. Document this expected decrease in drainage. B. Clamp the chest tube while assessing for air leaks. C. Milk the tube to remove any excessive blood clot buildup. D. Assess for kinks or dependent loops in the tubing.

D. Assess for kinks or dependent loops in the tubing. Rationale: The least invasive nursing action should be performed first to determine why the drainage has diminished (D). (A) is completed after assessing for any problems causing the decrease in drainage. (B) is no longer considered standard protocol because the increase in pressure may be harmful to the client. (C) is an appropriate nursing action after the tube has been assessed for kinks or dependent loops.

27. A patient with COPD is receiving oxygen at 2 L/min. While in the supine position for a bath, the patient complains of shortness of breath. What is the most appropriate first nursing action? A. Increase the flow of oxygen. B. Perform tracheal suctioning. C. Report this to the physician. D. Assist the patient to Fowler's position.

D. Assist the patient to Fowler's position. Breathing is easier in Fowler's position because it permits greater expansion of the chest cavity. If repositioning does not improve the situation, then oxygenation and physician reporting might be appropriate. The patient would not benefit from tracheal suctioning.

A patient with COPD is receiving oxygen at 2 L/min. While in the supine position for a bath, the patient complains of shortness of breath. What is the most appropriate first nursing action? A. Increase the flow of oxygen. B. Perform tracheal suctioning. C. Report this to the physician. D. Assist the patient to Fowler's position.

D. Assist the patient to Fowler's position. Breathing is easier in Fowler's position because it permits greater expansion of the chest cavity. If repositioning does not improve the situation, then oxygenation and physician reporting might be appropriate. The patient would not benefit from tracheal suctioning.

39. The nurse is scheduled to give a dose of ipratropium bromide by metered dose inhaler. The nurse would administer the right drug by selecting the inhaler with which of the following trade names? A. Vanceril B. Pulmicort C. AeroBid D. Atrovent

D. Atrovent The trade or brand name for ipratropium bromide, an anticholinergic medication, is Atrovent.

The nurse is scheduled to give a dose of ipratropium bromide by metered dose inhaler. The nurse would administer the right drug by selecting the inhaler with which of the following trade names? A. Vanceril B. Pulmicort C. AeroBid D. Atrovent

D. Atrovent The trade or brand name for ipratropium bromide, an anticholinergic medication, is Atrovent.

A client who has had an open cholecystectomy two weeks ago comes to the emergency department with complaints of nausea, abdominal distention, and pain. Which assessment should the nurse implement? A. Obtain a hemoccult of the client's stool B. Palpate the liver and spleen C. Perform a digital rectal exam D. Auscultate all quadrants of the abdomen

D. Auscultate all quadrants of the abdomen

7. Which of the following physical assessment findings in a patient with pneumonia best supports the nursing diagnosis of ineffective airway clearance? A. Oxygen saturation of 85% B. Respiratory rate of 28 C. Presence of greenish sputum D. Basilar crackles

D. Basilar crackles The presence of adventitious breath sounds indicates that there is accumulation of secretions in the lower airways. This would be consistent with a nursing diagnosis of ineffective airway clearance because the patient is retaining secretions.

Which of the following physical assessment findings in a patient with pneumonia best supports the nursing diagnosis of ineffective airway clearance? A. Oxygen saturation of 85% B. Respiratory rate of 28 C. Presence of greenish sputum D. Basilar crackles

D. Basilar crackles The presence of adventitious breath sounds indicates that there is accumulation of secretions in the lower airways. This would be consistent with a nursing diagnosis of ineffective airway clearance because the patient is retaining secretions.

The nurse administers an isotonic intravenous solution to a client in septic shock. Which parameter is most important for the nurse to monitor to determine if this treatment is effective? A. White blood cell count (WBC) B. Body temperature C. Hemoglobin and hematocrit D. Blood pressure

D. Blood pressure

After routine physical examination, the healthcare provider admits a woman with a history of Systemic Lupus Erythematosis (SLE) to the hospital because she has 3+ pitting ankle edema and blood in her urine. Which assessment finding warrants immediate intervention by the nurse? A. Dark, rust-colored urine B. Urine output 300 ml/hour C. Joint and muscle aches D. Blood pressure 170/91

D. Blood pressure 170/91

22. Respiratory acidosis is at highest risk in a patient with A. hypokalemia. B. pulmonary fibrosis. C. salicylate overdose. D. COPD.

D. COPD. Chronic respiratory acidosis is most commonly caused by COPD. Pulmonary fibrosis, hypokalemia, and salicylate overdose do not predispose a patient to respiratory acidosis. Hypokalemia can lead to cardiac dysrhythmias. Salicylate overdose results in central nervous system changes, and pulmonary fibrosis can result in respiratory arrest.

Respiratory acidosis is at highest risk in a patient with A. hypokalemia. B. pulmonary fibrosis. C. salicylate overdose. D. COPD.

D. COPD. Chronic respiratory acidosis is most commonly caused by COPD. Pulmonary fibrosis, hypokalemia, and salicylate overdose do not predispose a patient to respiratory acidosis. Hypokalemia can lead to cardiac dysrhythmias. Salicylate overdose results in central nervous system changes, and pulmonary fibrosis can result in respiratory arrest.

The home health nurse visits a client with heart failure (HF). Assessment findings include: temperature 97.6F, pulse 116 beats/minute, respiratory rate 36 breaths/minute, blood pressure 140/70, pulse oximeter 86% on 2 L/min of oxygen, and crackles are heard throughout the lung fields. Which intervention has the highest priority? A. Start an IV of normal saline (NS) at 100 ml/hr B. Assess for edema and weigh client C. Obtain a prescription for an increased oxygen rate D. Call 911 and prepare the client for transport

D. Call 911 and prepare the client for transport

At bedtime, an unlicensed assistive personnel (UAP) is positioning a client with obstructive sleep apnea syndrome (OSAS). The UAP elevates the head of the bed and encourages the client to turn on side. In supervising the UAP, what action should the nurse take? A. After leaving the room, discuss correct positioning with the UAP B. Reposition the client in a supine position with the feet elevated pillows C. Remind the UAP to pad the side rails to reduce risk for injury D. Confirm that the UAP has placed the call bell within reach of the client.

D. Confirm that the UAP has placed the call bell within reach of the client.

A client who is receiving an ACE inhibitor for hypertension calls the clinic and reports the recent onset of a cough to the nurse. What action should the nurse implement? A. Advise the client to come to the clinic immediately for further assessment. B. Instruct the client to discontinue use of the drug, and make an appointment at the clinic. C. Suggest that the client lear to accept the cough as a side effect to a necessary prescription. D. Encourage the client to keep taking the drug until seen by the HCP.

D. Cough is a common s/e of ACE inhibitors and is not an indication to discontinue the medication. (A) immediate evaluation is not needed. (B) an antihypertensive should not be stopped abruptly. (C) is demeaning since the cough may be disruptive to the client and other medications may produce results without the s/e.

A female client with severe renal impairment is receiving enoxaparin (Lovenox) 30 mg SUBQ BID. Which laboratory value due to enoxaparin should the nurse report to the healthcare provider? A. Calcium 9 mg/dl (9 mmol/L SI) B. Hemoglobin 12 grams/dl (120 mmol/L SI) C. Partial thromboplastin time (PTT) 30 seconds D. Creatinine clearance 25 ml/minute

D. Creatinine clearance 25 ml/minute

The nurse is assessing the emotional status of a client with Parkinson's disease. Which client finding is most helpful in planning goals to meet the client's emotional needs? A. Stares straight ahead without blinking B. Face does not convey any emotion C. Uses a monotone when speaking D. Cries frequently during the interview

D. Cries frequently during the interview

The healthcare provider prescribes Isosorbibide (Isordil) 40 mg every 8 hours for a male client with acute angina pectoris. Which finding should the nurse report to the healthcare provider prior to administering Isordil? A. Serum cholesterol of 200 mg/dl B. Hemoglobin of 14.8 grams/dl C. Takes enteric coated aspirin daily D. Currently takes sildenafil (Viagra)

D. Currently takes sildenafil (Viagra)

A 77-year-old female client is admitted to the hospital with confusion and anorexia of several days' duration. She has symptoms of nausea and vomiting and is currently complaining of a headache. The client's pulse rate is 43 beats/min. The nurse is most concerned about the client's history related to which medication? A. Warfarin (Coumadin) B. Ibuprofen (Motrin) C. Nitroglycerin (Nitrostat) D. Digoxin (Lanoxin)

D. Digoxin (Lanoxin) Rationale: Older persons are particularly susceptible to the buildup of cardiac glycosides, such as digoxin (Lanoxin) or digitoxin (medications derived from digitalis) (D), to a toxic level in their systems. Toxicity can cause anorexia, nausea, vomiting, diarrhea, headache, and fatigue. (A, B, and C) are unlikely to result in the symptoms described.

The father brings his pre-school-aged son to the rural urgent care clinic because the child fell from a horse earlier today. Which finding indicates to the nurse that further assessment is required for possible abuse or neglect? A. A dislocated shoulder and fractured wrist on X-ray B. Bruises, abrasions, and restricted movement in right shoulder, elbow, and wrist C. Father's presence during the child's assessment and physical examination D. Dry, peeling skin, ridged nails, and significantly underweight

D. Dry, peeling skin, ridged nails, and significantly underweight

The nurse is assessing a client who presents with jaundice. Which assessment finding is most important for the nurse to follow up? A. Urine specific gravity of 1.03 B. Frothy, tea-colored urine C. Clay-colored stools D. Elevated serum amylase and lipase levels

D. Elevated serum amylase and lipase levels Rationale: Obstructive cholelithiasis and alcoholism are the two major causes of pancreatitis, and elevated serum amylase and lipase levels (D) indicate pancreatic injury. (A) is a normal finding. (B and C) are expected findings related to jaundice.

The nurse is assessing a client who presents with jaundice. Which assessment finding is most important for the nurse to follow up? A. Urine specific gravity of 1.03 B. Frothy, tea-colored urine C. Clay-colored stools D. Elevated serum amylase and lipase levels

D. Elevated serum amylase and lipase levels Rationale: Obstructive cholelithiasis and alcoholism are the two major causes of pancreatitis, and elevated serum amylase and lipase levels indicate pancreatic injury. Option A is a normal finding. Options B and C are expected findings related to jaundice.

Which nursing action is necessary for the client with a flail chest? A. Withhold prescribed analgesic medications. B. Percuss the fractured rib area with light taps. C. Avoid implementing pulmonary suctioning. D. Encourage coughing and deep breathing.

D. Encourage coughing and deep breathing. Rationale: Treatment of flail chest is focused on preventing atelectasis and related complications of compromised ventilation by encouraging coughing and deep breathing (D). This condition is typically diagnosed in clients with three or more rib fractures, resulting in paradoxic movement of a segment of the chest wall. (C) should not be avoided because suctioning is necessary to maintain pulmonary toilet in clients who require mechanical ventilation. (A) should not be withheld. (B) should not be applied because the fractures are clearly visible on the chest radiograph.

A central venous catheter has been inserted via a jugular vein, and a radiograph has confirmed placement of the catheter. A prescription has been received for a medication STAT, but IV fluids have not yet been started. Which action should the nurse take prior to administering the prescribed medication? A. Assess for signs of jugular venous distention. B. Obtain the needed intravenous solution. C. Flush the line with heparinized solution. D. Flush the line with normal saline.

D. Flush the line with normal saline. Rationale: Medication can be administered via a central line without additional IV fluids. The line should first be flushed with a normal saline solution to ensure patency. Insufficient evidence exists on the effectiveness of flushing catheters with heparin. Option A will not affect the decision to administer the medication and is not a priority. Administration of the medication STAT is of greater priority than option B.

A client who is receiving an angiotensin-converting enzyme (ACE) inhibitor for hypertension calls the clinic and reports the recent onset of a cough to the nurse. Which action should the nurse implement? A. Advise the client to come to the clinic immediately for further assessment. B. Instruct the client to discontinue use of the drug and to make an appointment at the clinic. C. Suggest that the client learn to accept the cough as a side effect to a necessary prescription. D. Encourage the client to keep taking the drug until seen by the health care provider.

D. Encourage the client to keep taking the drug until seen by the health care provider. Rationale: Coughing is a common side effect of ACE inhibitors and is not an indication to discontinue the medication (D). Immediate evaluation is not needed (A). Antihypertensive medications should not be stopped abruptly (B) because rebound hypertension may occur. (C) is demeaning because the cough may be very disruptive to the client, and other antihypertensive medications may produce the desired effect without the adverse effect.

A client who is receiving an angiotensin-converting enzyme (ACE) inhibitor for hypertension calls the clinic and reports the recent onset of a cough to the nurse. Which action should the nurse implement? A. Advise the client to come to the clinic immediately for further assessment. B. Instruct the client to discontinue use of the drug and to make an appointment at the clinic. C. Suggest that the client learn to accept the cough as a side effect to a necessary prescription. D. Encourage the client to keep taking the drug until seen by the health care provider.

D. Encourage the client to keep taking the drug until seen by the health care provider. Rationale: Coughing is a common side effect of ACE inhibitors and is not an indication to discontinue the medication. Immediate evaluation is not needed. Antihypertensive medications should not be stopped abruptly because rebound hypertension may occur. Option C is demeaning because the cough may be very disruptive to the client, and other antihypertensive medications may produce the desired effect without the adverse effect.

The parents of a 6-year-old recently diagnosed with Duchenne muscular dystrophy tell the nurse that their child wants to continue attending swimming classes. How should the nurse respond? A. Explain that their child is too young to understand risks associated with swimming B. Provide a list of alternative activities that are less likely to cause the child to experience fatigue C. Suggest that the child can be encouraged to participate in a team sport to encourage socialization D. Encourage the parents to allow the child to continue attending swimming lessons with supervision

D. Encourage the parents to allow the child to continue attending swimming lessons with supervision

The healthcare provider prescribes digoxin (Lanoxin) 0.5 mg PO daily for a client with heart failure. When the nurse scans the medication label, "digoxin 0.25 mg/tablet," using an electronic scanner, a "pop-up" window in the electronic medical record indicates "Error in dose." What action should the nurse take? A. Request the pharmacy to deliver the correct dose B. Rescan the medication label until it registers C. Notify the healthcare provider of the error in the data D. Enter the value, 2 tablets, administered

D. Enter the value, 2 tablets, administered

After receiving a prescribed dose of quinapril (Accupril), losartan (Cozaar), and clonidine (Catapres), a female client tells the nurse that she usually takes Accupril at 0800, Cozaar at 1600, and clonidine at 2200 at home. The nurse informs the charge nurse that an error was made during the morning medication administration. What action should the charge nurse implement first? A. Implement orthostatic safety precautions B. Assess the client's level of consciousness C. Increase the client's fluid intake during the day D. Evaluate the clients blood pressure

D. Evaluate the clients blood pressure

22. The nurse is assigned to care for a patient who has anxiety and an exacerbation of asthma. Which of the following is the primary reason for the nurse to carefully inspect the chest wall of this patient? A. Observe for signs of diaphoresis B. Allow time to calm the patient C. Monitor the patient for bilateral chest expansion D. Evaluate the use of intercostal muscles

D. Evaluate the use of intercostal muscles The nurse physically inspects the chest wall to evaluate the use of intercostal (accessory) muscles, which gives an indication of the degree of respiratory distress experienced by the patient.

The nurse is assigned to care for a patient who has anxiety and an exacerbation of asthma. Which of the following is the primary reason for the nurse to carefully inspect the chest wall of this patient? A. Observe for signs of diaphoresis B. Allow time to calm the patient C. Monitor the patient for bilateral chest expansion D. Evaluate the use of intercostal muscles

D. Evaluate the use of intercostal muscles The nurse physically inspects the chest wall to evaluate the use of intercostal (accessory) muscles, which gives an indication of the degree of respiratory distress experienced by the patient.

A clinical trial is recommended for a client with metastatic breast cancer, but she refuses to participate and tells her family that she does not wish to have further treatments. The client's son and daughter ask the nurse to try to convince their mother to reconsider this decision. How should the nurse respond? A. ask the client with her children present if she fully understands the decision she has made B. Discuss success of clinical trials and ask the client to consider participating for one month C. Explain to the family that they must accept their mother's decision D. Explore the client's decision to refuse treatment and offer support

D. Explore the client's decision to refuse treatment and offer support

A central venous catheter has been inserted via a jugular vein, and a radiograph has confirmed placement of the catheter. A prescription has been received for a medication STAT, but IV fluids have not yet been started. Which action should the nurse take prior to administering the prescribed medication? A. Assess for signs of jugular venous distention. B. Obtain the needed intravenous solution. C. Flush the line with heparinized solution. D. Flush the line with normal saline.

D. Flush the line with normal saline. Rationale: Medication can be administered via a central line without additional IV fluids. The line should first be flushed with a normal saline solution (D) to ensure patency. Insufficient evidence exists on the effectiveness of flushing catheters with heparin (C). (A) will not affect the decision to administer the medication and is not a priority. Administration of the medication STAT is of greater priority than (B).

When planning patient teaching about emphysema, the nurse understands that the symptoms of emphysema are caused by which of the following? A. Hypertrophy and hyperplasia of goblet cells in the bronchi B. Collapse and hypoventilation of the terminal respiratory unit C. An overproduction of the antiprotease alpha1-antitrypsin D. Hyperinflation of alveoli and destruction of alveolar walls

D. Hyperinflation of alveoli and destruction of alveolar walls In emphysema, there are structural changes that include hyperinflation of alveoli, destruction of alveolar walls, destruction of alveolar capillary walls, narrowing of small airways, and loss of lung elasticity.

The nurse on a medical surgical unit is receiving a client from the postanesthesia care unit (PACU) with a Penrose drain. Before choosing a room for this client, which information is most important for the nurse to obtain? A. If suctioning will be needed for drainage of the wound B. If the family would prefer a private or semiprivate room C. If the client also has a Hemovac in place D. If the client's wound is infected

D. If the client's wound is infected Rationale: The fact that the client has a Penrose drain should alert the nurse to the possibility that the surgical wound is infected (D). Penrose drains provide a sinus tract or opening and are often used to provide drainage of an abscess. To avoid contamination of another postoperative client, it is most important to place any client with an infected wound in a private room. A Penrose drain does not require (A). Although (B) is helpful information, it does not have the priority of (D). A Hemovac (C) is used to drain fluid from a dead space and is not a determinant for the room assignment.

The nurse on a medical-surgical unit is receiving a client from the postanesthesia care unit (PACU) with a Penrose drain. Before choosing a room for this client, which information is most important for the nurse to obtain? A. If suctioning will be needed for drainage of the wound B. If the family would prefer a private or semiprivate room C. If the client also has a Hemovac in place D. If the client's wound is infected

D. If the client's wound is infected The fact that the client has a Penrose drain should alert the nurse to the possibility that the surgical wound is infected. Penrose drains provide a sinus tract or opening and are often used to provide drainage of an abscess. To avoid contamination of another postoperative client, it is most important to place any client with an infected wound in a private room. A Penrose drain does not require option A. Although option B is helpful information, it does not have the priority of option D. A Hemovac is used to drain fluid from a dead space and is not a determinant for the room assignment.

A young woman with multiple sclerosis just received several immunizations in preparation for moving into a college dormitory. Two days later, she reports to the nurse that she is experiencing increasing fatigue and visual problems. What teaching should the nurse provide? A. Plans to move into the dormitory need to be postponed for at least a semester B. These early signs of an infection may require medical treatment with antibiotics C. These are common side effects of the vaccines and will resolve in a few days D. Immunizations can trigger a relapse of the disease, so get plenty of extra rest

D. Immunizations can trigger a relapse of the disease, so get plenty of extra rest

What is the primary purpose for initiating nursing interventions that promote good nutrition, rest and exercise, and stress reduction for clients diagnosed with an HIV infection? A. Increase ability to carry out activities of daily living B. Promote a feeling of general well-being C. Prevent spread of infection to others D. Improve function of the immune system

D. Improve function of the immune system

Which is the primary purpose for initiating nursing interventions that promote good nutrition, rest and exercise, and stress reduction for clients diagnosed with an HIV infection? A. Increase ability to carry out activities of daily living B. Promote a feeling of general well-being C. Prevent spread of infection to others D. Improve function of the immune system

D. Improve function of the immune system

5. A nurse is performing assessment for a patient diagnosed with chronic obstructive pulmonary disease (COPD). Which of the following findings should the nurse expect to observe? A. Nonproductive cough B. Prolonged inspiration C. Vesicular breath sounds D. Increased anterior-posterior chest diameter

D. Increased anterior-posterior chest diameter An increased anterior-posterior diameter is a compensatory mechanism experienced by patients with COPD and is caused by air-trapping. Patients with COPD have a productive cough, often expectorating copious amounts of sputum. Because of air-trapping, patients with COPD experience a prolonged expiration because the rate of gas on exhalation takes longer to escape. Chest auscultation for patients with COPD often reveals wheezing, crackles, and other adventitious breath sounds.

A nurse is performing assessment for a patient diagnosed with chronic obstructive pulmonary disease (COPD). Which of the following findings should the nurse expect to observe? A. Nonproductive cough B. Prolonged inspiration C. Vesicular breath sounds D. Increased anterior-posterior chest diameter

D. Increased anterior-posterior chest diameter An increased anterior-posterior diameter is a compensatory mechanism experienced by patients with COPD and is caused by air-trapping. Patients with COPD have a productive cough, often expectorating copious amounts of sputum. Because of air-trapping, patients with COPD experience a prolonged expiration because the rate of gas on exhalation takes longer to escape. Chest auscultation for patients with COPD often reveals wheezing, crackles, and other adventitious breath sounds.

2. When assessing lab values on a patient admitted with septicemia, the nurse would expect to find: A. Increased platelets B. Decreased red blood cells C. Decreased erythrocyte sedimentation rate (ESR) D. Increased bands in the WBC differential (shift to the left)

D. Increased bands in the WBC differential (shift to the left) When infections are severe, such as in septicemia, more granulocytes are released from the bone marrow as a compensatory mechanism. To meet the increased demand, many young, immature polymorphonuclear neutrophils (bands) are released into circulation. WBCs are usually reported in order of maturity, with the less mature forms on the left side of a written report. Hence, the term "shift to the left" is used to denote an increase in the number of bands.

The nurse assesses a client who has been prescribed furosemide (Lasix) for cardiac disease. Which electrocardiographic change would be a concern for a client taking a diuretic? A. Tall, spiked T waves B. A prolonged QT interval C. A widening QRS complex D. Presence of a U wave

D. Presence of a U wave Rationale: A U wave is a positive deflection following the T wave and is often present with hypokalemia (low potassium level). Options A, B, and C are all signs of hyperkalemia.

When assessing lab values on a patient admitted with septicemia, the nurse would expect to find: A. Increased platelets B. Decreased red blood cells C. Decreased erythrocyte sedimentation rate (ESR) D. Increased bands in the WBC differential (shift to the left)

D. Increased bands in the WBC differential (shift to the left) When infections are severe, such as in septicemia, more granulocytes are released from the bone marrow as a compensatory mechanism. To meet the increased demand, many young, immature polymorphonuclear neutrophils (bands) are released into circulation. WBCs are usually reported in order of maturity, with the less mature forms on the left side of a written report. Hence, the term "shift to the left" is used to denote an increase in the number of bands.

The nurse is assessing a 75-year-old client for symptoms of hyperglycemia. Which symptom of hyperglycemia is an older adult most likely to exhibit? A. Polyuria B. Polydipsia C. Weight loss D. Infection

D. Infection Rationale: Signs and symptoms of hyperglycemia in older adults may include fatigue, infection (D), and evidence of neuropathy (e.g., sensory changes). The nurse needs to remember that classic signs and symptoms of hyperglycemia, such as (A, B, and C) and polyphagia, may be absent in older adults.

The nurse is assessing a 75-year-old client for symptoms of hyperglycemia. Which symptom of hyperglycemia is an older adult most likely to exhibit? A. Polyuria B. Polydipsia C.Weight loss D.Infection

D. Infection Rationale: Signs and symptoms of hyperglycemia in older adults may include fatigue, infection, and evidence of neuropathy (e.g., sensory changes). The nurse needs to remember that classic signs and symptoms of hyperglycemia, such as options A, B, and C and polyphagia, may be absent in older adults.

10. The blood bank notifies the nurse that the two units of blood ordered for an anemic patient are ready for pick up. The nurse should take which of the following actions to prevent an adverse effect during this procedure? A. Immediately pick up both units of blood from the blood bank. B. Regulate the flow rate so that each unit takes at least 4 hours to transfuse. C. Set up the Y-tubing of the blood set with dextrose in water as the flush solution. D. Infuse the blood slowly for the first 15 minutes of the transfusion.

D. Infuse the blood slowly for the first 15 minutes of the transfusion. Because a transfusion reaction is more likely to occur at the beginning of a transfusion, the nurse should initially infuse the blood at a rate no faster than 2 ml/min and remain with the patient for the first 15 minutes after hanging a unit of blood.

The blood bank notifies the nurse that the two units of blood ordered for an anemic patient are ready for pick up. The nurse should take which of the following actions to prevent an adverse effect during this procedure? A. Immediately pick up both units of blood from the blood bank. B. Regulate the flow rate so that each unit takes at least 4 hours to transfuse. C. Set up the Y-tubing of the blood set with dextrose in water as the flush solution. D. Infuse the blood slowly for the first 15 minutes of the transfusion.

D. Infuse the blood slowly for the first 15 minutes of the transfusion. Because a transfusion reaction is more likely to occur at the beginning of a transfusion, the nurse should initially infuse the blood at a rate no faster than 2 ml/min and remain with the patient for the first 15 minutes after hanging a unit of blood.

After placement of a left subclavian central venous catheter (CVC), the nurse receives report of the X-ray findings that indicate the CVC tip is in the client's superior vena cava. Which action should the nurse implement? A. Notify the healthcare provider of the need to reposition the catheter B. Remove the catheter and apply direct pressure for 5 minutes C. Secure the catheter using aseptic techniques D. Initiate intravenous fluids as prescribed

D. Initiate intravenous fluids as prescribed

Two unlicensed assistive personal (UAP) are arguing loudly in the hallway of the extended care facility about who will shower a male resident who defecated in his bed. What action is best for charge nurse to take? A. Shower the client with the help of a practical nurse B. Reassign the clients care to another staff member C. Document the conflict in the employee personnel files D. Instruct both UAP to shower the client immediately

D. Instruct both UAP to shower the client immediately

A client who is experiencing panic attacks receives a prescription for the benzodiazepine alprazolam (Xanax). Which instructions should the nurse provide the client? A. Explain that it may take up to two weeks before the anxiety starts to get better B. Instruct the client to notify the healthcare provider if tremors of the tongue occur C. Discuss the importance of obtaining monthly blood work to assess for toxicity D. Instruct the client on safety issues this medication causes drowsiness

D. Instruct the client on safety issues this medication causes drowsiness

The nurse is preparing to conduct discharge teaching for a client who had an anaphylactic reaction following administration of ampicillin (Omnipen-N). What instruction is essential for the nurse to provide this client prior to discharge? A. Inform the client that it is essential to take all of the prescribed ampicillin B. Teach the client how to self-administer epinephrine in case a reaction occurs again C. Tell the client to take medication with food to decrease the possibility of future reactions D. Instruct the client to wear a medic-alert bracelet so penicillin will not be given again

D. Instruct the client to wear a medic-alert bracelet so penicillin will not be given again

38. The nurse is evaluating whether a patient understands how to safely determine whether a metered dose inhaler is empty. The nurse interprets that the patient understands this important information to prevent medication underdosing when the patient describes which method to check the inhaler? A. Place it in water to see if it floats. B. Shake the canister while holding it next to the ear. C. Check the indicator line on the side of the canister. D. Keep track of the number of inhalations used.

D. Keep track of the number of inhalations used. It is no longer appropriate to see if a canister floats in water or not as research has demonstrated this is not accurate. The best method to determine when to replace an inhaler is by knowing the maximum puffs available per MDI and then replacing when those inhalations have been used.

The nurse determines that the patient is not experiencing adverse effects of albuterol (Proventil) after noting which of the following patient vital signs? A. Oxygen saturation 96% B. Respiratory rate of 18 C. Temperature of 98.4° F D. Pulse rate of 76

D. Pulse rate of 76 Albuterol is a β2-agonist that can sometimes cause adverse cardiovascular effects. These would include tachycardia and angina. A pulse rate of 76 indicates that the patient did not experience tachycardia as an adverse effect.

The nurse is evaluating whether a patient understands how to safely determine whether a metered dose inhaler is empty. The nurse interprets that the patient understands this important information to prevent medication underdosing when the patient describes which method to check the inhaler? A. Place it in water to see if it floats. B. Shake the canister while holding it next to the ear. C. Check the indicator line on the side of the canister. D. Keep track of the number of inhalations used.

D. Keep track of the number of inhalations used. It is no longer appropriate to see if a canister floats in water or not as research has demonstrated this is not accurate. The best method to determine when to replace an inhaler is by knowing the maximum puffs available per MDI and then replacing when those inhalations have been used.

The nurse plans to administer 1 teaspoon of a liquid medication to a toddler. What is the most accurate way to administer the medication? A. Give medication using a medication dropper B. Administer from a clean teaspoon C. Use a medicine cup to measure the dose D. Measure the medication in an oral syringe

D. Measure the medication in an oral syringe

The nurse is preparing a client for discharge from the hospital following a liver transplant. Which intervention is most important for the nurse to include in this client's discharge teaching plan? A. Keep a record of daily urinary output B. Report the onset of scleral jaundice C. Measure the abdominal girth daily D. Monitor for an elevated temperature

D. Monitor for an elevated temperature

A female client who has been taking diclofenac (Zipsor) for the past month is admitted with right upper quadrant tenderness, jaundice, and flu-like symptoms. She is also complaining of fatigue, diarrhea, and pruritus. Which intervention is most important to include in this client's plan of care? A. Review results of serum protein electrolytes B. Determine frequently of indigestion C. Evaluate intake and output ratios D. Monitor serum bilirubin levels

D. Monitor serum bilirubin levels

13. If a patient with arthritis develops iron-deficiency anemia, a nurse should ask about the patient's use of A. alcoholic beverages. B. stool softeners and laxatives. C. caffeinated foods and beverages. D. NSAIDs.

D. NSAIDs. NSAIDs decrease the level of vitamin C, which aids in the absorption of iron. These drugs also compete with folate and vitamin K and may cause gastritis. Excessive alcoholic beverage consumption can cause stomach irritation; alcohol would not be directly related to iron-deficiency anemia unless bleeding ulcers or gastritis were to occur. NSAID consumption, not stool softeners and laxative use, would be suspect for iron-deficiency anemia. Caffeinated foods and beverages can cause gastric irritation and discomfort but are not associated with iron-deficiency anemia.

A female client who received a nephrotoxic drug is admitted with acute renal failure and asks the nurse if she will need dialysis for the rest of her life. Which pathophysiologic consequence should the nurse explain that supports the need for temporary dialysis until acute tubular necrosis subsides? A. Azotemia B. Oliguria C. Hyperkalemia D. Nephron obstruction

D. Nephron obstruction Rationale: CKD is characterized by progressive and irreversible destruction of nephrons, frequently caused by hypertension and diabetes mellitus. Nephrotoxins cause acute tubular necrosis, a reversible acute renal failure, which creates renal tubular obstruction from endothelial cells that are sloughed or become edematous. The obstruction of urine flow will resolve (D) with the return of an adequate glomerular filtration rate and, when it does, dialysis will no longer be needed. (A, B, and C) are manifestations seen in the acute and chronic forms of kidney disease.

The nurse is assessing a client who presents with jaundice. Which assessment finding is the most significant indication that further follow up is needed? A. Urine specific gravity of 1.03 with a urine output of 500 ml in 8 hours B. Frothy, tea-colored urine C. Clay-colored stools and complaints of pruritus D. Serum amylase and lipase levels that are twice their normal levels

D. Obstructive cholelithiasis and alcoholism are the two major causes of pancreatitis, and an elevated serum amylase and lipase indicate pancreatic injury. (A) is a normal finding. (B & C) are expected findings for jaundice.

44. When caring for a patient with COPD, the nurse identifies a nursing diagnosis of imbalanced nutrition less than body requirements after noting a weight loss of 30 lb. Which of the following would be an appropriate intervention to add to the plan of care for this patient? A. Teach the patient to use frozen meals at home that can be microwaved. B. Provide a high-calorie, high-carbohydrate, nonirritating, frequent feeding diet. C. Order fruits and fruit juices to be offered between meals. D. Order a high-calorie, high-protein diet with six small meals a day.

D. Order a high-calorie, high-protein diet with six small meals a day.Because the patient with COPD needs to use greater energy to breathe, there is often decreased oral intake because of dyspnea. A full stomach also impairs the ability of the diaphragm to descend during inspiration, interfering with the work of breathing. Finally, the metabolism of a high carbohydrate diet yields large amounts of CO2, which may lead to acidosis in patients with pulmonary disease. For these reasons, the patient with emphysema should take in a high-calorie, high-protein diet, eating six small meals per day.

When caring for a patient with COPD, the nurse identifies a nursing diagnosis of imbalanced nutrition less than body requirements after noting a weight loss of 30 lb. Which of the following would be an appropriate intervention to add to the plan of care for this patient? A. Teach the patient to use frozen meals at home that can be microwaved. B. Provide a high-calorie, high-carbohydrate, nonirritating, frequent feeding diet. C. Order fruits and fruit juices to be offered between meals. D. Order a high-calorie, high-protein diet with six small meals a day.

D. Order a high-calorie, high-protein diet with six small meals a day.Because the patient with COPD needs to use greater energy to breathe, there is often decreased oral intake because of dyspnea. A full stomach also impairs the ability of the diaphragm to descend during inspiration, interfering with the work of breathing. Finally, the metabolism of a high carbohydrate diet yields large amounts of CO2, which may lead to acidosis in patients with pulmonary disease. For these reasons, the patient with emphysema should take in a high-calorie, high-protein diet, eating six small meals per day.

A practical nurse (PN) tells the charge nurse in a long-term facility that she does not want to be assigned to one particular resident. She reports that the male client keeps insisting that she is his daughter and begs her to stay in his room. What is the best managerial decision? A. Notify the family that the resident will have to be discharged if his behavior does not improve. B. Notify administration of the PN's insubordination and need for counseling about her statements. C. Ask the PN what she has done to encourage the resident to believe that she is his daughter. D. Reassign the PN until the resident can be assessed more completely for reality orientation.

D. Reassign the PN until the resident can be assessed more completely for reality orientation. Rationale: Temporary reassignment (D) is the best option until the resident can be examined and his medications reviewed. He may have worsening cerebral dysfunction from an infection or electrolyte imbalance. (A) is not the best option because the family cannot control the resident's actions. The administration may need to know about the situation, but not as a case of insubordination (B). Implying that the PN is somehow creating the situation is inappropriate until a further evaluation has been conducted (C).

A practical nurse (PN) tells the charge nurse in a long-term facility that she does not want to be assigned to one particular resident. She reports that the male client keeps insisting that she is his daughter and begs her to stay in his room. What is the best managerial decision? A. Notify the family that the resident will have to be discharged if his behavior does not improve. B.Notify administration of the PN's insubordination and need for counseling about her statements. C.Ask the PN what she has done to encourage the resident to believe that she is his daughter. D.Reassign the PN until the resident can be assessed more completely for reality orientation.

D. Reassign the PN until the resident can be assessed more completely for reality orientation. Rationale: Temporary reassignment is the best option until the resident can be examined and his medications reviewed. He may have worsening cerebral dysfunction from an infection or electrolyte imbalance. Option A is not the best option because the family cannot control the resident's actions. The administration may need to know about the situation, but not as a case of insubordination. Implying that the PN is somehow creating the situation is inappropriate until a further evaluation has been conducted.

A client with type 2 diabetes takes metformin (Glucophage) daily. The client is scheduled for major surgery requiring general anesthesia the next day. The nurse anticipates which approach to manage the client's diabetes best while the client is NPO during the perioperative period? A. NPO except for metformin and regular snacks B. NPO except for oral antidiabetic agent C. Novolin N insulin subcutaneously twice daily D. Regular insulin subcutaneously per sliding scale

D. Regular insulin subcutaneously per sliding scale Rationale: Regular insulin dosing based on the client's blood glucose levels (sliding scale) is the best method to achieve control of the client's blood glucose while the client is NPO and coping with the major stress of surgery (D). (A) increases the risk of vomiting and aspiration. (B and C) provide less precise control of the blood glucose level.

Following laser trabeculoplasty surgery for open-angle glaucoma, the client reports acute pain deep within the eye. What action should the nurse take? A. Apply bilateral eye shields to reduce photosensitivity B. Administer an antiemetic to prevent vomiting C. Begin postoperative prophylactic antibiotics D. Report the complaint of eye pain to the surgeon

D. Report the complaint of eye pain to the surgeon

An 18-year-old female client is admitted to the unit after ingesting an overdose of Phenobarbital (Luminal). She is unresponsive and ABG results are: pH 7.18, PaCO2 60 mmHg, and HCO3 26 mEq/L. Which interpretation of the client's ABG results by the nurse is accurate? A. Metabolic acidosis, uncompensated B. Respiratory acidosis, compensated C. Metabolic alkalosis, compensated D. Respiratory acidosis, uncompensated

D. Respiratory acidosis, uncompensated

A client with cirrhosis develops increasing pedal edema and ascites. Which dietary modification is most important for the nurse to teach this client? A. Avoid high-carbohydrate foods. B. Decrease intake of fat-soluble vitamins. C. Decrease caloric intake. D. Restrict salt and fluid intake.

D. Restrict salt and fluid intake. Rationale: Salt and fluid restrictions are the first dietary modifications for a client who is retaining fluid as manifested by edema and ascites (D). (A, B, and C) will not affect fluid retention.

The nurse receives the client's next scheduled bag of TPN labeled with the additive NPH insulin. Which action should the nurse implement? A. Hang the solution at the current rate. B. Refrigerate the solution until needed. C. Prepare the solution with new tubing. D. Return the solution to the pharmacy.

D. Return the solution to the pharmacy. Rationale: Only regular insulin is administered by the IV route, so the TPN solution containing NPH insulin should be returned to the pharmacy (D). (A, B, and C) are not indicated because the solution should not be administered.

The nurse plans to help an 18-year-old developmentally disabled female client ambulate on the first postoperative day. When the nurse tells her it is time to get out of bed, the client becomes angry and yells at the nurse. "Get out of here! I'll get up when I'm ready." Which response should the nurse provide? A. "Your healthcare provider has prescribed ambulation on the first postoperative day." B. "You must ambulate to avoid serious complications that are much more painful." C. "I know how you feel; you're angry about having to do this, but it is required." D. "I'll be back in 30 minutes to help you get out of bed and walk around the room."

D. Returning in 30 minutes provides a cooling off period, is firm, direct, nonthreatening, and avoids argument with the client. B is threatening. C. assumes what the client is feeling. A. avoids the nurse's responsibility to ambulate the client.

When preparing to start change-of-shift report, the charge nurse observes an unlicensed assistive personnel (UAP) walking in the hallway with a urine specimen that is not covered. After telling the UAP to cover the specimen, what intervention should the charge nurse implement? A. Ask the nurse educator to review infection control policies with all UAPs. B. Gather input from other staff about the quality of the UAP's performance. C. Direct the UAP to collect all specimens using universal precautions D. Review infection control guidelines with the UAP at the next opportunity

D. Review infection control guidelines with the UAP at the next opportunity

36. The nurse is assisting a patient to learn self-administration of beclomethasone two puffs inhalation q6hr. The nurse explains that the best way to prevent oral infection while taking this medication is to do which of the following as part of the self-administration techniques? A. Chew a hard candy before the first puff of medication. B. Ask for a breath mint following the second puff of medication. C. Rinse the mouth with water before each puff of medication. D. Rinse the mouth with water following the second puff of medication.

D. Rinse the mouth with water following the second puff of medication. The patient should rinse the mouth with water following the second puff of medication to reduce the risk of fungal overgrowth and oral infection.

The nurse is assisting a patient to learn self-administration of beclomethasone two puffs inhalation q6hr. The nurse explains that the best way to prevent oral infection while taking this medication is to do which of the following as part of the self-administration techniques? A. Chew a hard candy before the first puff of medication. B. Ask for a breath mint following the second puff of medication. C. Rinse the mouth with water before each puff of medication. D. Rinse the mouth with water following the second puff of medication.

D. Rinse the mouth with water following the second puff of medication. The patient should rinse the mouth with water following the second puff of medication to reduce the risk of fungal overgrowth and oral infection.

Which statement reflects the highest priority nursing diagnosis for an older client recently admitted to the hospital for a new-onset cardiac dysrhythmia? A. Diarrhea related to medication side effects B. Anxiety related to fear of recurrent anginal episodes C. Altered nutrition related to high serum lipid levels D. Risk for injury related to syncope and confusion

D. Risk for injury related to syncope and confusion Rationale: The loss of cardiac function in aging decreases cardiac output, so dysrhythmias, particularly tachycardias, are poorly tolerated. With onset of a tachycardic or bradycardic dysrhythmia, cardiac output is compromised further, placing the client at risk of syncope and falling, as well as confusion (D). (A) is of high priority but less so than maintaining client safety. Clients may experience (B) as a result of a newly diagnosed cardiac condition, but this nursing diagnosis does not have the priority of (D). (C) also does not have the priority of (D).

The nurse is assessing a 75-year-old male client for symptoms of hyperglycemia. Which symptom of hyperglycemia is an older adult most likely to exhibit? A. Polyuria B. Polydipsia C. Weight loss D. Infection

D. S/Sx of hyperglycemia in older adults may include fatigue, infection, and neuropathy (such as sensory changes). (A, B, C) are classic symptoms and may be absent in the older adult.

The unit manager of an acute care unit evaluates the time management skills of the nursing staff and determines that one staff nurse is consistently behind in meeting the needs of assigned clients. What action should the unit manager take? A. Plan to reassign some of the clients to another nurse the nest day B. Determine if the nurse is having personal problems that affect work. C. Request that the nursing supervisor meet with the nurse today D. Schedule a private meeting with the staff nurse as soon as possible

D. Schedule a private meeting with the staff nurse as soon as possible

An adolescent male is transferred from the medical unit to the mental health unit because his condition is stable after taking an assortment of prescription drugs. Based on the admission interview, the nurse determines that the client is still having suicidal ideations. What intervention is most important for the nurse to implement? A. Try to determine what life event precipitated the suicide attempt B. Place the client in lock-up until the psychiatrist releases him C. Reassure the client that he is in a safe place D. Search the client's belongings for potential weapons

D. Search the client's belongings for potential weapons

A client with alcohol-related liver disease is admitted to the unit. Which prescription should the nurse question as possibly inappropriate for the client? A. Vitamin K1 (AquaMEPHYTON) 5 mg IM daily B. High-calorie, low-sodium diet C. Fluid restriction to 1500 ml/day D. Pentobarbital (Nembutal sodium) 50 mg at bedtime for rest

D. Sedatives such as Nembutal are contraindicated for clients with liver damage and can have dangerous consequences. (A) is often prescribed since normal clotting mechanism is damaged. (B) is needed to restore energy. (C) Fluids are restricted to decrease ascites which often accompanies cirrhosis, particularly in later stages of the disease.

A female client who had a total thyroidectomy several weeks ago is admitted with myxedema coma. Which finding indicates that the client has been noncompliant with her postoperative treatment plan? A. Systolic blood pressure consistently greater than 160 B. Suppressed levels of thyroid stimulating hormone (TSH) C. Telemetry reveals atrial fibrillation D. Serum T3 and T4 levels below normal

D. Serum T3 and T4 levels below normal

18. In the case of pulmonary embolus from deep vein thrombosis, which of the following actions should the nurse take first? A. Notify the physician. B. Administer a nitroglycerin tablet sublingually. C. Conduct a thorough assessment of the chest pain. D. Sit the patient up in bed as tolerated and apply oxygen.

D. Sit the patient up in bed as tolerated and apply oxygen.The patient's clinical picture is consistent with pulmonary embolus, and the first action the nurse takes should be to assist the patient. For this reason, the nurse should sit the patient up as tolerated and apply oxygen before notifying the physician.

In the case of pulmonary embolus from deep vein thrombosis, which of the following actions should the nurse take first? A. Notify the physician. B. Administer a nitroglycerin tablet sublingually. C. Conduct a thorough assessment of the chest pain. D. Sit the patient up in bed as tolerated and apply oxygen.

D. Sit the patient up in bed as tolerated and apply oxygen.The patient's clinical picture is consistent with pulmonary embolus, and the first action the nurse takes should be to assist the patient. For this reason, the nurse should sit the patient up as tolerated and apply oxygen before notifying the physician.

After admitting a patient to the medical unit with a diagnosis of pneumonia, the nurse will verify that which of the following physician orders have been completed before administering a dose of cefotetan (Cefotan) to the patient? A. Serum laboratory studies ordered for AM B. Pulmonary function evaluation C. Orthostatic blood pressures D. Sputum culture and sensitivity

D. Sputum culture and sensitivityThe nurse should ensure that the sputum for culture and sensitivity was sent to the laboratory before administering the cefotetan. It is important that the organisms are correctly identified (by the culture) before their numbers are affected by the antibiotic; the test will also determine whether the proper antibiotic has been ordered (sensitivity testing). Although antibiotic administration should not be unduly delayed while waiting for the patient to expectorate sputum, all of the other options will not be affected by the administration of antibiotics.

A female client receives a prescription for alendronate sodium (Fosamx) to treat her newly diagnosed osteoporosis. What instruction should the nurse include in the client's teaching plan? A. Eat within 30 minutes of taking the medication B. Ingest an antacid 30 minutes prior to taking the tablet C. Consume a light snack with the medication D. Take on an empty stomach with a full glass of water

D. Take on an empty stomach with a full glass of water

An unlicensed assistive personnel (UAP) informs the nurse who is giving medications that a female client is crying. The client was just informed that she has a malignant tumor. What action should the nurse implement first? A. Provide the client with a PRN antianxiety medication and allow privacy for her to grieve B. instruct the UAP to notify the client's spiritual advisor of her need for counseling C. Ask another nurse to finish giving medications and attend to the client immediately D. Tell the client that the nurse will be back to talk to her after medications are given

D. Tell the client that the nurse will be back to talk to her after medications are given

An emergency department nurse is giving discharge instructions to the wife of a young adult male client who sustained a concussion after a fall. The nurse should provide the wife with what instruction as part of the discharge teaching plan? A. Encourage the wife to bring her husband back to the emergency department if he experiences headaches within the next 24 hours B. Teach the wife how to complete a Glasgow Scale (GCS) to do at home C. Provide written instructions on determining pupil constriction D. Tell the wife to bring her husband back to the emergency department if he has projectile vomiting or an unsteady gait

D. Tell the wife to bring her husband back to the emergency department if he has projectile vomiting or an unsteady gait

The nurse witnesses a baseball player receive a blunt trauma to the back of the head with a softball. What assessment data should the nurse collect immediately? A. Reactivity of deep tendon reflexes, comparing upper to lower extremities. B. Vital signs readings, excluding blood pressure if need equipment is unavailable. C. Memory of events that occurred before and after the blow to the head. D. Ability to spontaneously open the eyes before any tactile stimuli are given.

D. The LOC should be immediately established immediately after the head injury has occurred. Spontaneous eye opening (D) is a simple measure of LOC. (A) is not the best indicator of LOC. (B) is important but not the best indicator of LOC. (C) can be assessed after LOC has been established by assessing eye opening.

A nurse is caring for an elderly client who recently attempted suicide with a overdose of sedatives. Which conclusion regarding this client's achievement of normal development is accurate? A. Suicide attempts that occur in the elderly population are most likely due to declining physical health B. The client was unsuccessful in resolving Trust vs. Mistrust issues, resulting in anger turned inward C. Role confusion often occurs in the elderly due to various levels of dementia, which explains suicidal tenderness D. The adult who is unfulfilled at an advanced age fails to achieve ego integrity, and instead experiences despair

D. The adult who is unfulfilled at an advanced age fails to achieve ego integrity, and instead experiences despair

A client with a large pleural effusion undergoes a thoracentesis. Following the procedure, which observation warrants immediate intervention by the nurse? A. The client complains of pain at the insertion site B. The client's chest x-ray indicates decreased pleural effusion C. The client's arterial blood gases are pH 7.35; PaO2 85; PaCO2 35; HCO3 26 D. The client has asymmetrical chest wall expansion

D. The client has asymmetrical chest wall expansion

When developing a discharge teaching plan for a client after the insertion of a permanent pacemaker, the nurse writes a goal of "The client will verbalize symptoms of pacemaker failure." Which behavior indicates that the goal has been met? A. The client demonstrates the procedures to change the rate of the pacemak er using a magnet. B. The client carries a card in his wallet stating the type and serial number of the pacemaker. C. The client tells the nurse that it is important to report redness and tenderness at the insertion site. D. The client states that changes in the pulse and feelings of dizziness are significant changes.

D. The client states that changes in the pulse and feelings of dizziness are significant changes. Rationale: Changes in pulse rate and/or rhythm may indicate pacer failure. Feelings of dizziness may be caused by a decreased heart rate, leading to decreased cardiac output (D). The rate of a pacemaker is not changed by a client, although the client may be familiar with this procedure as explained by his health care provider (A). (B) is an important step in preparing the client for discharge but does not demonstrate knowledge of the symptoms of pacer failure. (C) indicates symptoms of possible incisional infection or irritation but do not indicate pacer failure.

The nurse assesses a patient with shortness of breath for evidence of long-standing hypoxemia by inspecting: A. Chest excursion B. Spinal curvatures C. The respiratory pattern D. The fingernail and its base

D. The fingernail and its base Clubbing, a sign of long-standing hypoxemia, is evidenced by an increase in the angle between the base of the nail and the fingernail to 180 degrees or more, usually accompanied by an increase in the depth, bulk, and sponginess of the end of the finger.

Twelve hours after chest tube insertion for hemothorax, the nurse notes that the client's drainage has decreased from 50 ml/hr to 5 ml/hr. What is the best inital action for the nurse to take? A. Document this expected decrease in drainage. B. Clamp the chest tube while assessing for air leaks. C. Milk the tube to remove any excessive blood clot build up. D. Assess for kinks or dependent loops in the tubing.

D. The least invasive action should be performed to assess the decrease in drainage. (A) is completed after assessing for and problems causing the decreased drainage. (B) is no longer protocol because the increased pressure may be harmful for the client. (C) is an appropriate nursing action after the tube has been assessed for kinks or dependent loops.

An older client is admitted with a diagnosis of bacterial pneumonia. The nurse's assessment of the client will most likely reveal which S/SX? A. Leukocytosis and febrile. B. Polycythemia and crackles. C. Pharyngitis and sputum production. D. Confusion and tachycardia.

D. The onset of pneumonia is the older may be signaled by general deterioration, confusion, increased heart rate or increased respiratory rate. (A, B, C) are often absent in the older with bacterial pneumonia.

A frail, elderly female with rheumatoid arthritis (RA) complains to the nurse that the weight of the sheets on her legs hurts all the time. Which action should the nurse implement? A. Soak her hands in warm water when resting B. Administer an analgesic at the hour of sleep C. Provide a soft blanket for covering the client D. Use a bed cradle to keep linens off her legs

D. Use a bed cradle to keep linens off her legs

After a 92-year-old client fractured a hip trying to get out of bed, a nurse is accused of failing to notify the healthcare provider that the client was disoriented. In determining whether the nurse is guilty, a jury would consider which standard? A. What the nurse was taught in school about similar client care situations B. What an experienced lawyer would advise to be done in a similar situation C. What a well-educated healthcare consumer would expect in the same situation D. What a reasonable and prudent nurse would have done in the same situation

D. What a reasonable and prudent nurse would have done in the same situation

The nurse is completing a neurological assessment. What observation indicates an abnormal pupil response? A. When shinning the light into the eye, the pupil contracts briskly B. The optic disc appears edematous and engorged C. As the nurse's finger is brought in closer to the eye, the pupils contract D. When shining the light into the right eye, the left pupil does not constrict

D. When shining the light into the right eye, the left pupil does not constrict

The nurse identifies the nursing diagnosis of activity intolerance for a patient with asthma. The nurse assesses for which of the following etiologic factor for this nursing diagnosis in patients with asthma? A. Anxiety and restlessness B. Effects of medications C. Fear of suffocation D. Work of breathing

D. Work of breathingWhen the patient does not have sufficient gas exchange to engage in activity, the etiologic factor is often the work of breathing. When patients with asthma do not have effective respirations, they use all available energy to breathe and have little left over for purposeful activity.

A client presents at the clinic with blepharitis. What instruction should the nurse provide for home care? A. use bilateral eye patches while sleeping to prevent injury to eyes B. wear sunglasses when out of doors to prevent photophobia C. apply cool moist compresses for 20 minutes followed by warm moist compresses D. apply warm moist compresses then gently scrub eyelids with dilute baby shampoo

D. apply warm moist compresses then gently scrub eyelids with dilute baby shampoo

To prevent atelectasis in an 82-year-old patient with a hip fracture, a nurse should A. supply oxygen. B. suction the upper airway. C. ambulate the patient frequently. D. assist the patient with aggressive coughing and deep breathing.

D. assist the patient with aggressive coughing and deep breathing. Decreased mobility after surgery in older adults creates the possibility of fluid buildup and retention in lung tissue. One of the primary goals of nursing intervention is to prevent atelectasis in a high-risk patient. Aggressive coughing and deep breathing can prevent atelectasis in the postoperative patient.

4. After a posterior nasal pack is inserted by a physician, the patient is very anxious and states, "I don't feel like I'm breathing right." The immediate intervention the nurse should initiate is to A. monitor ABGs. B. reassure the patient that this is normal discomfort. C. cut the pack strings and pull the packing out with a hemostat. D. direct a flashlight into the patient's mouth and inspect the oral cavity.

D. direct a flashlight into the patient's mouth and inspect the oral cavity. The nurse should inspect the oral cavity for the presence of blood, soft palate necrosis, and proper placement of the posterior plug. If the posterior plug is visible, the physician should be notified for readjustment of the packing. Reassurance, cutting the strings, and ABGs are not top priority interventions. The nurse needs further data before intervening.

After a posterior nasal pack is inserted by a physician, the patient is very anxious and states, "I don't feel like I'm breathing right." The immediate intervention the nurse should initiate is to A. monitor ABGs. B. reassure the patient that this is normal discomfort. C. cut the pack strings and pull the packing out with a hemostat. D. direct a flashlight into the patient's mouth and inspect the oral cavity.

D. direct a flashlight into the patient's mouth and inspect the oral cavity. The nurse should inspect the oral cavity for the presence of blood, soft palate necrosis, and proper placement of the posterior plug. If the posterior plug is visible, the physician should be notified for readjustment of the packing. Reassurance, cutting the strings, and ABGs are not top priority interventions. The nurse needs further data before intervening.

10. Anticoagulant therapy is used in the treatment of thromboembolic disease because anticoagulants can A. dissolve the thrombi. B. decrease blood viscosity. C. prevent absorption of vitamin K. D. inhibit the synthesis of clotting factors.

D. inhibit the synthesis of clotting factors. Anticoagulant therapy is based on the premise that the initiation or extension of thrombi can be prevented by inhibiting the synthesis of clotting factors or by accelerating their inactivation. The anticoagulants heparin and warfarin do not induce thrombolysis but effectively prevent clot extension.

The practical nurse (PN) reports to the charge nurse that the unlicensed assistive personal (UAP) did not adhere to the agency's fall prevention protocols when caring for a client at risk for falls. What action should the charge nurse implement? A. encourage the PN to complete an adverse occurrence report B. instruct the PN to supervise the UAP more close the next day C. plan to assign the UAP to more stable clients the next day D. meet with the UAP to discuss the observations made by the PN

D. meet with the UAP to discuss the observations made by the PN

31. A patient's ABGs include a PaO2 of 88 mm Hg and a PaCO2 of 38 mm Hg and mixed venous blood gases include a PvO2 of 40 mm Hg and PvCO2 of 46 mm Hg. These findings indicate that the patient has A. impaired cardiac output. B. unstable hemodynamics. C. inadequate delivery of oxygen to the tissues. D. normal capillary oxygen-carbon dioxide exchange.

D. normal capillary oxygen-carbon dioxide exchange. Normal venous blood gas values reflect the normal uptake of oxygen from arterial blood and the release of carbon dioxide from cells into the blood, resulting in a much lower PaO2 and an increased PaCO2. The pH is also decreased in mixed venous blood gases because of the higher PvCO2. Normal mixed venous blood gases also have much lower PvO2 and SvO2 than arterial blood bases. Mixed venous blood gases are used when patients are hemodynamically unstable to evaluate the amount of oxygen delivered to the tissue and the amount of oxygen consumed by the tissues.

A patient's ABGs include a PaO2 of 88 mm Hg and a PaCO2 of 38 mm Hg and mixed venous blood gases include a PvO2 of 40 mm Hg and PvCO2 of 46 mm Hg. These findings indicate that the patient has A. impaired cardiac output. B. unstable hemodynamics. C. inadequate delivery of oxygen to the tissues. D. normal capillary oxygen-carbon dioxide exchange.

D. normal capillary oxygen-carbon dioxide exchange. Normal venous blood gas values reflect the normal uptake of oxygen from arterial blood and the release of carbon dioxide from cells into the blood, resulting in a much lower PaO2 and an increased PaCO2. The pH is also decreased in mixed venous blood gases because of the higher PvCO2. Normal mixed venous blood gases also have much lower PvO2 and SvO2 than arterial blood bases. Mixed venous blood gases are used when patients are hemodynamically unstable to evaluate the amount of oxygen delivered to the tissue and the amount of oxygen consumed by the tissues.

7. The most appropriate position to assist a patient with chronic obstructive pulmonary disease (COPD) who is having difficulty breathing would be a A. high Fowler's position without a pillow behind the head. B. semi-Fowler's position with a single pillow behind the head. C. right side-lying position with the head of the bed at 45 degrees' elevation. D. sitting upright and forward position with arms supported on an over-the-bed table.

D. sitting upright and forward position with arms supported on an over-the-bed table.Sitting upright and leaning forward with arms supported on an over-the-bed table would be of most help to this patient, because it allows for expansion of the thoracic cage in all four directions (front, back, and two sides).

When administering oxygen to a patient with COPD with the potential for carbon dioxide narcosis, the nurse should A. never administer oxygen at a rate of more than 2 L/min. B. monitor the patient's use of oxygen to detect oxygen dependency. C. monitor the patient for symptoms of oxygen toxicity, such as paresthesias. D. use ABGs as a guide to determine what FIO2 level meets the patient's needs.

D. use ABGs as a guide to determine what FIO2 level meets the patient's needs. It is critical to start oxygen at low flow rates and then use ABGs as a guide to determine what FIO2 level is sufficient and can be tolerated.

The nurse is giving preoperative instructions to a 14-year-old client scheduled for surgery to correct a spinal curvature. Which statement by the client best demonstrates that learning has taken place? A."I will read all the teaching booklets you gave me before surgery." B."I have had surgery before, so I know what to expect afterward." C."All the things people have told me will help me take care of my back." D."Let me show you the method of turning I will use after surgery."

D."Let me show you the method of turning I will use after surgery." Rationale: The outcome of learning is best demonstrated when the client not only verbalizes an understanding but can also provide a return demonstration. A 14-year-old client may or may not follow through with option A, and there is no measurement of learning. Option B may help the client understand the surgical process, but the type of surgery may have been very different, with differing postoperative care. In option C, the client may be saying what the nurse wants to hear without expressing any real understanding of what to do after surgery

The nurse is preparing a 45-year-old client for discharge from a cancer center following ileostomy surgery for colon cancer. Which discharge goal should the nurse include in this client's discharge plan? A. Reduce the daily intake of animal fat to 10% of the diet within 6 weeks. B. Exhibit regular, soft-formed stool within 1 month. C. Demonstrate the irrigation procedure correctly within 1 week. D. Attend an ostomy support group within 2 weeks.

D.Attend an ostomy support group within 2 weeks. Rationale: Attending a support group will be beneficial to the client and should be encouraged because adaptation to the ostomy can be difficult. This goal is attainable and is measurable. Option A is not specifically related to ileostomy care. The client with an ileostomy will not be able to accomplish option B. Option C is not necessary.

A client with type 2 diabetes takes metformin (Glucophage) daily. The client is scheduled for major surgery requiring general anesthesia the next day. The nurse anticipates which approach to manage the client's diabetes best while the client is NPO during the perioperative period? A. NPO except for metformin and regular snacks B. NPO except for oral antidiabetic agent C. Novolin N insulin subcutaneously twice daily D. Regular insulin subcutaneously per sliding scale

D.Regular insulin subcutaneously per sliding scale Rationale: Regular insulin dosing based on the client's blood glucose levels (sliding scale) is the best method to achieve control of the client's blood glucose while the client is NPO and coping with the major stress of surgery. Option A increases the risk of vomiting and aspiration. Options B and C provide less precise control of the blood glucose level.

Which question should the nurse ask a client with systemic lupus erythematosus (SLE) in order to determine if the client is experiencing an impending disease exacerbation?

Have you had a fever in the past few weeks?

A nurse is caring for a client who is receiving a unit of packed RBCs. About 15 min following the start of the transfusion, the nurse notes that the client is flushed and febrile, and reports chills. To help confirm that the client is having an acute hemolytic transfusion reaction, the nurse should observe for which of the following manifestations? 1) Urticaria 2) Muscle pain 3) Hypotension 4) Distended neck veins

Hypotension

A nurse is reinforcing teaching about exercise with a client who has type 1 diabetes mellitus. Which of the following statements by the client indicates an understanding of the teaching? 1) "I will carry a complex carbohydrate snack with me when I exercise." 2) "I should exercise first thing in the morning before eating breakfast." 3) "I should avoid injecting insulin into my thigh if I am going to go running." 4) "I will not exercise if my urine is positive for ketones."

I will not exercise if my urine is positive for ketones

An older man with end stage COPD is brought to the ED by his wife because of increasing shortness ob breath and signs of respiratory distress. The client is using his portable oxygen at 2 L/minute per nasal cannula. During the admission process, which action should the nurse take first?

Inquire if the client or his wife has increased the oxygen flow

A nurse is contributing to the plan of care for a client who has labyrinthitis. Which of the following interventions should the nurse include in the plan? 1) Limit fluid intake.. 2) Monitor client's cardinal fields of vision. 3) Encourage ambulation. 4) Ensure the room is brightly lit

Monitor client's cardinal fields of vision

the healthcare provider prescribes the NSAID naproxen 500 mg by mouth twice a day for a client with OA. During a follow up visit one month later, the client tells the nurse "the pills don't seem to be working. They are not helping the pain at all." Which factor should influence the nurse response?

NSAIDs response is variable and another NSAID may be more effective.

After attending a class on reducing cancer risk factors, a client selects bran flakes with 2% milk and orange slices from a breakfast menu. In evaluating the client's learning, the nurse affirms that the client has made good choices and makes what additional recommendation? A. Switch to skim milk. B. Switch to orange juice. C. Add a source of protein. D. Add herbal tea.

Rationale: A Dietary recommendations to reduce cancer risk include reduced consumption of fats, with increased consumption of fruits, vegetables, and fiber. (A) promotes reduced fat consumption. Orange slices provide more fiber than orange juice (B, C, and D) are not standard recommendations for reducing cancer risk.

A resident in a long-term care facility is diagnosed with hepatitis B. Which intervention should the nurse implement with the staff caring for this client? A. Determine if all employees have had the hepatitis B vaccine series. B. Explain that this type of hepatitis can be transmitted when feeding the client. C. Assure the employees that they cannot contract hepatitis B when providing direct care. D. Tell the employees that wearing gloves and a gown are required when providing care.

Rationale: A Hepatitis B vaccine should be administered to all health care providers (A). Hepatitis A (not hepatitis B) can be transmitted by fecal-oral contamination (B). There is a chance that staff could contract hepatitis B if exposed to the client's blood and/or body fluids; therefore, (C) is incorrect. There is no need to wear gloves and gowns except with blood or body fluid contact (D).

A 63-year-old client with type 2 diabetes mellitus is admitted for treatment of an ulcer on the heel of the left foot that has not healed with wound care. The nurse observes that the entire left foot is darker in color than the right foot. Which additional symptom should the nurse expect to find? A. Pedal pulses will be weak or absent in the left foot. B. The client will state that the left foot is usually warm. C. Flexion and extension of the left foot will be limited. D. Capillary refill of the client's left toes will be brisk.

Rationale: A Symptoms associated with decreased blood supply are weak or absent pedal and tibial pulses (A). The client with diabetes experiences vascular scarring as a result of atherosclerotic changes in the peripheral vessels. This results in compromised perfusion to the dependent extremities, which further delays wound healing in the affected foot. Although flexion and extension may be limited (C), depending on the degree of damage, this is not always the case. (B and D) are signs of adequate perfusion of the foot, which would not be expected in this client.

A client with hypertension has been receiving ramipril (Altace), 5 mg PO, daily for 2 weeks and is scheduled to receive a dose at 0900. At 0830, the client's blood pressure is 120/70 mm Hg. Which action should the nurse take? A. Administer the prescribed dose at the scheduled time. B. Hold the dose and contact the health care provider. C. Hold the dose and recheck the blood pressure in 1 hour. D. Check the health care provider's prescription to clarify dose.

Rationale: A The client's blood pressure is within normal limits, indicating that the ramipril, an antihypertensive, is having the desired effect and should be administered (A). (B and C) would be appropriate if the client's blood pressure was excessively low (<100 mm Hg systolic) or if the client were exhibiting signs of hypotension such as dizziness. This prescribed dose is within the normal dosage range, as defined by the manufacturer; therefore, (D) is not necessary.

The nurse is counseling a healthy 30-year-old female client regarding osteoporosis prevention. Which activity would be most beneficial in achieving the client's goal of osteoporosis prevention? A. Cross-country skiing B. Scuba diving C. Horseback riding D. Kayaking

Rationale: A Weight-bearing exercise is an important measure to reduce the risk of osteoporosis. Of the activities listed, cross-country skiing (A) includes the most weight-bearing, whereas (B, C, and D) involve less.

The nurse is reviewing routine medications taken by a client with chronic angle closure glaucoma. Which medication prescription should the nurse question? A. Antianginal with a therapeutic effect of vasodilation B. Anticholinergic with a side effect of pupillary dilation C. Antihistamine with a side effect of sedation D. Corticosteroid with a side effect of hyperglycemia

Rationale: B Clients with angle-closure glaucoma should not take medications that dilate the pupil (B) because this can precipitate acute and severely increased intraocular pressure. (A, C, and D) do not cause increased intracranial pressure, which is the primary concern with angle-closure glaucoma.

Which condition should the nurse anticipate as a potential problem in a female client with a neurogenic bladder? A. Stress incontinence B. Infection C. Painless gross hematuria D. Peritonitis

Rationale: B Infection (B) is the major complication resulting from stasis of urine and subsequent catheterization. (A) is the involuntary loss of urine through an intact urethra as a result of a sudden increase in intraabdominal pressure. (C) is the most common symptom of bladder cancer. (D) is the most common and serious complication of peritoneal dialysis.

Which change in laboratory values indicates to the nurse that a client with rheumatoid arthritis may be experiencing an adverse effect of methotrexate (Mexate) therapy? A. Increase in rheumatoid factor B. Decrease in hemoglobin level C. Increase in blood glucose level D. Decrease in erythrocyte sedimentation rate (ESR; sed rate)

Rationale: B Methotrexate is an immunosuppressant. A common side effect is bone marrow depression, which would be reflected by a decrease in the hemoglobin level (B). (A) indicates disease progression but is not a side effect of the medication. (C) is not related to methotrexate. (D) indicates that inflammation associated with the disease has diminished.

Which abnormal laboratory finding indicates that a client with diabetes needs further evaluation for diabetic nephropathy? A. Hypokalemia B. Microalbuminuria C. Elevated serum lipid levels D. Ketonuria

Rationale: B Microalbuminuria (B) is the earliest sign of diabetic nephropathy and indicates the need for follow-up evaluation. Hyperkalemia, not (A), is associated with end-stage renal disease caused by diabetic nephropathy. (C) may be elevated in end-stage renal disease. (D) may signal the onset of diabetic ketoacidosis (DKA).

Client census is often used to determine staffing needs. Which method of obtaining census determination for a particular unit provides the best formula for determining long-range staffing patterns? A. Midnight census B. Oncoming shift census C. Average daily census D. Hourly census

Rationale: C An average daily census (C) is determined by trend data and takes into account seasonal and daily fluctuations, so it is the best method for determining staffing needs. (A) and (B) provide data at a certain point in time and that data could change quickly. It is unrealistic to expect to obtain an hourly census (D), and such data would only provide information about a certain point in time.

The nurse is completing an admission interview for a client with Parkinson's disease. Which question will provide additional information about manifestations that the client is likely to experience? A. "Have you ever experienced any paralysis of your arms or legs?" B. "Do you have frequent blackout spells?" C. "Have you ever been frozen in one spot, unable to move?" D. "Do you have headaches, especially ones with throbbing pain?"

Rationale: C Clients with Parkinson's disease frequently experience difficulty in initiating, maintaining, and performing motor activities. They may even experience being rooted to the spot and unable to move (C). Parkinson's disease does not typically cause (A, B, or D).

When developing a discharge teaching plan for a client after the insertion of a permanent pacemaker, the nurse writes a goal of "The client will verbalize symptoms of pacemaker failure." Which behavior indicates that the goal has been met? A. The client demonstrates the procedures to change the rate of the pacemaker using a magnet. B. The client carries a card in his wallet stating the type and serial number of the pacemaker. C. The client tells the nurse that it is important to report redness and tenderness at the insertion site. D. The client states that changes in the pulse and feelings of dizziness are significant changes.

Rationale: D Changes in pulse rate and/or rhythm may indicate pacer failure. Feelings of dizziness may be caused by a decreased heart rate, leading to decreased cardiac output (D). The rate of a pacemaker is not changed by a client, although the client may be familiar with this procedure as explained by his health care provider (A). (B) is an important step in preparing the client for discharge but does not demonstrate knowledge of the symptoms of pacer failure. (C) indicates symptoms of possible incisional infection or irritation but do not indicate pacer failure.

A central venous catheter has been inserted via a jugular vein, and a radiograph has confirmed placement of the catheter. A prescription has been received for a medication STAT, but IV fluids have not yet been started. Which action should the nurse take prior to administering the prescribed medication? A. Assess for signs of jugular venous distention. B. Obtain the needed intravenous solution. C. Flush the line with heparinized solution. D. Flush the line with normal saline.

Rationale: D Medication can be administered via a central line without additional IV fluids. The line should first be flushed with a normal saline solution (D) to ensure patency. Insufficient evidence exists on the effectiveness of flushing catheters with heparin (C). (A) will not affect the decision to administer the medication and is not a priority. Administration of the medication STAT is of greater priority than (B).

A client with a vaginal discharge and pruritus is diagnosed a yeast infection (Candidiasis) and is prescribed a 7-day course of an intravaginal tioconazole. What information should the practical nurse provide to the client about using this form of medication?

Remain recumbent for 5 to 15 minutes after insertion of the medication. The client should remain recumbent for 5 to 15 minutes after inserting the medication to facilitate absorption and to prevent loss of medication from the vagina.

An adult client is admitted to the Emergency Department with partial-thickness and full-thickness burns over 40% of the body surface area resulting from a car collision fire. While the health care provider and nurse are preparing to intubate the client, which intervention should the practical nurse (PN) do first?

Remove all the client's clothing, shoes, and jewelry. Interventions for moderate to severe burns of deep partial-thickness and full-thickness, once an airway and circulation is established, then the next thing is to remove all the victims clothing, shoes, and jewelry before the edema sets in and they become constricting, also it is possible to cause more severe burns by leaving clothing on.

A female client who works as a data entry clerk is concerned as to how her recent diagnosis of Raynaud's syndrome is going to affect her job performance. Which instruction should the nurse provide this client?

Use a space heater to keep the workspace warm

A nurse is reinforcing health teaching about skin cancer with a group of clients. Which of the following risk factors should the nurse identify as the leading cause of non-melanoma skin cancer? 1) Exposure to environmental pollutants 2) Sun exposure. 3) History of viral illness 4) Scars from a severe burn

Sun exposure

A client is diagnosed with fluid volume deficit. Which findings would the practical nurse document consistent with fluid volume deficit? (Select all that apply.)

Tachycardia Cool skin Decreased urine output Increased thirst Fluid volume deficit causes tachycardia because the body tries to compensate and pump blood efficiently. Cool skin is consistent with fluid volume deficit. Decreased urine output results from reduced fluid volume perfusing the kidneys. Thirst will be stimulated by the hypothalamus because of decreased fluid volume.

A nurse is reinforcing teaching to a client who is scheduled for an intravenous pyelogram. Which of the following should the nurse include in the teaching? 1) Omit your daily dose of aspirin. 2) Take a laxative the evening before the procedure. 3) Expect to be drowsy for 24 hr following the procedure. 4) You will feel cold chills after the dye has been injected.

Take a laxative the evening before the procedure

Twelve hours following orthopedic surgery on his leg, a male client begins to have acute chest pain and difficulty breathing. Which action should the nurse take first?

Take the client's vital signs and auscultate lungs sounds

A nurse is caring for a client who is postoperative following foot surgery and is not to bear weight on the operative foot. The nurse enters the room to discover the client hopped on one foot to the bathroom, using an IV pole for support. Which of the following actions should the nurse take? 1) Walk the client back to bed immediately and get the client a bedpan. 2) Tell the client to remain in the bathroom after toileting and obtain a wheelchair. 3) Warn the client she might have to be restrained if she gets up without assistance. 4) Keep the bathroom door open to ensure the client is okay.

Tell the client to remain in the bathroom after toileting and obtain a wheelchair

The practical nurse (PN) received report on their assignment of clients. In which order should the PN assess these clients?

The client with aphasia and right hemiplegia who is scheduled for gastrostomy tube (GT) placement today. The client who has chronic renal disease is scheduled for hemodialysis today and three times weekly. The client who has had GI bleeding but had a negative guaiac test for the last three stools. The client who is recovering from a left total knee replacement and who ambulates with a walker. The client with aphasia should be seen first because this client has safety risks related to limited mobility and communication and requires assessment before the GT is placed. The client with chronic renal disease should be seen next to evaluate the impact of fluid balance and potassium on cardiac function between dialysis treatments. The client with GI bleeding is stable and should be seen third to evaluate resolution of bleeding. The ambulatory client is progressing toward independence and is the least likely to need immediate attention.

A nurse is assisting with planning an immunization clinic for older adult clients. Which of the following information should the nurse plan to include about influenza? 1) Individuals at high risk should receive the live influenza vaccine. 2) Immunization for influenza should be repeated every 10 years. 3) The composition of the influenza vaccine changes yearly. 4) The influenza vaccine is necessary only for clients who have never had influenza

The composition of the influenza vaccine changes yearly

A client diagnosed with status asthmatic is admitted to the unit. Which breath sounds would the practical nurse anticipate to hear when auscultating the client's lungs?

Wheezes Wheezes are continuous, high-pitched musical or squeaking-type sounds. They are reflective of the narrowing of the airways as a result of the inflammation from the asthma. Wheezes are generally heard with expiration, but can be heard with inspiration in severe cases of asthma.

A client asks the practical nurse what type of food is the best to eat reduce their chances of getting colon cancer. Which type of food should the PN suggest to the client?

Whole grains According to the American Cancer Society, "studies suggest that fiber in the diet, especially from whole grains, may lower colorectal cancer risk."

An adult male with a history of myocardial infarction and hypertension loses control of his car during a tornado and walks into the emergency department with a head laceration. His vital signs are heart rate 120, respirations 24, blood pressure 150/90, capillary refill time 2 seconds. He is able to follow commands. According to the emergency triage system, which category should the nurse assign this client?

Yellow

The nurse working in a disaster area assesses an adult male who has partial-thickness burns on his lower legs, on approximately 10 % of his lower body. Which color of triage tag should the nurse place on this client?

Yellow

A client is currently receiving an infusion labeled 5% Dextrose Injection 500 mL with heparin sodium 25,000 units at 14 mL/hour. A prescription is received to change the rate of the infusion to heparin 1,000 units/hour. How many mL/hour should the nurse program the infusion pump?

__0.56 ML/hour_


Conjuntos de estudio relacionados

High School (Genelyn) Lesson 84 to 87 Course 18 Medical Office Procedures

View Set

Unit 3: Executive Branch and Bureaucracy

View Set

POS2041: Chapter 11 - Questions - Congress: Balancing National Goals and Local Interests Assignment

View Set